0% found this document useful (0 votes)
44 views255 pages

! Крок 1 Фарм 2007-2018 Іноз Англ

The document consists of a series of multiple-choice questions related to pharmacy and medicine, covering topics such as anatomy, biochemistry, pharmacology, and pathology. Each question presents a clinical scenario or scientific inquiry, followed by a list of possible answers. It serves as a preparatory resource for students or professionals in the field of pharmacy.

Uploaded by

cmlsefri
Copyright
© © All Rights Reserved
We take content rights seriously. If you suspect this is your content, claim it here.
Available Formats
Download as PDF, TXT or read online on Scribd
0% found this document useful (0 votes)
44 views255 pages

! Крок 1 Фарм 2007-2018 Іноз Англ

The document consists of a series of multiple-choice questions related to pharmacy and medicine, covering topics such as anatomy, biochemistry, pharmacology, and pathology. Each question presents a clinical scenario or scientific inquiry, followed by a list of possible answers. It serves as a preparatory resource for students or professionals in the field of pharmacy.

Uploaded by

cmlsefri
Copyright
© © All Rights Reserved
We take content rights seriously. If you suspect this is your content, claim it here.
Available Formats
Download as PDF, TXT or read online on Scribd
You are on page 1/ 255

Krok 1 Pharmacy 2007 1

1. During rocking a passenger became A. Stenocardia


seasick. High excitability of which B. Myocardium infarction
structures causes this phenomenon in the C. Myocarditis
first place? D. Endocarditis
E. Pericarditis
A. Vestibular receptors
B. Vasculomotor centre 7. A 54 y.o. patient with stomach ulcer
C. Vagus nerves complains about great weakness, dyspnea
D. Vomiting centre caused by the slightest physical exercise.
E. Sympathetic system Blood count: erythrocytes - 1, 44 · 1012 /l,
Hb- 66 g/l, colour index - 1,4. What
2. Microscopic examination of a root anemia are these changes of peripheral
cortex in the absorbing zone revealed that blood count typical for?
it consists mainly of multilayer living loose
parenchyma with starch granules. This is: A. B12 -deficiency
B. Iron-deficiency
A. Mesoderm C. Acute posthemorrhagic
B. Endoderm D. Acquired hemolytic
C. Exoderm E. Chronic posthemorrhagic
D. Collenchyme
E. Phellogen 8. What of the given reagents is applied
for determination of aldehyde group?
3. Final urine is generated as a result
of three sequential processes. Name the
most credible sequence:
A. Filtration, reabsorption, secretion
B. Secretion, filtration, reabsorption
C. Reabsorption, filtration, secretion
D. Secretion, reabsorption, filtration
E. - A. [Ag(NH3 )2 ]OH
B. Br2 (H2 O)
4. A plant under examination has a rhi- C. Ca(OH)2
zome, big pinnatisected leaves with sori D. Solution of KMnO4
and sporangia on their undersurface. E. 25% solution of H2 SO4
According to this data the plant should
be related to one of the following divisi- 9. What intracardiac compensation
ons: mechanism is actuated under conditions
of cardiac insufficiency and causes blood
A. P olypodiophyta volume overload?
B. P inophyta
C. Magnoliophyta A. Heterometric
D. Eguisetophyta B. Tachycardia
E. Lycopodiophyta C. Homeometric
D. Myocardium hypertrophy
5. Leaves of a plant under examination E. Increase of respiratory rate
have a distinct main nerve in the middle
with regularly diverging side nerves. What 10. An essential oil plant has a
type of nervation is it? tetraquetrous stem, flowers with bilabi-
ate corolla, its fruit is coenobium. These
A. Pinnate signs are typical for the following family:
B. Digitate
C. Arcwise A. Lamiaceae
D. Parallel B. P apaveraceae
E. Dichotomic C. P olygonaceae
D. Solanaceae
6. A 56 y.o. patient complains of periodical E. Scrophulariaceae
pain attacks in the heart area irradiating to
his left arm, sometimes to the left scapula. 11. Choose reagents for detection of nitri-
These pain attacks can be relieved by ni- te ions in presence of nitrate ions contai-
troglycerine. What heart patholohgy can ned in a pharmaceutical under examinati-
be suspected? on:

1
krok123.in.ua
Krok 1 Pharmacy 2007 2

A. Antipyrin and chlorohydrogen acid A. Pepsin


(diluted) B. Amylase
B. Iron (II) sulfate (diluted) and potassium C. Lipase
iodide D. Chemotrypsin
C. Iron (III) sulfate (concentrated) and E. Trypsin
potassium bromide
D. Iron (II) chloride 18. A child with evident hypotrophy has
edemata of lower extremities, ascites.
E. Iron (III) chloride What is the main factor of pathogenesis
12. A chemist in an analytical laboratory of cachectic edema?
needs to standardize a sodium hydroxide A. Drop of oncotic pressure of blood
solution. What primary standard solution plasm
can be used for this purpose? B. Rise of hydrostatic blood pressure
A. Oxalic acid C. Rise of oncotic pressure of intercellular
B. Acetic acid liquid
C. Chlorohydrogen acid D. Increased permeability of vascular wall
D. Sodium tetraborate E. Disorder of lymph outflow
E. Sodium chloride 19. What factors of humoral regulation sti-
13. Name the complex compound with mulate the function of respiratory centre
antitumour activity: the most actively?

A. [P t(NH3 )2 Cl2 ] A. Carbon dioxide


B. [Co(NH3 )5 NO3 ]Cl2 B. Adrenaline
C. Na4 [Sn(OH)3Cl3 ] C. Acetylcholine
D. [Cu(NH3 )4 (SCN)2 ] D. Thyroxine
E. K2 Na[Co(NO2 )6 ] E. Insulin

14. According to Hardy-Weinberg law 20. In order to keep vitality and stabili-
coagulating effect of coagulant ion is ty of eubiotics microorganisms in frozen
influenced by: state are dried under conditions of high
vacuum. What method is it?
A. Ion charge
B. Ion size A. Lyophilization
C. Adsorbability B. Pasteurization
D. Hydratability C. Tyndallization
E. Polarizability D. Inactivation
E. Hybridization
15. Name the type of reaction applied for
detection of F e3+ cation: 21. Hydrogen is characterized by the
following oxidation rates: −1; 0; +1. The
A. Complexing −1 oxidation rate hydrogen has in:
B. Precipitation
C. Hydrolysis A. Hydrides
D. Neutralization B. Acids
E. Renewing C. Hydroxides
D. Water
16. Pharmaceutists widely apply drugs in E. Acid salts
form of colloid-disperse systems. What
method of sol production is rated as physi- 22. What compounds entered into a reacti-
cal condensation? on if its products were nitrobenzene and
water:
A. Solvent substitution
B. Reduction
C. Oxidation
D. Hydrolysis
E. Double exchange
17. A patient was diagnosed with anacydic
gastritis. What enzyme activity will be
reduced?

2
krok123.in.ua
Krok 1 Pharmacy 2007 3

A.
A. Index of solution refraction
B. Angle of rotation of plane of polarized
light
C. Angle of total internal reflection of light
B. ray
D. Light ray angle
E. Angle of light refraction
27. In order to estimate antibiotic suscepti-
C. bility of a patient doctors introduced
him intracutaneously 0,2 ml of penicilli-
ne solution. Ten minutes after introducti-
on there appeared hyperemy and edema.
D. What type does this reaction relate to
(according to Coomb’s and Gell’s classi-
fication)?
A. Anaphylactic reaction
E. B. Cytotoxic reaction
C. Reaction of Arthus phenomenon type
D. Delayed-type hypersensitivity
E. Tuberculine reaction
28. Name inhibitory transmitters:
23. What disease of blood coagulation
system is based upon abrupt decelerati- A. GABA and glycin
on of blood coagulation due to disturbed B. Adrenaline and noradrenaline
formation of plasma thromboplastin (VIII C. Noradrenaline and dopamine
factor deficit)? D. Serotonin and glycin
E. Acetylcholine and GABA
A. Hemophilia
B. Thrombocytopenic purpura 29. Quantitative determination of
C. Hemorrhagic vasculitis pharmaceutical substances can be done
D. Symptomatic thrombocytopenia by method of acidimetry. Its titrant is the
E. Hemorrhagic purpura secondary standard solution of chloride
acid. What compound helps to determi-
24. To the membrane proteins that contact ne the precise concentration of chloride
with this or that biologically active acid?
substance transmitting information into
the cell belong: A. Sodium tetraborate
B. Oxalate acid
A. Receptor proteins C. Potassium dichromate
B. Pump proteins D. Sodium thiosulfate
C. Enzyme proteins E. Magnesium sulfate
D. Channel proteins
E. Glycocalix 30. Hop sprouts wind around a support
and climb upwards. That means that they
25. What segment of digestive tract are:
secretes digestive juice that has acid
reaction? A. Creeping
B. Recumbent
A. Stomach C. Arrect
B. Oral cavity D. Tenent
C. Small intestine E. Trailing
D. Large intestine
E. Esophagus 31. A patient was prescribed with
bile preparation for the purpose of
26. Concentration of ethyl alcohol in some improvement of rich food digestion. What
drug formulations and tinctures can be components of this preparation take part
determined by means of refractometry. in fat emulsification?
For this purpose the following characteri-
stic is measured:
3
krok123.in.ua
Krok 1 Pharmacy 2007 4

A. Bile acids
B. Cholesterol and its ethers A. Convection
C. Diglyceride B. Evaporation
D. Bilirubin-glucuronids C. Heat radiation
E. Higher fatty acids D. Heat conduction
E. Heat radiation and heat conduction
32. What carbon atoms in the given
compound are in the second valence state 37. What is molecular weight of a gas if its
of (sp2 -hybridization)? hydrogen density is 15?
A. 30 g/mole
B. 7,5 g/mole
C. 15 g/mole
D. 45 g/mole
E. 60 g/mole
A. 1 and 2 38. During an exam arterial pressure
B. 1 and 3 of a student rose and his heartbeat
C. 2 and 3 accelerated. Name the probable cause of
D. 3 and 4 this phenomenon:
E. 5 and 6
A. Increase of tonus of sympathetic
33. According to the requirements of nervous system
WHO and Pharmacopoeia different drug B. Extension of excitability threshold α
dosage forms of unsterile preparations and β adrenoreceptors
are allowed to have a certain quantity C. Increase of circulating blood volume
of bacteria and fungi. What quantity of D. Decrease of tonus of parasympathetic
saprophytic bacteria and fungi in 1 g (ml) nervous system
of a peroral preparation will ensure its E. Excretion of glucocorticoids
safety?
39. Titrant of chelatometry method is
A. 1000 bacteria and 100 mold fungi trilon B solution that forms complex
B. 500 bacteria and 50 mold fungi compounds with metal cations irrespecti-
C. 250 bacteria and 25 mold fungi ve of their valence at a ratio of:
D. 500 bacteria and 200 mold fungi
E. 1500 bacteria and 150 mold fungi A. 1 : 1
B. 1 : 3
34. It is known that digestion of proteins, C. 1 : 2
fats and carbohydrates happens due to D. 2 : 1
protease, lipase and amylase respectively. E. 3 : 1
What digestive juice contains all three
enzyme groups enough for digestion? 40. Name the type of reaction that takes
place during detection of ascorbic acid in
A. Juice of pancreas a preparation by iodometric method:
B. Saliva
C. Gastric juice A. Oxidation-reduction
D. Bile B. Acylation
E. Juice of large intestine C. Neutralization
D. Precipitation
35. Presence of chlorides in drinking water E. Complexing
can be detected by method of mercuri-
metry. One of the following solutions is 41. During quantitative estimation of
used as titrant: glucose by polarimetric method the
following factor is measured:
A. Hg(NO3)2
B. Hg2 (NO3 )2 A. Angle of rotation of polarized beam
C. HgCl2 plane
D. HgSO4 B. Coefficient of light refraction
E. Hg2 Cl2 C. Rate of polarized beam absorption by a
solution
36. Under conditions of high temperature D. Beam dispersion by a solution
of the environment a fan eases being E. Optical density of a solution
in premises because it intensifies heat
transfer by means of: 42. An important role during starvati-
4
krok123.in.ua
Krok 1 Pharmacy 2007 5

on diet belongs to gluconeogenesis that A. F eCl3


helps to maintain normal rate of glucose B. NaOH
in blood. Name the main substrate of this C. H2 SO4 (concentrated)
process: D. CH3 COOH (ice)
E. CH3 OH (H + )
A. Aminoacids
B. Cholesterine 47. What classification criterion
C. Nucleic acids incorporates the following types of
D. Bile acids anemias: posthemorrhagic, hemolytic
E. Acetone and anemia induced by disturbed
hematogenesis?
43. Interaction of catecholamines wi-
th β-adrenoreceptors increases the level A. Pathogenesis
of cyclic adenosine monophosphate B. Etiology
in tissue cells. Name an enzyme that C. Hematogenesis type
catalyzes reaction of cyclic adenosine D. Bone marrow regenerability
monophosphate generation: E. Colour index
A. Adenylate cyclase 48. Erythrocytes contain carbonic acid
B. Phosphodiesterase produced from CO2 and H2 O. What
C. Phosphatase enzyme ensures synthesis of carbonic acid
D. Guanylate cyclase in erythrocytes and its decomposition in
E. Creatine kinase pulmonary capillaries?
44. Bacteritic preparations are subdivided A. Carbonic anhydrase
into groups according to their purpose B. Alkaline phosphatase
and production principles. What group do C. Elastase
the preparations for initiation of active D. Lipase
immunity relate to? E. Amylase
A. Vaccines 49. Sanitary and bacteriological exami-
B. Immune sera nation of air in drug-store premises
C. Immunoglobulins revealed increased content of sanitary
D. Monoclonal antibodies representative microorganisms. What mi-
E. Bacteriophages croorganisms are these?
45. Antivitamins are substances of various A. Golden staphylococcus and hemolytic
structure that limit utilization of vitamins streptococcus
in an organism and have an opposite to B. Diphtheria and tuberculosis bacilli
them action. Name antivitamin of vitamin C. Colon and blue pus bacilli
K: D. Epidermal stafilococcus and sarcina
E. Enterococci and cytrobacter
A. Dicumarol
B. Sulfapyridasine 50. Synthesis of steroid hormons ari-
C. Deoxypyridoxine ses from a precursor that contains
D. Aminopterin cyclopentane perhydrophenantrene ring.
E. Isoniazid Name this precursor:
46. Salicylic acid relates to the phenol aci- A. Cholesterine
ds. Presence of phenolic hydroxyl can be B. Acetyl-CoA
proved by means of reaction with: C. Malonyl-CoA
D. Levulinic acid
E. Tyrosine
51. In pharmaceutical production
synthesis of preparations takes place
under various conditions. In what process
does the entropy stay unchanged?

5
krok123.in.ua
Krok 1 Pharmacy 2007 6

A. Adiabatic A. ∆G=0
B. Isothermal B. ∆F=0
C. Isochoric C. ∆H=0
D. Isobaric D. ∆U=0
E. Polytropic E. ∆S=0
52. Electrolyte solutions are medicinal 57. A patient had an attack of calculous
preparations. What is the maximum value cholecystitis that was accompanied by
of isotonic coefficient for MgSO4 soluti- saponated feces, steatorrhea. These
on? changes are the evidence of disturbance
of the following stage of lipometabolism:
A. 2
B. 4 A. Digestion and absorption
C. 3 B. Transport
D. 5 C. Intermediary metabolism
E. 7 D. Adipose tissue exchange
E. Depositing
53. Pulp of a needle leaf consists of living
tissue with internal ansiform outgrowths 58. Labels of some drugs have the followi-
of membrane. Along these outgrowths the ng inscription: "shake before use!". This
chloroplasts are placed. Name the type of warning is induced by:
this leaf’s parenchyma:
A. Sedimentation
A. Folded B. Coagulation
B. Spongy C. Solubility of disperse systems
C. Palisade D. Insolubility of disperse systems
D. Storage E. -
E. Aeriferous
59. Quantitative content of calcium chlori-
54. Bacteriological control of unsterile de can by measured by method of direct
drugs assumes the possibility of presence chelatometric titration. Choose an indi-
of some microorganism groups. What cator for registering the end point of ti-
groups are these? tration:
A. Sarcina A. Eriochrome black T
B. Colon bacillus B. Phenolphthalein
C. Blue pus bacillus C. Methyl red
D. Aurococcus D. Eosin
E. Hemolytic streptococcus E. Starch
55. Choose a reagent for production of an 60. A plant under examination has a
acetic acid hydrazide from ethyl acetate: storage root; its stems are ribbed and
channelled, hollow; leaves are many ti-
mes pinnatisected, leafstalk has a boot;
inflorescence is the compound umbel;
fruit is the cremocarp with essential oil
canaliculi in the pericarp. Such characteri-
stics are typical for the plants of the
following family:
A. H2 N  NH2
B. NH3 A. Apiaceae
C. H2 N  CH3 B. Solanaceae
D. C6 H5 NH2 C. F abaceae
E. C6 H5 NHNH2 D. Brassicaceae
E. Scrophulariaceae
56. What expression corresponds with
the state of chemical equilibrium under 61. One of fleshy fruits under exami-
constant pressure and temperature? nation is characterized by essential oil
exocarp, spongy mesocarp and overgrown
endocarp consisting of juice sacs. What
fruit was examined?

6
krok123.in.ua
Krok 1 Pharmacy 2007 7

A. Hesperidium A. Fortification and conduction


B. Pepo B. Growth and elongation
C. Cinarodium C. Absorption
D. Drupe D. Fissionable cells
E. Bacca E. Root cap
62. Sodium and potassium chlorides in 68. When fats get into an organism they
preparations can be detected by means are digested and absorbed. What products
of: of fat hydrolysis are absorbed in an intesti-
ne?
A. Argentometry, More’s method
B. Oxidation-reduction titration A. Glycerine, fatty acids
C. Alkalimetry B. Amino acids
D. Acidimetry C. Monosaccharides
E. Chelatometry D. Lipoproteids
E. Polypeptides
63. Adding of a diluted solution of
chlorohydrogen acid to a solution under 69. Analysis of urine composition
examination resulted in formation of whi- revealed changed concentration of sodi-
te caseous sediment. It is the evidence of um ions. Which of hormones provides
presence of the following ions: regulation of sodium ions reabsorption in
nephron canaliculi?
A. Silver
B. Ammonium A. Aldosterone
C. Iron (II) B. Vasopressin
D. Barium C. Somatostatin
E. Iodine D. Adrenaline
E. Parathormone
64. Which of the following cyclic
compounds is ranked with carbocyclic 70. When manufacturing the medicinal
compounds? preparations their yield can be raised by
correct choice of temperature conditions.
A. Benzene What equation determines dependence of
B. Furan equilibrium constant upon temperature
C. Tetrahydrofuran under constant pressure?
D. Pyridine
E. Hexane A. Isobars of chemical reaction
B. isotherms of chemical reaction
65. Addition reaction accompanied by C. Kirchhoff equation
cycle opening is typical for the folowing D. Isochors of chemical reaction
cycloalkane: E. Gibbs-Helmholtz equation
A. Cyclopropane 71. It is known that indirect bilirubin
B. Cyclohexane generated as a result of heme disintegrati-
C. Cyclopentane on is detoxicated in liver. What organic
D. Methyl cyclopentane compound takes part in bilirubin detoxi-
E. Cyclodecane cation in hepatocytes?
66. A sour cherry has shortened principal A. Uridine diphosphate glucuronic acid
axis of inflorescence, pedicles have nearly B. Urea
equal length and emerge like from the C. Mevalonic acid
same point. It is typical for the following D. Lactic acid
type of inflorescence: E. Glycin
A. Umbel 72. Pyridine can be characterized by
B. Corymb reactions of (SE ) electrophylic and (SN )
C. Truss nucleophylic substitution. Low reactivity
D. Ear of pyridine in SE reactions is caused by:
E. Anthodium
67. Section of Helianthus annuus root has
a secondary fascicular formation, it means
that the section was made in the zone of:
7
krok123.in.ua
Krok 1 Pharmacy 2007 8

A. Selivanov’s
B. Lugol’s
A. Electron-acceptor properties of ni- C. Fehling’s
trogen atom D. Chempure
B. Aromatic nature of pyridine nucleus E. Tollens’
C. Alkaline properties
D. Hybridization of carbon atoms 78. A smear from frothy and purulent
E. Cycle size vaginal discharges of a 42 y.o. woman was
stained by Romanovsky-Giemsa method.
73. Argentum nitrate solution was added Its analysis revealed some microorgani-
to a solution containing anions of the sms of flagellates class. What microorgani-
second analytical group. It resulted in ms were the most probably revealed?
generation of light yellow precipitate that
was unsoluble in nitric acid and partly A. T rihomonas vaginaslis
soluble in ammonium solution. What ani- B. Leishmania donovani
ons are contained in the solution? C. T rypanosoma gambiense
D. T rihomonas hominis
A. Bromide-ions E. Lamblia intestinalis
B. Iodide-ions
C. Chloride ions 79. During bacteriological examinati-
D. Sulfide-ions on of sputum of a child with choking
E. Arsenite-ions cough and fever there were revealed
glossy smooth colonies growing on casein-
74. It is known that bluish purple petal charcoal agar and reminding of mercury
coloration of a plant under examination drops. Microscopic examination revealed
varies up to pink or light pink accordi- short Gram-negative bacteria. What
ng to pH of cellular fluid of vacuole. It is microorganism was secured from the
caused by presence of: sputum?
A. Anthocyanins A. Bordetella pertussis
B. Carotins B. Haemophylus inf luenzae
C. Xanthophylls C. Corynebacterium dyphtheriae
D. Phycobilins D. Klebsiella pneumoniae
E. Chlorophylls E. Streptococcus pyogenes
75. Microscopic examination of a stem of 80. It was suspected that among workers
a perennial plant revealed integumentary of serum medications factory of regional
tissue of secondary origin that was formed hemotransfusion station there are carriers
as a result of activity of: of pathogenic aurococcus. What medium
should the material from nasopharynx of
A. Phellogen workers be inoculated of in order to reveal
B. Procambium aurococcous carriage?
C. Cambium
D. Pericycle A. Yolk-salt medium
E. Protoderm B. Endo agar
C. Beef-extract broth
76. Choose a reagent that can be used for D. Ressler’s medium
generation of propanol-2 from acetone: E. Blood agar
A. H2 81. According to Pharmacopoeia requi-
B. CH3 OH rements all drugs for topical administrati-
C. CH3 I on should be monitored for "microbi-
D. HCN ological purity". Detection of the followi-
E. HCOH ng microorganisms is the evidence of
77. Three enumerated test tubes contain uselessness of this drug group in medi-
solutions of glucose, fructose and starch. cal practice:
What reagent can help to detect fructose? A. Aurococci
B. Yeast fungi
C. Humicular staphylococci
D. Mold fungi
E. -

8
krok123.in.ua
Krok 1 Pharmacy 2007 9

82. A premises has high content of A. Platinum and chlorosilver


carbonic gas. A man who entered this B. Copper and glass
premises would experience the followi- C. Chingidron and zink
ng changes of his respiration (depth and D. Hydrogen and glass
rate): E. Antimonial and silver

A. Increased respiration rate and depth 88. Under certain conditions of qualitative
B. Decreased respiration rate and depth analysis K4 [F e(CN)6 ] is a specific reagent
C. Decreased respiration rate and to F e3+ cations. What colour is the preci-
increased respiration depth pitate?
D. Increased respiration rate and
decreased respiration depth A. Blue
E. Respiration would stay unchanged B. White
C. Brown
83. What class is represented by D. Red
nitroglycerine medication used for E. Black
stenocardia treatment?
89. During bacteriological examination of
A. Ester solutions prepared in a drug-store there
B. Nitrogen-containing alcohols appeared red colonies with metallic lustre
C. Ethers that grew on Endo agar. What bacteria
D. Nitroalkanes can they be?
E. Polyatomic alcohols
A. Escherichiae
84. What of the given compounds makes B. Shigellae
the reactions of electrophylic substitution C. Staphilococci
(SE ) the easiest? D. Streptococci
E. Salmonellae
A. Phenol
B. Toluol 90. In the technology of medicinal
C. Chlorobenzene preparation temperature and pressure are
D. Benzaldehyde sustained constant very often. What is this
E. Benzene sulfacid process called?

85. During studying a medicinal herbal A. Isobaric-isothermal


mixture a culture in form of black fluffy B. Isochoric-isothermal
film grew on the nutrient medium. Exami- C. Isobaric
nation of specimen smears revealed non- D. Isochoric
septate mycelium threads with globular E. Isothermal
thickenings at the tips. Name these mi-
croorganisms: 91. Technology of pharmaceutical
preparations requires sometimes that
A. Mucor some processes take place at low
B. Black molds fungus temperatures. In which solution will the
C. Candida crystallization be the first to begin provi-
D. Aspergill ded that solutions have equal molality?
E. Ray fungi
A. C6 H12 O6
86. Name a standard solution of B. NaCl
iodometric determination of reducing C. CaCl2
agents (direct titration)? D. Al2 (SO4)3
E. KBr
A. Solution of I2
B. Solution of KMnO4 92. What compound can be synthesized
C. Solution of Na2 S2 O3 from bromobenzene and bromoethane by
D. Solution of K2 Cr2 O7 Wurtz-Fittig’s reaction?
E. Solution of KI A. Ethylbenzene
87. Choose a pair of electrodes for F eSO4 B. Methylbenzene
determination by method of potenti- C. Bromoethyl benzene
ometric titration: D. O-Bromoethyl benzene
E. P-Diethylbenzene
93. What standard solution (titrant) is
9
krok123.in.ua
Krok 1 Pharmacy 2007 10

used in Folgard’s method of direct titrati- A. Diamide of carbonic acid


on? B. Malamic of carbonic acid
C. Ethyl ether of carbamic acid
A. Ammonium thiocyanate D. Diethyl ether of carbonic acid
B. Sodium chloride E. Dimethyl ether of carbonic acid
C. Silver nitrate
D. Potassium chromate 100. Refinement of glycerin that is a
E. Potassium dichromate component of many drug formulati-
ons can be done by means of activated
94. When studying a stem covered with carbon. What phenomenon underlies this
periderm a researcher came to conclusi- process?
on that gaseous exchange takes place
through: A. Adsorption
B. Cohesion
A. Lenticels C. Adhesion
B. Stomata D. Wetting
C. Pores E. Capillary condensation
D. Throughput cells
E. Hydatodes 101. What form of hypoxia can result from
shock and collapse?
95. In order to identify phenol and sali-
cylic acid we use a solution of: A. Circulatory
B. Respiratory
A. Sodium hydrogen carbonate C. Hypoxic
B. Ferrum chloride (III) D. Hemic
C. Sodium hydroxide E. Histic
D. Sodium chloride
E. Bromine 102. Intravenous injections are performed
with water solution CaCl2 with mass
96. An acid was added to the solutions of concentration 10%. What is the maximum
the given salts. In what case does the gas value of isotonic coefficient CaCl2 in a
emission take place? water solution?
A. Na2 CO3 A. 3
B. K2 SO4 B. 4
C. CuSO4 C. 2
D. Na2 SiO3 D. 5
E. Ca3 (P O4)2 E. 1
97. Quantitative content of iron (II) can be 103. Laboratorians from a physicochemi-
determined by non-indicator method of: cal laboratory prepared water solutions
of urea, glucose, sodium sulfate, alumini-
A. Permanganatometry um sulfate and sodium benzoate with the
B. Chelatometry same molar concentration. Which of the
C. Argentometry given solutions has the highest osmotic
D. Iodometry pressure under 2980K?
E. Nitritometry
A. Aluminium sulfate
98. One of the causes of optical activi- B. Urea
ty is that molecule structure contains an C. Glucose
organic compound: D. Sodium benzoate
E. Sodium sulfate
A. Asymmetric carbon atom
B. Double bond 104. During determination of fruit type
C. Triple bond Hypericum perf oratum it was found that:
D. Functional group the fruit is coebocarpous, dry, opens wi-
E. Asymmetry plane th valves and contains a big number of
seeds. Therefore the fruit of Hypericum
99. Urea is a derivative of carbonic acid. perf oratum is:
Choose the name that is adequate for
urea:

10
krok123.in.ua
Krok 1 Pharmacy 2007 11

A. Fruitcase applied. Solution of ammonium oxalate


B. Multifollicle was used as a precipitator. The gravimetric
C. Follicle form in this case is:
D. Coenobium
E. Aggregate achene A. Calcium oxide
B. Anhydrous calcium oxalate
105. A pharmaceutical factory recei- C. Monohydrous calcium oxalate
ved a batch of phytogenic raw materials D. Calcium carbonate
for manufacturing of phytomedications. E. Calcium hydroxide
What microbiological test should be appli-
ed in order to estimate quality of these 111. A patient ill with pheochromocytoma
raw materials? has high secretion of the following
hormone:
A. General amount of microorganisms in 1
g of raw material A. Adrenaline
B. Coli titer B. Glucagon
C. Coli index C. Insulin
D. Antibacterial activity D. Thyroxin
E. Pyrogens E. Somatotropin
106. First leukocytes that appear in the 112. What of the following metals di-
inflammation focus are: splaces hydrogen from hydrochloric acid?
A. Neutrophils A. Aluminium
B. Monocytes B. Copper
C. Eosinophils C. Mercury
D. Lymphocytes D. Aurum
E. Basophils E. Platinum
107. What medication is formed as a result 113. Some products of amino acid
of interaction of acid with acetic anhydri- decarboxylation are biologically active
de? substances. What CNS inhibition medi-
ator is formed by decarboxylation of
A. Aspirin glutamic acid?
B. Salicyl amide
C. Phenyl salicylate A. GABA
D. Benzyl salicylate B. Putrescine
E. Sodium salicylate C. Histamine
D. Cadaverine
108. Permanganatometric determination E. Asparagine
of H2 O2 can be done in a very acid medi-
um. What acid allows to produce medium 114. Choose an initial compound for one-
for permanganatometric determination? stage synthesis of phthalic acid:
A. H2 SO4 A. O-xylol
B. HCl B. Salicylic acid
C. HNO3 C. 1,2-dichlorobenzene
D. CH3 COOH D. 2-chlorobenzoic acid
E. H3 P O4 E. M-xylol
109. Coulometry is based upon 115. A patient with chronic calculous
measurement of electrics that is spent on cholecystitis complains of acute pains in
electrode reaction. What law underlies the right subcostal area, itch and icteri-
coulometric method? tiousness of skin, multiple punctuate
hemorrhages, saponated and discoloured
A. Faraday law feces (steatorrhea). What type of jaundice
B. Archimedes’ principle is it?
C. Newton law
D. Stokes law A. Mechanic
E. Bouguer-Lambert-Beer law B. Hemolytic
C. Parenchymatous
110. To determine mass concentration D. Hepatic
of calcium in a medical preparation the E. -
gravimetric precipitation method was
11
krok123.in.ua
Krok 1 Pharmacy 2007 12

116. Presence of pathogenic microorgani- A. 10−9 - 10−7 m


sms in the air can be detected by presence B. 10−7 - 10−4 m
of sanitary representative bacteria. C. 10−4 m
Choose bacteria that are indicators of di- D. 10−9 m
rect epidemiological danger: E. 10−9 - 10−4 m
A. Hemolytic streptococci 122. To prepare 50 g of hypertonic sodium
B. Sarcina chloride solution with mass concentration
C. Molds 10% it is necessary to take:
D. Yeast fungi
E. Micrococci A. 5 g of NaCl
B. 0,5 g of NaCl
117. A patient with low immunity, C. 1 g of NaCl
frequent colds is recommended to take D. 25 g of NaCl
ascorutine as a more effective drug than E. 50 g of NaCl
ascorbic acid. What constituent substance
of this preparation intensifies action of vi- 123. The 0,1 M solution of whi-
tamin C? ch substance has the smallest ion
concentration?
A. Vitamin P
B. Vitamin A A. CH3 COOH
C. Glucose B. HCl
D. Lactose C. CaCl2
E. Vitamin D D. H2 SO4
E. NaNO3
118. In order to prevent adipose liver
degeneration after viral hepatitis it is 124. In order to prepare 1 L of 0,1 М
necessary to prescribe the patient li- solution of sulfuric acid (M(H2 SO4) = 98
potropic factors. Name one of them: g/mole) it is necessary to take:
A. Choline A. 9,8 g of H2 SO4
B. Tryptophane B. 980 g of NaCl
C. Allopurinol C. 0,098 g of NaCl
D. Contrical D. 49 g of NaCl
E. Vicasol E. 98 g of NaCl
119. In order to determine CO2 content in 125. What vessels have the minimal linear
the air you can use: speed of sanguimotion?
A. Water solution of Ca(OH)2 A. Capillaries
B. Water solution of NaOH B. Aorta
C. CaO C. Arterioles
D. F e(OH)2 D. Veins
E. Crystalline NaOH E. Large arteries
120. Blood analysis of a patient revealed 126. Osmotic pressure is an importans
high content of the following enzymes: characteristic of biological liquids. Name
creatine kinase (MB-isoform), aspartate a solution in which the osmotic pressure
aminotransferase and LDH 1,2. What has nonconstant value:
pathology should be suspected in this
case? A. Sol of of argentum chloride
B. Clucose
A. Myocardium infarction C. Calcium sulfate
B. Muscular dystrophy D. Sodium chloride
C. Liver cirrhosis E. Magnesium sulfate
D. CNS affection
E. Pancreatitis 127. What salt does the expression K  =
Kw
Ka
for hydrolysis constant correspond wi-
121. Some medicinalpreparations are th?
colloid solutions. Particle size of colloid
disperse systems is in the range:

12
krok123.in.ua
Krok 1 Pharmacy 2007 13

A. NaCN A. Malignant tumour


B. Na2 SO4 B. Dystrophy
C. (NH4 )2 SO4 C. Degeneration
D. (NH4 )3 P O4 D. Benign tumour
E. CH3 COONH4 E. Regeneration
128. Choose reagents for detection of 134. Temperature coefficient of reaction
sulphate-ions in a solution that contai- rate equals 2. By how many times will the
ns carbonate-, sulfate-, thiosulfate-, reaction rate change if the temperature is
phosphate-anions: increased by 400 C?
A. Ba(NO3 )2 , HCl A. By 16 times
B. Ba(NO3 )2 , NaOH B. By 8 times
C. BaCl2 , H2 O C. By 4 times
D. CaCl2 , NH4 OH D. By 32 times
E. AgNO3 , HNO3 E. By 24 times
129. Some proteins in the human organi- 135. Before aniline nitrating it is first acidi-
sm have buffer properties. Which ami- fied in order to protect amino groups from
noacid allows hemoglobine to reveal its oxidation processes. What of the followi-
buffer properties in blood? ng reagents is used for this purpose?
A. Histidine A. (CH3 CO)2O
B. Alanine B. CH3 CHO
C. Isoleucine C. C2 H5 Cl
D. Valine D. HNO2
E. Threonine E. CHCl3 + NaOH
130. Thyroid hormones are related to ami- 136. Transport of some substances is
noacid derivatives. What aminoacid is the accompanied by utilization of metabolic
structure of these hormones based upon? energy (ATP energy). This process is
called:
A. Tyrosine
B. Proline A. Active transport
C. Tryptophane B. Simple diffusion
D. Serine C. Filtration
E. Glutamine D. Osmosis
E. Facilitated diffusion
131. Biochemical meaning of transami-
nation is that amino groups of different 137. What equation can be applied for
aminoacids are assembled in form of one calculation of surface tension of propionic
of aminoacids. What aminoacid is it? acid water solution?
A. Glutamine A. Shishkovsky’s
B. Asparaginic B. Freundlich’s equation
C. Valine C. Gibbs’ equation
D. Leucine D. Helmholtz-Smoluchowski
E. Arginine E. Rayleigh equation
132. A macroscopic alga of brown colour 138. What of the folowing compounds
with trunk, rhizoids and foliaceous part belongs to ketose representatives?
rich in alginates and iodine is ranked with
genus of: A. Fructose
B. Mannose
A. Laminaria C. Iodose
B. Chlorella D. Galactose
C. Chlamydomonas E. Talose
D. S irogira
E. Ulothrix 139. Preventive examination of a
woman revealed enlargement of her
133. What pathology of tissue growth is thyroid gland, exophthalmos, high body
characterized by cellular and tissue atypia temperature, increase of heart rate up
from the point of histomorphology? to 110 times per minute. It is advisable
13
krok123.in.ua
Krok 1 Pharmacy 2007 14

to determine content of the following A. Maceration


hormone in blood: B. Lignification
C. Mineralization
A. Thyroxine D. Sliming
B. Noradrenaline E. Gummosis
C. Adrenaline
D. Insulin 145. A vegetational microspecimen was
E. Cortisol treated with Sudan III solution. As a
result of it cell membranes turned pink
140. A plantation of medicinal plants was that means they contain:
affected by a disease that caused yellow
spots and necrosis areas on the leaves. A. Suberin
Juice of affected plants remains infecti- B. Cellulose
ous after passing through bacterial filter, C. Lignin
but after its inoculation of nutrient media D. Pectin
growth of causative agent is not regi- E. Hemicellulose
stered. What group of phytopathogenic
microorganisms does the causative agent 146. For dichromatometric determinati-
of this disease belong to? on of F eSO4 content in the solution wi-
th potentiometric fixation of equivalence
A. Viruses point the following indicator electrode is
B. Fungi applied:
C. Actinomycetes
D. Bacteria A. Platinum
E. Mycoplasmas B. Glass
C. Quinhydrone
141. Microscopic examination of a leaf D. Argental
revealed on its serratures some water E. Chloroargentic
stomata that serve the purpose of liquid-
drop moisture excretion, that is the 147. Content of potassium dichromate in
process of: a solution was determined by iodometric
method. Name the titrant of iodometric
A. Guttation method for oxidant determination:
B. Gaseous exchange
C. Internal secretion A. Sodium thiosulfate
D. Transpiration B. Sodium hydroxide
E. Photosynthesis C. Potassium iodide
D. Potassium permanganate
142. What osmotic pressure have medical E. Potassium bromate
solutions that are used as isotonic against
blood? 148. Ions of heavy metals are very toxic.
They block SH-groups that are a part of
A. 740 - 780 kPа active centre of enzymes. What is the type
B. 420 - 448 kPа of their inhibition mechanism?
C. 900 - 960 kPа
D. 600 - 670 kPа A. Noncompetitive
E. 690 - 720 kPа B. Allosteric
C. Competitive
143. Examination of a patient revealed D. Uncompetitive
neutrophilic leukocytosis with shift of E. Substrate
leukogram to the right. It is typical for:
149. ATP synthesis in glycolisis under
A. Acute inflammatory process anaerobic conditions takes place by
B. Chronic inflmmatory process means of substrate phosphorilation. In
C. Autoimmune process course of this process the energy of other
D. Allergy high-energy compounds is used. Name
E. Rheumatism such a substance:
144. Destruction of intercellular substance A. Phosphoenolpyruvate
and cell breakaway in overripe fleshy frui- B. Glucose 6-phosphate
ts is a result of: C. Lactate
D. Pyruvate
E. Glucose
14
krok123.in.ua
Krok 1 Pharmacy 2007 15

150. One of the common characteristics A. H2 S2 O7


of subfamily P runda representatives B. H2 S2 O5
(family Rosa) is that their fruit is: C. H2 S2 O8
D. H2 S
A. Drupe E. H2 S4 O6
B. Aggregate-accessory fruit
C. Bacca 156. What of the following compounds has
D. Pome acidofuge properties?
E. Pepo
A. Pyrrole
151. A patient with provisional diagnosis B. Pyrazole
"acute gastroenteritis"was admitted to C. Pyridine
infectious department. Inoculation of D. Pyrimidine
feces on bismuth-sulfite agar resulted in E. Imidazole
growth of black colonies with metallic
lustre. What microorganisms might they 157. A patient was diagnosed with ri-
be? ght lung cancer and doctors administered
him surgical treatment. After right-sided
A. Salmonella pulmonectomy the patient began to suffer
B. Shigella from evident dyspnea. What form of respi-
C. Escherichia ratory failure is it?
D. Yersinia
E. Brucella A. Pulmonary restrictive
B. Central
152. What substance can be used only as C. Peripheral
a reducing agent in oxidation-reduction D. Pulmonary obstructive
reactions? E. Thoracodiaphragmal

A. Na2 S 158. Essential oil glandules that consist


B. Na2 SO3 of 8 secretory cells placed in 2 lines and
C. Na2 SO4 4 tiers are typical for most plants of the
D. Na2 S2 O7 following family:
E. Na2 S2 O3
A. Asteraceae
153. Potassium permanganate KMnO4 B. Apiaceae
is applied in medicine as an antibacteri- C. Lamiaceae
al agent. What chemical properties of D. Rosaceae
KMnO4 provide its bactericidal action? E. Scrophulariaceae

A. Oxidative 159. Antibiotics are classified by sources


B. Acidic of production. Name an antibiotic of
C. Alkaline bacterial origin:
D. Reducing
E. Heat-activated dissociability A. Gramicidin
B. Penicillin
154. Solution with NaCl mass concentrati- C. Tetracycline
on of 0,95% is a part of a saline and can D. Lysozyme
be used in case of significant blood loss. E. Gentamycin
Name reaction of this solution’s medium:
160. What substances can be derived by
A. Neutral (pH = 7,0) alkaline hydrolysis of tripalmitine?
B. Acidic (pH < 7,0) A. Sodium palmitate and glycerin
C. Alkaline (pH > 7,0) B. Glycerin and palmitic acid
D. Very acidic (pH = 1,0) C. Palmitic acid and sodium hydroxide
E. Very alkaline (pH = 12,0) D. Glycerin and sodium stearate
E. Sodium palmitate and water
155. Choose a disulfate acid from the gi-
ven acids: 161. When a man is excited his salivation is
reduced, there is a sensation of dry mouth.
What mediator is excreted from nerve
endings that innervate salivary glands?

15
krok123.in.ua
Krok 1 Pharmacy 2007 16

A. Noradrenalin potassium ferricyanide (II) K4 [F e(CN)6 ]


B. Acetylcholine equals:
C. Serotonin
D. Histamine A. 6
E. GABA B. 2
C. 4
162. Roentgenological examination of D. 3
a patient revealed delayed transition of E. 8
contrast material from the stomach to the
duodenum. It is caused by disturbance of 168. A 50 y.o. patient suffers from hypovi-
the folowing function of digestive tract: taminosis C (scurvy) as a result of contini-
ous improper feeding. Reduced activity of
A. Evacuatory function of stomach which enzyme is the cause of connective
B. Secretory function tissue affection in this pathology?
C. Membrane digestion
D. Water absorption A. Proline hydroxylase
E. Protein digestion B. Alanine aminotransferase
C. Pyruvate carboxylase
163. Sulfation of naphthaline by D. Tryptophane hydroxylase
concentrated sulfuric acid at the E. Glutaminase
temperature over 1600 C leads to formati-
on of: 169. Decreased rate of B6 vitamin in di-
etary intake results in disorder of protein
A. 2-naphthaline sulfacid metabolism. What biochemical processes
B. 1-naphthaline sulfacid in the patient’s organism will become less
C. 3-naphthaline sulfacid active?
D. 4-naphthaline sulfacid
E. 5-naphthaline sulfacid A. Transamination
B. Oxidation-reduction
164. The second Konovalov’s law is appli- C. Phosphorilation
ed to azeotropic solutions that have D. Methylation
extreme points on phase diagrams and are E. Hydrolysis
called:
170. Examination of a patient revealed
A. Azeotropic mixtures symptoms of myocardial insufficiency.
B. Ideal solutions What is the possible cause of cardiac
C. Miscible in all proportions liquids insufficiency of myocardial type?
D. Partially miscible liquids
E. Mutually insoluble liquids A. Infectious myocarditis
B. Aorta coarctation
165. A patient complains of belting pain C. Pulmonary emphysema
in epigastrium. Examination revealed hi- D. Mitral stenosis
gh diastase content in urine, as well as E. Essential hypertension
undigested fat in feces. What pathology
are these occurences typical for? 171. Reaction of mercury excess with di-
luted nitric acid results in release of the
A. Acute pancreatitis following gas:
B. Gastritis
C. Infectious hepatitis A. NO
D. Acute appendicitis B. NH3
E. Enterocolitis C. N2
D. NO2
166. A patient was prescribed L-carnitine E. N2 O
preparation. This preparation provides
transmembrane transfer of the following 172. Chloride lime is used in medicine as
substances: a disinfectant. Its formula is:
A. Higher fatty acids A. CaCl(OCl)
B. Aminoacids B. CaCl2
C. Purine nucleotides C. Ca(ClO3 )2
D. Pyrimidine nucleotides D. Ca(ClO4 )2
E. Glucose E. CaOCl
167. Coordination number of iron in 173. Dehydregenases are enzymes
16
krok123.in.ua
Krok 1 Pharmacy 2007 17

that detach hydrogen atoms from the A. 4


substrate. What enzyme class is lactate B. 1
dehydrogenase related to? C. 2
D. 3
A. Oxidoreductases E. 5
B. Transferases
C. Hydrolases 179. Nitritometric determination of pri-
D. Isomerases mary aromatic amines in acidic medi-
E. Lipases um results in generation of the following
reaction product:
174. Skin diseases can be treated with
pastes. What class of disperse systems can A. Diazonium salt
the pastes be related to? B. Nitrosoamine
C. Nitrose arylenamine
A. Suspensions D. Nitrose antipyrine
B. Emulsions E. Azide
C. Powders
D. Foams 180. A medicinal plant under examinati-
E. Aerosols on has a pistil formed by a big number
of carpels, and a fruitcase that opens with
175. A patient has been suffering from di- small orifices. This is:
abetes mellitus for 10 years. He was deli-
vered to the hospital in grave condition. A. P apaver somnif erum
On the 2nd day of staying at the hospital B. Chelidonium majus
his condition has become abruptly worse: C. Zea mays
he lapsed into a coma, there appeared noi- D. Mentha piperita
sy deep breathing when deep inspirations E. Sanquisorba of f icinalis
took turns with forced expirations with
participation of expiratory muscles. What 181. A leaf of a plant under examination
form of respiratory impairment is it? has a membranous ocrea that envelops
the bottom of internode. Presence of such
A. Kussmaul’s respiration modified stipules is a diagnostic sign of the
B. Stenotic respiration following family:
C. Tachypnea
D. Cheyne-Stokes respiration A. Buckwheat
E. Biot’s respiration B. Gramineae
C. Rosaceae
176. A woman in labour was given a D. Legumes
preparation that activates contractions of E. Solanaceae
smooth uterine muscles. What hormone is
contained in this preparation? 182. A pharmaceutical company received
from a laboratory a delivery order of di-
A. Oxytocin agnostic medications used for studying
B. Gastrin antigenic properties of causative agent.
C. Secretin Name these preparations:
D. Angiotensin
E. Bradykinin A. Diagnostic sera
B. Allergens
177. Patients ill with tuberculosis take a C. Diagnosticums
drug that is an antivitamin of nicotinic D. Immunoglobulins
acid. Name this substance: E. Bacteriophages
A. Isoniazid 183. Nutrients are transported to a
B. Sulfanilamide bacterial cell by different mechanisms.
C. Acrichine One of them is facilitated diffusion that
D. Isoriboflavin is realized by special membrane carrier
E. Oxythiamine proteins. What are they called?
178. What is the maximum valence of A. Permeases
nitrogen taking into account the donor- B. Lyases
acceptor mechanism of covalent bond C. Oxidoreductases
formation? D. Isomerases
E. Ligases
17
krok123.in.ua
Krok 1 Pharmacy 2007 18

184. A patient was prescribed with an anti- A. 0, 9%


tumoral antibiotic that inhibits synthesis B. 0, 3%
of nucleic acids in the cells. What of the C. 0, 5%
following antibiotics has such a mechani- D. 1%
sm of action? E. 3%
A. Actinomycin 190. 3 years ago a patient was diagnosed
B. Tetracycline with chronic glomerulonephritis. Wi-
C. Nystatin thin last 6 months there have been
D. Lincomycin appearing edemata. What underlies their
E. Erythromycin development?
185. Natural fats have liquid or solid consi- A. Proteinuria
stence. What is the main cause of their B. Hyperaldosteronism
existence in this or that aggregate state: C. Indtroduction of nonsteroid anti-
inflammatory medications
A. Ratio of saturated and unsaturated D. Treatment with glucocorticoids
acids E. Vasopressin hyperproduction
B. Presence of hydrogen bonds
C. Molecule sizes 191. Quantitative determination of iodide
D. Molecule solvation can be done by method of:
E. Way of production
A. Oxidation-reduction titration
186. What thermodynamic value is a B. Acid-base titration
criterion of direction of spontaneous C. Chelatometry
processes under conditions of constant D. Spectrophotometry
volume and temperature? E. Precipitating titration
A. Helmholtz energy 192. Bile contains of bile acids. choose one
B. Entropy of them:
C. Gibbs energy
D. Chemical potential A. Cholic
E. Enthalpy B. Glutamine
C. Lactic
187. In course of an experiment a branch D. Arachidonic
of vagus that innervates heart is being sti- E. Pyruvic acid
mulated. What changes of heart activity
will appear in the first place? 193. A process that is characterized by
chemical interaction between adsorbate
A. Heart rate fall and adsorbent is called:
B. Heart force fall
C. Increase of heart rate A. Chemical adsorption
D. Increase of heart force B. Solvation
E. Rise of arterial pressure C. Absorption
D. Desorption
188. Introduction of a hormone into a E. Sedimentation
man’s organism resulted in increased
water reabsorption in kidneys, high 194. A set of elements form allotropic
vascular tone, rise of arterial pressure. modifications. Name an allotropic modi-
What hormone was introduced? fication of oxygen:

A. Vasopressin A. Ozone
B. Adrenaline B. Phosgene
C. Thyroxine C. Quartz
D. Aldosterone D. Corundum
E. Noradrenaline E. Diamond

189. In order to restore a man’s circulati- 195. Hydrogen compounds of which


ng blood volume he was transfused with element can form hydrogen bonds?
blood substitute - isotonic solution NaCl.
What is the concentration of this solution?

18
krok123.in.ua
Krok 1 Pharmacy 2007 19

A. F A. α-amylase
B. C B. Phosphatase
C. Si C. Enterokinase
D. P D. Chemotrypsin
E. I E. Lysozyme
196. Optic isomerism is possible for one of 199. Solution of a medicinal preparati-
the following compounds: on under examination contains cations
of magnesium (II) and aluminium (III).
A. Iodofluorochloromethane (CHJF Cl) What reagent will help to separate these
B. Methane (CH4 ) cations during analysis of this preparati-
C. Chloroform (CHCl3 ) on?
D. Dichloromethane (CH2 Cl2 )
E. Tetrachloromethane (CCl4 ) A. Alkali solution
B. Hydrogen peroxide in acidic medium
197. Stability of concentrated emulsi- C. Argentum nitrate solution
ons can be increased by adding surface- D. Hydrogen peroxide in ammoniac medi-
action substances and high-molecular um
compounds that are: E. Chloride acid solution
A. Emulsifiers 200. What method ensures reliable sterili-
B. Activators zation of biological liquids (serums, soluti-
C. Catalysts ons, enzymes, vitamines etc.) that cannot
D. Solvents be exposed to high temperatures?
E. Absorbents
A. Tyndallization
198. Pancreas secretes an enzyme that B. Dry heat
is aible to break up α − 1,4-glycosidic C. Flowing steam
linkages in a glycogen molecule. What D. Moist steam under pressure
enzyme is it? E. Flaming

19
krok123.in.ua
Krok 1 Pharmacy 2008 1

1. Generation of primary urine in ki- sturbance of the following process in ki-


dneys is induced by filtration in renal dneys:
corpuscles. What components of blood
plasma are absent in the primary urine? A. Glomerular filtration
B. Tubular secretion
A. Proteins C. Tubular reabsorption
B. Amino acids D. Tubular secretion and reabsorption
C. Glucose E. Renal blood flow
D. Urea
E. Ions 7. What reagent should be chosen in order
to detect presence of Ca2+ cation in a
2. Microscopical examination of pri- solution?
mary cortex of a root in its absorption
zone revealed that it consisted mainly of A. (NH4 )2 C2 O4
multilayer living loose parenchyma with B. HCl
amyloid granules. It is called: C. HNO3
D. KCl
A. Mesoderm E. NaBr
B. Endoderm
C. Exoderm 8. Particles of dispersed phase of an
D. Collenchyme emulsion are deformed and look as
E. Phellogene polyhedrons. What emulsion is it?
3. Morphological analysis of an A. High-concentrated
inflorescence revealed that its flowers B. Concentrated
were attached to the same axis at di- C. Diluted
fferent levels but due to different length D. Oil-in water
of peduncle they grew in the same plane. E. Water-in-oil
Such inflorescence is called:
9. A patient has neurasthenic syndrome,
A. Corymb diarrhea, dermatitis. This is associated wi-
B. Anthodium th deficiency of the following vitamin:
C. Glomus
D. Umbel A. Nicotinic acid
E. Spike B. Vitamin K
C. Vitamin D
4. A patient has high concentration of D. Folic acid
chylomicrons in blood, especially after E. Vitamin B12
taking fatty food. He has also type I
hyperlipoproteinemia that resulted from 10. During microbiological inspection of
deficiency of the following enzyme: crude drugs encapsulated bacteria were
revealed. What method was applied for
A. Lipoprotein lipase capsule detection?
B. Adenylate cyclase
C. Protein kinase A. Burry-Gins
D. Phospholipase C B. Ziehl-Neelsen
E. Prostaglandin synthetase C. Neisser
D. Gram
5. A chemist in analytical laboratory E. Ozheshko
needs to standardize solution of sodium
hydroxide. What primary standard soluti- 11. Pharmaceutical preparations of
on can be applied for this purpose? protein hydrolysate are applied for
parenteral proteinic feeding. Hydrolysates
A. Oxalic acid are of full value if they contain essential
B. Acetate acid amino acids. Which of the following ami-
C. Chloride acid no acids relates to the essential ones:
D. Sodium tatraborate
E. Sodium chloride A. Methionine
B. Cysteine
6. A female patient suffers from C. Alanine
chronic glomerulonephritis. Urine D. Serine
analysis revealed proteinuria, hematuria, E. Glycine
leukocyturia. Proteinuria indicates di-
12. An essential oil plant under examinati-
20
krok123.in.ua
Krok 1 Pharmacy 2008 2

on has tetraquetrous stalk, flowers with disperse systems are widely applied in
bilabiate corolla, coenobium fruit. These pharmaceutical practice. What method
characteristics are typical for the followi- of sol production relates to physical
ng family: condensation?
A. Lamiaceae A. Solvent substitution
B. Papaveraceae B. Reduction
C. Polygonaceae C. Oxidation
D. Solanaceae D. Hydrolysis
E. Scrophulariaceae E. Double exchange
13. Name a complex compound that has 19. A child with evident hypotrophy got
antitumoral activity: edemata on his lower extremities, asci-
tes. What is the main mechanism of
A. [P t(NH3 )2 Cl2 ] pathogenesis of cachectic edema?
B. [Co(NH3 )5 NO3 ]Cl2
C. Na4 [Sn(OH)3Cl3 ] A. Drop of oncotic pressure of blood
D. [Cu(NH3 )4 (SCN)2 ] plasma
E. K2 Na[Co(NO2 )6 ] B. Rise of hydrostatic blood pressure
C. Rise of oncotic pressure of intercellular
14. Solution applied as isotonic soluti- fluid
on should have the following osmotic D. Increased permeability of vascular wall
pressure: E. Disturbance of lymph outflow
A. 700 - 800 kPa 20. A patient has impaired mesopic vision,
B. 200 - 300 kPa his photopic vision is normal. What is the
C. 300 - 400 kPa probable cause of such vision anomaly?
D. 500 - 600 kPa
E. 900 - 1000 kPa A. Vitamin A deficiency
B. Hyperopia
15. According to Schultze-Hardy rule C. Cones disfunction
coagulating action of coagulant ion is D. Myopia
affected by: E. Vitamin D deficiency
A. Ion charge 21. Thiocyanatometry is based upon using
B. Ion size of secondary standard solution of potassi-
C. Adsorbability um thiocyanate that should be standardi-
D. Hydratability zed according to the following standard
E. Polarizability solution of:
16. What reaction is applied for detection A. Silver nitrate
of F e3+ cation? B. Hydrochloric acid
C. Sulfuric acid
A. Complexing D. Iron (II) sulfate
B. Precipitation E. Copper (II) nitrate
C. Hydrolysis
D. Neutralization 22. Concentration of potassium di-
E. Reduction chromate in a solution was determined
by means of iodometry. Name a titrant of
17. Dimethyl glyoxime entered into reacti- iodometric method for determination of
on with a solution that contained cati- strong oxidizer:
ons of the IV analytical group (acid-base
classification). The deposition turned cri- A. Sodium thiosulfate
mson. What cation caused this analytical B. Sodium hydroxide
effect? C. Potassium iodide
D. Potassium permanganate
A. Nickel cation (II) E. Potassium bromate
B. Mercury cation (II)
C. Copper cation (II) 23. Solution of potassium chromate was
D. Cadmium cation (II) added to a solution under examination.
E. Cobalt cation (II) As a result of it some yellow deposition
settled down. This deposition cannot be
18. Drugs in form of colloid- dissolved in acetic acid. This means that
21
krok123.in.ua
Krok 1 Pharmacy 2008 3

the solution under examination contains A. Sodium tetraborate


cations of: B. Oxalic acid
C. Potassium dichromate
A. Barium D. Sodium thiosulfate
B. Calcium E. Magnesium sulfate
C. Sodium
D. Cobalt 29. For determination of nitrate ions di-
E. Magnesium phenylamine was added to the soluti-
on under examination. The following
24. Enzyme hyaluronidase breaks changes were observed:
down hyaluronic acid thus increasing
intercellular permeability. Which vitamin A. Generation of blue solution
strengthens vascular walls and inhibits B. Generation of yellow deposition
activity of hyaluronidase? C. Generation of blue deposition
D. Generation of brown gas
A. P E. Emergence of a typical smell
B. A
C. B1 30. A patient was prescribed a bile
D. B2 preparation for better digestion of fatty
E. D food. What components of this preparati-
on cause fat emulsification?
25. A patient underwent an operation.
After it he was prescribed glycosami- A. Bile acids
noglycan that has coagulating action. B. Cholesterol and its ethers
Specify this substance: C. Diglycerides
D. Bilirubinglucuronids
A. Heparin E. Bile pigments
B. Keratan sulfate
C. Hyaluronic acid 31. Antibiotics can be classified accordi-
D. Chondroitin-6-sulfate ng to various principles. According to the
E. Chondroitin-4-sulfate action mechanism cephalosporins relate
to the following group:
26. Examination of an inflorescence of
sweet flag Acorus calamus L. revealed A. Inhibitors of cell wall synthesis
that it was encircled with a covering leaf B. Inhibitors of protein synthesis
(spathe) and small sessile flowers grew C. Inhibitors of respiratory processes
compactly on the thickened pulpy axis. D. Inhibitors of oxidative phosphorilation
Such inflorescence is called: E. Inhibitors of cytoplasmic membrane
synthesis
A. Ear
B. Glomus 32. Plant pathogenic microorganisms
C. Spike relate to various groups. Which of them
D. Umbel causes diseases of medicinal plants most
E. Corymb often?
27. What substance can act as both oxidant A. Fungi
and reducer in oxidation-reduction reacti- B. Viruses
ons? C. Bacteria
D. Actinomycetes
A. SO2 E. Micoplasma
B. SO3
C. CO2 33. It is known that proteins, fats and
D. P bO2 carbohydrates are digested by means of
E. CrO3 proteases, lipases and amylases, respecti-
vely. Which of digestive juices contains all
28. Quantitative determination of these groups of enzymes enough for di-
pharmaceutical substances can be done gestion?
by means of acidimetry. Its titrant is
the secondary standard solution of A. Pancreatic juice
hydrochloric acid. According to which B. Saliva
compound the precise concentration of C. Gastric juice
hydrochloric acid can be determined? D. Bile
E. Juice of large intestine
22
krok123.in.ua
Krok 1 Pharmacy 2008 4

34. Qualitative reaction for determinati- A. Convection


on of Cr(V I) compounds is origination B. Liquid evaporation
of chromium oxide-diperoxide that stains C. Heat radiation
ether layer with blue. What is formula of D. Heat conduction
this chromium compound? E. Heat radiation and conduction
A. CrO5 40. Under conditions of high exterior
B. CrO3 temperature and dry climate heat emissi-
C. Cr2 O3 on will become more intense by means of:
D. CrO
E. NaCrO2 A. Evaporation
B. Radiation
35. Concentrated nitric acid and crystalli- C. Convection
ne lead dioxide were added to a solution D. Conduction
under examination. The solution turned E. -
crimson. This analytical effect indicates
presence of: 41. During an exam a student got high
arterial pressure and palpitation. What ist
A. Manganese (II) the probable cause of this phenomenon?
B. Bismuth (III)
A. Increased tonus of sympathetic nervous
C. Iron (III) system
D. Chromium (III) B. Low excitability threshold of α and β
E. Tantum (II) adrenoreceptors
C. Increased volume of circulating blood
36. Determination of chlorides in D. Decreased tonus of parasympathetic
potable water can be done by means of nervous system
mercurymetry. The following solution is E. Secretion of glucocorticoids
used as a titrant:
42. During preventive examination of a
A. Hg(NO3)2 patient a doctor revealed considerable
B. Hg2 (NO3 )2 weakening of patellar-tendon reflex. What
C. HgCl2 part of CNS might be affected?
D. HgSO4
E. Hg2 Cl2 A. Spinal cord
B. Metencephalon
37. What reagent enables detection of C. Mesencephalon
phenolic hydroxyl? D. Thalamencephalon
A. F eCl3 E. Cerebellum
B. Ag(NH3 )2 OH 43. Gluconeogenesis plays an important
C. NaNO2 (HCl) part in maintaining normal glucose rate in
D. Cu(OH)2 blood during starvation. Name the main
E. - substrate of this process:
38. It is required to diminish pump functi- A. Amino acids
on of patient’s heart. This can be done B. Cholesterol
by means of blockers of the following C. Nucleic acids
membrane cytoreceptors: D. Bile acids
A. β-adrenoreceptors E. Acetone
B. Nicotinic cholinoreceptors 44. A 40 year old woman has been sufferi-
C. Muscarinic cholinoreceptors ng from profuse uterine bleedings for
D. α-adrenoreceptors a long time. Blood count: Нb- 90 g/l,
E. Dopamine receptors
erythrocytes - 3, 9·1012 /l, colour index - 0,6.
39. Under conditions of high exterior What is the main cause of hypochromic
temperature a ventilating fan can relieve anemia?
staying in the premises because it intesifi-
es heat emission by means of: A. Iron loss with blood
B. Increased consumption of iron
C. Nonassimilability of iron
D. Deficiency of vitamin B12
E. Insufficient iron content in food ration
23
krok123.in.ua
Krok 1 Pharmacy 2008 5

45. During sanitary and bacteriological limbs is increased, their spinal reflexes
examination of air in a drugstore it was are intensified, reflex zones are increased.
revealed that the air had high concentrati- What type of CNS disorder is it?
on of sanitary meaningful microorgani-
sms. What microorganisms are these? A. Central paralysis
B. Peripheral paralysis
A. Staphylococcus aureus and hemolytic C. Spinal shock
streptococcus D. Atonic paralysis
B. Diphtheritic and tuberculous bacilli E. Reflex paralysis
C. Colibacilli and blue pus bacilli
D. Epidermal staphylococcus and Sarcina 51. Bacteriological control of unsterile
E. Enterococci and Citrobacter drugs allows presence of small quantity of
some microorganism groups. What group
46. Steroid hormones are synthesi- is meant?
zed out of a precursor that contai-
ns cyclopentanoperhydrophenanthrene. A. Sarcina
Name this precursor: B. Colon bacillus
C. Bacillus pynocyaneus
A. Cholesterol D. Staphylococcus aureus
B. Acetyl-CoA E. Hemolytic streptococcus
C. Malonyl-CoA
D. Levulinic acid 52. As a result of reduced water
E. Tyrosine reabsorption in nephron tubules daily di-
uresis of a patient has increased up to 10
47. In the pharmaceutical production litres. This might be caused by reduced
processes of drug synthesis take place secretion of the following hormone:
under different conditions. Entropy stays
unchanged in the following process: A. Vasopressin
B. Aldosterone
A. Adiabatic C. Parathormone
B. Isothermal D. Thyrocalcitonin
C. Isochoric E. Insulin
D. Isobaric
E. Polytropic 53. A product of complete acetylation of
glycerine relates to the following class of
48. Pulp of a needle leaf consists of living organic compounds:
tissue with inner ansiform protuberances
of membrane and chloroplasts along A. Ester
them. What is type of this leaf’s B. Ether
parenchyma? C. Ketone
D. Acetal
A. Plicate E. Phenol
B. Spongioid
C. Palisade 54. Name a product of ester condensation
D. Storage of acetaldehyde (Tishchenko reaction):
E. Aeriferous
49. A patient suffering from gastric ulcer
for a long time has dramatic emaciation,
skin pallor, appetite loss, aversion to meat
products. Biopsy of mucous membrane
of stomach revealed cellular atypia. What A. Ethyl acetate
pathology are these symptoms typical for? B. Acetone
C. Crotonic aldehyde
A. Malignant tumour of stomach D. Malonic acid
B. Benign tumour of stomach E. Acetoacetic aldehyde
C. Polyposis
D. Hypertrophic gastritis 55. Water-soluble vitamins take coenzyme
E. Helminthic invasion form in an organism. Thiamine di-
phosphate is coenzyme of the following
50. A patient had cerebral haemorrhage vitamin:
that made impossible active motions of
left arm and leg. Muscle tone of these
24
krok123.in.ua
Krok 1 Pharmacy 2008 6

A. B1 Generation of a brown ring indicates


B. B2 presence of:
C. C
D. B6 A. Nitrate ions
E. B12 B. Acetate ions
C. Carbonate ions
56. Excess of ammonia was added to a D. Oxalate ions
solution under examination. The solution E. Phosphate ions
turned bright blue. This indicates presence
of the following ions: 62. What disorders are possible as a result
of thyroid insufficiency during infancy?
A. Copper
B. Silver A. Cretinism
C. Lead B. Nanism
D. Bismuth C. Gigantism
E. Mercury (II) D. Basedow’s disease
E. Itsenko-Cushing syndrome
57. One of the examined soft fruits is
characterized by essential-oil exocarp, 63. A patient suffers from jaundi-
spongioid mesocarp and overgrown ce. Examination revealed that blood
endocarp that consists of juice saccules. plasm had high concentration of indirect
What fruit was under examination? reacting (free) bilirubin, feces and uri-
ne had high concentration of stercobi-
A. Hesperidium lin, concentration of direct reacting
B. Pepo (conjugated) bilirubin was normal. What
C. Multicoccus type of jaundice is it?
D. Drupe
E. Bacca A. Hemolytic
B. Neonatal jaundice
58. Determination of sodium and potassi- C. Parenchymatous
um chlorides in pharmaceuticals can be D. Gilbert’s disease
done by means of: E. Obstructive
A. Argentometry, Mohr method 64. Which of the following cyclic
B. Reduction-oxidation titration compounds relates to the carbocyclic
C. Alkalimetry ones:
D. Acidimetry
E. Chelatometry A. Benzol
B. Furan
59. Diluted solution of hydrochloric acid C. Tetrahydrofuran
was added to a solution under exami- D. Pyridine
nation. This resulted in origin of white E. Hexane
caseous deposition. This is the evidence
of presence of following ions: 65. Ring-opening addition reactions are
typical for the following cycloalkane:
A. Silver
B. Ammonium A. Cyclopropane
C. Iron (II) B. Cyclohexane
D. Barium C. Cyclopentane
E. Iodine D. Methylcyclopentane
E. Cyclodecane
60. What reagent helps to distinguish
glycerine from ethanol? 66. Bromination proceeds with generation
of tribromoderivative in presence of the
A. Cu(OH)2 following substituent X:
B. SOCl2
C. HNO3 (concentrated), in presence of
H2 SO4 (concentrated)
D. P Cl3
E. P Cl5
61. A solution under examination
was added to the solution of F eSO4
in presence of concentrated H2 SO4 .
25
krok123.in.ua
Krok 1 Pharmacy 2008 7

production. What equation determines


dependence of equilibrium constant from
A. Х = OH the temperature under constant pressure?
B. Х = COOH
C. Х = NO2 A. Isobaric lines of chemical reaction
D. Х = CHO B. Isotherms of chemical reaction
E. Х = SO3 H C. Kirchhoff equation
D. Isochores of chemical reaction
67. It is known that rhizome and roots of E. Gibbs-Helmholtz equation
Inula helenium have cavities without disti-
ncts inner boundaries filled with essential 73. A herbaceous plant under exami-
oils. They are called: nation has segmented lacticifers with
anastomoses filled with white latex. This
A. Lysigenous receptacles is typical for:
B. Schizogenous receptacles
C. Resin ducts A. Taraxacum officinale
D. Segmented lacticifers B. Urtica dioica
E. Nonsegmented lacticifers C. Chelidonium majus
D. Anethum graveolens
68. Gastric juice of a patient has decreased E. Thymus vulgaris
concentration of enzymes. What secretory
cells of stomach display disfunction? 74. What compound will be produced as a
result of interaction of aniline with nitrite
A. Chief cells of glands acid?
B. Parietal cells of glands
C. Gland mucocytes
D. Cells of tegumental epithelium
E. G-cells
69. On the root section of Helianthus
annuus a secondary fascicular structure
was found. This means that the section
was made in the zone of:
A. Fixation and conduction
B. Growth and distension
C. Absorption
D. Dividing cells A.
E. Root cap (pileorhiza)
70. Fatty food is digested by means of
several digestive juices. Which of them
enables fat emulsification?
A. Bile B.
B. Saliva
C. Intestinal juice
D. Gastric juice
E. Pancreatic juice
71. Pharmacological effect of enterosgel
(hydrogel of methylosilicic acid) is based C.
upon the following phenomenon that is
typical for disperse systems:
A. Adsorption
B. Adhesion D.
C. Cohesion
D. Moistening
E. Desorption
E.
72. Yield of medical products can
be enhanced by proper choice of
temperature conditions during their
26
krok123.in.ua
Krok 1 Pharmacy 2008 8

75. A few minutes afer repeated cardiac insufficiency has got soft ti-
introduction of penicillin a patient got ssue edemata on his shins. What is the
dyspnea, tongue numbness, hyperemia leading pathogenetic factor of edema
and then skin pallor. The patient also lost development?
consciousness. What is the cause of such a
grave condition? A. Rise of hydrostatic pressure in capillari-
es
A. Anaphylactic shock B. Drop of osmotic pressure in blood
B. Serum sickness plasma
C. Hemolytic anemia C. Rise of oncotic pressure in tissues
D. Acute glomerulonephritis D. Drop of hydrostatic pressure in capi-
E. Bronchial asthma llaries
E. Rise of osmotic pressure in tissues
76. Cations of the third analytical group
(acid-base classification) can be isolated 81. It is known that depending on pH
in course of systematic analysis by means of cellular fluid petal coloration can vary
of the following group reagent: from blue-and-violet to pink and light pi-
nk. This is caused by presence of:
A. 1 M solution of sulfate acid in presence
of ethanol A. Anthocyanins
B. 1 M solution of potassium chromate B. Carotins
C. 0,1 M solution of sodium carbonate C. Xanthophylls
D. 0,1 M solution of ammonium oxalate D. Phycobilins
E. 1 M solution of ammonium carbonate E. Chlorophylls
77. Solution of potassium iodide was 82. Which of the following plants has
added to the solution acidated with sulfate pome fruit?
acid that contained anions of the third
analytical group. Release of free iodine A. Sorbus aucuparia
is observed. What anion are present in the B. Prunus domestica L.
solution? C. Amygdalus communis
D. Rosa majalis
A. Nitrite ion E. Prunus padus
B. Carbonate ion
C. Sulfate ion 83. Bacteriological examination for
D. Bromide ions bacteria carrying of drugstore workers
E. Acetate ions revealed that one of the pharmacists
had bacteria of genus Staphylococcus.
78. During gravimetric determination What morphological pecularities of mi-
of mass fraction of sulfate ions in the crobal cell arrangement are typical for this
magnesium sulfate preparation precipi- genus?
tation is performed by means of bari-
um chloride solution. Precipitated barium A. They are arranged in form of bunch of
sulfate should be rinsed with: grapes
B. They are arranged in form of a chain
A. Diluted solution of sulfate acid C. They are arranged isolatedly
B. Distilled water D. They are arranged in pairs
C. Solution of barium chloride E. They are arranged in tetrads
D. Solution of sodium sulfate
E. Solution of hydrochloride acid 84. Crude herbal drugs must be examined
for yeastlike fungi. What agar can ensure
79. A patient has bradycardia, moderate development of these microorganisms so
hypotension, decrease of basal metaboli- that associating microflora will grow very
sm, edemata. What abnormality can slowly or won’t grow at all?
induce such syndrome?
A. Sabouraud’s peptone agar
A. Thyroid hypofunction B. Endo agar
B. Parathyroid hypofunction C. Meat infusion agar
C. Thyroid hyperfunction D. Milk-salt agar
D. Parathyroid hyperfunction E. Blood agar
E. Adrenal hypofunction
85. A patient took maximally deep breath.
80. A patient suffering from chronic Air volume being in lungs under these
27
krok123.in.ua
Krok 1 Pharmacy 2008 9

conditions is called: on. It revealed 5 small roundish coloni-


es with zone of hemolysis around them.
A. Total lung capacity Inoculations were made on the following
B. Vital lung capacity cultural medium:
C. Tidal volume
D. Residual volume A. Blood agar
E. Inspiratory reserve volume B. Endo agar
C. Meat infusion agar
86. Choose a reagent that can be used for D. Egg yolk and salt agar
production of propanol − 2 out of acetone: E. Lewin’s agar
A. H2 91. Which of the following solutions of
B. CH3 OH the same molality has the highest boiling
C. CH3 I temperature?
D. HCN
E. HCOH A. Al2 (SO4 )3 solution
B. K3 [F e(CN)6 ] solution
87. What reagent enables differentiation C. Saccharose solution
of the following pair of compounds? D. CaCl2 solution
E. NaCl solution
92. During bacteriological analysis of
solutions prepared in a pharmacy some
red colonies with metallic glitter have
grown on Endo agar. What microbes were
revealed?
A. Foelling’s reagent
B. NaHSO3 A. Escherichia
C. NH2 − NHC6 H5 B. Shigella
D. HCN C. Staphylococci
E. H2 N − OH D. Streptococci
E. Salmonella
88. Stable contraction of myofibrilla
of muscle fibers takes place due to 93. Most technological processed in
accumulation of the following ions in the pharmaceutics run in heterogenous
cytoplasm: systems. How many phases has an eutectic
composition under eutectic temperature
A. Calcium of two-component system?
B. Potassium
C. Sodium A. 3
D. Magnesium B. 2
E. Hydrogen C. 5
D. 4
89. For production of phenol ether it is E. 1
necessary to cause reaction of sodium
phenoxide with: 94. Micelle solutions of surfactants are
applied in pharmaceutical production as
stabilizers and solubilizers. What soluti-
on of colloidal surfactants will have the
greatest value of critical concentration of
micelle formation?
A. C9 H19 SO3 Na
B. C14 H29 SO3 Na
C. C16 H33 SO3 Na
A. CH3 Cl D. C12 H25 SO3 Na
B. CH3 OH E. C10 H21 SO3 Na
C. CH4
D. CH3 NH2 95. Pharmaceutic preparation collargol is
E. CH3 C ≡ N a colloid silver solution containing a high-
molecular compound. What is the functi-
90. Examination of air state in drugstore on of this compound?
premises for preparation of injection
drugs was done by method of sedimentati-
28
krok123.in.ua
Krok 1 Pharmacy 2008 10

A. It enhances aggregative stability edemata development?


B. It induces coagulation
C. It facilitates sedimentation A. Hypoalbuminemia
D. It reduces aggregative stability B. Hypoglobulinemia
E. It increases dispersion degree C. Hypocholesterolemia
D. Hypokalemia
96. Which of the following reactions is E. Hypoglycemia
addition reaction?
100. Apical bud of a sprout stops its
development early and growth is realized
due to two lateral buds placed opposite
one another under the apex. Such ramifi-
A. cation is called:
A. Pseudodichotomic
B. Equidichotomic
B. C. Monopodial
D. Nonequidichotomic
E. Bush

C. 101. Choose a diazonium salt among the


given compounds:

D. A.

B.

E.
C.

97. Choose the most stable complex ion


on the ground of values of instability
constants:
A. [F e(CN)6 ]3− Kн = 1 · 10−31
B. [Ag(CN)2 ]− Kн = 1 · 10−21
C. [Ag(NH3 )2 ]+ Kн = 5, 89 · 10−8 D.
D. [Ni(CN)4 ]2− Kн = 1 · 10−22
E. [Co(NH3 )6 ]2+ Kн = 4, 07 · 10−5
98. Change of acid-base properties in the
compounds MnO → MnO2 → Mn2 O7
answers to the following regularity:
E.
A. Acidic properties become stronger
B. Basic properties become stronger
C. Acid-base properties stay unchanged 102. Introduction of glucocorticoi-
D. Acidic properties diminish ds induces strengthening of glucose
E. - concentration in blood. Which of the
following processes will be activated in
99. A 38 year old patient had hepatitis but liver?
didn’t give up alcohol. There appeared
symptoms of hepatocirrhosis along with
ascites and edemata of his lower limbs.
What changes in blood are main factor of
29
krok123.in.ua
Krok 1 Pharmacy 2008 11

A. Gluconeogenesis A. Ascorutinum
B. Glycogenolysis B. Thiamine hydrochloride
C. Oxidation of fatty acids C. Cyanocobalamin
D. Ketogenesis D. Nicotinic acid
E. Glycolysis E. Pyridoxine hydrochloride
103. An electrode composed by scheme 109. What form of hypoxia develops duri-
Au3+ | Au relates to the following type: ng shock and collapse?
A. I type electrodes A. Circulatory
B. II type electrodes B. Respiratory
C. III type electrodes C. Hypoxic
D. Oxidation-reduction electrodes D. Hemic
E. Ion-selective electrodes E. Tissue
104. You need to prepare ammoniac 110. Employees of a physicochemical
buffer solution. For this purpose you laboratory prepared water solutions of
should add the following solution to the urea, glucose, sodium sulfate, aluminium
water solution of ammonia: sulfate and sodium benzoate all of which
had the same molar concentration. What
A. Solution of ammonium chloride solution has the highest osmotic pressure
B. Solution of chloride acid under 298o K?
C. Solution of sulfate acid
D. Solution of potassium chloride A. Aluminium sulfate
E. Solution of sodium sulfate B. Urea
C. Glucose
105. Alkadiene is a name for aliphatic D. Sodium benzoate
carbohydrates with double bonds. Choose E. Sodium sulfate
a general formula for homologous series
of alkadienes: 111. Potassium dichromate K2 Cr2 O7 is
applied as oxidant in acidic medium. What
A. Cn H2n−2 is the product of reduction of dichromate-
B. Cn H2n+2 ion Cr2 O72− under these conditions?
C. Cn H2n
D. Cn H2n+ 1 A. Cr3+
E. Cn H2n−1 B. Cr(OH)3
C. Cr(OH)2
106. Urea is a derivative of carbonic acid. D. [Cr(OH)6]3−
Choose a denomination of urea: E. Cr2 O3
A. Diamide of carbonic acid 112. Blood analysis revealed rise of acti-
B. Monoamide of carbonic acid vity of LDH1 , LDH2 , aspartate ami-
C. Ethylic ether of carbamic acid notransferase, kreatine phosphokinase-
D. Diethylic ether of carbonic acid MB. Biochemical disorder is observed in
E. Dimethylic ether of carbonic acid the following organ:
107. Choose a pair of substances that A. Heart
can be used for standardization of 0,1 M B. Skeletal muscles
solution of KMnO4 : C. Kidneys
A. Na2 C2 O4 , H2 C2 O4 D. Liver
B. K2 CO3 , CH3 COOH E. Pancreas
C. CH3 COOK, H2 C2 O4 113. Limited swelling of gelatine comes to
D. KHC2O4 , HCOOH
E. Na2 C2 O4 , CH3 COOH unlimited one (solution formation) under
the following conditions:
108. A patient complains about gingival
haemorrhage, petechial haemorrhages. A. Heating
What vitamin preparation should be B. Cooling
recommended? C. In presence of SO42 ions
D. In presence of Cl ions
E. If pH medium matches with isoelectric
point

30
krok123.in.ua
Krok 1 Pharmacy 2008 12

114. What compound will be produced 119. You are given 0,05 M solution of
during reduction of methyl ethyl ketone? versene. What is standard substance for
standardization of this solution?
A. Metallic zinc
B. Sodium tetraborate
C. Sodium hydroxide
D. Oxalic acid
E. Potassium dichromate
A. secondary-butyl alcohol
B. Butanol-1 120. A 56 year old patient complains
C. Isobutyl alcohol about limitation of movements and pain
D. tertiary-butyl alcohol in hand joints, mainly at night. Objecti-
E. Propanol-2 vely: there is a disfiguring painful swelling
of affected joints. Blood and urine have
115. A female patient has been treated wi- high concentration of uric acid. What di-
th antibiotics for a long time. Thereafter sease has developed?
examination of smears form vaginal
secretion revealed oval cells with well- A. Gout
defined nucleus, some cells gemmate. B. Pellagra
What preparations can help to confirm C. Phenylketonuria
the diagnosis "candidosis"? D. Alkaptonuria
E. Tyrosinosis
A. Antifungal
B. Antibacterial 121. Electronic microscopy of a cell
C. Antichlamydial revealed mitochondrial destruction. What
D. Antiviral processes are disturbed?
E. Antiprotozoal A. ATP synthesis
116. One of mass production drugs is B. Protein biosynthesis
produced by inactivation of bacterial C. Glycolysis
exotoxin by formalin. What is this drug D. Synthesis of nucleic acids
for? E. Fat synthesis

A. For active immunization 122. What changes will be observed in


B. For serodiagnostic assay the isolated heart after introduction of
C. For passive immunization adrenaline into perfusion solution?
D. For toxinemia treatment A. Heart rate rise
E. For immunocorrection B. Heart rate fall
117. Digestion of proteins in the digesti- C. Heart force fall
ve tract is a complex process of their D. Conduction reduction
hydrolysis till peptides and free amino aci- E. Excitability reduction
ds. What enzymes decompose proteins in 123. Analytical indication of effect of
the duodenum? potassium iodide solution upon unstai-
A. Trypsin, chemotrypsin ned oxidizing anions in presence of
B. Enterokinase, lipase chloroform is:
C. Amylase, maltase A. Brown stain of free iodine
D. Pepsin, gastricsin B. Settling down of white deposition
E. Lipase, phospholipase C. Change of aggregate state
118. Permanganatometry enebles D. Emission of gas bubbles
determination of H2 O2 in high-acidity E. Origination of deposition and its soluti-
medium. What acid can be used for on in reagent excess
production of such medium? 124. Benzoic acid enters into benzene ring
A. H2 SO4 reaction with the following reagent:
B. HCl
C. HNO3
D. CH3 COOH
E. H3 P O4

31
krok123.in.ua
Krok 1 Pharmacy 2008 13

A. HNO3 (k) + H2 SO4 (k)


B. NaOH
C. P Cl3
D. NH3 : t
E. P2 O5 A.
125. A patient diagnosed with botuli-
sm was admitted to the infectious di-
sease hospital. What medication should
be applied in the first place?
A. Antitoxic serum B.
B. Anatoxin
C. Antibiotics
D. Sulfanilamides
E. Nitrofurans
C.
126. Inoculation of hen’s embryos is the
main method of detection of influenza
virus. In order to neutralize associated
bacterial flora in the material under
examination (nasopharyngeal lavage) it
is necessary to add beforehand:
D.
A. Antibiotics
B. Eubiotics
C. Fluorescent serum
D. Leukocytic interferon
E. Ant-influenza gamma globulin
E.
127. Interaction of lactic acid with SOCl2
excess will result in generation of the
following compound: 128. A patient 42 year old sufferi-
ng from chronic calculous cholecysti-
tis complains about acute pain in
the right subcostal area, itching and
skin icteritiousness, multiple petechi-
al haemorrhages, saponified and light-
coloured feces (steatorrhea). What type
of icterus is it?
A. Mechanic
B. Hemolytic
C. Parenchymatous
D. Cythemolytic
E. Hepatocellular
129. Positive sol of iron hydroxide was
generated by method of hydrolysis. What
coagulating ion will have the lowest
coagulation threshold?
A. Phosphate
B. Sulfate
C. Chloride
D. Nitrate
E. Bromide
130. In pharmaceutical synthesis both si-
mple and complex reactions are applied.
Specify the order of the simple reaction of
type 2A + B = 3D:

32
krok123.in.ua
Krok 1 Pharmacy 2008 14

A. 3 analysis revealed high concentration of


B. 2 uric acid and its salts. This state is caused
C. 1 by metabolic disorder of the following
D. 0 substances:
E. 0,5
A. Purines
131. In order to determine CO2 in air the B. Pyrimidines
following substance can be applied: C. Porphyrines
D. Cholesterol
A. Water solution Ca(OH)2 E. Phospholipids
B. Water solution NaOH
C. CaO 137. Albinism is characterized by lacking
D. F e(OH)2 formation of melanin in an organism. This
E. Crystalline NaOH disease is caused by metabolic disorder of
the following amino acid:
132. Anatomico-histochemical analysis
of a petiole revealed living parenchyma A. Phenylalanine
cells with cellulose, angular thickened B. Methionine
membranes under the epiderm and above C. Alanine
the fascicle. This is typical for: D. Glutargine
E. Asparagine
A. Angular collenchyma
B. Spongy perenchyma 138. A boy is 4 year old. Glucose
C. Lamellar collenchyme concentration in blood plasma is 12 milli-
D. Lacunar collenchyme mole/l. This might be caused by deficiency
E. Bast fibers of the following hormone:
133. A compound undeer examination A. Insulin
contains cations of iron (III) and copper B. Glucagon
(II). What group reagent can separate C. Cortisol
these cations? D. Somatotropin
E. Adrenocorticotropin
A. Concentrated ammonia solution
B. Solution of sodium hydroxide and 139. You have to carry out a qualitati-
hydrogen peroxide ve analysis. What substance will originate
C. Concentrated solution of hydrochloride from chromium ions under the influence
acid of group reagent excess (solution of sodi-
D. Solution of sodium hydroxide um hydroxide) upon cations of the IV
E. Concentrated solution of sulfuric acid analytical group?
134. Heme (a constituent part of A. Sodium hexahydroxochromate (III)
hemoglobin) is a complex iron compound. B. Chromium (III) hydroxide
It relates to the following type of complex C. Chromium (III) oxide
compounds: D. Chromium (II) hydroxide
E. Chromium (II) oxide
A. Chelate complex
B. Acidocomplex 140. Which of the given bases is a weak
C. Aquacomplex electrolyte?
D. Cationic complex
E. Hydroxocomplex A. Mg(OH)2
B. Ca(OH)2
135. Which compound was generated due C. Ba(OH)2
to nonpolar covalent type of chemical D. NaOH
bond? E. KOH
A. H2 141. During study of pharmaceutical
B. KCl substances pH rate can be determi-
C. NH4 Cl ned by method of potentiometry. What
D. KI electrode can be used as an indicator duri-
E. H2 S ng measuring of pH solution?
136. Patient’s joints are enlarged, look
like thickened disfigured knots. Blood
33
krok123.in.ua
Krok 1 Pharmacy 2008 15

A. Glass ons of pharmaceutical substances have


B. Copper molal concentration of 0,1 mole/kg. Whi-
C. Silver-chloride ch solution has the maximal boiling-point
D. Calomel elevation?
E. Zinc
A. Sodium acetate
142. A 40 year old patient complains B. Glucose
about general weakness, headache, body C. Nicotinic acid
temperature rise, cough with sputum, D. Ethanol
dyspnea. After examination his illness was E. Ascorbic acid
diagnosed as focal pneumonia. What type
of hypoxia is observed? 147. Thyroid hormones are derivatives of
amino acids. What amino acid underlies
A. Respiratory the structure of these hormones?
B. Circulatory
C. Hemic A. Tyrosine
D. Tissue B. Proline
E. Hypoxic C. Tryptophan
D. Serine
143. If concentrated HNO3 interreacts wi- E. Glutamine
th copper, it can be reduced to the followi-
ng compound: 148. Which of the following sterilizati-
on methods ensures total death of mi-
A. NO2 croorganisms and their spores during one-
B. NO time thermal processing of an object?
C. N2 O
D. N2 A. Autoclaving
E. NH4 NO3 B. Boiling
C. Tyndallization
144. The given reaction is called: D. Pasteurization
E. -
149. Manganese and chlorine demonstrate
the most similar properties when they
have the following oxidation number:
A. +7
B. +3
C. 0
D. +4
E. +2
150. Choose a generalized reaction that
A. Acylation will help to reveal an amino group in the
B. Esterification following compounds:
C. Addition
D. Removal
E. Regrouping
145. A 58 year old patient complained
about persistent rise of arterial pressure.
Clinical examination revealed chronic
renal disease accompanied by disturbance
of renal blood flow. Rise of arterial A. Isonitrile assay
pressure was induced by activation of the B. Diazotization
following regulatory system: C. Azo dye generation
D. Alkylation
A. Renin-angiotensin E. Acylation
B. Parasympathetic nervous
C. Sympathetic nervous 151. 150 ml of meat broth was introduced
D. Sympathoadrenal into the gastric cavity of an experimental
E. Hypothalamo-pituitary-adrenal dog through the feeding tube. This will
result in rapid rise of concentration of the
146. All the undermentioned water soluti- following hormone in the animal’s blood:
34
krok123.in.ua
Krok 1 Pharmacy 2008 16

A. Gastrin A. HCl
B. Vasointestinal polypeptide B. HCN
C. Neurotensin C. H2 CO3
D. Somatostatin D. CH3 COOH
E. Insulin E. H2 SO3
152. What denomination corresponds wi- 157. What is oxidation number of the
th the given formula? central atom in the compound H[AuCl4 ]?
A. +3
B. 0
C. +1
D. +2
E. +4
A. Benzo [b] pyridine 158. A child got burn on his hand caused
B. Benzo [b] pyrone-4 by hot water. Burn skin is bright red. What
C. Imidoazopyrimidine disturbance of local blood circulation is it?
D. Pyrazinopyrimidine
E. Benzothiazole A. Arterial hyperemia
B. Venous hyperemia
153. According to the IUPAC C. Stasis
nomenclature the given compound has D. Thrombosis
the following denomination: E. Embolism
159. A plantation of medicinal plants was
affected by a disease that caused yellow
spots and necrotic areas on leaves. Jui-
ce of affected plants remains infectious
even after bacterial filtration but after its
A. 1, 2, 3 − propanetriol inoculation on cultural medium growth of
B. 1 − propanol causative agent wasn’t observed. Causati-
C. 2 − propanol ve agent of this disease relates most
D. 1 − propanethiol probably to the following group of plant
E. 1, 2 − propanediol pathogenic microorganisms:
154. A 13 year old child complains A. Viruses
about poor appetite, pain in the right B. Fungi
subcostal area. Microscopical examinati- C. Actinomycetes
on of duodenal contents revealed big D. Bacteria
pyriform cells with two nuclei. What mi- E. Mycoplasma
croorganism was revealed?
160. It is known that a peroral drug contai-
A. Lamblia ns over 1 billion of living microbal cells
B. Trichomonad per 1 millilitre. Nonetheless the drug was
C. Amoeba accepted as applicable. What drug group
D. Trypanosoma does it relate to?
E. Toxoplasma
A. Eubiotics
155. Systems relate to colloid-disperse B. Antibiotics
ones if size of their particles is within the C. Vitamins
following range: D. Sulfanilamides
E. Immunostimulants
A. 10−9 – 10−7 m
B. 10−7 – 10−4 m 161. Microscopical examination of a leaf
C. > 10−4 m revealed water stomata on its serration.
D. ≶ 10−9 m These stomata are for exudation of liquid-
E. 10−9 – 10−4 m drop moisture. This process is called:
156. 0,1 M solution of the following acid
has the highest concentration of hydrogen
ions:
35
krok123.in.ua
Krok 1 Pharmacy 2008 17

A. Guttation A. Ischemia
B. Gas exchange B. Thrombosis
C. Internal secretion C. Embolism
D. Transpiration D. Arterial hyperemia
E. Photosynthesis E. Venous hyperemia
162. A man has symptoms of cardi- 167. Choose an appropriate indicator for
ovascular atherosclerosis. The most fixation of titration end point in method
probable characteristic of this state will of bromatometry:
be growth of the following biochemical
value: A. Methyl red
B. Phenolphthalein
A. Concentration of low-density li- C. Starch
poproteins D. Methyl blue
B. Concentration of high-density li- E. Tropeolin 00
poproteins
C. Concentration of chylomicrons 168. Decarboxylation of 5-hydroxytryptophane
D. LDH5 activity gives origin to a certain biogenic ami-
E. Activity of pancreatic lipase ne with vasoconstrictive action. What bi-
ogenic amine is it?
163. Presence of the following ion of d-
elements in solutions can be exploited by A. Serotonin
means of K4 [F e(CN)6 ]: B. Histamine
C. Gamma-aminobutyric acidc
A. F e3+ D. Putrescine
B. Zn2+ E. Cadaverine
C. Cr 3+
D. Ni2+ 169. An annual plant of the Asteraceae
E. Cu2+ family has tripartite leaves, apical
anthodia with tubular flowers, flat
164. Sol is one of drug forms. What achenocarps that are tenent due to 2-3
happens if sols are fused with oppositely bristly serratures. This plant is:
charged granules?
A. Bidens tripartita
A. Mutual coagulation B. Chamomilia recutita
B. Thixotropy C. Centaurea cyanus
C. Sedimentation D. Echinacea purpurea
D. Contraction E. Artemisia vulgaris
E. Lyophilization
170. Crop production includes cultivation
165. A patient has obstructive respiratory of medicinal essential oil plants that don’t
failure. Name a disease that is usually grow in Ukraine wildely, namely Mentha
accompanied by such type of respiratory piperita, Ortosiphon stamineus, and also:
failure:
A. Salvia officinalis
A. Bronchial asthma B. Origanum vulgare
B. Pneumonia C. Leonurus cardiaca
C. Exudative pleuritis D. Thymus serpyllum
D. Pneumoconiosis E. Leonurus quinquelobatus
E. Pneumothorax
171. A patient was admitted to the infecti-
166. A patient is 54 year old. After intense ous department of a hospital. His provisi-
emotional stress he felt strong pain behi- onal diagnosis was "acute gastroenteritis".
nd his breastbone irradiating to his left Inoculation of feces on bismuth-sulfite
arm and left part of his neck. He felt also agar induced growth of black colonies wi-
death anxiety and broke into a cold sweat. th metallic glitter. What microorganisms
Nitroglycerine relieved pain. Name a di- should you think of?
sturbamce of local blood circulation in
heart that has developed in this case: A. Salmonellae
B. Escherichia
C. Shigella
D. Yersinia
E. Brucella
36
krok123.in.ua
Krok 1 Pharmacy 2008 18

172. Potassium permanganate KMnO4 is


used in medicine as a bactericidal drug.
What chemical properties of KMnO4
determine its bactericidal action?
A. Oxidative
B. Acidic
C. Basic A.
D. Reducing
E. Heat-activated dissociability
173. Sodium hydrogen arsenate
Na2 HAsO4 · 7H2 O is applied in medici-
ne as general health-improving and tonic
drug. What type of salts does it relate to?
A. Acid salts B.
B. Neutral salts
C. Basic salts
D. Double salts
E. Mixed salts
174. Oxidation of menthol by potassium
dichromate in sulfuric acid (chrome mi-
xture) results in production of: C.

D.

E.

175. Isoelectric point of protein equals


8,3. Electrophoretic mobility of protein
macromolecule will be equal zero if pH
value is:
A. 8,3
B. 7,0
C. 11,5
D. 2,3
E. 4,7
176. A male patient who suffers from
chronic intestinal obstruction has intensi-
fied putrefaction of proteins in the colon.
What toxic substance originates from
tryptophane in this case?

37
krok123.in.ua
Krok 1 Pharmacy 2008 19

A. Indole
B. Bilirubin A. Proline hydroxylase
C. Lactate B. Alanine aminotransferase
D. Kreatine C. Pyruvate carboxylase
E. Glucose D. Tryptophane hydroxylase
E. Glutaminase
177. A patient takes blocker of muscari-
nic cholinoreceptors of parasympathetic 183. Low rate of vitamin B6 in the dietary
nerve organ synapses. What changes of intake leads to disturbance of protein
heart activity will be observed? metabolism. What biochemical processes
in the patient’s organism will become less
A. Heart rate rise active?
B. Heart rate and heart force fall
C. Heart rate fall A. Transamination
D. Heart force fall B. Reduction-oxidation
E. Prolongation of atrioventricular delay C. Phosphorilation
D. Methylation
178. Anxious condition can be characteri- E. Hydrolysis
zed by reduced salivation and sense of
dry mouth. What mediator is exuded out 184. During blowing up a baloon a boy
of nerve terminals innervating salivary took maximally deep and prolonged inspi-
glands? rations and expirations. Thereafter he
felt slight dizziness. What is the probable
A. Noradrenaline cause of this phenomenon?
B. Acetylcholine
C. Serotonin A. Drop of pCO2 in blood
D. Histamine B. Rise of pCO2 in blood
E. GABA C. Bronchi constriction
D. Arterial pressure rise
179. What mediator provides informati- E. Drop of pO2 in blood
on transmission from nerve terminations
of motoneurons to the fibers of skeletal 185. A child has got a burn. Burnt skin is
muscles? hyperemic, there are small vesicles full of
transparent fluid. What type of fluid is it?
A. Acetylcholine
B. Adrenaline A. Serous exudate
C. Noradrenaline B. Hemorrhagic exudate
D. Serotonin C. Purulent exudate
E. GABA D. Transsudate
E. Putrid exudate
180. Bacca fruit is typical for the following
representative of Solanaceae family: 186. As a result of reaction of mercury
excess with diluted nitric acid the followi-
A. Atropa belladonna ng gas will escape:
B. Hyoscyamus niger
C. Datura stramonium A. NO
D. Nicotiana tabacum B. NH3
E. Datura innoxia C. N2
D. N2 O
181. A male patient has pain in the right E. -
subcostal area, acholic feces. Decolourati-
on of feces is caused by deficiency of: 187. A biological system (living organism)
exchanges material and energy with the
A. Stercobilin environment. What system does it relate
B. Hemoglobin to?
C. Bilirubin
D. Bile acids A. Open, heterogenous
E. Skatole B. Isolated, heterogenous
C. Closed, homogenous
182. A patient is 50 years old. Ad a result D. Closed, heterogenous
of continuous improper feeding he got E. Open, homogenous
hypovitaminosis C. Lesion of connecti-
ve tissue is caused by low activity of the 188. A patient has been suffering from di-
following enzyme: abetes mellitus for 10 years. He was deli-
38
krok123.in.ua
Krok 1 Pharmacy 2008 20

vered to a hospital in grave condition. the following family:


On the 2nd day of treatment his condi-
tion grew significantly worse: he lapsed A. Polygonaceae
into a coma, there appeared noisy deep B. Gramineae
breathing. Deep inspirations took turns C. Rosaceae
with forced expirations with assistance of D. Legumes
expiratory muscles. What form of respi- E. Solanaceae
ration disorder is it?
194. A bacterial cell obtains nutrients by
A. Kussmaul’s respiration different ways. One of them is the facili-
B. Stenotic respiration tated diffusion that is realized by speci-
C. Tachypnea al membrane carrier proteins. What are
D. Cheyne-Stokes respiration these proteins called?
E. Biot’s respiration
A. Permeases
189. What is maximal valency of ni- B. Lyases
trogen in consideration of donor-acceptor C. Oxidoreductases
mechanism of covalent bond? D. Isomerizing enzymes
E. Ligases
A. 4
B. 1 195. Name type of bond between
C. 2 complementary bases:
D. 3
E. 5
190. What cations added to the solution of
potassium iodide form orange-red deposi-
tion that is soluble in reagent excess and
builds up a colourless solution?
A. Mercury (II)
B. Mercury (I)
C. Bismuth
D. Antimony (V)
E. Lead
191. Microscopical examination of
transverse section of a root revealed A. Hydrogen bond
investing tissue consisting of thin-walled, B. Covalent π-bond
closely joining cells with root fibrilla. This C. Ionic bond
tissue is called: D. Covalent σ-bond
E. Semipolar bond
A. Epiblem
196. You need to specify a monocarpous
B. Root cap (pileorhiza) one-seeded fruit with hard scleroid
C. Periderm endocarp and soft mesocarp. This fruit is:
D. Endoderm
E. Epiderm A. Monodrupe
B. Legume
192. Monopodial inflorescences of C. Silique
plantain (spike) and maize (ear) have one D. Capsule
trait in common: their flowers are placed E. Bacca
on the well-developed principal axis. This
is typical for the following inflorescences: 197. One of the herbarium specimens of
medicinal plants relates to the Asteraceae
A. Simple botrioid family. This plant is:
B. Complex botrioid
C. Cymose A. Arctica lappa
D. Aggregate B. Atropa belladonna
E. Thyrsoid C. Cassia acutifolia
D. Urtica dioica
193. A leaf has glumaceous ochrea. It E. Rubus idaeus
clasps bottom of internode and is a modi-
ficated stipule. This is diagnostic sign of 198. Tritane relates to:
39
krok123.in.ua
Krok 1 Pharmacy 2008 21

to separate these cations during analysis


of this drug?
A. Alkali solution
B. Solution of hydrogen peroxide in acidic
medium
C. Solution of silver nitrate
D. Ammonia solution
E. Solution of chloride acid
A. Multinuclear arenes with isolated 200. What method ensures reliable sterili-
benzene cycles zation of biological fluids (sera, solutions,
B. Multinuclear arenes with condensated
benzene cycles enzymes, vitamines etc.) that can’t stand
C. Mononuclear arenes high temperatures?
D. Alkanes A. Tyndallization
E. Alkenes B. Dry-heat sterilization
199. A drug solution under examination C. Flowing steam
D. Moist steam under pressure
contains cations of magnesium (II) and E. Flaming
aluminium (III). Which reagent can help

40
krok123.in.ua
Krok 1 Pharmacy 2009 1

1. Microscopical examination of pri- A. ATP synthesis


mary cortex of a root in its absorption B. Glycogen synthesis
zone revealed that it consisted mainly of C. Amino acid synthesis
multilayer living loose parenchyma with D. Lipide synthesis
amyloid granules. It is called: E. Glucose synthesis
A. Mesoderm 7. Particles of dispersed phase of an
B. Endoderm emulsion are deformed and look as
C. Exoderm polyhedrons. What emulsion is it?
D. Collenchyme
E. Phellogene A. High-concentrated
B. Concentrated
2. A patient has been administered a C. Diluted
competitive inhibitor of cholinesterase. D. Oil-in water
Name it: E. Water-in-oil
A. Proserin 8. A drug which inhibits ATP synthesis in
B. Aspirin a cell has been used during an experiment.
C. Sodium diclophenac What type of transmembrane transport
D. Indometacin will be disturbed?
E. Allopurinol
A. Active
3. A 30 year-old patient suffering from B. Diffusion
pulmonary tuberculosis, has been prescri- C. Osmosis
bed isoniazid. Continuous taking of this D. Filtration
drug may lead to the deficiency of the E. Facilitated diffusion
following vitamin:
9. Increase in secretion of hydrochloric
A. Pyridoxine acid in the stomach of an experimental
B. Tocopherol animal can be provoked by subcutaneous
C. Cobalamin injection of the following gastrointestinal
D. Ergocalciferol hormone:
E. Retinol
A. Gastrin
4. Active form of one of the sulphur- B. Secretin
containing amino acids can be used C. Cholecystokinin
as a methyl group donor for the drug D. Somatostatin
methylation. Specify this amino acid: E. Motilin
A. Methionine 10. Protein digestion in the stomach is
B. Glycin carried out by pepsin secreted in form
C. Glutamine of an inactive pepsinogen. Pepsinogen is
D. Tyrosine converted to pepsin by the removal of the
E. Glutamate N-terminal peptide that is provoked by:
5. A chemist in analytical laboratory A. Perchloric acid
needs to standardize solution of sodium B. Sulfuric acid
hydroxide. What primary standard soluti- C. Acetic acid
on can be applied for this purpose? D. Bile acids
E. Amino acids
A. Oxalic acid
B. Acetate acid 11. In order to keep eubiotics viable and
C. Chloride acid stable, frozen microorganisms are dried
D. Sodium tatraborate out under the conditions of high vacuum.
E. Sodium chloride This method is called:
6. A patient has taken a large dose of a A. Lyophilization
barbiturate hypnotic (amytal) that inhibi- B. Pasteurization
ts NAD-dependent dehydrogenase of the C. Tyndallization
respiratory chain. What process running D. Inactivation
in the mitochondria will be disturbed? E. Hybridization
12. Cardiac diseases are treated wi-
th cocarboxylase preparation. This
41
krok123.in.ua
Krok 1 Pharmacy 2009 2

preparation is the coenzymatic form of 18. Nitrogen (I) oxide (N2 O) is applied for
the following vitamin: inhalation narcosis. It is obtained by heati-
ng of:
A. B1
B. B6 A. NH4 NO3
C. B12 B. NH3
D. C C. Cu(NO3 )2
E. P D. NH4 OH
E. NaNO3
13. During starvation the normal rate of
glucose in blood is sustained due to the 19. Quantitative determination of
gluconeogenesis stimulation. Which of pharmaceutical substances can be done
the following substances can be used as by means of acidimetry. Its titrant is
a source for glucose synthesis? the secondary standard solution of
hydrochloric acid. According to which
A. Alanine compound the precise concentration of
B. Adenine hydrochloric acid can be determined?
C. Ammonia
D. Nicotinamide A. Sodium tetraborate
E. Urea B. Oxalic acid
C. Potassium dichromate
14. It is known that infectious type B D. Sodium thiosulfate
hepatitis is a systemic disease caused by E. Magnesium sulphate
the type B hepatitis virus and characteri-
zed by a predominant liver affection. 20. Analysis of a patient’s urine revealed
Choose from the below given list the increased concentration of the uric acid.
drugs for the etiotropic therapy of this The patient was prescribed allopurinol.
infection: What is the biochemical mechanism of its
action?
A. Acyclovir
B. Penicillin A. Xanthine oxidase inhibition
C. Tetracycline B. Cyclooxigenase activation
D. Sulfanilamides C. Desaminase inhibition
E. Fluoroquinolones D. Phosphorylase inhibition
E. Nucleosidase inhibition
15. Choose a non-salt-forming oxide from
the following compounds: 21. Physical exercise results in an increase
in thermogenesis due to an increase in
A. N2 O heat production in the following structure:
B. CuO
C. P2 O5 A. Skeletal muscles
D. SO3 B. Heart
E. Na2 O C. Lungs
D. Liver
16. What substance can act as both oxi- E. Brain
dant and reducer in oxidation-reduction
reactions? 22. Plant pathogenic microorganisms
relate to various groups. Which of them
A. SO2 causes diseases of medicinal plants most
B. SO3 often?
C. CO2
D. P bO2 A. Fungi
E. CrO3 B. Viruses
C. Bacteria
17. Oxidative deamination of biogenic D. Actinomycetes
amines in the tissues is catalyzed by the E. Micoplasma
following enzyme:
23. It is known that proteins, fats and
A. Monoaminooxidase carbohydrates are digested by means of
B. Aspartate transaminase proteases, lipases and amylases, respecti-
C. Alanine transaminase vely. Which of digestive juices contains all
D. Decarboxylase these groups of enzymes enough for di-
E. Acetylcholinesterase gestion?
42
krok123.in.ua
Krok 1 Pharmacy 2009 3

A. Distal convoluted tubuli and receiving


A. Pancreatic juice tubes
B. Saliva B. Proximal convoluted tubuli
C. Gastric juice C. Henle’s loops
D. Bile D. Descending limbs of Henle’s loops
E. Juice of large intestine E. Ascending limbs of Henle’s loops
24. Colloidal protection is used whi- 29. Under conditions of high exterior
le manufacturing drug preparations. temperature a ventilating fan can relieve
Name the preparation of colloidal silver staying in the premises because it intesifi-
protected by proteins: es heat emission by means of:
A. Protargol A. Convection
B. Festal B. Liquid evaporation
C. Enzymtal C. Heat radiation
D. Argentum D. Heat conduction
E. Collagen E. Heat radiation and conduction
25. It is known that the unconjugated 30. In the pharmaceutical production
bilirubin being the product of heme processes of drug synthesis take place
catabolism is detoxicated in liver. Which under different conditions. Entropy stays
compound is involved into the bilirubin unchanged in the following process:
detoxication within the hepatocytes?
A. Adiabatic
A. Glucuronic acid B. Isothermal
B. Urea C. Isochoric
C. Mevalonic acid D. Isobaric
D. Lactic acid E. Polytropic
E. Glycin
31. Specify the complexing agent for a
26. An adult presents with systemic arteri- complex compound K2 [HgI4]:
al pressure at the rate of 160/100 mm Hg.
This might be caused by the increased A. Hg 2+
concentration of the following hormone B. K +
in blood: C. I −
D. HgI42−
A. Adrenalin E. K2 [HgI4]
B. Aldosterone
C. Glucagon 32. Analysis of a patient’s urine showed an
D. Cortisol increase in Na+ ions concentration and a
E. Thyroxin decrease is K + ions concentration. This
might be caused by the reduced secretion
27. It is required to diminish pump functi- of the following hormone:
on of patient’s heart. This can be done
by means of blockers of the following A. Aldosterone
membrane cytoreceptors: B. Insulin
C. Thyroxine
A. β-adrenoreceptors D. Hydrocortisone
B. Nicotinic cholinoreceptors E. Prolactin
C. Muscarinic cholinoreceptors
D. α-adrenoreceptors 33. Choose a reagent for synthesis of
E. Dopamine receptors acetic acid hydrazide from ethyl acetate:
28. Taking vasopressin resulted in a
decrease in diuresis. The reason for it
is increased water reabsorption in the
following renal tubuli:

43
krok123.in.ua
Krok 1 Pharmacy 2009 4

A. H2 N − NH2 A. Hesperidium
B. N 3 B. Pepo
C. H2 N − CH3 C. Multicoccus
D. C6 H5 NH2 D. Drupe
E. C6 H5 NHNH2 E. Bacca
34. When computing quantities of 39. Enzymes (biological catalysts) are
adjuvant substances required to make li- used as pharmacologic preparations.
quid drug forms isotonic, the values of What is the mechanism of enzyme acti-
isotonic quotients are used. What is the on in the biochemical reactions?
quotient for zinc sulphate if known that
it dissociates completely in an aqueous A. They reduce the energy of reaction
solution? activation
B. They increase the energy of reaction
A. 2 activation
B. 0 C. They inhibit the reaction process
C. 1 D. They change the constant of the reacti-
D. 3 on rate
E. 4 E. They change the reaction order
35. After a stomach resection a pati- 40. After a solution had been heated wi-
ent presented with weakness, skin pallor, th (NH4 )2 S2 O8 in presence of AgNO3 , it
face puffiness, enlargement of liver and turned crimson. What ions were present
spleen. Analysis of the peripheral blood in the solution?
revealed megaloblasts and megalocytes;
hyperchromatism (colour index - 1,3). A. Mn2+
What type of anaemia is observed in this B. F e3+
patient? C. F e2+
D. Co2+
A. B12 -deficient E. Cu2+
B. Haemolytic
C. Hypoplastic 41. A solution under examination
D. Iron-deficient was added to the solution of F eSO4
E. Toxic in presence of concentrated H2 SO4 .
Generation of a brown ring indicates
36. Water-soluble vitamins take coenzyme presence of:
form in an organism. Thiamine di-
phosphate is coenzyme of the following A. Nitrate ions
vitamin: B. Acetate ions
C. Carbonate ions
A. B1 D. Oxalate ions
B. B2 E. Phosphate ions
C. C
D. B6 42. Specify standard substances used
E. B12 for standardization of titrant soluti-
ons (NaOH, KOH) in the alkalimetric
37. Quantitative determination of calcium method:
chloride is carried out by method of direct
chelatometric titration. Choose an indi- A. Oxalic and succinic acids
cator for fixation of the titration endpoint: B. Acetic and succinic acids
C. Formic and acetic acids
A. Eriochrome black T D. Sulphanilic and oxalic acids
B. Phenolphthalein E. Sulphanilic and salicylic acids
C. Methyl red
D. Eosin 43. Irritation of the sympathetic nerve
E. Starch in an experimental dog induces quanti-
tative and qualitative alterations in the
38. One of the examined soft fruits is saliva composition. What alterations are
characterized by essential-oil exocarp, induced?
spongioid mesocarp and overgrown
endocarp that consists of juice saccules.
What fruit was under examination?

44
krok123.in.ua
Krok 1 Pharmacy 2009 5

A. Little saliva, a lot of enzymes


B. A lot of saliva, a lot of enzymes
C. Little saliva, few enzymes A.
D. A lot of saliva, few enzymes
E. A lot of saliva, no enzymes
44. Kinetic methods are used for determi-
nation of drug stability. What is the order
of reaction if its rate constant equals to B.
c−1 ?
A. First
B. Zero C.
C. Fractional
D. Second
E. Third
D.
45. Underneath the stem epidermis some
layers of living perenchymal cells were
found. The cells contained chloroplasts
and had cellulose membranes with thi- E.
ckened angles. This tissue is called:
A. Angular collenchyme 48. Choose the carbocation among the gi-
B. Lacunar collenchyme ven intermediate reactive particles:
C. Lamellar collenchyme
D. Storage parenchyme
E. Chlorophyll-containing parenchyme
A.
46. Which of the following oxides is the
anhydride of nitric acid?
A. N2 O3 B.
B. N2 O5
C. N2 O4
D. NO
E. NO2 C.
47. What is the product of ethyl alcohol-
acetic aldehyde reaction? D.

E.

49. 1 M sulphuric acid solution was added


to the solution under study. This resulted
in formation of white sediment that was
soluble in the alkalies. This indicated that
the solution contains:
A. Plumbum cations
B. Calcium cations
C. Barium cations
D. Argentum cations
E. Mercury (I) cations
50. Argentum nitrate solution was added
to a solution containing anions of the
second analytical group. This resulted in
formation of light-yellow sediment that
was insoluble in the nitric acid and partly
45
krok123.in.ua
Krok 1 Pharmacy 2009 6

soluble in the ammonia solution. What A. Total lung capacity


anions were present in the solution? B. Vital lung capacity
C. Tidal volume
A. Bromide ions D. Residual volume
B. Iodide ions E. Inspiratory reserve volume
C. Chloride ions
D. Sulphide ions 56. Choose a reagent that can be used for
E. Arsenite ions production of propanol − 2 out of acetone:
51. A patient has bradycardia, moderate
hypotension, decrease of basal metaboli-
sm, edemata. What abnormality can
induce such syndrome?
A. Thyroid hypofunction A. H2 (Ni)
B. Parathyroid hypofunction B. CH3 OH
C. Thyroid hyperfunction C. CH3 I
D. Parathyroid hyperfunction D. HCN
E. Adrenal hypofunction E. HCOH
52. Most cases of alimentary starvati- 57. Galactose belongs to the aldehyde
on are accompanied by development alcohols and similarly to aldehydes
of evident edemata. What is the leadi- interacts with hydrocyanic acid (HCN)
ng pathogenetic mechanism of edemata according to the following mechanism:
development in this case?
A. Fall of oncotic pressure of blood plasma
B. Rise of hydrostatic pressure in the
capillaries
C. Fall of hydrostatic pressure in the tissues
D. Rise of oncotic pressure in the
intercellular fluid
E. Fall of osmotic pressure in the
intercellular fluid
53. A patient suffering from pleuritis
underwent pleural puncture. There was
obtained a transparent odourless liquid.
What type of exudate was obtained?
A. Serous A. AN
B. Haemorrhagic B. SN 1
C. Purulent C. SN 2
D. Fibrinous D. AE
E. Putrefactive E. SR
54. Examination of a medicinal plant 58. Identify the succinimide (succinic acid
revealed that its underground organ had imide) among the given compounds:
nodes, internodes, cataphylls, gemmae
and secondary roots. Therefore, this
underground organ is:
A. Rhizome
B. Storage root
C. Root bulb
D. Stolon
E. Tuber
55. A patient took a maximal deep breath.
Air volume being present in lungs under
these conditions is called:

46
krok123.in.ua
Krok 1 Pharmacy 2009 7

vision:
A. Bryophyta
B. Lycopsida
C. Equisetophyta
D. Pteridophyta
E. Gymnospermae
A.
62. A fruit under examination is
pseudomonocarpic, with woody pericarp
and one seed. The seed cuticle remains
unfused with the pericarp. Such fruit is
called:
A. Nut
B. B. Cremocarp
C. Achenocarp
C. H2 NOC − CH2 − CH2 − CONH2 D. Caryopsis
D. H2 NOC − CH2 − CH2 − CH2 − CONH2 E. Pseudomonocarpic drupe
63. The birch has compound inflorescences
with drooping main axis bearing dichasia
composed of unisexual cells. Therefore,
this inflorescence is called:

E. A. Ament
B. Raceme
C. Spadix
59. Ammonia is generated in different ti- D. Spike
ssues and organs and then transported to E. Glomus
liver for detoxication and conversion into
urea. What amino acid transports it from 64. A female patient bitten by a stray dog
skeletal muscles to liver? came to a surgery. Wide lacerated wounds
were localized on the patient’s face. What
A. Alanine treatment-and prevention aid should be
B. Histidine rendered in order to prevent rabies?
C. Glycin
D. Serine A. Immunization with the antirabic vacci-
E. Valine ne
B. Combined antibiotic therapy
60. For production of phenol ether it is C. Hospitalization, injection of diphtheria-
necessary to cause reaction of sodium pertussis-tetanus vaccine
phenoxide with: D. Hospitalization, medical surveillance
E. Urgent injection of normal gamma-
globulin
65. It is suspected that the workers
of a serum drugs plant at a regional
hemotransfusion station are carriers of
pathogenic staphylococcus aureus. In
order to detect staphylococcus carriage,
the material from the nasopharynx of the
A. CH3 Cl workers should be inoculated into the
B. CH3 OH following medium:
C. CH4
D. CH3 NH2 A. Egg-yolk-salt agar
E. CH3 C ≡ N B. Endo agar
C. Meat infusion broth
61. A higher nonvascular plant has di- D. Kessler medium
stinct alternation of dominant sexual E. Blood agar
(gametophyte) and reduced asexual
(sporophyte) generations. This indicates 66. Analysis of sputum obtained from
that the plant belongs to the following di- a patient with suspected pneumonia
47
krok123.in.ua
Krok 1 Pharmacy 2009 8

revealed gram-positive diplococci. They 71. Which of the following indices of


were slightly elongated, with the pointed the external respiration characterizes the
opposite ends. What microorganisms were maximum volume of air that a person can
revealed in the sputum? exhale after maximum inhalation?
A. Streptococcus pneumoniae A. Lung vital capacity
B. Staphylococcus aureus B. Total lung capacity
C. Klebsiella pneumoniae C. Functional residual capacity
D. Neisseria meningitidis D. Expiratory reserve volume
E. Streptococcus pyogenes E. Respiratory volume
67. Examination of air state in drugstore 72. The air in a room has increased
premises for preparation of injection concentration of carbonic acid. What
drugs was done by method of sedimentati- respiratory changes (depth and rate) will
on. It revealed 5 small roundish coloni- be observed in a person after entering this
es with zone of hemolysis around them. room?
Inoculations were made on the following
cultural medium: A. Increase in respiration rate and depth
B. Decrease in respiration rate and depth
A. Blood agar C. Decrease in respiration depth and
B. Endo agar increase in respiration rate
C. Meat infusion agar D. Increase in respiration depth and
D. Egg yolk and salt agar decrease in respiration rate
E. Lewin’s agar E. There will be no respiratory changes
68. According to the Pharmacopoeia 73. Vitamin A is quickly oxidized in the
requirements, all drugs for topical admi- open air and hereupon looses its bi-
nistration should be tested for “microbi- ological activity. What component of the
ological purity”. Inapplicability of this foodstuffs mainly prevents the oxidation
drug group in the medical practice is indi- of the vitamin?
cated by presence of the following mi-
croorganisms: A. Tocopherol
B. Nicotinic acid
A. Staphylococcus aureus C. Common salt
B. Yeast fungi D. Protein
C. Saprophytic staphylococci E. Fat
D. Mold fungi
E. Sarcinae 74. Specify the standardized solutions
used for direct and back titration of
69. A pharmacy produced a batch of vials reducing agents in the iodometric method:
with glucose diluent for injections. What
is the best way for their sterilization? A. I2 , Na2 S2 O3
B. K2 Cr2 O7 , Na2 S2 O3
A. Autoclave sterilization by flowing steam C. I2 , KI
(fractional method) D. KMnO4 , KI
B. Autoclave sterilization under 2 E. K2 Cr2 O7 , I2
atmosphere pressure
C. Dry-heat sterilization 75. By heating aniline with concentrated
D. X-ray exposure sulphuric acid the following compound
E. UV exposure can be obtained:

70. When producing some liquid drug


forms, it is necessary to take into account
their osmotic pressure. The highest
osmotic pressure is characteristic for the
0,1 M solution of the following substance:
A. AlCl3
B. Glucose
C. Saccharose
D. CaCl2
E. KNO3

48
krok123.in.ua
Krok 1 Pharmacy 2009 9

A. Saturated calomel
B. Quinhydrone
C. Glass
A. D. Hydrogen
E. Zinc
79. Sodium nitrite is used in medicine as
a vasodilating drug against stenocardia.
NaNO2 acts as reducer with the following
B. compound:
A. KMnO4
B. H2 S
C. NH3
C. D. KI
E. NaHCO3
80. Most technological processed in
pharmaceutics run in heterogenous
D. systems. How many phases has an eutectic
composition under eutectic temperature
of two-component system?
A. 3
B. 2
C. 5
E. D. 4
E. 1
76. During the quantitative analysis carri- 81. Sulphur (IV) oxide is a constituent
ed out under the primary conditions, part of one of the most harmful envi-
a specific reagent to F e3+ cations is ronment pollutants called toxic smog.
K4 [F e(CN)6 ]. Their interaction gives a When dissolved in water, sulphur (IV) oxi-
precipitate of the following colour: de forms the following acid:
A. Blue A. Sulphurous
B. White B. Sulphuric
C. Brown C. Hydrosulphuric
D. Red D. Thiosulfate
E. Black E. Tetrathionate
77. From a patient with the symptoms 82. Micelle solutions of surfactants are
of acute meningitis the spinal fluid was applied in pharmaceutical production as
taken. Its smears contained gram-negative stabilizers and solubilizers. What soluti-
diplococci within the leukocytes and on of colloidal surfactants will have the
outside them. Which microorganism is the greatest value of critical concentration of
most likely cause of the disease? micelle formation?
A. Neisseria meningitidis A. C9 H19 SO3 Na
B. Haemophilus influenzae B. C14 H29 SO3 Na
C. Streptococcus pneumoniae C. C16 H33 SO3 Na
D. Candida albicans D. C12 H25 SO3 Na
E. Escherichia coli E. C10 H21 SO3 Na
78. Potentiometric method of pH 83. In the pharmaceutical industry, the
determination is regarded as the most micelle-forming solutions of surface-
universal and enters into the Nati- active substances are used for producti-
onal Pharmacopeia of Ukraine. Which on of water-soluble preparations out of
electrode is used as a reference electrode? water-insoluble substances, for example
vitamins A and E. The critical concentrati-
on of micelle formation has the lowest
value in the solutions of the following
substances:
49
krok123.in.ua
Krok 1 Pharmacy 2009 10

89. Which electronic configuration of


A. C17 H35 COONa valence electrons is corresponding to an
B. C12 H25 COONa element of the 4th period, VI group, main
C. C13 H27 COONa subgroup?
D. C15 H31 COONa
E. C11 H23 COONa A. 4s2 4p4
B. 4s1 3d5
84. Specify the titration method, in which C. 6s2 6p2
a standardized titrant solution is gradually D. 6s2 5d2
added to the solution under study until a E. 3s2 3p4
titration endpoint is reached:
90. According to the mass action law,
A. Direct titration velocity of process 2SO2 (g) + O2 (g) =
B. Back titration 2SO3 (g) is expressed as:
C. Indirect titration
D. Substitution titration A. k [SO2 ]2 x [O2 ]
E. Residue titration B. [2SO2 ] x [O2 ]
85. The given reaction is called: C. k [SO2 ] x [O2 ]
D. k [SO2 ] + [O2 ]
6 12 6 −enzymes
−−−−→ 22 5  + 22 ? E. [SO2 ]2 + [O2 ]
91. What saturated heated solution is used
for transformation of sulphates BaSO4 ,
A. Alcohol fermentation of glucose SrSO4, CaSO4 to carbonates during the
B. Glucose hydrolysis systematic analysis?
C. Glucose oxidation
D. Lactic-acid fermentation of glucose A. Na2 CO3
E. Glucose reduction B. CaCO3
C. (NH4 )2 CO3
86. Which of the following formulas D. MgCO3
corresponds to the electronic configurati- E. CO2
on of Cu atom?
92. Which indicatorless method enables
10
A. [Ar] 3d 4s 1 quantitative determination of iron (II)
B. [Ar] 3d9 4s2 content?
C. [Ar] 3d6 4s2 A. Permanganatometry
D. [Ar] 3d8 4s2 B. Chelatometry
E. [Ar] 3d7 4s2 C. Argentometry
D. Iodometry
87. Early pregnancy test involves analysis E. Nitritometry
of a woman’s urine. Pregnancy is
ascertained by presence of the following 93. Thresholds of coagulation of a drug sol
hormone: by electrolytes MgSO4 , NaCl, Al(NO3 )3
are equal to 0,81; 51,0; 0,095 millimole/l
A. Chorionic gonadotropin correspondingly. Which electrolyte ion
B. Estriol has the maximal coagulating effect?
C. Aldosterone
D. Testosterone A. Al3+
E. Progesterone B. Mg 2+
C. Na+
88. Microbiological assay of a peppermint D. Cl−
tincture established its discrepancy with E. SO42−
the Pharmacopoeia requirements. It was
found to contain pathogenic microflora. 94. Iodoform when stored decomposes
The reason for such a conclusion was the spontaneously into free iodine. Which
presence of the following microflora: thermochemical function is a criterion for
this process direction when V and T are
A. Blue pus bacillus constant?
B. Yeast fungi
C. Epidermal staphylococcus
D. Mold fungi
E. Micrococci
50
krok123.in.ua
Krok 1 Pharmacy 2009 11

A. Helmholtz energy F ion Cr2 O72− under these conditions?


B. Entropy S
C. Enthalpy H A. Cr3+
D. Gibbs energy G B. Cr(OH)3
E. Intrinsic energy U C. Cr(OH)2
D. [Cr(OH)6]3−
95. Glycosidic (hemiacetal) hydroxyl in E. Cr2 O3
a molecule of α-D-glucose pyranose is
bonded to the following carbon atom: 100. Medicinal plants collected at a
plantation included a lot of plants with
mosaic-coloured leaves. What is the most
likely causative agent of such infection?
A. Phytopathogenic viruses
B. Phytopathogenic bacteria
C. Toxic substances of the soil
D. Microscopic mites
E. Nematode worms
101. A pharmaceutic factory recei-
ved a batch of crude herbal drugs for
A. C1 phytomedicines production. To evaluate
B. C2 quality of these crude drugs it is necessary
C. C3 to determine:
D. C4
E. C6 A. Total number of microorganisms pro 1
g of thecrude drug
96. Microscopic examination of a potato B. Coli titer
tuber showed some cell inclusions that C. Coli index
become blue-violet as affected by Lugol’s D. Antimicrobial
iodine solution. These inclusions are: E. Pyrogens
A. Starch granules 102. The leukocytes that are the first to
B. Aleurone grains appear in a focus of inflammation are
C. Drops of fatty oil called:
D. Insulin crystals
E. Calcium oxalate crystals A. Neutrophils
B. Monocytes
97. Chlorophyll, the green pigment of C. Eosinophils
plants, is a chelate compound. Specify the D. Lymphocytes
chelating ion in the chlorophyll: E. Basophils
A. Mg 2+ 103. The most active component in the
B. F e3+ aniline acylation reaction is:
C. Mn2+
D. F e2+
E. Ni2+
98. Prevailing plants of a foliage forest are
monoecious high trees coated with thi-
ck dark-grey rind with deep cracks. Their
leaves are short-petiolar, pinnatilobate.
Their fruit is acorn. Therefore, the domi-
nating species is:
A. Quercus robur
B. Robinia pseudoacacia
C. Aesculus hippocastanum
D. Tilia cordata
E. Betula verrucosa
99. Potassium dichromate K2 Cr2 O7 is
applied as oxidant in acidic medium. What
is the product of reduction of dichromate-
51
krok123.in.ua
Krok 1 Pharmacy 2009 12

A. Calcium chloride
B. Anhydrous calcium oxalate
C. Monohydrous calcium oxalate
A. D. Calcium carbonate
E. Calcium hydroxide
107. A man’s tip of tongue was processed
with an anesthetic solution. Therefore he
B. will loose the sense of the following taste:
A. Sweet
B. Bitter
C. Sour
D. Salty
E. Bitter and salty

C. 108. Anti-inflammatory effect of a


number of drugs is caused by the inhibiti-
on of arachidonic acid release. This acid is
the precursor of:
A. Prostaglandins
D. B. Uric acid
C. Urea
D. Haem
E. Cholesterol
109. Dark-violet colour appears, when
E. F eCl3 solution is added to the followi-
ng substance:
104. Staphylococci were isolated in
pure culture from a patient with sepsis.
These were staphylococci producing
betalactamase. Such property should be
taken into account when:
A. Choosing an antibiotic for treatment
B. Determining biiochemical properties
C. Determining the strain pathogenicity
D. Differentiating specific types of
staphylococci
E. Choosing optimal conditions for culti-
vation
105. It is required to determine amount of
sodium salicylate in a solution. What ti-
trimetric method can be applied for the
quantitative determination of aromatic
compounds?
A. Bromometry
B. Mercurimetry
C. Cerimetry
D. Argentometry
E. Chelatometry
106. In order to determine mass fraction
of calcium in a pharmaceutical preparati-
on, gravimetric method was applied.
Ammonium oxalate solution was used as
a precipitating agent. What is the gravi-
metric form in this case?
52
krok123.in.ua
Krok 1 Pharmacy 2009 13

A.
A.

B. B.

C.
C.

D.
D.

E.

E. 112. A female patient consulted a doctor


about leg pain that arises usually toward
the evening; feet and shins edemata.
110. Inoculation of hen’s embryos is the Objectively: leg skin is cyanotic, cold to
main method of detection of influenza the touch. What type of peripheral ci-
virus. In order to neutralize associated rculation disorder does the patient present
bacterial flora in the material under with?
examination (nasopharyngeal lavage) it
is necessary to add beforehand: A. Venous hyperaemia
B. Arterial hyperaemia
A. Antibiotics C. Ischaemia
B. Eubiotics D. Stasis
C. Fluorescent serum E. Thrombosis
D. Leukocytic interferon
E. Ant-influenza gamma globulin 113. A patient 42 year old sufferi-
ng from chronic calculous cholecysti-
111. Interaction of lactic acid with SOCl2 tis complains about acute pain in
excess will result in generation of the the right subcostal area, itching and
following compound: skin icteritiousness, multiple petechi-
al haemorrhages, saponified and light-
coloured feces (steatorrhea). What type
of icterus is it?
A. Mechanic
B. Hemolytic
C. Parenchymatous
D. Cythemolytic
E. Hepatocellular
114. Specify the relevant indicators for fi-
xation of the titration endpoint when usi-
53
krok123.in.ua
Krok 1 Pharmacy 2009 14

ng nitritometric method: forms a propionic aldehyde as a result of


alkaline hydrolysis (H2 O, OH − )?
A. Tropeolin 00 + methylene blue
B. Methylene blue
C. Methylene orange
D. Starch solution
E. Diphenylamine
A.
115. Presence of the pathogenic mi-
croorganisms in the air can be prognosti-
cated according to the content of sanitary-
indicative bacteria. Which bacteria indi- B.
cate immediate epidemiologic danger?
A. Haemolytic streptococci
B. Sarcinae
C. Mold fungi C.
D. Yeast fungi
E. Micrococci
116. To prevent fatty degeneration of liver
after viral hepatitis, a patient should be
administered lipotropic factors. Indicate D.
one of them:
A. Choline
B. Tryptophane
C. Allopurinol
D. Contrical E.
E. Vicasol
117. Hybridization of one s− and 2 120. During thin-layer chromatography
−orbitals leads to formation of three s2 - of novocaine, the developed plate
hybrid orbitals. Specify the angle between represented a stain 3 cm away from the
these orbitals: start line, and the length of solvent front
was 10 cm. What is the Rf value of
A. 120o novocaine?
B. 180o
C. 109o A. 0,3
D. 90o B. 0,4
E. 104, 5o C. 0,5
D. 0,6
118. Toluol is converted to the benzoic E. 0,7
acid under the following conditions:
121. For quantitative determination of
some drugs the solutions of sulfuric and
perchloric acids are applied. Which of
the following oxides are the anhydrides
of these acids?
A. SO3 , Cl2 O7
B. SO2 , Cl2 O
C. SO3 , ClO2
D. SO2 , Cl2 O7
A. Oxidation with potassium E. SO2 , Cl2 O7
permanganate
B. Heating with sulphuric acid 122. A patient suffers from the cerebral
C. Hydrogen peroxide action at a room atherosclerosis. Blood count showed
temperature hyperlipoproteinemia. You will most li-
D. Sodium hydroxide action at a room kely observe increase in the concentration
temperature of the following plasma lipoprotein class:
E. Boiling in the open air
119. Which of the following compounds
54
krok123.in.ua
Krok 1 Pharmacy 2009 15

A. Low-density lipoproteins A. CH2 Cl2


B. High-density lipoproteins B. C2 H5 Cl
C. Chylomicrons C. CH2 O
D. Globulin complexes with steroid D. CH3 COOH
hormones E. NaNH2
E. Fatty acid complexes with albumines
128. Pharmaceutical practice involves use
123. Inflammatory processes in the gall of microheterogeneous systems with li-
bladder exert negative influence on the quid disperse medium and solid disperse
colloidal properties of bile. This may lead phase. Such drug form is:
to gallstone formation. One of the causes
of their formation is the crystallization of A. Suspension
the following substance: B. Foam
C. Powder
A. Cholesterol D. Aerosol
B. Albumine E. Emulsion
C. Haemoglobin
D. Urates 129. What is the number of degrees of
E. Oxalates freedom for the salol-camphor system,
provided that both components crystallize
124. A 55-year-old woman with renal fai- from the melt simultaneously?
lure has arterial pressure at the rate of
170/100 mm Hg. Stable pressure rise is A. 0
caused by hyperactivity of the following B. 1
system: C. 2
D. 3
A. Renin-angiotensin-aldosterone E. - 1
B. Sympathoadrenal
C. Hypothalamo-pituitary 130. Choose the reagents for detection of
D. Central nervous the sulphate ions in a solution containing
E. Kallikrein-kinin carbonate, sulphate and phosphate ions:
125. What method of titrimetric analysis A. Ba(NO3 )2 , HCl
can be applied for the quantitative B. Ba(NO3 )−2 , NaOH
determination of sulphuric acid by means C. BaCl2 , H2 O
of the potassium hydroxide solution? D. CaCl2 , NH4 OH
E. AgNO3 , HNO3
A. Alkalimetry
B. Acidimetry 131. A big brown alga has a stipe, rhizoids
C. Oxidation-reduction and laminae rich in alginates and iodine.
D. Precipitation It belongs to the following genus:
E. Complexation
A. Laminaria
126. General formula of alkynes is B. Chlorella
n H2n−2. Isomerous alkynes fall into the C. Chlamydomonas
following compound class: D. Spirogira
E. Ulothrix
A. Alkadienes
B. Alkenes 132. Aetiological factors for the infecti-
C. Cycloalkanes ous diseases are often microorganisms wi-
D. Mononuclear arenes th various ultrastructure. Which of the
E. Multinuclear arenes following microorganism groups relates
to the eucariots?
127. Diphenylmethane can be derived
from benzol by means of the following A. Protozoa
reagent: B. Viruses
C. Viroids
D. Prions
E. Scotobacteria
133. After a birth trauma a newborn
presents with limited movements of
the right upper extremity, hyporeflexia,
myatrophy. These changes relate to the
55
krok123.in.ua
Krok 1 Pharmacy 2009 16

following type of motor dysfunctions: 138. Alpha-cells of pancreas stimulate


synthesis of the glucagon hormone that is
A. Peripheric (atonic) paralysis involved into the carbohydrate metaboli-
B. Central paralysis sm. It has the following effect on liver
C. Myasthenia processes:
D. Bulbar paralysis
E. Neuritis A. Activates glycogenolysis
B. Activates alcoholic fermentation
134. Temperature quotient of the reaction C. Inhibits glycogenolysis
velocity is equal to 2. In how many times D. Inhibits glycolysis
does the reaction velocity change, if the E. Activates lypogenesis
temperature changes by 40o C?
139. Sol Al(OH)3 was derived by processi-
A. In 16 times ng a freshly made Al(OH)3 precipitate
B. In 8 times with a small amount of HCl solution.
C. In 4 times Sol production bases upon the following
D. In 32 times phenomenon:
E. In 24 times
A. Chemical peptization
135. Surface-active substances are widely B. Chemical condensation
used in the drug production. Specify a C. Rinsing with a solvent
surface-active substance present at the D. Mechanic dispersing
aqueous solution-air interface: E. Physical condensation
A. Butyric acid 140. What is the molecular weight of an
B. Saccharose undefined gas knowing that its density is
C. NaCl 20 relative to the density of hydrogen?
D. NaOH
E. HCl A. 40 g/mole
B. 10 g/mole
136. What type of tautomerism is typical C. 20 g/mole
for the given compound? D. 30 g/mole
E. 50 g/mole
141. How much sodium hydroxide is
required to prepare 500 g of 10% sodi-
um hydroxide solution?
A. 50 g
B. 0,5 g
C. 5 g
D. 10 g
E. 25 g
A. Nitro-aci-nitro tautomerism 142. A 37-year-old man was admitted
B. Carbonyl-enol tautomerism to a hospital with an attack of bronchi-
C. Cyclo-oxo tautomerism al asthma. What respiration type will be
D. Amine-imine tautomerism observed in this patient?
E. Keto-enol tautomerism
A. Expiratory dyspnea
137. Microscopic examination of a ficus B. Inspiratory dyspnea
leaf revealed in some cells of its epidermis C. Apnoea
a protrusion of the cell membrane with an D. Gasping respiration
accumulation of crystals that dissolve in E. Hyperpnoea
the hydrochloric acid and release carbonic
acid gas. This structure is called: 143. After a bacteriological analysis a
tableted medication was considered to be
A. Cystolith inapplicable, though its general microbi-
B. Raphide al contamination was within the norm.
C. Druse The reason for such a conclusion was the
D. Single crystal presence of the following microorganisms:
E. Styloid

56
krok123.in.ua
Krok 1 Pharmacy 2009 17

A. Enterobacteria observed in the following solvent:


B. Mold fungi
C. Actinomycetes A. Water
D. Micrococci B. Benzol
E. Sarcinae C. Ethyl alcohol
D. Chloroform
144. An essential oil plant under exami- E. Acetone
nation has a square stem, flowers with
bilabiate corolla, coenobium fruit. These 150. A patient suffering from the bone
characteristics are typical for the followi- marrow form of radiation sickness was
ng family: found to have the following changes
in his hemogram: leukocytes - 2 · 109 /l,
A. Lamiaceae lymphopenia, erythrocytes - 3, 0 · 1012 /l,
B. Papaveraceae Hb- 52 g/l, thrombocytes - 105 · 109 /l,
C. Polygonaceae reduced blood coagulation. These changes
D. Solanaceae are typical for the following stage of the
E. Scrophulariaceae radiation sickness:
145. Concentration of different ions within A. Fastigium
the cytoplasm of a neurocyte has been B. Latent period
measured during an experiment. The hi- C. Prodromal period
ghest ion concentration was observed in: D. Solution
A. K + E. Relapse
B. Na+ 151. Immediate-type allergies are
C. Cl− characterized by degranulation of the ti-
D. Ca2+ ssue basophils that secrete biologically
E. HCO 3− active substances. One of such substances
146. Specify the molecular formula of the is:
oxide whose conjugate is permanganic A. Histamine
acid: B. Acetylcholine
A. Mn2 O7 C. Plasminogen
B. Mn3 O4 D. Hageman’s factor
C. Mn2 O3 E. Thromboxane
D. MnO2 152. A 56-year-old female patient
E. MnO complains about a fast growing hard
147. A patient presents with weakening neoplasm in the mammary gland that
of the inhibitory processes of CNS whi- appeared a month ago. Objectively: the
ch is associated with disturbed producti- formation is fused with the surroundi-
on of gamma-aminobutyric acid. What ng tissues, it is uneven, slightly painful.
substance is the GABA precursor? What are the peculiarities favouring the
infiltrative growth of a malignant tumour?
A. Glutamate
B. Tryptophane A. Lack of contact inhibition
C. Methionine B. Intensified chalone formation
D. Valine C. Intensified contact inhibition
E. Glycin D. Intensified formation of tight contacts
E. Rise of embryonal antigens
148. Drugs in form of colloidal-and-
disperse systems are widely spread in the 153. During the field practice a student
pharmaceutical practice. What method found a plant with disk-shaped structure
of sol production is based upon the of its rachis, sessile flowers and husk. This
phenomenon of physical condensation? inflorescence is called:

A. Solvent substitution A. Anthodium


B. Reduction B. Spike
C. Oxidation C. Spadix
D. Hydrolysis D. Glomus
E. Double exchange E. Raceme

149. The best swelling of gelatine will be 154. As a result of staining of a plant mi-
57
krok123.in.ua
Krok 1 Pharmacy 2009 18

croslide with Sudan III solution the cell


membranes turned pink. This indicates
the presence of:
A. Suberin
B. Cellulose
C. Lignin A. HCN
D. Pectin B. NaOH
E. Hemicellulose C. SOCl2
D. F eCl3
155. After a plant microslide had been E. CH3 OH (H + )
processed with phloroglucinol together
with concentrated hydrochloric acid, the 160. Which of the following compounds
cell membranes turned crimson red. This has acidophobic properties?
indicates presence of: A. Pyrrole
A. Lignin B. Pyrazole
B. Pectin C. Pyridine
C. Cellulose D. Pyrimidine
D. Hemicellulose E. Imidazole
E. Suberin 161. What reagent can demonstrate
156. Nitritometric determination of presence of an aldehyde group in a
compounds containing primary aromatic furfural molecule?
amino group can be carried out under the
following conditions:
A. With observation of all the mentioned
conditions
B. At a temperature up to 10o C
C. With adding of the crystalline KBr
(catalyst) A. [Ag(NH3 )2 ]OH
D. Chloric acid excess B. (CH3 CO)2 O
E. Slow titration C. NaNO2
D. NH3
157. What electrode is used as indicator E. NaOH
during dichromatometric determination
of F eSO4 in a solution provided that fi- 162. A patient present’s with Kussmaul’s
xation of the equivalence point is done by respiration, acetone smell from the
a potentiometric method? mouth; low tonus of eyeballs, myotic
pupils, dry skin, polyuria, glycosuria,
A. Platinum hyperglycemia. Such symptom complex
B. Glass is typical for the following coma:
C. Quinhydrone
D. Silver A. Diabetic
E. Silver chloride B. Hepatic
C. Alimentary dystrophic
158. Inflorescence of Ledum palustre has D. Hypoglycemic
a significantly shortened rachis, connivent E. Adrenal
nodes, pedicles of the quite similar length.
This inflorescence is called: 163. Heart automatism is possible due
to the atypical cardiomyocytes formi-
A. Umbel ng the cardiac conduction system. What
B. Glomus part of this system is the primary cardiac
C. Bostryx pacemaker?
D. Spike
E. Ament A. Sinoatrial node
B. Purkinje’s fibers
159. Reaction of pyruvic acid in which C. Atrioventricular node
the ketone functional group takes place D. His’ bundle
proceeds with the following reagent: E. His’ bundle branches
164. Essential oil glandules consisting of 8
58
krok123.in.ua
Krok 1 Pharmacy 2009 19

secretory cells arranged in two rows and


four tiers can be be found in most plants
of the following family:
A. Asteraceae
B. Apiaceae
C. Lamiaceae A. AN nucleophylic addition
D. Rosaceae B. AE electrophylic addition
E. Scrophulariaceae C. SE electrophylic addition
D. SN nucleophylic substitution
165. To relax skeletal muscles during E. SR radical substitution
complex surgeries, curarelike substances
are applied. These substances block the 170. The highest bromination rate will be
following structure: observed for the following compound:
A. Neuromuscular synapses
B. Basal ganglions
C. Red nuclei of the mesencephalon
D. Synaptic structures of the spinal cord
E. Vegetative ganglions
A.
166. Anxious condition can be characteri-
zed by reduced salivation and sense of
dry mouth. What mediator is exuded out
of nerve terminals innervating salivary
glands? B.
A. Noradrenaline
B. Acetylcholine
C. Serotonin
D. Histamine
E. GABA
167. A patient complains about an C.
increase in heart rate, hyperperspirati-
on, irritability, sleeplessness. He has been
presenting with these symptoms for the
latest six months. They indicate the
hyperfunction of the following endocri-
ne gland: D.
A. Thyroid gland
B. Pancreas
C. Adrenal glands
D. Sexual glands
E. Thymus
168. What mediator provides informati-
on transmission from nerve terminations E.
of motoneurons to the fibers of skeletal
muscles?
171. Examination of five herbarium speci-
A. Acetylcholine mens of medicinal plants showed that one
B. Adrenaline of them belonged to the legume family,
C. Noradrenaline namely:
D. Serotonin
E. GABA A. Glycyrhiza glabra
B. Atropa belladonna
169. What is the mechanism of addition C. Hyoscyamus niger
reaction of ethanol to acetaldehyde? D. Datura stramonium
E. Solanum dulcamara
172. Bacca fruit is typical for the following
representative of Solanaceae family:
59
krok123.in.ua
Krok 1 Pharmacy 2009 20

A. Transamination
A. Atropa belladonna B. Reduction-oxidation
B. Hyoscyamus niger C. Phosphorilation
C. Datura stramonium D. Methylation
D. Nicotiana tabacum E. Hydrolysis
E. Datura innoxia
178. Before diving experienced divers first
173. A patient was found to have an take several deep breaths. They do it in
increase in total bilirubin concentration order to:
in plasma at the expense of indirect bi-
lirubin; high rate of stercobilin in feces A. Remove as much as possible CO2
and urine; normal rate of direct bilirubin. B. Reduce functional residual capacity of
What jaundice is it? lungs
C. Increase lung vital capacity (LVC)
A. Haemolytic D. Increase total lung capacity (TLC)
B. Mechanic E. Increase respiratory volume (RV)
C. Gilbert’s syndrome
D. Parenchymatous 179. As a result of reaction of mercury
E. Physiological excess with diluted nitric acid the followi-
ng gas will escape:
174. A patient has been administered
L-carnitine. This preparation ensures A. NO
transmembrane transfer of the following B. NH3
substances: C. N2
D. N2 O
A. Higher fatty acids E. -
B. Amino acids
C. Purine nucleotides 180. A 47-year-old patient with an arm
D. Pyrimidine nucleotides injury was delivered to a hospital in pain
E. Glucose shock condition. Objectively: the pati-
ent is in grave condition, with mental
175. Coordination number of iron in confusion; integuments are moist, pale,
the potassium hexacyanoferrate (II) acrocyanotic. There are also tachypnea,
K4 [F e(CN)6 ] is: fall in the arterial pressure, tachycardia.
A. 6 What type of hypoxia is prevailing in this
B. 2 patient?
C. 4 A. Circulatory
D. 3 B. Haemic
E. 8 C. Tissue
176. A patient is 50 years old. Ad a result D. Respiratory
of continuous improper feeding he got E. Substrate
hypovitaminosis C. Lesion of connecti- 181. A patient suffering from the essenti-
ve tissue is caused by low activity of the al hypertension presents with an increase
following enzyme: in the arterial pressure up to 180/110 mm
A. Proline hydroxylase Hg; dyspnea, cyanosis, tachycardia; heart
B. Alanine aminotransferase borders are dilated to the left, in lungs
C. Pyruvate carboxylase moist rales are present. What signs of
D. Tryptophane hydroxylase urgent compensation for cardiac failure
E. Glutaminase are observed?

177. Low rate of vitamin B6 in the dietary A. Tachycardia


intake leads to disturbance of protein B. Arterial pressure rise
metabolism. What biochemical processes C. Cyanosis
in the patient’s organism will become less D. Dyspnea
active? E. Myogenic dilatation
182. A patient has been suffering from di-
abetes mellitus for 10 years. He was deli-
vered to a hospital in grave condition.
On the 2nd day of treatment his condi-
tion grew significantly worse: he lapsed
60
krok123.in.ua
Krok 1 Pharmacy 2009 21

into a coma, there appeared noisy deep line with ordinate axis of the equilibrium
breathing. Deep inspirations took turns diagram of a two-component system:
with forced expirations with assistance of
expiratory muscles. What form of respi- A. С = 0
ration disorder is it? B. С = 2
C. С = 1
A. Kussmaul’s respiration D. С = –1
B. Stenotic respiration E. С = 3
C. Tachypnea
D. Cheyne-Stokes respiration 188. After taking phenacetin a pati-
E. Biot’s respiration ent complained about sore throat
and impossibility of deglutition. An
183. The product of potassium otolaryngologist made a dignosis of
permanganate reduction in the neutral necrotic angina. In blood: Hb- 130 g/l,
medium has the following chemical erythrocytes - 4, 5 · 1012 /l, leukocytes -
formula and colour: 3, 0 · 109 /l, among them lymphocytes -
75%, neutrophils - 10%, eosinophils - 5%,
A. MnO2 , brown monocytes - 10%. What type of white
B. MnO2 , green blood cell disorder is it?
C. K2 MnO4 , green
D. K2 MnO4 , violet A. Neutropenia
E. MnSO4 , colourless B. Neutrophilia
C. Monocytosis
184. What is maximal valency of ni- D. Eosinophylia
trogen in consideration of donor-acceptor E. Lymphopenia
mechanism of covalent bond?
189. 3 years ago a patient was diagnosed
A. 4 with chronic glomerulonephritis. The pati-
B. 1 ent has got multiple edemata within the
C. 2 last 6 months. What is the cause of their
D. 3 development?
E. 5
A. Proteinuria
185. Microscopy of a leaf epidermis B. Hyperaldosteronism
of Convallaria majalis showed that the C. Injection of non-steroidal anti-
stomata had four accessory cells. Two of inflammatory preparations
them were lateral, and two other were D. Glucocorticoid treatment
polar. What type of stomatal mechanism E. Vasopressin hyperproduction
is it?
190. One of the herbarium specimens of
A. Tetracytic medicinal plants relates to the Asteraceae
B. Diacytic family. This plant is:
C. Anisocytic
D. Anomocytic A. Arctica lappa
E. Paracytic B. Atropa belladonna
C. Cassia acutifolia
186. Medical examination of a dairymaid D. Urtica dioica
revealed affection of the locomotive E. Rubus idaeus
system, vision impairment, disorder of the
nervous and other systems. To confirm the 191. Examination of a medicinal herb
diagnosis the patient was referred for a revealed that its leaves were divided down
serological assay (Wright’s reaction) and to the base of the leaf blade with segments
Burnet’s skin allergy test. What was the radiating from a common point in a fan
provisional diagnosis? manner. These leaves are:
A. Brucellosis A. Palmatisected
B. Tularemia B. Pinnatisected
C. Anthrax C. Palmatipartite
D. Rheumatism D. Pinnatipartite
E. Leptospirosis E. Palmatilobate
187. Specify the number of degrees of 192. What is the primary standard for
freedom for intersection of the liquidus standardization of Hg2 (NO3 )2 solution?
61
krok123.in.ua
Krok 1 Pharmacy 2009 22

A. Iodine fluorochloromethane (CHIF Cl)


A. Sodium chloride B. Methane (CH4 )
B. Sodium bromide C. Chloroform (CHCl3)
C. Sosium sulphate D. Dichloromethane (CH2 Cl2 )
D. Sodium hydroxide E. Tetrachloromethane (CCl4 )
E. Sodium dichromate
197. What analytical effect is observed
193. After the diluted solution of when potassium cation is being determi-
hydrochloric acid had been added to the ned by the sodium hexanitrocobaltate
solution under examination, the white
caseous precipitate settled down. This (III) solution?
indicates presence of the following ions: A. Yellow crystalline precipitate
A. Silver B. White crystalline precipitate
B. Ammonium C. Yellow colouring of the solution
C. Iron (II) D. Black crystalline precipitate
D. Barium E. Red crystalline precipitate
E. Iodine 198. Epidemic of influenza was
194. Tritane relates to: announced in a town. Which drug can
be recommended for the nonspecific
prophylaxis of influenza?
A. Leukocytic interferon
B. Anti-influenza vaccine
C. Antibiotics
D. Anti-influenza immunoglobulin
E. Anti-influenza serum
199. A drug solution under examination
A. Multinuclear arenes with isolated contains cations of magnesium (II) and
benzene cycles aluminium (III). Which reagent can help
B. Multinuclear arenes with condensated to separate these cations during analysis
benzene cycles of this drug?
C. Mononuclear arenes
D. Alkanes A. Alkali solution
E. Alkenes B. Solution of hydrogen peroxide in acidic
medium
195. Name the process characterized C. Solution of silver nitrate
by a chemical interaction between an D. Ammonia solution
adsorbate and an adsorbent: E. Solution of chloride acid
A. Chemical adsorption 200. Quantitative determination of
B. Solvation pharmaceutical substances can be carri-
C. Absorption ed out by method of alkalimetry using 0,1
D. Desorption M sodium hydroxide solution as a titrant.
E. Sedimentation Precise concentration of sodium hydroxi-
de can be determined according to:
196. Optical isometry can be applied to
the following compounds: A. Oxalic acid
B. Sodium tetraborate
C. Potassium dichromate
D. Sodium thiosulphate
E. Ammonium hydroxide

62
krok123.in.ua
Krok 1 Pharmacy 2010 1

1. A man got an injection of curareli- Name it:


ke substance causing the relaxation of all
skeletal muscles. What is its mechanism of A. Proserin
action? B. Aspirin
C. Sodium diclophenac
A. Block of cholinergic receptors of D. Indometacin
postsynaptic membrane E. Allopurinol
B. Disturbance of acetylcholine synthesis
C. Block of Ca2+ -channels of presynaptic 7. A 30 year-old patient suffering from
membrane pulmonary tuberculosis has been prescri-
D. Disturbance of cholinesterase synthesis bed isoniazid. Continuous taking of this
E. Disturbance of acetylcholine secretion drug may lead to the deficiency of the
following vitamin:
2. As a result of rocking a passenger has
developed the sea sickness. The reflexes A. Pyridoxine
causing the development of this sickness B. Tocopherol
are provoked by the excitement of the C. Cobalamin
following receptors: D. Ergocalciferol
E. Retinol
A. Vestibular
B. Visual 8. Examination of the lower limbs of a 40-
C. Tactile year-old patient with coronary artery di-
D. Gustatory sease and vascular disease of lower limbs
E. Olfactory (obliterating endarteritis) revealed skin
pallor and dystrophy, local temperature
3. Production of primary urine in kidneys decrease, sense shock, pain. The patient
is induced by filtration in renal corpuscles. is likely to have the following disorder of
What components of blood plasma are the peripheral blood circulation:
absent in the primary urine?
A. Obstruction ischemia
A. Proteins B. Compression ischemia
B. Amino acids C. Angiospastic ischemia
C. Glucose D. Venous hyperaemia
D. Urea E. Arterial hyperaemia
E. Ions
9. A 45-year-old patient complains of
4. Microscopic examination of primary nausea, foul-smelling eructation, periodic
cortex of a root in its absorption zone vomiting, meteorism. Fractional analysis
revealed that it consisted mainly of of the secretory function of stomach
multilayer loose living parenchyma with revealed the absence of hydrochloric aci-
amyloid granules. It is called: ds and some enzymes. The patient has the
following pathology of the gastrointesti-
A. Mesoderm nal tract:
B. Endoderm
C. Exoderm A. Achylia
D. Collenchyme B. Hypochlorhydria
E. Phellogene C. Hypoacidic state
D. Achlorhydria
5. Morphological analysis of an E. Anacidic state
inflorescence revealed that its flowers
were attached to the same axis at different 10. Particles of dispersed phase of an
levels but due to the various length of emulsion are deformed and look as
peduncle they grew in the same plane. polyhedrons. What emulsion is it?
Such inflorescence is called:
A. High-concentrated
A. Corymb B. Concentrated
B. Anthodium C. Diluted
C. Glomus D. Oil-in water
D. Umbel E. Water-in-oil
E. Spike
11. Oxidizing properties of free halogens
6. A patient has been administered a increase in the following group:
competitive inhibitor of cholinesterase.
63
krok123.in.ua
Krok 1 Pharmacy 2010 2

A. I2 , Br2 , Cl2 , F2 A. Collagen


B. F2 , Cl2 , Br2 , I2 B. Prothrombin
C. Cl2 , F2 , I2 , Br2 C. Fibrinogen
D. Br2 , F2 , I2 , Cl2 D. Albumin
E. I2 , Cl2 , Br2 , F2 E. Ceruloplasmin
12. Dimethyl glyoxime entered into reacti- 17. It is known that infectious type B
on with a solution that contained cati- hepatitis is a systemic disease caused by
ons of the IV analytical group (acid-base the type B hepatitis virus and characteri-
classification). The deposition turned cri- zed by a predominant liver affection.
mson. What cation caused this analytical Choose from the below given list the
effect? drugs for the etiotropic therapy of this
infection:
A. Nickel cation (II)
B. Mercury cation (II) A. Acyclovir
C. Copper cation (II) B. Penicillin
D. Cadmium cation (II) C. Tetracycline
E. Cobalt cation (II) D. Sulfanilamides
E. Fluoroquinolones
13. A child with evident hypotrophy got
edemata on his lower extremities, asci- 18. In compliance with the requirements
tes. What is the main mechanism of of National Pharmacopoeia of Ukrai-
pathogenesis of cachectic edema? ne the following drugs should be steri-
le: eye drops, parenteral drugs as well
A. Drop of oncotic pressure of blood as substances and additives used in their
plasma production. What method is applied for
B. Rise of hydrostatic blood pressure control of their sterility?
C. Rise of oncotic pressure of intercellular
fluid A. Membrane filtration
D. Increased permeability of vascular wall B. Filter paper discs
E. Disturbance of lymph outflow C. Serial dilution
D. Agar diffusion
14. Silver nitrate is used in ophthalmology E. Two-phase fermentative
as an antibacterial and anti-inflammatory
agent. AgNO3 can be produced as a 19. What substance can act as both oxi-
result of interaction between the followi- dant and reducer in oxidation-reduction
ng substances: reactions?

A. Ag + HNO3 A. SO2
B. SO3
B. AgCl + NH4 NO3
C. CO2
C. Ag + KNO3
D. Ag2 O + KNO3 D. P bO2
E. CrO3
E. AgCl + NaNO3
20. Heparin is the direct-acting anti-
15. Solution of Trilon B is the ti-
trant in chelatometry. It makes complex coagulant that reduces blood coagulati-
on and prevents thrombosis. Its action is
compounds with metal cations irrespecti-
based upon the following phenomenon:
ve of their valency with the following
proportion: A. Protective power of colloids
A. 1:1 B. Syneresis
C. Thixotropy
B. 1:3
C. 1:2 D. Micelle formation
E. Dialysis
D. 2:1
E. 3:1 21. Examination of a patient revealed
an increase in low-density lipoprotein
16. High-grade deficit of the ascorbic acid
causes development of scorbutus. This concentration in blood serum. The pati-
ent can be expected to have the following
pathology develops due to the disturbed
synthesis of the following connective ti- disease:
ssue protein:

64
krok123.in.ua
Krok 1 Pharmacy 2010 3

A. Atherosclerosis A. Paraplegia
B. Pneumonia B. Tetraplegia
C. Glomerulonephritis C. Monoplegia
D. Acute pancreatitis D. Hemiplegia
E. Gastritis E. Paraparesis
22. Nitrogen (I) oxide (N2 O) is applied for 28. In course of an experiment the experi-
inhalation narcosis. It is obtained by heati- menters are stimulating a sympathetic
ng of: nerve responsible for heart innervation.
What changes in cardiac activity can be
A. NH4 NO3 expected?
B. NH3
C. Cu(NO3 )2 A. Increase in heart rate and force
D. NH4 OH B. Decrease in heart force
E. NaNO3 C. Decrease in heart rate
D. Deceleration of excitement conduction
23. After examination a patient has E. Increase in heart rate
been diagnosed with alkaptonuria. This
pathology is caused by the deficit of the 29. Sodium arsenate solution can be di-
following enzyme: stinguished from the arsenite solution by
means of the following reagent:
A. Homogentisic acid oxidase
B. Diamine oxidase A. Magnesia mixture
C. Acetylcholinesterase B. Potassium sulphate
D. Thyroxin hydroxylase C. Potassium nitrate
E. Monoamine oxidase D. Sodium chloride
E. Sodium fluoride
24. Plant pathogenic microorganisms
relate to various groups. Which of them 30. Passive and active transport of
causes diseases of medicinal plants most substances is realized through the cell
often? membrane. Name the type of active
transport by which the membrane changes
A. Fungi its structure:
B. Viruses
C. Bacteria A. Endocytosis
D. Actinomycetes B. Osmosis
E. Micoplasma C. Filtration
D. Diffusion
25. Calcium hydrogen sulphide E. Facilitated diffusion
hexahydrate is often used in the
cosmetological practice. Specify the the 31. Disperse systems are widely used in
formula of his salt: the pharmaceutical practice. The evidence
of colloidal state is the passing of light
A. Ca(HS)2 · 6H2 O through the system. In this case the beam
B. CaS · 6H2 O of light:
C. CaSO3 · 6H2 O
D. Ca(HSO3)2 · 6H2 O A. Is diffused in form of light cone
E. CaSO4 · 6H2 O B. Is reflected
C. Is adsorbed
26. Iodometric determination of D. Is refracted
formaldehyde in formaline can be done E. Penetrates into the particle
by the back titration. Iodine surplus is ti-
trated with the standard solution of: 32. Qualitative determination of the
following compound is accompanied by
A. Sodium thiosulphate blue stain of the ether layer:
B. Sodium nitrate
C. Sodium sulphate A. H2 O2
D. Sodium carbonate B. Cl2
E. Sodium phosphate C. Na2 HP O4
D. MnSO4
27. As a result of spine injury a female E. F eSO4
patient has no voluntary movements of
her lower limbs. This disorder is called: 33. Corolla of the origanum flower is
zygomorphic, sympetalous and consists of
65
krok123.in.ua
Krok 1 Pharmacy 2010 4

a tube and two limbs. The upper limb is in presence of concentrated H2 SO4 .
bilobate and the lower is trilobate. Such Formation of a brown ring indicates
corolla is called: presence of:
A. Bilabiate A. Nitrate ions
B. Unilabiate B. Acetate ions
C. Lingulate C. Carbonate ions
D. Thimble-like D. Oxalate ions
E. - E. Phosphate ions
34. Water-soluble vitamins take coenzyme 39. A patient complains of pain behind
form in an organism. Thiamine di- the breastbone on the left, perspiration
phosphate is the coenzyme of the followi- and palpitation. Which of the followi-
ng vitamin: ng enzymes should be found in blood
in order to confirm the diagnosis of
A. B1 myocardium infarction?
B. B2
C. C A. AspAT, CPK, LDH-1
D. B6 B. AlAT, aldolase, LDH-4
E. B12 C. Amylase, alkaline phosphatase, AlAT
D. Acid phosphatase, LDH-5, LDH-4
35. Biochemical function of water-soluble E. α-fetoprotein, aldolase, CPK
vitamins depends on their ability to turn
into the coenzymatic forms. Specify the 40. For tuberculosis prevention the
coenzymatic form of the vitamin B2 (ri- newborns got an injection of a vaccine.
boflavin): What vaccine was used?

A. FMN (flavin mononucleotide) A. BCG


B. NAD+ (nicotinamide adenine di- B. Mantoux
nucleotide) C. DTaP vaccine
C. TMP (thiamine monophosphate) D. Anatoxin
E. Oral polio vaccine (Sabin vaccine)
D. TDP (thiamine diphosphate)
E. PALP (pyridoxal phosphate) 41. Kinetic methods are used for determi-
nation of drug stability. What is the order
36. Enzymes (biological catalysts) are of reaction if its rate constant equals to
used as pharmacologic preparations. c−1 ?
What is the mechanism of enzyme acti-
on in the biochemical reactions? A. First
B. Zero
A. They reduce the energy of reaction C. Fractional
activation D. Second
B. They increase the energy of reaction E. Third
activation
C. They inhibit the reaction process 42. Examination of a root revealed a ti-
D. They change the constant of the reacti- ssue that has root fibrils and doesn’t have
on rate stomata and cuticle. What tissue is it?
E. They change the reaction order
A. Epiblema
37. Filter paper impregnated with solution B. Epiderm
of cobalt (II) nitrate and a solution under C. Periderm
examination makes blue ash when burned D. Endoderm
down. This is the evidence of presence of E. Exoderm
the following ions:
43. During identification of a perennial
A. Al3+ herb of Ranunculaceaе family it was found
B. Cr 3+ to have: apical flowers of regular form
C. Ni2+ up to 6 cm in diameter; 5 downy violet-
D. Sb3+ and-green calyx lobes of irregular serrate
E. Zn2+ form; up to 20 bright yellow glossy petals
without nectarostigma. What plant is it?
38. A solution under examination
was added to the solution of F eSO4
66
krok123.in.ua
Krok 1 Pharmacy 2010 5

A. Аdonis vernalis A. Thyroid hypofunction


B. Helleborus purpurascens B. Parathyroid hypofunction
C. Ranunculus acris C. Thyroid hyperfunction
D. Delphinium elatum D. Parathyroid hyperfunction
E. Aconitum napellus E. Adrenal hypofunction
44. A section of beet root has several 49. In compliance with the requirements
layers of cambium that form additional of WHO and Pharmacopoeia of Ukraine
conducting bundles. What is the structure the number of microorganisms in 1 ml of
of the given root? ear drops shoud not exceed the following
number of microbial cells (bacteria and
A. Secondary, polycambial fungi):
B. Secondary monocambial
C. Primary, polycambial A. 100
D. Primary, monocambial B. 10
E. Transitional, monocambial C. 1000
D. 10 000
45. What is the final product of methane E. 100 000
chlorination?
50. Drug quality is estimated by a number
of factors including the "microbiologic
purity". What drugs are allowed to
include a greater number of saprophytic
bacteria as compared to other drug forms?
A. Tetrachloromethane A. Tinctures
B. Chloroform B. Aerosols
C. Chloroethanol C. Suppositories
D. Ethane D. Eye drops
E. Chloromethane E. Injection solutions
46. A solution contaning anions of the 51. Choose the reagent that can be used
second analytical group has been blended for acetone cyanohydrin production:
with the solution of argentum nitrate. This
resulted in formation of black precipitate
insoluble in the ammonia solution and
soluble in the diluted nitric acid at heating.
What anions are present in the solution?
A. Sulphide ions
B. Iodide ions
C. Chloride ions
D. Bromide ions A. HCN
E. Arsenite ions B. H2 N  OH
C. H2 N  NH2
47. 0,1 M solution of potassium D. H2 N − NH − C6 H5
permanganate is used as a titrant in E. H2 N − CH3
permanganatometry. The solution is
prepared like the secondary standard 52. Which reagent allows to distinguish
solution and standardized according to: propine (CH3 − C ≡ CH) from propene
(CH3 − CH = CH2 )?
A. Ammonia oxide
B. Potassium dichromate A. [Ag(NH3 )2 ]OH
C. Sodium chloride B. Br2
D. Sodium carbonate C. HCl
E. Calcium oxide D. Cu(OH)2
E. Cl2
48. A patient has bradycardia, moderate
hypotension, decrease of basal metaboli- 53. What compound is produced as a
sm, edemata. What disorder can induce result of reaction:
such syndrome?

67
krok123.in.ua
Krok 1 Pharmacy 2010 6

A. Hydroxylation
B. Oxidation
C. Methylation
D. Acetylation
E. Reduction
57. For production of phenol ether it is
necessary to cause reaction of sodium
phenoxide with:
A.

B.

C.

A. CH3 Cl
D. B. CH3 OH
C. CH4
D. CH3 NH2
E. CH3 C ≡ N
58. Polysaccharide cellulose consists of the
E. remains of the following monosaccharide:

54. Stable contraction of myofibrilla


of muscle fibers takes place due to
accumulation of the following ions in the
cytoplasm:
A. Calcium
B. Potassium
C. Sodium
D. Magnesium
E. Hydrogen
55. A 45-year-old woman has frequent A. β-D-glucopyranose
uterine haemorrhages, she presents with B. α-D-glucopyranose
general weakness, dyspnea, tachycardia, C. β-D-fructopyranose
cardiac pain. In blood: erythrocytes - D. α-D-fructofuranose
E. β-D-glucofuranose
3 · 109 /l, haemoglobin - 70 g/l, colour
index - 0,7. The smear contains mostly 59. A higher nonvascular plant has di-
hypochromic erythrocytes, microcytes. stinct alternation of dominant sexual
Specify the type of anaemia accrding to (gametophyte) and reduced asexual
its mechanism of development: (sporophyte) generations. This indicates
that the plant belongs to the following di-
A. Iron-deficiency
vision:
B. B12 -folate-deficiency
C. Haemolytic A. Bryophyta
D. Minkowsky-Shauffard disease B. Lycopsida
E. Protein-deficiency C. Equisetophyta
56. Single-oxygenase system of D. Pteridophyta
membranes of endoplasmic hepatocyte E. Gymnospermae
reticulum includes flavoprotein NADF- 60. It is known that a seed without
cytochrome, R-450-reductase and R-450- endosperm and perisperm has its nutri-
cytochrome. It stimulates inactivation of ents accumulated in:
biologically active substances or neutrali-
zation of toxic compounds by catalyzing
the reaction of:
68
krok123.in.ua
Krok 1 Pharmacy 2010 7

A. Embryo cotyledons 66. Aqueous-alcocholic mixtures are wi-


B. Embryo root dely used in the medical and pharmaceuti-
C. Embryo stalk cal practice. They relate to the azeotropes.
D. Gemma What is the peculiarity of azeotropic mi-
E. Seed coat xtures?
61. Specify the standardized solutions A. They produce a vapor of the same
used for direct and back titration of composition as the mixture
reducing agents in the iodometric method: B. They don’t mix together
C. They interact with each other
A. I2 , Na2 S2 O3 D. They don’t interact with each other
B. K2 Cr2 O7 , Na2 S2 O3 E. They mix together at a critical
C. I2 , KI temperature
D. KMnO4 , KI
E. K2 Cr2 O7 , I2 67. Iron (II) sulfate is a part of drugs used
in treatment of iron deficiency anemia.
62. Determination of medications contai- F eSO4 enters into reaction with one of
ning cations of magnesium and calcium the following compounds:
is done by trilonometric titration. What
type of chemical reaction takes place in A. KMnO4
this case? B. HCl
C. CO2
A. Complexation D. F eCl2
B. Oxidation-reduction E. NaCl
C. Electrophylic substitution
D. Alkylation 68. Micelle solutions of surfactants are
E. Precipitation applied in pharmaceutical production as
stabilizers and solubilizers. What soluti-
63. During the qualitative analysis under on of colloidal surfactants will have the
the influence of group reagent NaOH greatest value of critical concentration of
upon the aluminium ions the following micelle formation?
substance is produced:
A. C9 H19 SO3 Na
A. Sodium hexahydroxoaluminate B. C14 H29 SO3 Na
B. Aluminium hydroxide C. C16 H33 SO3 Na
C. Sodium metaaluminate D. C12 H25 SO3 Na
D. Basic aluminium salts E. C10 H21 SO3 Na
E. Aluminium oxide
69. During analysis of cations of the IV
64. Presence of arsenic in the raw materi- analytic group Zn cations can be detected
al used in pharmaceutical production can under certain conditions with the followi-
be detected by means of Marsh test. Duri- ng reagent:
ng the test tha compond of arsenic with
hydrogen is produced. What is the oxidati- A. Dithizone
on number of arsenic in this compound? B. Ammonia solution
C. Alkali
A. –3 D. Alkali metal carbonates
B. +3 E. Dimethylglyoxime
C. +5
D. –5 70. Choose the most stable complex ion
E. +1 on the ground of values of instability
constants:
65. Most technological processes in
pharmaceutics run in heterogenous A. [F e(CN)6 ]3− Kн = 1 · 10−31
systems. How many phases has an eutectic B. [Ag(CN)2 ]− Kн = 1 · 10−21
composition at the eutectic temperature
C. [Ag(NH3 )2 ]+ Kн = 5, 89 · 10−8
in the two-component system?
D. [Ni(CN)4 ]2− Kн = 1 · 10−22
A. 3 E. [Co(NH3 )6 ]2+ Kн = 4, 07 · 10−5
B. 2
C. 5 71. Presence of which ion of d-elements in
D. 4 the solutions can be detected by means of
E. 1 K4 [F e(CN)6 ]?

69
krok123.in.ua
Krok 1 Pharmacy 2010 8

A. F e3+ structures in form of a stack of flattened


B. F e2+ membrane cisterns and vesicles were
C. Zn2+ found. What organelles are these?
D. Cr 3+
E. Cu2+ A. Golgi apparatus
B. Endoplasmic reticulum
72. What gas is produced as a result of C. Plastids
interaction of concentrated nitric acid wi- D. Mitochondrions
th sulphur? E. Microbodies
A. NO2 78. The particles of dispersed phase of a
B. H2 ready drug emulsion are sized 10−6 m. The
C. N2 given drug form relates to the following
D. N2 O type of disperse systems (according to the
E. NH3 dispersion degree classification):
73. Early pregnancy test involves analysis A. Microheterogeneous system
of a woman’s urine. Pregnancy is B. Heterogeneous system
ascertained by presence of the following C. Coarse-dispersion system
hormone: D. Colloidal disperse system
A. Chorionic gonadotropin E. Ultramicroheterogeneous system
B. Estriol 79. In oxidation-reduction reactions
C. Aldosterone potassium permanganate KMnO4 acts
D. Testosterone only as an oxidizer. When the reaction
E. Progesterone takes place in the acidic medium, the
crimson solution becomes discoloured.
74. While detecting Co2+ ions in presence Specify the product of MnO4− -ion reducti-
of F e3+ the following ions should be on in the acidic medium:
added to the solution in order to mask
F e3+ ions: A. Mn2+
B. MnO2
A. Fluoride ions C. MnO42−
B. Chloride ions D. [Mn(OH)2 ]
C. Bromide ions E. [Mn(OH)4 ]
D. Nitrite ions
E. Sulphate ions 80. Under certain conditions high-
molecular substances make gellies that
75. This substance can be produced from are widely used in drug production. What
the calcium carbide. It discolours bromine process takes place during jelly ageing?
water and makes metal derivatives. What
compound is it? A. Syneresis
B. Thixotropy
A. Acetylene C. Swelling
B. Ethylene D. Solvatation
C. Ethane E. Diffusion
D. Orenol
E. Aniline 81. Decarboxylation of the amino acid hi-
stidine results in formation of histamine
76. The compound C7 H8 O relates to the in the cells. Neutralization of this biogenic
derivatives of aromatic hydrocarbons, amine takes place due to the following
doesn’t stain with F eCl3 , the product of its enzyme:
oxidation is benzoic acid. What compound
is it? A. Diaminooxidase (DAO)
A. Benzyl alcohol B. Monoaminooxidase (MAO)
B. Methylphenyl alcohol C. Catalase
C. o-cresol D. Aminotransferase
D. m-cresol E. Aminopeptidase
E. p-cresol 82. Chlorophyll, the green pigment of
77. During examination of a plant cell plants, is a chelate compound. Specify the
under the electron microscope some chelating ion in the chlorophyll:

70
krok123.in.ua
Krok 1 Pharmacy 2010 9

A. Mg 2+
B. F e3+ A. Glycol
C. Mn2+ B. Ketone
D. F e2+ C. Carboxylic acid
E. Ni2+ D. Aldehyde
E. Epoxide
83. A flower has the androecium consisti-
ng of two long and two short stamens. 89. Study of the antibioticogram of the
Therefore the flower’s androecium is: pure salmonella culture revealed multiple
antibiotic resistance. What factor might
A. Didynamous have caused this effect?
B. Tetradynamous
C. Diadelphous A. R-plasmids
D. Tetradelphous B. Chromosomal mutations
E. Polyadelphous C. F-plasmids
D. Temperate phages
84. Potassium dichromate K2 Cr2 O7 is E. Transposons
applied as oxidant in acidic medium. What
is the product of reduction of dichromate- 90. After a 5-year-old child has been
ion Cr2 O72− under these conditions? brought home from the kindergarten he
presented with weakness, headache, body
A. Cr3+ temperature rise up to 37, 5o C. What peri-
B. Cr(OH)3 od of disease develompent is the case?
C. Cr(OH)2
A. Prodromal
D. [Cr(OH)6]3− B. Latent
E. Cr2 O3 C. Incubative
85. In course of an experiment a dog has D. Recovery
been injected a preparation that reduces E. Fastigium
secretory and motor activity of stomach. 91. It is required to determine the amount
What preparation is it? of sodium salicylate in a solution. What ti-
A. Atropine trimetric method can be applied for the
B. Histamine quantitative determination of aromatic
C. Secretin compounds?
D. Acetylcholine A. Bromometry
E. Gastrin B. Mercurimetry
86. What ion mechanism is responsible for C. Cerimetry
the development of depolarization phase D. Argentometry
of action potential? E. Chelatometry

A. Sodium influx into the cell 92. When copper (II) hydroxide enters
B. Sodium outflux into reaction with alkali, complex
C. Potassium influx into the cell compounds are produced. What is the
D. Potassium outflux coordination number of copper in such
E. Calcium influx into the cell compounds?

87. Study of an isolated heart shows that it A. 4


keeps on contracting even after removal B. 5
from the body. This effect owes to the C. 6
following peculiarity of the myocardium: D. 3
E. 2
A. Automatism
B. Excitability 93. Interaction of aniline with excess of
C. Conductivity bromine water resulted in formation of
D. Contractility white precipitate. What substance was
E. Adiaphoria produced?

88. What product is obtained in Wagner


reaction during oxidation of alkenes with
potassium permanganate in the aqueous
medium?
71
krok123.in.ua
Krok 1 Pharmacy 2010 10

A. 2,4,6-tribromaniline A. o-xylol
B. 2,4-dibromaniline B. Salicylic acid
C. 2,6-dibromaniline C. 1,2-dichlorobenzene
D. 2-bromaniline D. 2-chlorobenzoic acid
E. 4-bromaniline E. m-xylol
94. For treatment of the psychosis a pati- 99. Interaction of lactic acid with SOCl2
ent was administered the neuroleptic excess will result in production of the
aminazine. The main way of its bi- following compound:
otransformation in the organism is
induction of microsomal oxidation. Speci-
fy the principal component of this system:
A. Cytochrome R-450
B. Cytochrome C
C. Cytochrome oxidase
D. NAD-dehydrogenase
E. CoQ-reductase
95. Every year during the plant blossomi-
ng a female patient develops acute
catarrhal inflammation of conjuncti- A.
va and nasal mucosa that is the cli-
nical presentation of an allergy. These
symptoms relate to the following type of
allergic reactions:
A. Anaphylactic
B. Cytotoxic B.
C. Immune complex
D. Cell-mediated
E. Cellular dysfunction
96. A patient is suspected to have the C.
typhoid fever. What method of laboratory
diagnostics would be the most appropri-
ate for confirmation of this diagnosis in
the first week of disease?
A. Hemoculture identification
B. Urine culture identification D.
C. Myeloculture identification
D. Biliculture identification
E. Coproculture identification
97. Inoculation of hen’s embryos is the
main method of detection of influenza E.
virus. In order to neutralize associated
bacterial flora in the material under
examination (nasopharyngeal lavage) it 100. A group of alpinists climbing to
is necessary to add beforehand: the top had their blood tested. The test
revealed erythrocytosis and increase in
A. Antibiotics hemoglobin rate. What type of hypoxia
B. Eubiotics caused the stimulation of erythropoiesis
C. Fluorescent serum in the bone marrow?
D. Leukocytic interferon
E. Ant-influenza gamma globulin A. Hypoxic
B. Combined
98. Choose the initial compound for one- C. Hemic
stage synthesis of phthalic acid: D. Circulatory
E. Tissue
101. A 42-year-old patient suffering from
72
krok123.in.ua
Krok 1 Pharmacy 2010 11

chronic calculous cholecystitis complains A. 0,85 g


of acute pain in the right subcostal area, B. 8,5 g
itching and skin icteritiousness, multiple C. 4,5 g
petechial haemorrhages, saponified and D. 0,45 g
light-coloured feces (steatorrhea). What E. 5,0 g
type of icterus is it?
107. Which element has the same valence
A. Mechanic in both hydrogen compound and higher
B. Hemolytic oxide?
C. Parenchymatous
D. Cythemolytic A. Carbon
E. Hepatocellular B. Phosphorus
C. Selenium
102. Microscopic examination of leaf D. Bromine
serration revealed secretory structures E. Argon
secreting some liquid. What are these
structures called? 108. Toluol is converted to the benzoic
acid under the following conditions:
A. Hydatodes
B. Nectaries
C. Stomata
D. Glandules
E. Osmophores
103. In pharmaceutical synthesis both si-
mple and complex reactions are applied.
Specify the order of the simple reaction of
type 2A + B = 3D: A. Oxidation with potassium
A. 3 permanganate
B. 2 B. Heating with sulphuric acid
C. 1 C. Hydrogen peroxide action at a room
D. 0 temperature
E. 0,5 D. Sodium hydroxide action at a room
temperature
104. Choose a reduction-oxidation E. Boiling in the open air
method for the quantitative determinati-
on of iron (II) salts in a solution that 109. Some drugs are colloid solutions.
contains hydrochloric acid: Colloidal disperse systems are the systems
whose particles are sized within the range
A. Dichromatometry of:
B. Iodometry
C. Permanganatometry A. 10−9 - 10−7 m
D. Nitritometry B. 10−7 - 10−4 m
E. Ascorbinometry C. 10−4 m
D. 10−9 m
105. In order to prevent adipose E. 10−9 - 10−4 m
degeneration of liver after the viral
hepatitis a patient should be administered 110. The thyroid gland synthesizes a
lipotropins. Name one of them: hormone that lowers the rate of Ca2+
concentration in blood thus facilitating its
A. Choline deposition in bones. What hormone is it?
B. Tryptophan
C. Allopurinol A. Calcitonin
D. Contrykal B. Thyroxin
E. Vicasol C. Triiodthyronine
D. Adrenaline
106. Pharmaceutical practice widely appli- E. Parathormone
es isotonic solution of sodium chlori-
de. How much sodium chloride is to be 111. Which of the following compounds
takenin order to prepare 100 g of the forms a propionic aldehyde as a result of
isotonic solution? alkaline hydrolysis (H2 O, OH − )?

73
krok123.in.ua
Krok 1 Pharmacy 2010 12

A. Cholesterol
B. Albumine
C. Haemoglobin
D. Urates
A. E. Oxalates
115. What method of titrimetric analysis
can be applied for the quantitative
B. determination of sulphuric acid by means
of the potassium hydroxide solution?
A. Alkalimetry
B. Acidimetry
C. C. Oxidation-reduction
D. Precipitation
E. Complexation
116. Ions of which chemical element have
an impact on the electrolytic balance of
D. cerebral tissues. What salt of this element
is used for treating the psychic disorders?
A. Li, Li2 CO3
B. Cl, NaCl
C. I, KI
E. D. Ca, CaCl2
E. Mg, MgSO4
112. In order to choose an indicator duri- 117. Drug production commonly involves
ng the acid-bace titration a titration curve the processes of adsorption and ion
is made which is the dependence of: exchange. What ion is selectively
adsorbed from the aqueous solution based
A. pH solution from the volume of the on silver chloride crystal?
added titrant
B. pH solution from the concentration of A. Ag +
the added titrant B. H +
C. pH solution from the volume of the C. NO 3
solution under analysis D. Cu2+
D. Concentration of the solution under E. OH 
analysis from pH solution
E. pH solution from the temperature 118. Osmotic pressure is an important
characteristic of biologic fluids. Osmotic
113. Nephron is the structural and functi- pressure variates with time in the followi-
onal unit of the kidneys. The process of ng solution:
filtration takes place in the following part
of it: A. Silver chloride sol
B. Glucose
A. Bowman’s capsule C. Calcium sulphate
B. Henle’s loop D. Sodium chloride
C. Collecting tubule E. Magnesium sulphate
D. Proximal tubule
E. Distal tubule 119. 3-aminopropane acid is included in
pantothenic acid which is a component of
114. Inflammatory processes in the gall coenzyme A. What reaction takes place in
bladder exert negative influence on the course of heating of this acid?
colloidal properties of bile. This may lead
to gallstone formation. One of the causes A. Elimination (detachment)
of their formation is the crystallization of B. Substitution
the following substance: C. Addition
D. Rearrangement
E. Reduction
120. The given reaction is called:
74
krok123.in.ua
Krok 1 Pharmacy 2010 13

125. Microscopic examination of ground


tissue of a small branch revealed cork and
felloderm. These are the derivates of:
A. Phellogen
B. Cambium
C. Procambium
D. Protoderm
E. Pericycle
126. Aetiological factors for the infecti-
ous diseases are often microorganisms wi-
A. Acylation th various ultrastructure. Which of the
B. Esterification following microorganism groups relates
C. Addition to the eucariots?
D. Removal
E. Regrouping A. Protozoa
B. Viruses
121. A patient diagnosed with acute C. Viroids
abdomen was delivered to the hospi- D. Prions
tal. A doctor suspected acute appendici- E. Scotobacteria
tis and ordered urgent blood test. What
factor would be the evidence of acute 127. For the technology of drug producti-
inflammation in this patient? on the pressure, temperature and
concentration are of great importance.
A. Leukocytosis What process is accelerated in case of
B. Leukopenia temperature decrease?
C. Eosinophilia
D. Erythrocytosis A. Exothermic
E. Erythropenia B. Endothermic
C. Adiabatic
122. Bacteria may contain not only D. Isochoric
chromosomal but also nonchromosomal E. Isobaric
hereditary elements called plasmids.
Presence of plasmid genes can show itself 128. Before nitration of aniline it is usually
by: acidified in order to protect amino groups
from oxidation. Which of the following
A. Multiple drug resistance reagents is used for this purpose?
B. Stain resistance
C. Physical factor resistance A. (CH3 CO)2O
D. Sporogenesis ability B. CH3 CHO
E. Mobility C. C2 H5 Cl
D. HNO2
123. The method of "accelerated drug E. CHCl3 + NaOH
ageing"used for determination of drug
shelf life is based upon: 129. Amides are weak NH-acids. They
make salts as a result of interaction with
A. Van’t Hoff’s rule one of the given reagents:
B. Fajans’ rule
C. Planck’s postulate A. NaNH2 (Na met.)
D. Ostwald law B. NaOH (H2 O)
E. Raoult law C. P2 O5 (to )
D. NaOBr (Br2 + NaOH)
124. Transamination is the biochemical E. LiAlH4
process in which amino groups of different
amino acids take form of one of the amino 130. Six-membered nitrogen-containing
acids. What amino acid is it? heterocyclic compounds exhibit basic
properties. Which compound has the
A. Glutamic strongest basic properties?
B. Glycine
C. Valine
D. Leucine
E. Arginine
75
krok123.in.ua
Krok 1 Pharmacy 2010 14

A. Piperazine 136. It is required to measure the nitrogen


B. Pyridine metabolism in a person under observati-
C. Pyrimidine on who is recovering from continuous
D. Pyrazine starvation. What result is most likely to
E. Pyridazine be expected?
131. Microscopic examination of a ficus A. Decrease in nitrogen secretion
leaf revealed in some cells of its epidermis B. Nitrogen equilibrium
a protrusion of the cell membrane with an C. Negative nitrogen balance
accumulation of crystals that dissolve in D. Acetonemia
the hydrochloric acid and release carbonic E. -
acid gas. This structure is called:
137. It is required to increase the secreti-
A. Cystolith on of gastric juice in an experimental
B. Raphide dog with stomach fistula. What should be
C. Druse introduced into the stomach?
D. Single crystal
E. Styloid A. Meat broth
B. White bread
132. Aminotransferases are the enzymes C. Milk
that transfer an amino group from one D. Dried bread
compound to another. What compound is E. Sour cream
the acceptor of amino groups?
138. After a bacteriological analysis a
A. α-ketoglutaric acid tableted medication has been found to be
B. Acetone inapplicable, though its general microbi-
C. Lactic acid al contamination was within the norm.
D. Succinic acid The reason for such a conclusion was the
E. Butyric acid presence of the following microorganisms:
133. A composition under examination A. Enterobacteria
contains ions of Cl , Br  and I  in equi- B. Mold fungi
molar quantities. The sequence of precipi- C. Actinomycetes
tate formation in course of argentometric D. Micrococci
titration will be determined by: E. Sarcinae
A. Solubility product of the corresponding 139. From a medicinal herb a certain
silver halogenides phytopathogenic microorganism was
B. Value of oxidation-reduction potentials secured. In the nutrient medium it forms
C. Way of titration - either back or direct "fried egg"colonies. What is the most li-
D. Value of corresponding ion mobility kely agent?
E. Ionic strength of solution
A. Mycoplasma
134. When chlorine is passed through the B. Yeast fungi
cold solution of potassium hydroxide the C. Actinomycetes
following compound are produced: D. Nocardia
E. Pseudomonades
A. KCl, KClO, H2 O
B. KCl, KClO2 , H2 O 140. While examining structure of a root
C. KCl, H2 O the students payed attention to an area
D. KClO3 , KClO, H2 O where the superficial cells formed root fi-
E. KClO3 , H2 O brils. What root zone is it?
135. A 37-year-old man was admitted A. Suction
to a hospital with an attack of bronchi- B. Cell division
al asthma. What respiration type will be C. Extension
observed in this patient? D. Conduction
E. Pileorhiza
A. Expiratory dyspnea
B. Inspiratory dyspnea 141. Choose benzo-1,4-diazepine from the
C. Apnoea given heterocyclic compounds:
D. Gasping respiration
E. Hyperpnoea
76
krok123.in.ua
Krok 1 Pharmacy 2010 15

A. HCl, NaOH
B. HCl, H2 SO4
C. KOH, NaOH
A. D. NaOH, KCl
E. H2 SO4, K2 SO4
146. Nitritometric determination of
compounds containing primary aromatic
B. amino group can be carried out under the
following conditions:
A. With observation of all the mentioned
conditions
C. B. At a temperature up to 10o C
C. With adding of the crystalline KBr
(catalyst)
D. Chloric acid excess
E. Slow titration
D.
147. Under anaerobic conditions duri-
ng glycolysis ATP is synthesized by
the way of substrate phosphorylation.
E. This process uses energy of other high-
energy compounds. Specify one of such
compounds:
142. What is the osmotic pressure of medi-
cinal solutions used as blood isotonics? A. Phosphoenol pyruvate
B. Glucose 6-phosphate
A. 740 - 780 kPa C. Lactate
B. 420 - 448 kPa D. Pyruvate
C. 900 - 960 kPa E. Glucose
D. 600 - 670 kPa
E. 690 - 720 kPa 148. A patient with acute pneumonia has
an edema and hardening of pulmonary ti-
143. Under what conditions the limited ssue. What cells are the first to infiltrate
swelling of gelatine turns into the unlimi- the inflammation zone and provide the
ted one? effective protection from the bacterial
infection?
A. Heating
B. Cooling A. Neutrophils
C. In presence of P O43− ions B. Monocytes
C. Thrombocytes
D. In presence of Cl− ions D. Eosinophils
E. In presence of H + ions whose E. Basophils
concentration is equal to their concentrati-
on in the isoelectric point 149. Heart automatism is possible due
to the atypical cardiomyocytes formi-
144. A 57-year-old worker at an asphalt ng the cardiac conduction system. What
plant complains of weakness, cough wi- part of this system is the primary cardiac
th blood-streaked sputum, chest pain. pacemaker?
He has been diagnosed with lung cancer.
What is the first stage of carcinogenesis? A. Sinoatrial node
B. Purkinje’s fibers
A. Transformation C. Atrioventricular node
B. Promotion D. His’ bundle
C. Activization E. His’ bundle branches
D. Progression
E. Induction 150. Examination of a patient revealed
reddening of oral mucosa, cracks on
145. Choose a pair of titrants for the quali- the lips and mouth corners, face skin
tative determination of ammonia in a dryness and desquamation, conjunctiva
solution by the method of back titration: inflammation, vasculature invasion into
the cornea. The possible cause of this
77
krok123.in.ua
Krok 1 Pharmacy 2010 16

pathology is the deficit of the following of bile inflow to the duodenum. This wi-
vitamin: ll cause the failure of hydrolysis of the
following substances:
A. B2
B. C A. Fats
C. E B. Carbohydrates
D. K C. Proteins
E. D D. Fats and carbohydtares
E. Proteins and carbohydrates
151. During the practical training the
students placed the isolated frog’s heart 156. As a result of sulfonation of
into a solution. This caused the cardiac naphthalene with concentrated sulfuric
arrest in diastole. What solution was the acid at a temperature over 160o C the
heart placed into? following substance is produced:
A. 3% solution of KCl A. 2-naphthalensulfonic acid
B. 1% solution of NaCl B. 1-naphthalensulfonic acid
C. 3% solution of NaCl C. 3-naphthalensulfonic acid
D. 1% solution of CaCl2 D. 4-naphthalensulfonic acid
E. 0,1% solution of MgCl2 E. 5-naphthalensulfonic acid
152. Which atoms of carbon in the given 157. One of the plants under examinati-
compound on has a zygomorphic flower and papili-
onaceous corolla. This plant is called:
A. Melilotus officinalis
B. Mentha piperita
C. Valeriana officinalis
are in the second valence state (sp2 - D. Urtica dioica
hybridization)? E. Rosa canina

A. 1 and 2 158. An elderly woman complains of twi-


B. 1 and 3 light vision impairment. Which of the
C. 2 and 3 following vitamins should be admini-
D. 3 and 4 stered in this case?
E. 5 and 6
A. A
153. To relax skeletal muscles during B. C
complex surgeries, curarelike substances C. E
are applied. These substances block the D. D
following structure: E. P P

A. Neuromuscular synapses 159. A patient is 50 years old. Ad a


B. Basal ganglions result of continuous improper diet he has
C. Red nuclei of the mesencephalon developed hypovitaminosis C. Lesion of
D. Synaptic structures of the spinal cord connective tissue is caused by low activity
E. Vegetative ganglions of the following enzyme:

154. It is known that the digestion of A. Proline hydroxylase


proteins, fats and carbohydrates is possi- B. Alanine aminotransferase
ble due to the protease, lipase and C. Pyruvate carboxylase
amylase respectively. What digestive jui- D. Tryptophane hydroxylase
ce contains the enough supply of all the E. Glutaminase
groups of enzymes?
160. Before diving experienced divers first
A. Pancreatic take several deep breaths. They do it in
B. Saliva order to:
C. Gastric
D. Bile
E. Gastric juice and bile
155. Lligation of the common bile duct in
an experimental animal results in block
78
krok123.in.ua
Krok 1 Pharmacy 2010 17

A. Remove as much as possible CO2 al hypertension presents with an increase


B. Reduce functional residual capacity of in the arterial pressure up to 180/110 mm
lungs Hg; dyspnea, cyanosis, tachycardia; heart
C. Increase lung vital capacity (LVC) borders are dilated to the left, in lungs
D. Increase total lung capacity (TLC) moist rales are present. What signs of
E. Increase respiratory volume (RV) urgent compensation for cardiac failure
are observed?
161. During the preventive medical
examination a doctor revealed a signifi- A. Tachycardia
cant weakening of patellar reflex in one B. Arterial pressure rise
of the patients. What part of the central C. Cyanosis
nervous system is likely to be affected? D. Dyspnea
E. Myogenic dilatation
A. Spinal cord
B. Hindbrain 167. A medicinal herb under examination
C. Midbrain has the capsule fruit with lacticifers and
D. Interbrain small openings. This herb is called:
E. Cerebellum
A. Papaver somniferum
162. Systematic and intensive physi- B. Chelidonium majus
cal exercise causes reduction of fat C. Zea mays
concentration in the adipose tissues. It is D. Mentha piperita
released from the cells into the blood in E. Sanquisorba officinalis
form of:
168. When a smear is stained by Burry-
A. Free fatty acids and glycerine Gins method a mucous structure that
B. Chylomicrons is tightly bound with the cellular wall
C. Lipoproteins of bacteria and has well-defined outer
D. Ketone bodies boundaries can be detected. This element
E. Glucose of a bacteria cell is called:
163. Production of a number of drugs A. Capsule
requires sterile isotonic solution. Choose B. Spore
the optimal method of its sterilization: C. Filaments
D. Ribosomes
A. Steam under pressure sterilization E. Episomes
B. Dry heat sterilization
C. Boiling 169. Sanitary-biologic examination of
D. Direct flame sterilization air in a drugstore revealed a sanitary-
E. Pasteurization indicative microorganism. Name it:
164. Manganese tetrachloride is very A. Staphylococcus aureus
unstable. It can be easily decomposed B. Colon bacillus
into: C. Fecal enterococcus
D. Alpha-haemolytic streptococcus
A. MnCl2 + Cl2 E. Citrobacter
B. Mn + Cl2
C. Cl2 170. A patient presents with fever, chi-
D. Mn ll and cough. From his sputum the ovoid
E. MnCl3 + Cl2 Gram-negative bipolar-stained bacilli wi-
th a delicate capsule were secured. What
165. Glass electrode is commonly used for is the most likely diagnosis?
pH measurement in the biologic media,
fluid drug forms etc. What type does the A. Plague
glass electrode relate to? B. Tuberculosis
C. Leptospirosis
A. Ion selective electrode D. Brucellosis
B. I type electrode E. Toxoplasmosis
C. Reduction-oxidation electrode
D. II type electrode 171. A drugstore received a supply of a
E. Gas electrode drug that is widely used for treatment of
many virus diseases since it is not virus
166. A patient suffering from the essenti- specific. What drug is it?
79
krok123.in.ua
Krok 1 Pharmacy 2010 18

177. A citrus fruit is characterized by the


A. Interferon glandular exocarp, spongiose mesocarp
B. Remantadin and overgrown endocarp consisting of jui-
C. Metisazone ce sacs. Such fruit is called:
D. Immunoglobulin
E. Vaccine A. Hesperidium
B. Legume
172. A patient was administered an antibi- C. Pod
otic of animal origin for the corneal ulcer D. Drupe
treatment. What is it called? E. Bacca
A. Lysozyme 178. You need to specify a monocarpous
B. Chlorophyllipt one-seeded fruit with hard scleroid
C. Nystatin endocarp and soft mesocarp. This fruit is:
D. Imanin
E. Gramicidin A. Drupe
B. Legume
173. What kind of isomerism is typical for C. Silique
the oleic acid? D. Capsule
E. Bacca
A. Cis-trans-stereoisomerism
B. Optic 179. One of the herbarium specimens of
C. Keto-enol tautomerisn medicinal plants relates to the Asteraceae
D. Enantiomerism family. This plant is:
E. Lactim-lactam tautomerism
A. Arctica lappa
174. As a result of hyperventilation a B. Atropa belladonna
student has developed dizziness. What C. Cassia acutifolia
blood changes are the primary cause of D. Urtica dioica
this effect? E. Rubus idaeus
A. Decrease in CO2 concentration 180. The section of a sunflower seed has
B. pH increase been treated with Sudan III solution that
C. Increase in CO2 concentration caused pink-and-orange staining. This is
D. Increase in O2 concentration the evidence of presence of:
E. Decrease in O2 concentration
A. Fatty oil
175. A Rh-positive child of a Rh-negative B. Protein
woman (secundapara) has yellow skin, C. Starch
pathologic reflexes, convulsions. The child D. Inulin
has an increased rate of indirect bilirubin E. Cellulose
in blood. What type of jaundice is it?
181. Determination of sodium chloride by
A. Haemolytic Folgard’s method involves the following
B. Hepatic with violation of bilirubin techniques:
capture
C. Hepatic with violation of bilirubin A. Back titration, argentometry
conjugation B. Direct titration, argentometry
D. Hepatic with violation of bilirubin C. Substitute titration
excretion D. Back titration, mercurimetry
E. Mechanic E. Direct titration, mercurimetry
176. After a road accident a patient has 182. Tritane relates to:
the arterial pressure at the rate of 70/40
mm Hg and daily diuresis at the rate of
about 300 ml. What is the mechanism of
oliguria development in this case?
A. Decrease in glomerular filtration
B. Increase in glomerular filtration
C. Decrease in tubular reabsorption
D. Increase in tubular reabsorption
E. Decrease in tubular secretion
80
krok123.in.ua
Krok 1 Pharmacy 2010 19

A. Multinuclear arenes with isolated A. No more than 50 bacteria and fungi in


benzene cycles total
B. Multinuclear arenes with condensated B. No more than 500 bacteria and fungi in
benzene cycles total
C. Mononuclear arenes C. No more than 1000 bacteria and fungi
D. Alkanes in total
E. Alkenes D. No more than 1000 bacteria and 100
fungi
183. Specify the reagent that allows to E. No more than 500 bacteria and 50 fungi
produce liquid soap as a result of alkaline
fat hydrolysis (saponification): 188. A warmly dressed child has spent
a considerably long time out of doors.
A. K2 CO3 This resulted in body temperature elevati-
B. NaOH on and general weakness development.
C. CaO What form of thermoregulation disorder
D. P bO is observed in this case?
E. NaHCO3
A. Exogenous hyperthermia
184. Many elements have allotropic modi- B. Endogenous hyperthermia
fications. Specify the allotropic modificati- C. Fever
on of oxygen: D. Heat shock
E. Centrogenous hyperthermia
A. Ozone
B. Phosgene 189. A patient with pneumosclerosis has
C. Quartz blood pH at the rate of 7,34. Analysis of
D. Corundum gas formula of blood showed hypercapnia.
E. Diamond Urine analysis revealed the increase in its
acidity. What form of acid-base disbalance
185. What substance can be identified is the case?
by method of acid-base titration and
oxidation-reduction titration? A. Gaseous acidosis
B. Secretory alkalosis
A. Oxalate acid C. Gaseous alkalosis
B. Sodium sulphate D. Non-gaseous alkalosis
C. Calcium nitrate E. Non-gaseous acidosis
D. Sodium hydroxide
E. Ammonium chloride 190. Specify the different-ligand complex
compound that is used as an antitumour
186. The pancreas secretes an enzyme that drug:
is able to hydrolyze α-1,4-glycosidic li-
nkages in a glycogen molecule. Specify A. [P t(NH3 )2 Cl2 ]
this enzyme: B. [Co(NH3 )5 NO3 ]Cl2
C. Na4 [Sn(OH)3Cl3 ]
A. α-amylase D. [Cu(NH3 )4 (SCN)2 ]
B. Phosphatase E. K2 Na[Co(NO2 )6 ]
C. Enterokynase
D. Chemotrypsin 191. A solution containing the cations of
E. Lysozyme the V analytic group (acid-base classifi-
187. A 6-month old child has been admi- cation) has been taken for the analysis.
nistered a peroral drug. What is the maxi- The solution of sodium hydroxostannite
mal number of bacteria and fungi that is has been added to the composition which
permissible in 1 g of this drug in compli- resulted in formation of black depositi-
ance with the requirements of WHO and on. This is the evidence of presence of the
Pharmacopoeia? following cation:
A. Bi3+
B. F e2+
C. Sb3+
D. F e3+
E. Mg 2+
192. If the amount of a high-molecular
substance added to a sol is very small,
81
krok123.in.ua
Krok 1 Pharmacy 2010 20

then the decrease in stability is possible. A. Ag


This phenomenon is called: B. CuO
C. NaCl
A. Sensibilization D. Ca
B. Solubilization E. [Mg(NO3 )2 ]
C. Mutual coagulation
D. Protective power of colloids 197. Choose a plant whose apical sprouts
E. Colloid coagulation are used in medical practice for sedative
drug production:
193. Which of the following adsorbents
is the most effective for adsorption of a A. Leonurus cardiaca
substance from the aqueous solution? B. Glycyrrhiza glabra
C. Digitalis purpurea
A. Activated carbon D. Ledum palustre
B. Silica gel E. Fagopyrum sagittatum
C. Quartz
D. Bolus alba 198. In course of the systematic analysis
E. Gypsum separation of cations of the V and VI
analytic groups (according to the acid-
194. Characteristic peculiarity of base classification) is carried out under
mechanic plant tissues is that they consi- the action of excess of:
st mainly of dead cells, but there is one
type of mechanic tissues consisting of li- A. Concentrated ammonia solution
ving cells. Which of the listed mechanic B. Sodium hydroxide solution
tissues contains the living protoplast? C. Hydrochloric acid solution
D. Potassium hydroxide solution
A. Collenchyme E. Sulphuric acid solution
B. Scleroids
C. Libriform 199. Transport form of lipids in blood are
D. Perivascular fibers lipoproteins. Cholesterol is transported to
E. Phloem fibers the liver mostly in form of:
195. What cations relate to the I analytic A. High-density lipoproteins
group according to the acid-base classifi- B. Low-density lipoproteins
cation? C. Very-low-density lipoproteins
D. Interferons
A. Sodium, potassium, ammonium E. Albumins
B. Calcium, strontium, barium
C. Silver, lead, nickel 200. Nonsteroid anti-inflammatory drugs
D. Aluminium, magnesium, zinc are used in medical practice for treati-
E. Potassium, barium, bismuth ng the rheumatoid arthritis, osteoporosis,
inflammatory dseases of the connective
196. Which of the following substances tissue. These preparations inhibit the acti-
does the concentrated sulphuric acid react vity of the following enzyme:
with along with production of SO2 ?
A. Cyclooxygenase
B. Hexokinase
C. Succinate dehydrogenase
D. Aminotransferase
E. Xanthine oxidase

82
krok123.in.ua
Krok 1 Pharmacy 2011 1

1. A man got an injection of curareli- N-terminal peptide that is provoked by:


ke substance causing the relaxation of all
skeletal muscles. What is its mechanism of A. Perchloric acid
action? B. Sulfuric acid
C. Acetic acid
A. Block of cholinergic receptors of D. Bile acids
postsynaptic membrane E. Amino acids
B. Disturbance of acetylcholine synthesis
C. Block of Ca2+ -channels of presynaptic 7. Hydrogen is characterized by the
membrane following oxidation rates: −1; 0; +1. The
D. Disturbance of cholinesterase synthesis −1 oxidation rate hydrogen has in:
E. Disturbance of acetylcholine secretion A. Hydrides
2. Production of primary urine in kidneys B. Acids
is induced by filtration in renal corpuscles. C. Hydroxides
What components of blood plasma are D. Water
absent in the primary urine? E. Acid salts

A. Proteins 8. High-grade deficit of the ascorbic acid


B. Amino acids causes development of scorbutus. This
C. Glucose pathology develops due to the disturbed
D. Urea synthesis of the following connective ti-
E. Ions ssue protein:

3. Microscopic examination of primary A. Collagen


cortex of a root in its absorption zone B. Prothrombin
revealed that it consisted mainly of C. Fibrinogen
multilayer loose living parenchyma with D. Albumin
amyloid granules. It is called: E. Ceruloplasmin

A. Mesoderm 9. Cardiac diseases are treated wi-


B. Endoderm th cocarboxylase preparation. This
C. Exoderm preparation is the coenzymatic form of
D. Collenchyme the following vitamin:
E. Phellogene A. B1
4. Particles of dispersed phase of an B. B6
emulsion are deformed and look like C. B12
polyhedrons. What emulsion is it? D. C
E. P
A. High-concentrated
B. Concentrated 10. What substance can act as both oxi-
C. Diluted dant and reducer in oxidation-reduction
D. Oil-in water reactions?
E. Water-in-oil A. SO2
5. A child with evident hypotrophy got B. SO3
edemata on his lower extremities, asci- C. CO2
tes. What is the main mechanism of D. P bO2
pathogenesis of cachectic edema? E. CrO3

A. Drop of oncotic pressure of blood 11. Examination of a patient revealed


plasma an increase in low-density lipoprotein
B. Rise of hydrostatic blood pressure concentration in blood serum. The pati-
C. Rise of oncotic pressure of intercellular ent can be expected to have the following
fluid disease:
D. Increased permeability of vascular wall A. Atherosclerosis
E. Disturbance of lymph outflow B. Pneumonia
6. Protein digestion in the stomach is C. Glomerulonephritis
carried out by pepsin secreted in form D. Acute pancreatitis
of an inactive pepsinogen. Pepsinogen is E. Gastritis
converted to pepsin by the removal of the
12. Nitrogen (I) oxide (N2 O) is applied for
83
krok123.in.ua
Krok 1 Pharmacy 2011 2

inhalation narcosis. It is obtained by heati-


ng of: A. Juice of pancreas
B. Saliva
A. NH4 NO3 C. Gastric juice
B. NH3 D. Bile
C. Cu(NO3 )2 E. Juice of large intestine
D. NH4 OH
E. NaNO3 18. Plant pathogenic microorganisms
relate to various groups. Which of them
13. For determination of nitrate ions di- causes diseases of medicinal plants most
phenylamine was added to the soluti- often?
on under examination. The following
changes were observed: A. Fungi
B. Viruses
A. Generation of blue solution C. Bacteria
B. Generation of yellow deposition D. Actinomycetes
C. Generation of blue deposition E. Micoplasma
D. Generation of brown gas
E. Emergence of a typical smell 19. Iodometric determination of
formaldehyde in formaline can be done
14. A patient was prescribed a bile by the back titration. Iodine surplus is ti-
preparation for better digestion of fatty trated with the standard solution of:
food. What components of this preparati-
on cause fat emulsification? A. Sodium thiosulphate
B. Sodium nitrate
A. Bile acids C. Sodium sulphate
B. Cholesterol and its ethers D. Sodium carbonate
C. Diglycerides E. Sodium phosphate
D. Bilirubinglucuronids
E. Bile pigments 20. It is required to diminish pump functi-
on of patient’s heart. This can be done
15. According to the requirements of by means of blockers of the following
WHO and Pharmacopoeia different drug membrane cytoreceptors:
dosage forms of unsterile preparations
are allowed to have a certain quantity A. β-adrenoreceptors
of bacteria and fungi. What quantity of B. Nicotinic cholinoreceptors
saprophytic bacteria and fungi in 1 g (ml) C. Muscarinic cholinoreceptors
of a peroral preparation will ensure its D. α-adrenoreceptors
safety? E. Dopamine receptors

A. 1000 bacteria and 100 mold fungi 21. Electrolyte solutions are medicinal
B. 500 bacteria and 50 mold fungi preparations. What is the maximum value
C. 250 bacteria and 25 mold fungi of isotonic coefficient for MgSO4 soluti-
D. 500 bacteria and 200 mold fungi on?
E. 1500 bacteria and 150 mold fungi A. 2
16. Antibiotics can be classified accordi- B. 4
ng to various principles. According to the C. 3
action mechanism cephalosporins relate D. 5
to the following group: E. 7

A. Inhibitors of cell wall synthesis 22. A patient had cerebral haemorrhage


B. Inhibitors of protein synthesis that made impossible active motions of
C. Inhibitors of respiratory processes left arm and leg. Muscle tone of these
D. Inhibitors of oxidative phosphorilation limbs is increased, their spinal reflexes
E. Inhibitors of cytoplasmic membrane are intensified, reflex zones are increased.
synthesis What type of CNS disorder is it?

17. It is known that digestion of proteins, A. Central paralysis


fats and carbohydrates happens due to B. Peripheral paralysis
protease, lipase and amylase respectively. C. Spinal shock
What digestive juice contains all three D. Atonic paralysis
enzyme groups enough for digestion? E. Reflex paralysis
84
krok123.in.ua
Krok 1 Pharmacy 2011 3

23. Corolla of the origanum flower is What is the mechanism of enzyme acti-
zygomorphic, sympetalous and consists of on in the biochemical reactions?
a tube and two limbs. The upper limb is
bilobate and the lower is trilobate. Such A. They reduce the energy of reaction
corolla is called: activation
B. They increase the energy of reaction
A. Bilabiate activation
B. Unilabiate C. They inhibit the reaction process
C. Lingulate D. They change the constant of the reacti-
D. Thimble-like on rate
E. - E. They change the reaction order
24. Structure of proteins includes protei- 29. Depressurization of the cabin at an
nogenic amino acids. What is the positi- altitude of 19 km led to instantaneous
on of the amino group in the structure of death of pilots. What is its cause?
these amino acids?
A. Explosive decompression
A. α-position B. Hematencephalon
B. β-position C. Myocardial infarction
C. γ-position D. Bleeding
D. δ-position E. Respiratory centre paralysis
E. -position
30. After a girl had accidentally eaten
25. Biochemical function of water-soluble inedible mushrooms she was admitted
vitamins depends on their ability to turn to the resuscitation unit with symptoms
into the coenzymatic forms. Specify the of impaired consciousness, arterial
coenzymatic form of the vitamin B2 (ri- hypotension, anuria, hyperazotemia.
boflavin): What kind of renal dysfunction is it?

A. FMN (flavin mononucleotide) A. Acute renal failure


B. NAD+ (nicotinamide adenine di- B. Acute glomerulonephritis
nucleotide) C. Acute pyelonephritis
C. TMP (thiamine monophosphate) D. Urolithiasis
E. Urine acid diathesis
D. TDP (thiamine diphosphate)
E. PALP (pyridoxal phosphate) 31. Filter paper impregnated with solution
of cobalt (II) nitrate and a solution under
26. A plant has ribbed and hollow stems, examination forms blue ash when burned
sheathing pinnatisected leaves; compound down. This is the evidence of presence of
umbel inflorescence; fruit with essential the following ions:
oil tubules. These features are typical for
the representatives of the following fami- A. Al3+
ly: B. Cr 3+
C. Ni2+
A. Apiaceae D. Sb3+
B. Solanaceae E. Zn2+
C. Fabaceae
D. Brassicaceae 32. After a solution had been heated wi-
E. Scrophulariaceae th (NH4 )2 S2 O8 in presence of AgNO3 , it
turned crimson. What ions were present
27. One of the examined soft fruits is in the solution?
characterized by essential-oil exocarp,
spongioid mesocarp and overgrown A. Mn2+
endocarp that consists of juice saccules. B. F e3+
What fruit was under examination? C. F e2+
A. Hesperidium D. Co2+
B. Pepo E. Cu2+
C. Multicoccus 33. A solution under examination
D. Drupe was added to the solution of F eSO4
E. Bacca in presence of concentrated H2 SO4 .
28. Enzymes (biological catalysts) are Formation of a brown ring indicates
used as pharmacologic preparations. presence of:
85
krok123.in.ua
Krok 1 Pharmacy 2011 4

up to 6 cm in diameter; 5 downy violet-


A. Nitrate ions and-green calyx lobes of irregular serrate
B. Acetate ions form; up to 20 bright yellow glossy petals
C. Carbonate ions without nectarostigma. What plant is it?
D. Oxalate ions
E. Phosphate ions A. Аdonis vernalis
B. Helleborus purpurascens
34. For tuberculosis prevention the C. Ranunculus acris
newborns got an injection of a vaccine. D. Delphinium elatum
What vaccine was used? E. Aconitum napellus
A. BCG 39. A herb under analysis relates to
B. Mantoux the Malvaceae family and is used as an
C. DTaP vaccine expectorant and coating agent. The stem
D. Anatoxin is erect, with simple palmate three to five
E. Oral polio vaccine (Sabin vaccine) lobed leaves, large pink flowers growing in
short panicles. The herb has schizocarpic
35. Bromination proceeds with generation fruit - a capsule. Identify the plant:
of tribromoderivative in presence of the
following substituent X: A. Althaea officinalis
B. Fragaria vesca
C. Potentilla erecta
D. Tussilago farfara
E. Thymus serpyllum
40. What is the final product of methane
chlorination?

A. Х = OH
B. Х = COOH
C. Х = NO2 A. Tetrachloromethane
D. Х = CHO B. Chloroform
E. Х = SO3 H C. Chloroethanol
D. Ethane
36. Gastric juice of a patient has decreased E. Chloromethane
concentration of enzymes. What secretory
cells of stomach display disfunction? 41. 1M sulphuric acid solution was added
to the solution under study. This resulted
A. Chief cells of glands in formation of white sediment that was
B. Parietal cells of glands soluble in the alkalies. This indicated that
C. Gland mucocytes the solution contains:
D. Cells of tegumental epithelium
E. G-cells A. Plumbum cations
B. Calcium cations
37. Yield of medical products can C. Barium cations
be enhanced by proper choice of D. Argentum cations
temperature conditions during their E. Mercury (I) cations
production. What equation determines
dependence of equilibrium constant from 42. A solution contains cations of zinc
the temperature under constant pressure? and aluminum. Specify the reagent that
enables to detect cations of zinc in this
A. Isobaric lines of chemical reaction solution:
B. Isotherms of chemical reaction
C. Kirchhoff equation
D. Isochores of chemical reaction
E. Gibbs-Helmholtz equation
38. During identification of a perennial
herb of Ranunculaceaе family it was found
to have: apical flowers of regular form
86
krok123.in.ua
Krok 1 Pharmacy 2011 5

A. Potassium hexacyanoferrate (II) soluti-


on
B. Sodium hydroxide solution
C. Cobalt nitrate Co(NO3 )2
D. The excess of 6M sodium hydroxide in
presence of hydrogen peroxide
E. Sulfuric acid solution
43. The polarographic method is
commonly used for the analysis
of inorganic cations and anions. A.
Electroreduction of the analyzed ions is
performed by using:
B.
A. Dropping mercury electrode
B. Platinum electrode
C. Antimony electrode C.
D. Calomel electrode
E. Silver electrode
44. What disorder of local circulation is D.
characterized by pallor, local temperature
drop, pain, local sensitivity disorder,
reduction in the volume of the organ?
A. Ischemia E.
B. Venostasis
C. Thrombosis
D. Embolism 48. Cellular and plasma mediators play
E. Arterial hyperemia an important part in the pathogenesis of
45. Choose the reagent that can be used secondary alteration during inflammati-
for acetone cyanohydrin production: on. What mediators are produced in the
blood plasma?
A. Bradykinin
B. Histamine
C. Leukotrienes
D. Prostaglandins
E. Lysosomal factors
49. A 73-year-old patient had been admi-
tted to a hospital with closed fracture of
A. HCN his right femur. Suddenly his condition
B. H2 N  OH deteriorated, the patient was diagnosed
C. H2 N  NH2 with vascular embolism. What type of
D. H2 N − NH − C6 H5 embolism is observed most often in pati-
E. H2 N − CH3 ents with the fractures of tubular bones?
46. Which reagent allows to distinguish A. Fatty
propine (CH3 − C ≡ CH) from propene B. Air
(CH3 − CH = CH2 )? C. Tissue
D. Retrograde
A. [Ag(NH3 )2 ]OH E. Gas
B. Br2
C. HCl 50. Stable contraction of myofibrilla
D. Cu(OH)2 of muscle fibers takes place due to
E. Cl2 accumulation of the following ions in the
cytoplasm:
47. What compound is formed as a result
of reaction:

87
krok123.in.ua
Krok 1 Pharmacy 2011 6

A. Calcium asexual (sporophyte) generation. This


B. Potassium indicates that the plant relates to the:
C. Sodium
D. Magnesium A. Bryophyta
E. Hydrogen B. Lycopodiophyta
C. Equisetophyta
51. A 45-year-old woman has frequent D. Pteroid
uterine haemorrhages, she presents with E. Gymnosperms
general weakness, dyspnea, tachycardia,
cardiac pain. In blood: erythrocytes - 55. A patient has a necrotizing phlegmon
3 · 109 /l, Hb- 70 g/l, colour index - 0,7. of his lower extremity. A doctor suspects
The smear contains mostly hypochromic a gas gangrene. Microscopy reveals gram-
erythrocytes, microcytes. Specify the type positive bacilli. In order to confirm the
of anaemia accrding to its mechanism of diagnosis further bacteriological tests
development: should include inoculation of the material
into the following nutrient medium:
A. Iron-deficiency
B. B12 -folate-deficiency A. Kitt-Tarozzi medium
C. Haemolytic B. Endo agar
D. Minkowsky-Shauffard disease C. Levine agar
E. Protein-deficiency D. Meat-peptone agar
E. Milk-salt agar
52. For production of phenol ether it is
necessary to cause reaction of sodium 56. A female patient bitten by a stray dog
phenoxide with: came to a surgery. Wide lacerated wounds
were localized on the patient’s face. What
treatment-and prevention aid should be
rendered in order to prevent rabies?
A. Immunization with the antirabic vacci-
ne
B. Combined antibiotic therapy
C. Hospitalization, injection of diphtheria-
pertussis-tetanus vaccine
A. CH3 Cl D. Hospitalization, medical surveillance
B. CH3 OH E. Urgent injection of normal gamma-
C. CH4 globulin
D. CH3 NH2
E. CH3 C ≡ N 57. With an attachment of −CH2 group to
a hydrocarbon radical the surface activity
53. Polysaccharide cellulose consists of the of surfactants increases (maximally) by:
remains of the following monosaccharide:
A. 3,5 times
B. 2,5 times
C. 1,5 times
D. 4,5 times
E. 5,5 times
58. Specify two compounds that can be
present in a solution at the same time:
A. Al(NO3 )3 and HCl
B. Ba(OH)2 and CO2
C. NaOH and P2 O5
A. β-D-glucopyranose D. CuSO4 and BaCl2
B. α-D-glucopyranose E. AgNO3 and HCl
C. β-D-fructopyranose
D. α-D-fructofuranose 59. Specify the standardized solutions
E. β-D-glucofuranose used for direct and back titration of
reducing agents in the iodometric method:
54. A higher avascular plant shows clear
alternation of generations with the domi-
nant sexual (gametophyte) and reduced
88
krok123.in.ua
Krok 1 Pharmacy 2011 7

A. I2 , Na2 S2 O3 65. In the pharmaceutical industry, the


B. K2 Cr2 O7 , Na2 S2 O3 micelle-forming solutions of surface-
C. I2 , KI active substances are used for producti-
D. KMnO4 , KI on of water-soluble preparations out of
E. K2 Cr2 O7 , I2 water-insoluble substances, for example
vitamins A and E. The critical concentrati-
60. Specify the standard solution (titrant) on of micelle formation has the lowest
for the iodometric determination of oxi- value in the solutions of the following
dants: substances:
A. Na2 S2 O3 A. C17 H35 COONa
B. KMnO4 B. C12 H25 COONa
C. I2 C. C13 H27 COONa
D. K2 Cr2 O7 D. C15 H31 COONa
E. KBrO3 E. C11 H23 COONa
61. In a surgical unit an outbreak 66. Pharmaceutic preparation collargol is
of purulent infections has been regi- a colloid silver solution containing a high-
stered. The infections are caused by molecular compound. What is the functi-
Staphylococcus aureus with multiple resi- on of this compound?
stance to antibiotics. What plasmid has
provided this property? A. It enhances aggregative stability
B. It induces coagulation
A. R C. It facilitates sedimentation
B. F D. It reduces aggregative stability
C. Col E. It increases dispersion degree
D. T ox
E. Hly 67. What kind of standard solution (ti-
trant) is used according to Folgard’s direct
62. Iron (II) sulfate is a part of drugs used titration method?
in treatment of iron deficiency anemia.
F eSO4 enters into reaction with one of A. Ammonium thiocyanate
the following compounds: B. Sodium chloride
C. Silver nitrate
A. KMnO4 D. Potassium chromate
B. HCl E. Potassium dichromate
C. CO2
D. F eCl2 68. Presence of which ion of d-elements in
E. NaCl the solutions can be detected by means of
K4 [F e(CN)6 ]?
63. Treatment of a wound with hydrogen
peroxide is accompanied by the release of A. F e3+
some gas bubbles from the solution. What B. F e2+
kind of gas is it? C. Zn2+
D. Cr 3+
A. Oxygen E. Cu2+
B. Hydrogen
C. Ozone 69. Temporary hardness of water is caused
D. Carbon dioxide by presence of the following calcium and
E. Nitrogen magnesium salts in the natural water:
64. Micelle solutions of surfactants are A. Hydrogen carbonate
applied in pharmaceutical production as B. Sulfates
stabilizers and solubilizers. What soluti- C. Chlorides
on of colloidal surfactants will have the D. Nitrates
greatest value of critical concentration of E. Phosphates
micelle formation?
70. Which of these chemical substances
A. C9 H19 SO3 Na CANNOT act as an excitatory
B. C14 H29 SO3Na neurotransmitter in the central nervous
C. C16 H33 SO3 Na system?
D. C12 H25 SO3Na
E. C10 H21 SO3 Na
89
krok123.in.ua
Krok 1 Pharmacy 2011 8

A. Glycine forms of monosaccharides with alcohols


B. Serotonin in presence of an acid catalyst the followi-
C. Noradrenaline ng compounds are formed:
D. Substance P
E. Dopamine
71. Which of the following presentations
can be explained by the functional effects
of adrenaline?
A. Relaxation of bronchial muscles
B. The decrease in heart rate
C. Dilatation of the skin vessels
D. Intensification of stomach and intesti-
nes contraction
E. High uropoiesis
72. To quantify the adsorption at the solid-
gas interface, the following equation can
be used:
A. Freundlich A. Glycosides
B. Gibbs B. Esters
C. Shishkovski C. Osazones
D. Helmholtz-Smoluchowski D. Acids
E. Rayleigh E. Oxoacids
73. Iron (III) hydroxide is produced as a 76. During examination of a plant cell
result of interaction of: under the electron microscope some
structures in form of a stack of flattened
A. F eCI3 with NaOH membrane cisterns and vesicles were
B. F e2 O3 with NaOH found. What organelles are these?
C. F eCI3 with H2 O
D. F e2 O3 with H2 O A. Golgi apparatus
E. F e with NaOH B. Endoplasmic reticulum
C. Plastids
74. What compound is formed as a result D. Mitochondrions
of the intermolecular dehydration of E. Microbodies
ethanol?
77. Analysis of the cerebrospinal fluid of
a child with signs of purulent lesion of
brain tunics revealed gram-negative bean-
shaped diplococci. What presumptive di-
agnosis can be made on the basis of the
analysis results?
A. C2 H5 − O − C2 H5 A. Meningitis
B. C2 H5 − O − SO3 H B. Gonorrhea
C. CH2 = CH2 C. Cholera
D. Plague
E. Anthrax
78. As a result of the interaction of excess
D. alkali with amphoteric metals the followi-
ng compound is formed:
A. Hydroxo complexes
B. Oxides
C. Hydroxides
E. D. Neutral salts
E. Basic salts

75. As a result of the reaction of cyclic 79. Chlorophyll, the green pigment of
plants, is a chelate compound. Specify the
90
krok123.in.ua
Krok 1 Pharmacy 2011 9

chelating ion in chlorophyll:


A. Mg 2+ A. secondary-butyl alcohol
B. F e3+ B. Butanol-1
C. Mn2+ C. Isobutyl alcohol
D. F e2+ D. tertiary-butyl alcohol
E. Ni2+ E. Propanol-2
80. In order to enhance the solubility of 85. What medication is formed as a result
the individual components of a number of interaction of salicylic acid with acetic
of liquid drug formulations, the colloidal anhydride?
surface-active substances are added. What
physico-chemical phenomenon underlies
this process?
A. Solubilization
B. Coagulation
C. Extraction
D. Diffusion
E. Sedimentation
81. A flower has the androecium consisti-
ng of two long and two short stamens.
Therefore the flower’s androecium is:
A. Didynamous A. Aspirin
B. Tetradynamous B. Salicyl amide
C. Diadelphous C. Phenyl salicylate
D. Tetradelphous D. Benzyl salicylate
E. Polyadelphous E. Sodium salicylate
82. Potassium dichromate K2 Cr2 O7 is 86. Digestion of proteins in the digesti-
applied as an oxidant in acidic medi- ve tract is a complex process of their
um. What is the product of reduction hydrolysis till peptides and free amino aci-
of dichromate-ion Cr2 O72− under these ds. What enzymes decompose proteins in
conditions? the duodenum?
A. Cr 3+ A. Trypsin, chemotrypsin
B. Cr(OH)3 B. Enterokinase, lipase
C. Cr(OH)2 C. Amylase, maltase
D. [Cr(OH)6]3− D. Pepsin, gastricsin
E. Cr2 O3 E. Lipase, phospholipase
83. Mass serological diagnosis of HIV 87. After a 5-year-old child has been
infection is made by means of enzyme- brought home from the kindergarten he
linked immunosorbent assay techniques. presented with weakness, headache, body
What standard component of the reaction temperature rise up to 37, 5o C. What peri-
must be adsorbed on the solid phase of od of disease develompent is the case?
the test system?
A. Prodromal
A. HIV antigens B. Latent
B. Monoclonal HIV antibodies C. Incubative
C. Enzyme-marked HIV antibodies D. Recovery
D. Specific immunoglobulins E. Fastigium
E. Substrates to determine enzyme activity
88. Iodimetry involves use of standard
84. What compound will be produced solutions of iodine and Na2 S2 O3 . What
during reduction of methyl ethyl ketone? substance is used to standardize the sodi-
um thiosulfate solution?

91
krok123.in.ua
Krok 1 Pharmacy 2011 10

A. K2 Cr2 O7
B. NaCl
C. N2 B4 O7
D. K2 CO3
E. As2 O3
89. It is required to determine the amount
of sodium salicylate in a solution. What ti- A.
trimetric method can be applied for the
quantitative determination of aromatic
compounds?
A. Bromometry
B. Mercurimetry
C. Cerimetry
D. Argentometry B.
E. Chelatometry
90. A 56 year-old patient complains about
limitation of movements and pain in
hand joints, mainly at night. Objectively:
there is a disfiguring painful swelling of
affected joints. Blood and urine have high C.
concentration of uric acid. What disease
has developed?
A. Gout
B. Pellagra
C. Phenylketonuria
D. Alkaptonuria D.
E. Tyrosinosis
91. What elements of IIB group exhibit
amphoteric properties?
A. Zinc only
B. Zinc and cadmium
C. Cadmium and mercury
D. All elements E.
E. Mercury only
92. Interaction of aniline with of bromine 93. Analytical indication of effect of
water resulted of white precipitate. What potassium iodide solution upon unstai-
substance was produced? ned oxidizing anions in presence of
chloroform is:
A. Brown stain of free iodine
B. Settling down of white deposition
C. Change of aggregate state
D. Emission of gas bubbles
E. Origination of deposition and its soluti-
on in reagent excess
94. Every year during the plant blossomi-
ng a female patient develops acute
catarrhal inflammation of conjuncti-
va and nasal mucosa that is the cli-
nical presentation of an allergy. These
symptoms relate to the following type of
allergic reactions:

92
krok123.in.ua
Krok 1 Pharmacy 2011 11

A. Anaphylactic
B. Cytotoxic
C. Immune complex
D. Cell-mediated
E. Cellular dysfunction
95. A female patient consulted a doctor
about leg pain that arises usually toward
the evening; feet and shins edemata.
Objectively: leg skin is cyanotic, cold to A. Oxidation with potassium
the touch. What type of peripheral ci- permanganate
rculation disorder does the patient present B. Heating with sulphuric acid
with? C. Hydrogen peroxide action at a room
temperature
A. Venous hyperaemia D. Sodium hydroxide action at a room
B. Arterial hyperaemia temperature
C. Ischaemia E. Boiling in the open air
D. Stasis
E. Thrombosis 100. Which of the following compounds
forms a propionic aldehyde as a result of
96. Presence of the pathogenic mi- alkaline hydrolysis (H2 O, OH − )?
croorganisms in the air can be prognosti-
cated according to the content of sanitary-
indicative bacteria. Which bacteria indi-
cate immediate epidemiologic danger?
A. Haemolytic streptococci A.
B. Sarcinae
C. Mold fungi
D. Yeast fungi
E. Micrococci B.
97. A patient with low immunity, frequent
colds is recommended to take ascoruti-
ne as a more effective drug than ascorbic
acid. What constituent substance of this C.
preparation intensifies action of vitamin
C?
A. Vitamin P
B. Vitamin A
C. Glucose D.
D. Lactose
E. Vitamin D
98. Determination of ∆Boiling poi-
nt of water-alcohol mixtures is the
pharmacopoeial method of quantitative E.
determination of alcohol. Which method
enables to determine ∆Boiling point?
101. The 0,1 M solution of whi-
A. Ebullioscopy ch substance has the smallest ion
B. Cryoscopy concentration?
C. Enteroscopy
D. Osmometry A. CH3 COOH
E. Conductometry B. HCl
C. CaCl2
99. Toluol is converted to the benzoic acid D. H2 SO4
under the following conditions: E. NaNO3
102. Inflammatory processes in the gall
bladder exert negative influence on the
colloidal properties of bile. This may lead
to gallstone formation. One of the causes
93
krok123.in.ua
Krok 1 Pharmacy 2011 12

of their formation is the crystallization of


the following substance: A. Phellogen
B. Cambium
A. Cholesterol C. Procambium
B. Albumine D. Protoderm
C. Haemoglobin E. Pericycle
D. Urates
E. Oxalates 109. Chooes the reagent that can be used
for acylation of amines:
103. Which of the given bases is a weak
electrolyte?
A. Mg(OH)2
B. Ca(OH)2
C. Ba(OH)2
D. NaOH
E. KOH
A. (CH3 CO)2O
104. Bacteria may contain not only B. CH3 CHO
chromosomal but also nonchromosomal C. C2 H5 Cl
hereditary elements called plasmids. D. HNO2
Presence of plasmid genes can show itself E. CHCl3 + NaOH
by:
110. What type of tautomerism is typical
A. Multiple drug resistance for the given compound?
B. Stain resistance
C. Physical factor resistance
D. Sporogenesis ability
E. Mobility
105. The method of "accelerated drug
ageing"used for determination of drug
shelf life is based upon:
A. Van’t Hoff’s rule
B. Fajans’ rule
C. Planck’s postulate
D. Ostwald law A. Nitro-aci-nitro tautomerism
E. Raoult law B. Carbonyl-enol tautomerism
C. Cyclo-oxo tautomerism
106. Choose the reagents for detection of D. Amine-imine tautomerism
the sulphate ions in a solution containing E. Keto-enol tautomerism
carbonate, sulphate and phosphate ions:
111. Aminotransferases are the enzymes
A. Ba(NO3 )2 , HCl that transfer an amino group from one
B. Ba(NO3 )−2 , NaOH compound to another. What compound is
C. BaCl2 , H2 O the acceptor of amino groups?
D. CaCl2 , NH4 OH
E. AgNO3 , HNO3 A. α-ketoglutaric acid
B. Acetone
107. Transamination is the biochemical C. Lactic acid
process in which amino groups of different D. Succinic acid
amino acids take form of one of the amino E. Butyric acid
acids. What amino acid is it?
112. Alpha-cells of pancreas stimulate
A. Glutamic synthesis of the glucagon hormone that is
B. Glycine involved into the carbohydrate metaboli-
C. Valine sm. It has the following effect on liver
D. Leucine processes:
E. Arginine
108. Microscopic examination of ground
tissue of a small branch revealed cork and
felloderm. These are the derivates of:
94
krok123.in.ua
Krok 1 Pharmacy 2011 13

A. Activates glycogenolysis class of immunoglobulins prevails in this


B. Activates alcoholic fermentation preparation?
C. Inhibits glycogenolysis
D. Inhibits glycolysis A. IgG
E. Activates lypogenesis B. IgA
C. IgM
113. Sol Al(OH)3 was produced as a result D. IgE
of treatment of freshly prepared Al(OH)3 E. IgD
precipitate with a small amount of HCl
solution. What phenomenon underlies the 119. Specify the product that results from
sol production? the interaction of 2 moles of HCl with 1
mole of propyne:
A. Chemical peptization
B. Chemical condensation
C. Washing with a solvent
D. Mechanical dispersion
E. Physical condensation
114. What is oxidation number of the
central atom in the compound H[AuCl4]? A.
A. +3
B. 0
C. +1 B.
D. +2
E. +4
115. A 37-year-old man was admitted
to a hospital with an attack of bronchi- C.
al asthma. What respiration type will be
observed in this patient?
A. Expiratory dyspnea D.
B. Inspiratory dyspnea
C. Apnoea
D. Gasping respiration
E. Hyperpnoea
E.
116. It is required to measure the nitrogen
metabolism in a person under observati-
on who is recovering from continuous 120. Specify the name of the compound
starvation. What result is most likely to according to the IUPAC nomenclature:
be expected?
A. Decrease in nitrogen secretion
B. Nitrogen equilibrium
C. Negative nitrogen balance
D. Acetonemia
E. -
117. It is required to increase the secreti-
on of gastric juice in an experimental
dog with stomach fistula. What should be A. 2,3,5-Trimethyl heptadiene-3,4
introduced into the stomach? B. 3,5,6,6-Tetramethyl heptadiene-3,4
C. 3,5,6,6-Trimethyl hepten-3
A. Meat broth D. 2-Ethyl-4,5-dimethyl hexadiene-2,3
B. White bread E. 2-Ethyl-4,5,5-trimethyl pentadiene-2,3
C. Milk
D. Dried bread 121. Which salt should be dissolved in
E. Sour cream water in order to increase the concentrati-
on of hydrogen ions?
118. Anti-tetanus gamma globulin is
produced by hyperimmunization of
donors with tetanus anatoxin. What
95
krok123.in.ua
Krok 1 Pharmacy 2011 14

A. ZnCl2 a solution was determined by iodometric


B. NaNO3 method. Name the titrant of iodometric
C. KCl method for oxidant determination:
D. Na3 CO2
E. Na2 S A. Sodium thiosulfate
B. Sodium hydroxide
122. Drugs in form of colloidal-and- C. Potassium iodide
disperse systems are widely spread in the D. Potassium permanganate
pharmaceutical practice. What method E. Potassium bromate
of sol production is based upon the
phenomenon of physical condensation? 128. Under anaerobic conditions duri-
ng glycolysis ATP is synthesized by
A. Solvent substitution the way of substrate phosphorylation.
B. Reduction This process uses energy of other high-
C. Oxidation energy compounds. Specify one of such
D. Hydrolysis compounds:
E. Double exchange
A. Phosphoenol pyruvate
123. Under what conditions the limited B. Glucose 6-phosphate
swelling of gelatine turns into the unlimi- C. Lactate
ted one? D. Pyruvate
E. Glucose
A. Heating
B. Cooling 129. Fruits of the Apiaceae family can
C. In presence of P O43− ions be identified on the basis of a set of
D. In presence of Cl− ions morphological features and presence of
E. In presence of H + ions whose the following formation in the pericarp:
concentration is equal to their concentrati-
on in the isoelectric point A. Essential oil tubules
B. Resin ducts
124. As a result of staining of a plant mi- C. Articulated lacticifers
croslide with Sudan III solution the cell D. Non-articulated lacticifers
membranes turned pink. This indicates E. Wax strips with stomata
the presence of:
130. A patient was diagnosed with ri-
A. Suberin ght lung cancer and doctors administered
B. Cellulose him surgical treatment. After right-sided
C. Lignin pulmonectomy the patient began to suffer
D. Pectin from evident dyspnea. What form of respi-
E. Hemicellulose ratory failure is it?

125. Choose a pair of titrants for the quali- A. Pulmonary restrictive


tative determination of ammonia in a B. Central
solution by the method of back titration: C. Peripheral
D. Pulmonary obstructive
A. HCl, NaOH E. Thoracodiaphragmal
B. HCl, H2 SO4
C. KOH, NaOH 131. During the practical training the
D. NaOH, KCl students placed the isolated frog’s heart
E. H2 SO4 , K2 SO4 into a solution. This caused the cardiac
arrest in diastole. What solution was the
126. The analytical effect of reaction of heart placed into?
potassium hexacyanoferrate (II) solution
with iron (III) ions is: A. 3% solution of KCl
B. 1% solution of NaCl
A. Formation of blue precipitate C. 3% solution of NaCl
B. Formation of white precipitate D. 1% solution of CaCl2
C. Formation of blue precipitate and its E. 0,1% solution of MgCl2
dissolution in the excess of the reagent
D. Effervescence 132. A patient takes blocker of muscari-
E. Characteristic smell nic cholinoreceptors of parasympathetic
nerve organ synapses. What changes of
127. Content of potassium dichromate in heart activity will be observed?
96
krok123.in.ua
Krok 1 Pharmacy 2011 15

138. Perchromic acid formed as a result


A. Heart rate rise of chromium oxidation is unstable and di-
B. Heart rate and heart force fall ssolves in aqueous solutions. What solvent
C. Heart rate fall is used for its extraction?
D. Heart force fall
E. Prolongation of atrioventricular delay A. Isoamyl alcohol and ether
B. Chloroform
133. A patient complains about an C. Benzene
increase in heart rate, hyperperspirati- D. Nitrobenzene
on, irritability, sleeplessness. He has been E. Ethanol
presenting with these symptoms for the
latest six months. They indicate the 139. When a smear is stained by Burry-
hyperfunction of the following endocri- Gins method a mucous structure that
ne gland: is tightly bound with the cellular wall
of bacteria and has well-defined outer
A. Thyroid gland boundaries can be detected. This element
B. Pancreas of a bacteria cell is called:
C. Adrenal glands
D. Sexual glands A. Capsule
E. Thymus B. Spore
C. Filaments
134. Examination of five herbarium speci- D. Ribosomes
mens of medicinal plants showed that one E. Episomes
of them belonged to the legume family,
namely: 140. Sanitary-biologic examination of
air in a drugstore revealed a sanitary-
A. Glycyrhiza glabra indicative microorganism. Name it:
B. Atropa belladonna
C. Hyoscyamus niger A. Staphylococcus aureus
D. Datura stramonium B. Colon bacillus
E. Solanum dulcamara C. Fecal enterococcus
D. Alpha-haemolytic streptococcus
135. Iron in the +6 oxidation state acts E. Citrobacter
only as an oxidant because:
141. The product of full acetylation of
A. It is in the maximum oxidation state glycerol relates to the following class of
B. It has 5 outer shell electrons organic compounds:
C. It falls into VIIIB group
D. It is a d-element
E. It is found in the fourth period
136. Before diving experienced divers first
take several deep breaths. They do it in
order to:
A. Remove as much as possible CO2
B. Reduce functional residual capacity of
lungs
C. Increase lung vital capacity (LVC)
D. Increase total lung capacity (TLC)
E. Increase respiratory volume (RV) A. Ester
B. Ether
137. A woman in labour has been gi- C. Ketone
ven a drug that activates contractions D. Acetal
of the smooth muscles of uterus. Which E. Phenol
hormone is a part of this drug?
142. Specify the number of degrees of
A. Oxytocin freedom of the intersection of liquidus li-
B. Gastrin ne with the ordinate axis of diagram of a
C. Secretin two-component system state:
D. Angiotensin
E. Bradykinin

97
krok123.in.ua
Krok 1 Pharmacy 2011 16

A. C = 0 147. Specify the reagents enabling to prove


B. C = 2 the presence of a primary amino group
C. C = 1 in a molecule of n-aminobenzoic acid by
D. C = -1 means of isonitrile test:
E. C = 3
A. CHCl3 , NaOH
143. What thermodynamic potential is B. Br2 , H2 O
the criterion for the direction of a C. NaHCO3
spontaneous process at constant volume D. KMnO4
and temperature? E. I2 , NaOH
A. Helmholtz energy 148. Specify the reaction which proves the
B. Entropy acidic properties of phenol:
C. Gibbs energy
D. Chemical potential
E. Enthalpy
144. A citrus fruit is characterized by the
glandular exocarp, spongiose mesocarp
and overgrown endocarp consisting of jui-
ce sacs. Such fruit is called: A. Phenol + sodium hydroxide
A. Hesperidium
B. Legume
C. Pod
D. Drupe
E. Bacca
B. Phenol + bromine
145. What is the mechanism of brominati-
on of toluene aromatic ring?

C. Phenol + nitric acid

D. Phenol + hydrogen

A. SE
B. AE
C. SR
D. SN
E. AN E. Phenol + chloromethane
146. A patient fell ill the day before, the
disease is acute with a predominance of
general toxic symptoms. With an account
for the epidemic situation in the city,
the doctor diagnosed the patient with
influenza A. What emergency etiotropic 149. Specify the electronic effects of the
treatment must be administered to this carboxyl group (−COOH) in a molecule
patient? of benzoic acid:
A. Rimantadine A. −I, −M
B. Oxolinic ointment B. −I
C. Gentamicin C. +I, −M
D. Inactivated influenza vaccine D. +I
E. Human gamma globulin E. −I, +M
98
krok123.in.ua
Krok 1 Pharmacy 2011 17

150. Butanol-1 (n-butyl alcohol) and 2-


methyl-1-propanol (isobutyl alcohol) are A. Mycoplasma
the isomers: B. Viruses
C. Bacteria
A. Of carbon chain D. Fungi
B. Position isomers E. Protozoa
C. Optical
D. Geometric 156. Molar mass equivalent to barium
E. Of functional group hydroxide (M(Ba(OH)2) = 171 g/mol) is:
151. A leaf of a plant under examinati- A. 85,5 g/mol
on has a membranous ochrea wrapped B. 34 g/mol
around the internode base. Presence of C. 42,8 g/mol
such modified stipules is the diagnostic D. 57 g/mol
feature of the following family: E. 232 g/mol
A. Polygonaceae 157. Sanitary microbiological analysis
B. Gramineae of the indoor air of a pharmacy carri-
C. Rosaceae ed out in summer revealed presence
D. Legumes of Streptococcus haemolyticus and
E. Solanaceae Streptococcus viridians at the rate of 40
microorganisms per 1 m3 . Specify the mi-
152. A solution containing the cations of crobiological characteristic of the air:
the V analytic group (acid-base classifi-
cation) has been taken for the analysis. A. Contaminated
The solution of sodium hydroxostannite B. Within the permissible limits
has been added to the composition which C. Almost pure
resulted in formation of black depositi- D. Pure
on. This is the evidence of presence of the E. These microorganisms are not the
following cation: determinants of the air quality

A. Bi3+ 158. Each stem node of white deadnettle


B. F e2+ (Lamium album) has two leaves which
C. Sb3+ grow perpendicularly to the leaves of the
D. F e3+ previous node. Such leaf arrangement is
E. Mg 2+ called:

153. Which of the following adsorbents A. Cross-opposite


is the most effective for adsorption of a B. Spiral
substance from the aqueous solution? C. Verticillate
D. Rosette
A. Activated carbon E. Leaf mosaic
B. Silica gel
C. Quartz 159. A 3,5-year-old child has been di-
D. Bolus alba agnosed with dysbacteriosis in form
E. Gypsum of critical reduction of gram-positive
anaerobic bacteria and increased number
154. Choose a plant whose apical sprouts of staphylococci and yeast fungi. Whi-
are used in medical practice for sedative ch preparation should be used for the
drug production: correction of dysbacteriosis?
A. Leonurus cardiaca A. Bifidumbacterin
B. Glycyrrhiza glabra B. Colibacterin
C. Digitalis purpurea C. Coli-Proteus bacteriophage
D. Ledum palustre D. Furazolidone
E. Fagopyrum sagittatum E. Lactoglobulin
155. Examination of procured medicinal 160. One of saliva functions is the bacteri-
herbs grown in a warm climate revealed cidal one. It can be fulfilled due to the
their affection in form of yellowing, following substance:
overgrowth of lateral shoots, dwarfism,
delayed fruiting. Which organisms can
cause such changes?
99
krok123.in.ua
Krok 1 Pharmacy 2011 18

A. Lysozyme A. Hot water


B. Amylase B. 0,2 M solution of sodium carbonate
C. Bradykinin C. Saturated solution of sodium carbonate
D. Maltase D. 2M sulfuric acid solution
E. Mucin E. Ethanol
161. Morphological analysis of leaves 166. A student had analyzed an
revealed that each vein runs along the inflorescence and found out that the
lamina separately and the veins join flowers on the developed main axis were
together only at the top of the lamina. set one by one, and due to the vari-
This kind of venation is called: ous length of pedicels they were located
almost in the same plane, so they formed:
A. Arcuate
B. Pinnate A. Corymb
C. Dichotomous B. Glomus
D. Palmate C. Anthodium
E. - D. Volute
E. Umbel
162. During identification of an unknown
salt the colorless part of the burner flame 167. A 50-year-old patient complains of
turned yellow and green. What cation was having dyspnea under a considerable
the salt formed by? physical stress, leg edemata. Examinati-
on reveals chronic myocarditis and ci-
A. Ba2+ rculatory failure. What is the evidence of
B. Ca2+ cardiac decompensation in the patient?
C. Sr 2+
D. Na+ A. Decreased cardiac output
E. K + B. Increased blood flow velocity
C. Increased vascular resistance
163. A pharmaceutical analyst has to D. Decreased venous pressure
identify potassium acetate. He can prove E. Increased hydrostatic pressure in the
the presence of potassium cation in the lumen of blood vessels
analyzed substance by means of the
following solution: 168. Study of secretory function
of stomach revealed a decrease in
A. Tartrate acid hydrochloric acid concentration in gastric
B. Sodium hydroxide juice. This must cause hypoactivity of the
C. Potassium permanganate following enzyme:
D. Iron (III) chloride
E. Formate acid A. Pepsin
B. Hexokinase
164. A pharmacist studies the coagulati- C. Amylase
on process. He adds the minimum D. Lipase
concentration of the electrolyte to a E. Dipeptidase
sol. Coagulation takes place when this
concenration is exceeded. The minimum 169. In order to reduce heart rate during
concentration of the electrolyte is called: tachycardia alternative medicine doctors
recommend to induce artificial vomiting.
A. Coagulation threshold Specify the efferent nerve of this reflex:
B. Sedimentation threshold
C. Sensitivity threshold A. Vagus
D. The threshold of the adsorption- B. Glossopharyngeal nerve
solvation sensitivity C. Sympathetic (abdominal) nerve
E. Coagulating power D. Hypoglossus
E. Phrenic nerve
165. Reaction of a group reagent with the
cations of the 2nd analytical group results 170. Saffron propagates vegetatively - via
in precipitation of P bCl2 which can be di- corms which are a modification of . . .
ssolved in:

100
krok123.in.ua
Krok 1 Pharmacy 2011 19

A. Underground shoot A. Triglyceride lipase


B. Main root B. Phospholipase C
C. Above-ground shoot C. Phospholipase A2
D. Lateral roots D. Phospholipase A1
E. Additional roots E. Glycogen phosphorylase
171. Mass fraction of F e2+ ions in Mohr’s 176. After a 40-year-old patient had
salt can be determined by gravimetric changed his body position from vertical
sedimentation method using: to horizontal one his heart rate dropped
from 70 to 65 bpm. This reaction was
A. NH4 OH caused by the following reflex:
B. Na2 S
C. K3 P O4 A. Depressor reflex
D. BaCl2 B. Pressor reflex
E. ZnCl2 C. Goltz reflex
D. Aschner reflex
172. In order to generate hydrogen by E. Bainbridge reflex
means of Kipp’s apparatus the following
reagents should be used: 177. The microflora of the colon plays an
important part in the process of digestion.
A. Zinc and diluted sulfuric acid What vitamins does it synthesize?
B. Aluminum and potassium chloride
solution A. Vitamins K and B group
C. Magnesium and concentrated sulfuric B. Vitamin C
acid C. Vitamin P P
D. Hydrogen peroxide D. Vitamin E
E. The compound of hydrogen and carbon E. Vitamin A
173. Specify a halogen whose reaction wi- 178. Styloids are big single elongate-
th hydrogen causes an explosion: prismatic needle-like crystals. They are
mostly typical for the following plants:
A. Fluorine
B. Bromine A. Monocotyledonous
C. Chlorine B. Dicotyledonous
D. Astatine C. Gymnospermous
E. Iodine D. Lycopodiophyta
E. Equisetophyta
174. Inhibition of the synthesis of bile aci-
ds from cholesterol in liver of an experi- 179. The listed below drugs can be used to
mental animals has caused maldigestion correct acid-base and ionic balance. Provi-
of lipids. What is the role of these acids in ded that their molar concentration is the
the enteral lipidic metabolism? same, the following solution will have the
maximum value of ionic strength:
A. They emulsify dietary lipids
B. They keep balance of alkaline envi- A. Calcium chloride
ronment in the gut B. Potassium chloride
C. They participate in the synthesis of C. Potassium iodide
lipids D. Sodium chloride
D. They are part of LDL E. Sodium fluoride
E. They activate the formation of chylomi-
crons 180. Examination of a patient revealed
an increase in ammonia and citrulline
175. Pheochromocytoma provokes concentration in blood, a decrease in urea
hypersecretion of adrenaline and concentration in urine as well as citrulli-
noradrenaline. The concentration of free nuria. This condition is caused by the defi-
fatty acids is higher than normal. In this ciency of the following enzyme:
case hyperlipidemia is caused by activati-
on of the following enzyme: A. Arginine-succinate synthetase
B. Glutamine synthetase
C. Ornithine carbamoyl transferase
D. Glutaminase
E. Arginine-succinate lyase
181. A patient with atherosclerosis has
101
krok123.in.ua
Krok 1 Pharmacy 2011 20

been prescribed Linaetholum containi- with skin burns, impaired vision. He has
ng essential fatty acids. Which of the been diagnosed with albinism. It is caused
following acids is an obligatory part of by the deficiency of the following enzyme:
the preparation?
A. Tyrosinase
A. Linolenic B. DOPA-oxidase
B. Palmitic C. Phenylalanine hydroxylase
C. Crotonic D. Ornithine carbamoyl transferase
D. Stearic E. Arginase
E. Oleic
187. The properties in a series of oxides:
182. What cation of the 4th analytical Al2 O3 - SiO2 - P2 O5 - SO3 - Cl2 O7 change
group is present in a solution, if it is known in the following way:
that the reaction with a group reagent
causes formation of yellow precipitate? A. The properties increase from left to
right
A. Cr 3+ B. The properties decrease from left to
B. Zn2+ right
C. Sn2+ C. The properties first increase and then
D. Al3+ decrease
E. Sn(IV ) D. The properties do not change
E. The properties first decrease, then
183. A patient complains of general increase
weakness, dyspnea, palpitation. Exami-
nation revealed inflammation of the 188. Select the fruit that meets the descri-
mucous membrane of tongue, lips, especi- ption: monocarpic, dry, polyspermous, can
ally in the corners of mouth; inflammati- be split apart only in the ventral suture.
on and increased vascularization in the The seeds are located along the ventral
external membrane of eye. What is the li- suture:
kely cause of this pathological condition:
A. Follicle
A. Hypovitaminosis B2 B. Coccus
B. Hypovitaminosis A C. Fleshy stone fruit
C. Hypovitaminosis C D. Dry stone fruit
D. Hypervitaminosis A E. Follicetum
E. Hypervitaminosis B1
189. A patient with current coronary heart
184. Examination of a patient revealed an disease who had had two myocardial
increase in 17-ketosteroid concentration in infarctions of left ventricular wall presents
urine. Hydroxylation of 17-ketosteroids is with bubbling breathing and dyspnea.
possible with the enzymes of the following Pulmonary auscultation reveals moist
system: rales. What kind of heart failure is it?
A. Microsomal oxidation A. Left ventricular
B. Krebs cycle B. Right ventricular
C. Protein synthesis system C. Compensated
D. Pentose phosphate cycle D. Subcompensated
E. Ornithine cycle E. Combined
185. It is necessary to carry out preventive 190. A healthy 45-year-old man is sitti-
vaccination of a student group because ng in a chair reading a newspaper. What
of an occurrence of diphtheria. Which muscles ensure breathing in a sitting posi-
preparation should be used for the creati- tion?
on of the artificial active immunity?
A. Diaphragm and external intercostal
A. Diphtheria anatoxin muscles
B. Specific immunoglobulin B. Scalenes
C. DTP vaccine C. Diaphragm and internal intercostal
D. Inactivated bacteria vaccine muscles
E. Anti-diphtheria serum D. Internal intercostal muscles and straight
muscles of abdominal wall
186. A patient consulted a doctor about E. Sternocleidomastoid muscles
intolerance to the sun rays. He presents
102
krok123.in.ua
Krok 1 Pharmacy 2011 21

191. Parents of a 11-year-old boy noti- 196. Humoral immune response to an


ced that he is far behind his peers in antigen results in generation of antibodi-
the physical development. After the X- es produced by plasmacytes. Plasmacytes
ray an endocrinologist revealed that the arise as a result of immunostimulated di-
growth zones of tubular bones had already vision from the following cells of immune
closed. Under these conditions, the intake system:
of growth hormone can result in the
development of: A. B-lymphocytes
B. Monocytes
A. Acromegaly C. Granulocytes
B. Gigantism D. T-helpers
C. Dwarfism E. T-killers
D. Cretinism
E. Myxedema 197. It is known that the leaves of
Eucalyptus globulus have cavities with
192. A 46-year-old patient was found to well-defined internal boundaries and fi-
have hyperactivity of creatine kinase in lled with essential oils. They are called:
blood serum. What kind of pathology
should be suspected? A. Schizogenous cavities
B. Non-articulated lacticifers
A. Myocardial infarction C. Schizolysigenous cavities
B. Acute pancreatitis D. Articulated lacticifers
C. Chronic hepatitis E. Lysigenous cavities
D. Haemolytic anemia
E. Renal failure 198. Examination of a patient revealed
toxic hepatitis developed on the
193. Colloidal dispersion systems include background of the use of medicines. This
systems whose particle size is within the diagnosis can be confirmed by the activity
range: of the following enzyme of blood serum:
A. 10−9 - 10−7 m A. Alanine amino transferase
B. 10−7 - 10−4 m B. Creatine phosphokinase
C. > 10−4 m C. Pyruvate dehydrogenase
D. < 10−9 m D. Maltase
E. 10−9 - 10−4 m E. Malate dehydrogenase
194. Drugs that block certain channels 199. A patient has a mental disorder due
can prevent the transmission of exci- to the insufficient synthesis of gamma-
tation from presynaptic membrane to aminobutyric acid in the brain. Such
the postsynaptic memebrane of synapse. pathological changes might be caused by
What channels are blocked? the deficiency of the following vitamin:
A. Calcium A. Pyridoxine
B. Sodium B. Tocopherol
C. Potassium potential-dependent C. Cyanocobalamin
D. Potassium ATP-dependent D. Folic acid
E. Chlorine E. Riboflavin
195. A patient with obstructive jaundi- 200. How many stages has a system consi-
ce presents with bradycardia, low arteri- sting of molten salts NaCl and CaCl2 ,
al pressure, itching, irritability, asthenia. and being in balance with crystals of the
What is the cause of these presentations? respective salts?
A. Cholemia A. 3
B. Anacholia B. 4
C. Hypercholesterolemia C. 1
D. Hypocholesterolemia D. 2
E. Hyperbilirubinemia E. The number of phases changes with time

103
krok123.in.ua
Krok 1 Pharmacy 2012 1

1. Microscopic examination of
absorption zone of primary root cortex
revealed that it consisted mainly of
loose multilayer living parenchyma with
amyloid granules. It is called: A.
A. Mesoderm
B. Endoderm
C. Exoderm
D. Collenchyme B.
E. Phellogene
2. An excess of concentrated ammoni-
um hydroxide is a group reagent for the
cations of the VI analytical group (acid- C.
base classification) Co2+ , Ni2+ , Cd2+ ,
Cu2+ , Hg 2+ . In this case the following
substances are formed:
A. Water-soluble ammonia complexes D.
B. Hydroxides of acid-soluble cations
C. Stained, water-insoluble compounds
D. Hydroxides of alkali-soluble cations
E. Hydroxides of the cations insoluble in
the excess of ammonium hydroxide
E.
3. Both external and internal indicators
are used in the following titrimetric
method of analysis 5. Estimation of temperature of phase
transition at different pressures is of
A. Nitritometry great practical importance for modern
B. Alkalimetry pharmaceutical industry and can be done
C. Chelatometry by applying:
D. Permanganatometry
E. Argentometry A. Clapeyron-Clausius equation
B. Trouton’s rule
4. Which of the listed reactions indicates C. Gibbs’ phase rule
the basic properties of pyridine? D. Mendeleev-Clapeyron equation
E. Konovalov law
6. A patient with ischemic heart disease
has been administered inosine, which is an
intermediate metabolite in the synthesis
of:
A. Purine nucleotides
B. Metalloproteins
C. Lipoproteins
D. Glycoproteins
E. Ketone bodies
7. Examination of the lower limbs of a
40-year-old patient with coronary artery
disease and vascular disease of the
lower limbs (obliterating endarteritis)
revealed skin pallor and dystrophy, local
temperature decrease, sense shock, pain.
The patient is likely to have the following
disorder of the peripheral blood circulati-
on:

104
krok123.in.ua
Krok 1 Pharmacy 2012 2

A. Obstruction ischemia A. Constant of chemical reaction rate


B. Compression ischemia B. Chemical reaction rate
C. Angiospastic ischemia C. Endpoint of a reaction
D. Venous hyperaemia D. Change in the reactants concentration
E. Arterial hyperaemia E. Change in the concentration of the
reaction products
8. A patient has been found to have sugar
in the urine. Blood glucose is normal. 11. Amylolytic enzymes catalyze the
Arterial pressure is normal. What is the hydrolysis of polysaccharides and oli-
mechanism of glycosuria development in gosaccharides. They have an effect upon
this case? the following chemical bond:
A. Disturbance of glucose reabsorption in A. Glycosidic
the nephron tubules B. Hydrogen
B. Insulin deficiency C. Peptide
C. Hyperfunction of adrenal medulla D. Amide
D. Hyperfunction of thyroid gland E. Phosphodiester
E. Hyperfunction of adrenal cortex
12. Which of the following names
9. What compound is formed by heating corresponds with the formula:
α-hydroxypropionic acid?

A. Nicotinic acid amide


B. Isonicotinic acid amide
C. Picolinic acid amide
D. Salicylic acid amide
E. Anthranilic acid amide
A.
13. What indicator is used for the quanti-
tative determination of sodium carbonate
in a preparation by the method of acid-
base titration?
A. Methyl orange
B. Murexide
C. Methylene blue
B. D. Diphenylamine
E. Ferroin
14. Depressive states can be treated by
means of drugs inhibiting the enzyme that
C. inactivates biogenic amines. Specify this
enzyme:
A. MAO (monoamine oxidase)
B. LDH (lactate dehydrogenase)
D.
C. CPK (creatine phosphokinase)
D. AST (aspartate aminotransferase)
E. ALT (alanine aminotransferase)
E. 15. Solutions of some electrolytes are
used as medications. What is the maxi-
mum value of the isotonic coefficient for
10. Rates of chemical reactions of the MgSO4 solution?
same order are compared by:

105
krok123.in.ua
Krok 1 Pharmacy 2012 3

A. 2 A. pH 3,2-3,5
B. 4 B. pH 6,5
C. 3 C. pH 7,0
D. 5 D. pH 9,0
E. 7 E. pH 0,5-1,0
16. What substance blocks the conducti- 21. After taking phenacetin a pati-
on of excitation in the neuromuscular ent developed acute sore throat,
synapses? body temperature rise. Examination
allowed doctors to make a diagnosis
A. Curare of necrotic angina and agranulocytosis.
B. Noradrenaline Agranulocytosis can be characterized by
C. Adrenaline a decrease in the amount of the following
D. Somatostatin WBCs:
E. Aspartate
A. Neutrophils
17. The labels of some medications have B. Eosinophils
an inscription: Shake before use! This C. Basophils
warning is caused by: D. Lymphocytes
E. Monocytes
A. Sedimentation
B. Coagulation 22. A newborn born to an Rh-negative
C. Solubility of disperse systems mother (3rd pregnancy) presents with
D. Insolubility of disperse systems progressing jaundice, symptoms of CNS
E. None of the above excitation, anemia. What type of jaundice
is it?
18. Which of these formulas corresponds
with acetoacetic acid? A. Hemolytic
B. Parenchymatous
C. Obstructive
A. D. Parasitic
E. Toxic
23. A plant under examination has papili-
B. onaceous flower. This plant belongs in the
family:
A. Fabaceae
C. B. Scrofulariaceae
C. Ranunculaceae
D. Lamiaceae
E. Asteraceae
D.
24. Van’t Hoff’s rule is used for determini-
ng the shelf life of drugs. The temperature
coefficient of the rate of most chemical
E. reactions lies within the following range:
A. 2-4
19. Blood serum electrophoresis revealed B. 2-3
interferon. This protein is in the following C. 1-3
fraction: D. 3-4
E. 1-5
A. γ-globulins
B. α1 -globulins 25. In an emergency situation a scuba di-
C. α2 -globulins ver has quickly risen from the depths to
D. β-globulins the surface, which is against the rule. He
E. Albumins is unconscious, presents with respiratory
failure and cardiac activity disorder as a
20. Proteolytic enzymes of gastric juice result of decompression sickness. What
exhibit maximum activity in the medium complication may develop in the scuba di-
with the following pH: ver?

106
krok123.in.ua
Krok 1 Pharmacy 2012 4

A. Gas embolism this purpose:


B. Fat embolism
C. Air embolism A. Entropy
D. Cellular embolism B. Gibbs energy
E. Thromboembolism C. Helmholtz energy
D. Internal energy
26. A patient has been hospitalized for E. Enthalpy
chronic heart failure. Objectively: skin
and mucous membranes are cyanotic, the 31. Histochemical test for fixed oils wi-
patient has tachycardia, tachypnea. What th sudan III results in the following stain
type of hypoxia has developed in the pati- colour:
ent?
A. Pink and orange
A. Circulatory B. Blue and violet
B. Anemic C. Lemon-yellow
C. Hemic D. Raspberry-red
D. Tissue E. Black and purple
E. Hypoxic
32. The strongest acid among the
27. The solid residue obtained after hydrohalic acids is:
evaporation of the sample solution makes
the colorless flame of burner turn yellow, A. Hydroiodic
and when watched through a blue glass, it B. Hydrofluoric
looks purple. What cations are present in C. Hydrochloric
the solid residue? D. Hydrobromic
E. -
A. Na+ , K +
B. a2+ , K + 33. A solution contains cations of zinc and
C. Na+ , Sr 2+ aluminum. Specify the reagent that allows
D. Li+ , Ba2+ to detect cations of zinc in this solution:
E. Na+ , Ca2+
A. Potassium hexacyanoferrate (II) soluti-
28. Amino group of p-aminobenzoic acid on
is involved into reaction with the followi- B. Sodium hydroxide solution
ng reagent: C. Cobalt nitrate Co(NO3 )2
D. Excess of 6M sodium hydroxide in
presence of hydrogen peroxide
E. Sulfuric acid solution
34. In a solution containing cations of
copper (II) and zinc, the copper cations
A. HCl can be identified by means of the excess
B. NH4 OH of the following reagent:
C. NaOH
D. CH3 COONa A. 6M ammonia solution
E. KCN B. 2M sulfuric acid solution
C. 6M potassium hydroxide solution
29. Urine analysis revealed a decrease D. 2M hydrochloric acid solution
in sodium ion concentration. Whi- E. 2M solution of ammonium carbonate
ch hormone provides an enhanced
reabsorption of sodium ions in the 35. Concentration of magnesium sulfate
convoluted nephron tubules? in a drug can be determined by
complexometric titration. Choose an indi-
A. Aldosterone cator to detect the end point of titration:
B. Vasopressin
C. Somatostatin A. Chromogen black
D. Adrenaline B. Phenolphtalein
E. Acetylcholine C. Methyl orange
D. Eosin
30. Thermodynamic calculations allow E. -
us to determine the possibility and di-
rection of spontaneous processes. In an 36. Choose a pair of electrodes for
isolated system the change of the followi- potentiometric pH measurement of a
ng thermodynamic function is used for solution:
107
krok123.in.ua
Krok 1 Pharmacy 2012 5

bacteriostatic agents. The mechanism of


A. Glass and silver chloride antimicrobial action of sulfonamides is
B. Calomel and silver chloride based on their structural similarity to:
C. Quinhydrone and antimonial
D. Mercury sulphate and silver chloride A. Para-aminobenzoic acid
E. Glass and antimonial B. Glutamic acid
C. Folic acid
37. Microscopic examination of a perenni- D. Nucleic acid
al stem revealed integumentary tissue of E. Antibiotics
secondary origin that was formed as a
result of cell division of: 43. Specify the reaction conditions (medi-
um, to ) in the standardization of potassi-
A. Phellogen um permanganate solution by sodium
B. Procambium oxalate solution:
C. Cambium
D. Pericycle A. Acidic, heating
E. Protoderma B. Neutral, heating
C. Alkaline, heating
38. Heart rate of a person at rest is 40/min. D. Acidic, cooling
What structure is the pacemaker of heart E. Neutral, cooling
in this man?
44. Which of the acids with the same
A. Atrioventricular node concentration has the highest ionization
B. Sinoatrial node degree (α)?
C. His’ bundle
D. His’ bundle branches A. HCOOH К=1, 74 · 10−4
E. Purkinje fibers B. CH3 COOH К=1, 74 · 10−5
39. The volume of air exhaled by a C. NH2 CH2 COOH К=1, 70 · 10−10
healthy person during quiet breathing was D. C6 H5 COOH К=6, 3 · 10−5
measured with a spirometer, it was 0,5 li- E. NH2 (CH2 )2 COOH К=2, 6 · 10−11
ter. What is this volume called? 45. Select the correct name for the given
A. Tidal volume compound:
B. Inspiratory reserve volume
C. Expiratory reserve volume
D. Vital capacity of lungs
E. Residual volume
40. A 40-year-old patient has developed
polyuria (10-12 liters per day) and
polydipsia induced by damage to the
hypothalamo-hypophyseal tract. What
hormone deficiency causes such di-
sorders?
A. 1,5-Dinitronaphthalene
A. Vasopressin B. 1,6-Dinitronaphthalene
B. Oxytocin C. 4,8-Dinitronaphthalene
C. Corticotropin D. 2,7-Dinitronaphthalene
D. Somatotropin E. 4,9-Dinitronaphthalene
E. Thyrotropin
46. Acylated amino group acts as a substi-
41. The rate of a chemical reaction does tuent of the following type:
not depend on the concentration of the
reactants. Specify the order of such reacti-
on:
A. Zeroth
B. First
C. Second
D. Third
E. Fraction
42. Sulfonamides are widely used as
108
krok123.in.ua
Krok 1 Pharmacy 2012 6

medium-pH the buffer solutions are used.


A. Type I Specify a composition of substances that
B. Type II DOES NOT HAVE buffer properties:
C. Type I and II at the same time
D. Acetanilide does not take part in the SE A. NaOH + NaCl
reactions B. CH3 COOH + CH3 COONa
E. Impossible to determine C. NH4 Cl + NH3 ·H2 O
D. HCOOH + HCOONa
47. Halide ions in drugs are determined by E. NaH2 P O4 + Na2 HP O4
titration based upon the reaction of:
53. What solution can be determined
A. Precipitation by photocolorimetric method by self-
B. Oxidation-reduction absorbance?
C. Substitution
D. Acid-base A. Potassium chromate
E. Complexing B. Potassium chloride
C. Potassium sulphate
48. In the qualitative analysis which D. Potassium nitrate
involves precipitation of sulphates of the E. Potassium phosphate
third analytical group cations (Ca2+ , Sr 2+ ,
Ba2+ ) the solubility of sulphates can be 54. Uric acid is a derivative of:
reduced by adding:
A. Ethyl alcohol
B. Distilled water
C. Benzene
D. Chloroform
E. Amyl alcohol
49. Lithium carbonate is used in medici- A. Purine
ne for the prevention and treatment of B. Indole
psychoses of different etiology. Li2 CO3 C. Pyrazine
can react with the following compound: D. Pyrazole
E. Pyridine
A. HCl
B. NaCl 55. What class of reactions does this
C. LiNO3 reaction relate?
D. KNO3
E. KCl
50. Drugs are commonly analyzed
by means of potentiometric pH
measurement. Which of the electrodes A. Addition
can be used for measuring the solution B. Substitution
pH? C. Reduction
D. Oxidation
A. Glass
B. Standard hydrogen E. Rearrangement
C. Zinc 56. While studying a stem covered with
D. Calomel periderm, the researcher realized that gas
E. Chlorine-silver exchange takes place through . . .
51. The ability of reagent to ensure a A. Lenticels
stable analytical effect in the interaction B. Stomata
with the analyzed substance is characteri- C. Pores
zed by: D. Non-suberized (conducting) cells
E. Hydatodes
A. Reaction sensitivity
B. Reaction selectivity 57. Which of the ligands is bidentate?
C. Reaction specificity
D. Reagent amount
E. -
52. To maintain a certain value of
109
krok123.in.ua
Krok 1 Pharmacy 2012 7

A. Ethylenediamine 63. A patient consulted a doctor about


B. Thiocyanate ion sunburns, decreased visual acuity. His hair,
C. Cyanide ion skin and eyes are not pigmented. He has
D. Pyridine been diagnosed with albinism. The patient
E. Hydroxide ion presents with the following enzyme defici-
ency:
58. An expression for the hydrolysis
constant A. Tyrosinase
Kg = (KacidK·K
w
base )
B. Arginase
corresponds with the following salt: C. Carbonic anhydrase
D. Histidine decarboxylase
A. (NH4 )2 S E. Hexokinase
B. NaCN
C. F e(NO3 )3 64. The causative agent of botulism causes
D. Li2 S severe food poisoning. Specify the most
E. NH4 Cl characteristic morphological feature of
botulism causative agent:
59. The conversion MnO4− → MnO2
represents the following reaction: A. Gram-positive bacillus with subtermi-
nal spore
A. Reduction in neutral medium B. Thick gram-positive non-spore-forming
B. Oxidation in acidic medium bacillus
C. Reduction in acidic medium C. Gram-positive bacillus with terminal
D. Oxidation in alkaline medium spore
E. Reduction in alkaline medium D. Thin mobile bacillus with central spore
E. Thick gram-positive bacillus without
60. The high energy from thermal dissoci- spores and flagella
ation of CO molecule (the binding energy
of 1075 kJ) results from: 65. After the Cannizzaro reaction for
benzaldehyde the following compound is
A. Triple bond between the atoms of obtained:
oxygen and carbon
B. Covalent bond
C. Ionic bond
D. High polarity of the molecule
E. Hydrogen bond
61. The sodium-potassium pump functi-
oning in a cell is responsible for the
transport of the following ions across the
membrane:
A. Sodium ions out of the cell, potassium
ions into the cell
B. Calcium ions into the cell, potassium
ions out of the cell
C. Sodium ions into the cell, potassium
ions out of the cell
D. Chlorine ions into the cell, potassium
ions out of the cell
E. Sodium ions into the cell, chlorine ions
out of the cell
62. In terms of water-air interface, the
following substance acts as a surface-
active substance:
A. Valeric acid
B. HCl
C. NaOH
D. Urea
E. -

110
krok123.in.ua
Krok 1 Pharmacy 2012 8

67. Fatty degeneration of liver is prevented


by lipotropic substances. Which of the
following substances relates to them?
A. A. Methionine
B. Cholesterol
C. Bilirubin
D. Glycine
E. Glucose
B. 68. Alkaline reaction is typical for the
solution of the following salt:
A. Na2 S
B. Na2 SO4
C. C. KCl
D. CuCl2
E. F eCl3
69. Nitrite ions in presence of nitrate ions
can be detected by means of:
D.
A. Crystalline antipyrine in presence of
dilute HCl
B. Crystalline sodium thiosulfate
C. Dimethylglyoxime
D. Crystalline iron (III) sulfate
E. E. Diphenylcarbazone
70. Which of the following substances
66. Specify the reaction, through which relates to colloidal surface-active
salicylic acid can be synthesized: substances?
A. Potassium oleate
B. Iodine
C. Sodium chloride
D. Polyethylene
E. Gelatin
A.
71. The cause of optical activity is
the presence of the following organic
compound in the molecular structure:
A. Asymmetric carbon atom
B. Double bond
B. C. Triple bond
D. Functional group
E. Plane of symmetry
72. Given the ability of iodine to dissolve
in nonpolar solvents, determine the type
of chemical bond in an I2 molecule:
C. A. Nonpolar covalent
B. Ionic
C. Polar covalent
D. Metal
E. Intermolecular interaction
73. Aqueous solution of CaCl2 with mass
D. concentration 10% is used for intravenous
injections. What is the maximum value
E. - of isotonic coefficient of CaCl2 in an
aqueous solution?
111
krok123.in.ua
Krok 1 Pharmacy 2012 9

A. 3
B. 4
C. 2
D. 5 A.
E. 1
74. Cryoscopic constants of water,
benzene, chloroform, acetic acid and B.
camphor equal to 1,86; 5,12; 4,9; 3,9;
40,0 respectively. Which of these solvents
should be selected for the most accurate
determination of the molar mass of a
drug substance (nonelectrolyte) by the
cryoscopic method?
A. Camphor C.
B. Chloroform
C. Acetic acid
D. Benzene
E. Water
D.
75. Which of the listed carbonyl
compounds gives a positive iodoforme
reaction?

E.

A.
77. For the quantitative analysis of ethanol
the gas chromatography was used. Which
parameter was measured?
B. A. Peak height or area
B. Retention time
C. Retention volume
D. Peak width
E. Peak width at half height
C.
78. Potassium permanganate reacting with
hydrogen peroxide in acidic medium acts
as:
D. A. Oxidant
B. Reductant
C. Disproportionation agent
D. Oxidant and reductant
E. Does not act either as an oxidant, or as
E. a reductant
79. Specify the number of electrons
76. Which of these reactions can be used involved into formation of the isolated
to identify the primary amino group? conjugated system in the pyrimidine
molecule:

112
krok123.in.ua
Krok 1 Pharmacy 2012 10

A. 6 A. Unsaturated carboxylic acids


B. 4 B. Lactones
C. 10 C. Lactides
D. 2 D. Dicarboxylic acid
E. 8 E. Saturated monocarboxylic acids
80. A patient has an increased 85. The pH of 0,001 M of hydrochloric
concentration of hippuric acid in the uri- acid solution is:
ne. This acid is the product of benzoic
acid detoxification in the liver of. In the A. 3
human body benzoic acid is formed from B. 0
the following amino acid: C. 10
D. 7
A. Phenylalanine E. 5
B. Succinate
C. Lactate 86. Which of the listed biologically acti-
D. Aspartate ve compounds inhibits the secretion of
E. Malate pancreatic juice?
81. A group of alpinists climbing to the A. Atropine
top had their blood tested. The test B. Acetylcholine
revealed erythrocytosis and an increase C. Insulin
in hemoglobin rate. What type of hypoxia D. Gastrin
caused the stimulation of erythropoiesis E. Secretin
in the bone marrow?
87. What type of tautomerism is typical for
A. Hypoxic the given compound?
B. Combined
C. Hemic
D. Circulatory
E. Tissue
82. Quite often, the soil may contain a
number of pathogenic microorganisms.
The causative agents of the following di-
sease may exist in the soil for a long time:
A. Anthrax
B. Diphtheria A. Nitro-aci-nitro tautomerism
C. Viral hepatitis B. Carbonyl-enol tautomerism
D. Pertussis C. Cyclo-oxo tautomerism
E. Dysentery D. Amine-imine tautomerism
E. Keto-enol tautomerism
83. Pathogenic microorganisms are
characterized by presence of aggression 88. The molar mass of calcium hydroxide
enzymes that determine their virulence. equivalent (M(Ca(OH)2 ) = 74 g/mol) is:
Select an aggression enzyme:
A. 37 g/mol
A. Hyaluronidase B. 19 g/mol
B. Carbohydrase C. 32 g/mol
C. Transferase D. 74 g/mol
D. Oxidase E. 148 g/mol
E. Lyase
89. Bacteriological inspection of disi-
84. By heating β-hydroxy acids the nfection quality at a pharmacy revealed
following substance is formed: a microorganism in an utility room (in the
sink). The microorganism has the followi-
ng properties: mobile nonspore-forming
gram-negative bacteria that form capsular
substance, grow well on ordinary nutrient
media, secrete the blue-green pigment.
This microorganism is most likely to be
of the following genus:
113
krok123.in.ua
Krok 1 Pharmacy 2012 11

of its conversion?
A. Pseudomonas
B. Proteus A. Alpha-glycerolophosphate
C. Clostridium B. Pyruvate
D. Shigella C. Lactate
E. Vibrio D. Choline
E. Acetyl coenzyme A
90. Microbiological analysis of medicinal
raw materials revealed capsular bacteria. 96. When ammonia enters into reacti-
What stain method was used to detect the on with acids, this results in formation
capsules? of ammonium salts. Which properties of
ammonia characterize this process?
A. Gin’s
B. Ziehl-Neelsen’s A. Ability to accept the hydrogen ions
C. Neisser’s B. Reductive properties
D. Gram’s C. Acidic properties
E. Ozheshko’s D. Oxidative properties
E. Ability to hydrolyze
91. The analyzed plant has hollow ribbed
stems, compound umbel inflorescence, 97. A patient had been diagnosed wi-
schizocarpic fruit (cremocarp) and is rich th right lung cancer and administered
in essential oils, which is a characteristic surgical treatment. After right-sided
of: pulmonectomy the patient presented wi-
th evident dyspnea. What form of respi-
A. Apiaceae ratory failure has developed in this pati-
B. Fabaceae ent?
C. Ericaceae
D. Brassisaceae A. Pulmonary restrictive
E. Asteraceae B. Central
C. Peripheral
92. Specify the colour of phenolphthalein D. Pulmonary obstructive
in the sodium sulfide solution: E. Thoracodiaphragmal
A. Crimson 98. What wave of ECG characterizes the
B. Colourless spread of excitation throughout the heart
C. Blue atria?
D. Yellow
E. Green A. P
B. R
93. In pharmaceutical technology an C. Q
important part is played by pressure, D. T
temperature, concentration. The reacti- E. S
on yield can be increased by lowering the
temperature of the following process: 99. Cardiac tones are the outer acoustic
manifestations of heart functioning. What
A. Exothermic is the cause of the II tone?
B. Endothermic
C. Isochoric A. Closure of the semilunar valves
D. Isobaric B. Closure of the cuspid valves
E. Adiabatic C. Vibration of the ventricle walls
D. Vibration of the atrium walls
94. To identify a drug by thin-layer E. Chest movements
chromatography the following parameter
is used: 100. In accordance with the requirements
of the pharmacopoeia, the non-sterile
A. Rf medicinal preparations may include mi-
B. n croorganisms. What micro-organisms
C. E, mV MUST NOT be present in them?
D. I, A
E. Kp
95. The intracellular metabolism of
glycerol starts with its activation. What
compound is formed in the first reaction
114
krok123.in.ua
Krok 1 Pharmacy 2012 12

A. Enterobacteria A. Plague
B. Ascomycetes B. Tuberculosis
C. Micrococci C. Leptospirosis
D. Mold fungi D. Brucellosis
E. Sarcinae E. Toxoplasmosis
101. While performing finger-nose test 106. Choose a name that corresponds to
the examinee could not touch the tip the formula: CH3 − C ≡ N:
of his nose with his fingertip having his
eyes closed. What structure of the central A. Acetic acid nitrile
nervous system is damaged? B. Acetamide
C. Acetic anhydryde
A. Cerebellum D. Acetoxime
B. Quadrigeminal plate E. Ethyl isocyanide
C. Cortex
D. Spinal cord 107. What data is required to determine
E. Thalamus the activation energy?
102. A patient has obstruction of the A. Constants of reaction rate at two
common bile duct. Which of these temperatures
substances is usually found in urine in B. Thermal energy of the reaction
such cases? C. Energy change of the system
D. Internal energy of the system
A. Bilirubin E. Reaction order
B. Ketone bodies
C. Uric acid 108. A patient with systemic lupus
D. Creatinine erythematosus has developed a diffuse
E. Glucose renal affection accompanied by protei-
nuria, hypoproteinemia, massive edema.
103. A continuous stay in the mountai- What is the mechanism of proteinuria
ns causes an increase of blood oxygen development in this case?
capacity. What is the possible reason for
this phenomenon? A. Autoimmune affection of glomeruli
B. Inflammation of renal tubules
A. Development of functional C. Ischemic affection of tubules
erythrocytosis D. Blood protein increase
B. Increase of P O2 rate in the air E. Affection of urinary tracts
C. Increase of P CO2 rate in the air
D. Decrease in respiratory rate and depth 109. What reactions are used in the
E. Development of gas acidosis methods of permanganatometry, di-
chromatometry, iodometry?
104. A student had to analyze an axi-
al plant organ characterized by radi- A. Oxidation-reduction
al symmetry, unlimited growth, positi- B. Precipitation
ve geotropism. It provided nutrition, C. Complexation
vegetative propagation, anchorage of D. Neutralization
plant in the soil. This organ was identi- E. Hydrolysis
fied as . . .
110. In order to bind hydrogen ions during
A. Root the identification of potassium ions with
B. Stem tartaric acid the following solution is used:
C. Leaf
D. Rhizome A. Sodium acetate
E. Seed B. Sodium hydroxide
C. Ammonia
105. A patient presents with fever, chi- D. Sulfuric acid
ll and cough. From his sputum the ovoid E. Hydrochloric acid
Gram-negative bipolar-stained bacilli wi-
th a delicate capsule were isolated. What 111. According to the Paneth-Fajans rule,
is the most likely diagnosis? the ion preferably adsorbed from a soluti-
on on the surface of a solid crystalline
adsorbent is the ion, which:

115
krok123.in.ua
Krok 1 Pharmacy 2012 13

A. Is included in the crystal lattice of the A. SE


adsorbent B. AE
B. Is not included in the crystal lattice of C. SR
the adsorbent D. SN
C. Does not form a sparingly soluble E. AN
compound with one of the lattice ions
D. Forms an easily soluble compound with 116. This scheme of nitroalkane synthesis
one of the lattice ions is called the reaction of:
E. Forms a sparingly soluble compound
with one of the lattice ions
112. Some medications are produced by
hydrolysis of corresponding neutral salts.
From the salts listed below, select the one
that WILL NOT succumb to hydrolysis:
A. Konovalov
A. Na2 SO4 B. Zinin
B. NaHCO3 C. Kucherov
C. AlCl3 D. Tishchenko
D. Bi(NO3 )3 E. Chichibabin
E. Na2 SO3
117. A hospital admitted a patient with
113. Which of the following reactions is arterial hypertension induced by renal
required in order to obtain an azo dye out artery stenosis, complaints of persistent
of an aromatic amine? nausea and headache. The main element
A. Diazotization and azo compound in the pathogenesis of hypertension is the
B. Reduction and diazotization activation of the following system:
C. Diazotization and interaction with A. Renin-angiotensin
potassium cyanide B. Hypothalamic-pituitary
D. Salt formation and nitration C. Kallikrein-kinin
E. Alkylation and nitrosation D. Sympathoadrenal
114. A solution containing calcium and E. Parasympathetic
magnesium cations is titrated with Trilon 118. Nitrating mixture is a mixture of
B solution. Complexometric titration of concentrated acids:
these cations requires the following medi-
um: A. HNO3 + H2 SO4
B. H3 P O4 + H2 SO4
A. Ammonium buffer solution C. HCl + H2 SO4
B. Formate buffer solution D. HNO3 + HCl
C. Neutral medium E. H3 P O4 + HCl
D. Acidic solution
E. Acetate buffer solution 119. Bacterioscopic examination of
chancre material revealed some mobile,
115. What is the mechanism of brominati- long, convoluted microorganisms with 8-
on of toluene aromatic nucleus? 12 regular coils. These features are typical
for:
A. Treponema
B. Borrellia
C. Leptospira
D. Vibrios
E. Campylobacter
120. Before a surgical operation, a
surgeon treated his hands with an alcohol-
containing solution. Which group of drugs
does this solution relate to?

116
krok123.in.ua
Krok 1 Pharmacy 2012 14

A. Antiseptics has blood pH at the rate of 7,34.


B. Disinfectants Analysis of gas formula of blood showed
C. Sterilizing solutions hypercapnia. Urine analysis revealed an
D. Detergents acidity increase. What form of acid-base
E. Surface-active substances disbalance is the case?
121. In order to establish the possible A. Gaseous acidosis
contamination of a medication with fungi, B. Secretory alkalosis
a nutrient medium was inoculated, whi- C. Gaseous alkalosis
ch resulted in growth of large cream-like D. Non-gaseous alkalosis
colonies. What nutrient medium was used E. Non-gaseous acidosis
in this case?
127. The study of the main root
A. Sabouraud ontogenesis shows that it has developed
B. Lowenstein-Jensen from:
C. Roux
D. Loeffler A. Radicle
E. Finn-2 B. Apical meristem
C. Pericycle
122. Aniline can be converted into water- D. Lateral meristem
soluble salt by treatment with a solution E. Intercalary meristem
of:
128. The end product of starch hydrolysis
A. Hydrochloric acid is:
B. Sodium hydroxide
C. Sodium sulfate A. D-glucose
D. Ethanol B. D-fructose
E. Dimethylamine C. Saccharose
D. Maltose
123. As a rule, the maximum oxidation E. D-galactose
number of an element is:
129. What reagent can help to distinguish
A. Group number in the periodic system between starch and glucose?
B. Subgroup number in the periodic
system A. I2
C. Period number B. Br2
D. Row number C. KMnO4
E. - D. K2 Cr2 O7
E. F eCl3
124. It is known that some chemical
compounds uncouple the tissue respirati- 130. Which of the following solutions wi-
on and oxidative phosphorylation. Name th the same molar concentration has the
one of these compounds: maximum osmotic pressure?
A. 2,4-dinitrophenol A. Aluminum nitrate
B. Carbon monoxide B. Glucose
C. Antimycin A C. Sodium chloride
D. Lactic acid D. Magnesium sulfate
E. Acetyl-CoA E. Potassium iodide
125. A warmly dressed child has spent 131. Pharmacopoeia test reaction for
a considerably long time out of doors. determination of benzoate ions is the
This resulted in body temperature elevati- interaction with the following solution:
on and general weakness development.
What form of thermoregulation disorder A. Iron (III) chloride
is observed in this case? B. Potassium chloride
C. Resorcinol
A. Exogenous hyperthermia D. Acetic anhydride
B. Endogenous hyperthermia E. Diphenylamine
C. Fever
D. Heat shock 132. If the amount of high-molecular
E. Centrogenous hyperthermia substance added to the sol is very small, it
may not increase but decrease its stability.
126. A patient with pneumosclerosis This phenomenon is called:
117
krok123.in.ua
Krok 1 Pharmacy 2012 15

A. Reagin (anaphylactic)
A. Sensibilization B. Cytotoxic
B. Solubilization C. Immunocomplex
C. Mutual coagulation D. Cell-mediated
D. Colloidal protection E. Stimulating
E. Sol habituation
138. When determining the changes in
133. In order to increase the inhibitory membrane permeability during an acti-
processes in the CNS the pharmacological on potential it was established that duri-
agents are used that cause the following ng the depolarization phase the following
process on the postsynaptic membranes: movement predominates:
A. Hyperpolarization A. Movement of Na+ into the cell
B. Depolarization B. Movement of Na+ out of the cell
C. Afterdepolarization C. Movement of K + into the cell
D. Activation of sodium channels D. Movement of K + out of the cell
E. Activation of calcium channels E. Movement of Cl− into the cell
134. A 70-year-old patient has been found 139. Spore and pollen analysis revealed in
to have atherosclerosis of heart and the pollen some tetrahedral spores with a
brain vessels. Examination revealed the semi-circular base and a reticular surface,
changes in the lipid profile. Pathogenesis which may belong to:
of atherosclerosis is greatly influenced by
an increase in the following lipoproteins A. Lycopodiophyta
rate: B. Equisetiphyta
C. Bryophyta
A. Low-density lipoprotein D. Polypodiophyta
B. Very-low-density lipoproteins E. Pinophyta
C. Intermediate-density lipoproteins
D. High-density lipoprotein 140. In the practice of harvesting herbal
E. Chylomicrons raw material of Asteraceae family the term
"flowers"means both individual flowers
135. Calendula officinalis which a and inflorescences. However, the notion
representative of the aster fami- of "flowers"is botanically correct only for:
ly is characterized by the following
inflorescence type: A. Centaurea cyanus
B. Gnaphalium uliginosum
A. Flowerhead C. Arnica montana
B. Umbel D. Echinops ritro
C. Catkin E. Bidens tripartita
D. Glome
E. Cyme 141. What working solutions (titrants) are
used in the method of precipitation titrati-
136. Which medicinal plant of the on - Folgard method?
Asteraceae family has only disk flowers
in the flowerhead? A. AgNO3 and NH4 SCN
B. H2 SO4 and NaOH
A. Three-part beggarticks (Bidens triparti- C. Na2 S2 O3 and K(I3 )
ta) D. KMnO4 and KBrO3
B. Dandelion (Taraxacum officinale) E. HClO4 and KOH
C. Echinacea purpurea
D. Cornflower (Centaurea cyanus) 142. In which of these reactions hydrogen
E. Common yarrow (Achillea millefolium) acts as an oxidizing agent?

137. After eating strawberries a child A. 2Na + H2 → 2NaH


presented with itchy red spots on the skin B. Cl2 + H2 → 2HCl
(hives). According to the classification of C. CuO + H2 → H2 O + Cu
Coombs and Jell this reaction relates to D. N2 + 3H2 → 2NH3
the following type of allergic reactions: E. F2 + H2 → 2HF
143. A perennial herbaceous plant has
ascending quadrangular stem and opposi-
tely arranged leaves. The flowers with bi-
labiate corolla are zygomorphic, bisexual,
118
krok123.in.ua
Krok 1 Pharmacy 2012 16

arranged in whorls in the leaf axils. The properties of phytopathogenic micro-


fruit type is coenobium. The described organisms are due to the following
medicinal plant relates to the following enzymes:
botanic family:
A. Hydrolytic
A. Lamiaceae B. Isomerase
B. Asteraceae C. Transferase
C. Poaceae D. Oxidoreductase
D. Brassicaceae E. Lyase
E. Rosaceae
149. The medicinal plants growing on a
144. Dysbiosis can be treated with plantations were found to have mosaic
drugs that contain living representati- patterns on leaves. What microorganisms
ves of normal microflora as well as caused this affection?
their metabolic products. Select the
microorganisms that are used for the A. Phytopathogenic viruses
production of such drugs: B. Phytopathogenic bacteria
C. Phytopathogenic fungi
A. Bifidus bacteria D. Protozoa
B. Staphylococcus aureus E. Rickettsiae
C. Proteus
D. Providencia 150. Bacteria eventually become resi-
E. Yersinia stant to antibacterial agents. Resistance of
gram-positive bacteria to penicillin antibi-
145. The primary structure of nucleic aci- otics is caused by:
ds is a polynucleotide chain which has
a certain composition and order of the A. Beta-lactamase production
nucleotides. What bonds stabilize this B. Permeability of the cell wall
structure? C. Active synthesis of peptidoglycan
D. Active transport of antibiotic
A. 3’,5’-phosphodiester E. Protein synthesis
B. Peptide
C. Glycosidic 151. A 28-year-old male got a burn that
D. Disulfide caused an increase in spontaneous secreti-
E. Amide on of gastric juice. It is associated with
secretion of the following substance:
146. While determining the type and
characteristics of conducting bundles of A. Histamine
axial organs one should take into account B. Secretin
the positional relation between phloem C. Gastric inhibitory peptide
and xylem and . . . D. Cholecystokinin-Pancreozymin
E. Serotonin
A. Cambium
B. Procambium 152. Blood pressure is regulated by a
C. Collenchyme number of biologically active compounds.
D. Pericycle What peptides that enter the bloodstream
E. Phellogen can affect the vascular tone?
147. Growth of some cancer cells is caused A. Kinins
by a certain growth factor. Treatment of B. Leukotrienes
leukemia involves applying an enzyme C. Enkephalins
that destroys this essential factor. Speci- D. Iodothyronines
fy this enzyme: E. Endorphins
A. Asparaginase 153. Mycothallus of the fungus under
B. Glutaminase study consists of a stipe, pileus, lamellar
C. Succinate dehydrogenase hymenophore. This fungus belongs in the
D. Citrate synthetase class:
E. Aspartate aminotransferase
148. Medicinal plants infected by
microorganisms cannot be used in
the pharmaceutical industry. Invasive
119
krok123.in.ua
Krok 1 Pharmacy 2012 17

A. Basidiomycetes A. Bi(OH)3
B. Ascomycetes B. Sb(OH)3
C. Zygomycetes C. As(OH)3
D. Deuteromycetes D. H3 P O3
E. Oomycetes E. -
154. A laboratory received a sample of 159. What is the equivalent of Al(OH)3
water used in drug production for sanitary in the reaction Al(OH)3 + 2HCl =
and virological analysis. What group of vi- Al(OH)Cl2 + 2H2 O?
ruses will indicate faecal contamination of
water and thus the need for its additional A. 1/2 mol
purification? B. 1/3 mol
C. 1 mol
A. Picornaviridae D. 2 mol
B. Herpesviridae E. 3 mol
C. Orthomyxoviridae
D. Retroviridae 160. Food rich in carbohydrates at fi-
E. Flaviviridae rst increases the blood glucose and then
decreases its rate due to the insulin acti-
155. Specify the indicator of the protective on. What process is activated by this
properties of high-molecular compounds hormone?
of body that promote the keeping of calci-
um, phosphate and carbonate in blood A. Synthesis of glycogen
plasma: B. Gluconeogenesis
C. Breakdown of glycogen
A. Protective value D. Breakdown of proteins
B. Coagulation threshold E. Breakdown of lipids
C. Critical micelle concentration
D. Hydrophilic-lipophilic balance 161. A patient has developed
E. Volume of sol coagulated by 1 mol of megaloblastic anemia on a background
the electrolyte substance of alcoholic cirrhosis. The main cause of
anemia in this patient is the following vi-
156. After an insulin injection a 45-year- tamin deficiency:
old woman with a long history of di-
abetes mellitus has developed weakness, A. Folic acid
paleness, palpitation, anxiety, double visi- B. Lipoic acid
on, numbness of lips and the tip of tongue. C. Biotin
Blood glucose is at the rate of 2,5 mmol/l. D. Thiamin
What complication has developed in the E. Pantothenic acid
patient?
162. Inflorescence of greater plantain
A. Hypoglycemic coma grows out at apex, the main axis is long,
B. Hyperosmolar coma and flowers are sessile. This type of
C. Hyperglycemic coma inflorescence is called:
D. Hyperketonemic coma
E. Uremic coma A. Spike
B. Panicle
157. Morphological analysis of poplar C. Spadix
inflorescence showed that it is a simple D. Capitulum
monopodial inflorescence: main axis is E. Thyrsus
drooping, the flowers are sessile, uni-
sexual. Specify the type of inflorescence: 163. In order to identify the cations
of zinc (II) an analytical chemist used
A. Catkin the reagent solution of hexacyanoferrate
B. Head (II) potassium (Pharmacopeia reaction).
C. Capitulum What colour precipitate is formed in this
D. Cyme reaction?
E. Panicle
A. White
158. Specify the compound with the most B. Yellow
pronounced basic properties: C. Black
D. Green
E. Red
120
krok123.in.ua
Krok 1 Pharmacy 2012 18

164. A patient was found to have an in the urine of a young man. In what
increased blood serum LDH-1 activity. In case a healthy person may present a sli-
which organ is the pathological process ght proteinuria?
localized?
A. After exercise
A. Heart B. In the resting state
B. Liver C. During sleep
C. Kidneys D. In the state of psychoemotional exci-
D. Stomach tation
E. Muscles E. After overeating
165. Diaphoretic herbal tea includes di- 170. A male patient was found to have
chasial cymes with light-yellow, oblong, hypovitaminosis P P . What amino acid
wing-like, squamelliferous perianth. The taken with food may partially compensate
flowers are fragrant, yellowish. These the vitamin P P deficiency?
inflorescences belong to:
A. Tryptophan
A. Tilia cordata B. Phenylalanine
B. Viburnum opulus C. Valine
C. Robinia pseudoacacia D. Arginine
D. Mentha piperita E. Methionine
E. Padus avium
171. Emulsions are classified according
166. A patient with alcoholic cirrhosis to the volume concentration of dispersed
complains of general weakness, dyspnea. phase. An emulsion with the concentrati-
He has been found to have decreased on at the rate of 0,1-74,0% vol. relates to
blood pressure, ascites, enlargement of the following group of emulsions:
superficial veins of the anterior abdominal
wall, esophageal varices, splenomegaly. A. Concentrated
What hemodynamic disorder is observed B. Diluted
in the patient? C. Highly concentrated
D. Direct
A. Portal hypertension E. Reversible
B. Left ventricular failure
C. Right ventricular failure 172. Choose a reaction, in which a basic
D. Heart failure salt is formed:
E. Collapse
A. F e(OH)3 + 2HCl
167. A patient with tuberculosis has been B. F e(OH)3 + 3KCl
prescribed some anti-TB preparations. C. 2NaOH + H2 SO4
Which of the following chemotherapeutic D. KOH + H2 SO4
drugs has an effect on the tuberculosis E. NaOH + HCl
pathogen?
173. Ammonia solution has been added to
A. Ftivazide the solution under examination. A black
B. Furacilinum precipitate fell out. This indicates the
C. Methisazonum presence of the following cations in the
D. Sulfadimezinum solution:
E. Phthalylsulfathiazole
A. Mercury (I)
168. What segment of a nephron contains B. Copper (II)
liquid with a maximum concentration of C. Iron (III)
glucose under normal conditions? D. Iron (II)
E. Silver (I)
A. Poximal tubules
B. Medullary thick ascending limb of loop 174. The causative agents of intesti-
of Henle nal infections can grow at refrigerator
C. Inner medullary portion of thin temperatures, which may cause infection
descending limb of loop of Henle in people. What type of temperature opti-
D. Distal convoluted tubule mum do these microorganisms relate to?
E. Inner medullary collecting duct
169. Laboratory analysis revealed protein
121
krok123.in.ua
Krok 1 Pharmacy 2012 19

A. Psychrophilic sic acid. Development of this disease is


B. Mesophilic associated with disorder of the following
C. Thermophilic amino acid metabolism:
D. Anthropophilic
E. Necrophilic A. Tyrosine
B. Tryptophan
175. What type of conducting bundles is C. Alanine
characteristic of all root zones of one- D. Methionine
seeded plants? E. Asparagine
A. Radical 181. Antidepressants can increase the
B. Central phloem concentartion of catecholamines in the
C. Central xylem synaptic cleft. What is the mechanism of
D. Bilateral action of these drugs?
E. Collateral
A. Inhibition of monoamine oxidase
176. A man who had been struck in the B. Activation of monoamine oxidase
epigastric region had a heart arrest. What C. Inhibition of xanthine oxidase
caused such changes in the cardiac activi- D. Activation of acetylcholinesterase
ty? E. Inhibition of acetylcholinesterase
A. Increased vagal tonus 182. A patient was taken to a hospital wi-
B. Adrenaline release th acute food poisoning caused by home-
C. Increased sympathetic tonus made canned mushrooms. The product
D. Angiotensin II release analysis revealed some microorganisms
E. Histamine release that develop only in the absence of
oxygen. What microorganisms caused the
177. A 45-year-old patient with a gastric poisoning?
ulcer needs the reduction of HCl secreti-
on. Which drug provides this effect due to A. Obligate anaerobes
inhibition of the proton pump? B. Facultative anaerobes
C. Microaerophiles
A. Omeprazole D. Obligate aerobes
B. Atropine E. Capnophiles
C. Quamatel
D. Benzohexonium 183. During a survey of the sanitary state
E. Proglumide of an environment object perfringens titre
was determined. What object was studi-
178. When chyme enters the duodenum, it ed?
stimulates the secretion of gastrointestinal
hormones. Which hormone is responsible A. Soil
for release of enzymes being included in B. Outdoor air
digestive juices? C. Water from an open water reservoir
D. Tap water
A. Cholecystokinin-pancreozymin E. Indoor pharmacy air
B. Secretin
C. Glucagon 184. According to the Rayleigh equation,
D. Somatostatin the intensity of scattered light is inversely
E. Calcitonin proportional to the wavelength of:
179. Loop of Henle is involved in the A. Incident light (fourth power)
mechanism of urine formation. What B. Incident light (second power)
process takes place in its descending porti- C. Incident light (fifth power)
on? D. Incident light (third power)
A. Reabsorption of water E. Incident light
B. Reabsorption of water and electrolytes 185. Quantitative analysis of zinc salts
C. Reabsorption of Na+ is performed by method of trilonometry.
D. Reabsorption of l− What indicator is used for this purpose?
E. Reabsorption of Ca2+
180. Alkaptonuria is characterized by an
excessive urinary excretion of homogenti-
122
krok123.in.ua
Krok 1 Pharmacy 2012 20

A. Eriohrome black-T 191. A common species of the Pinaceae


B. Phenolphtalein family is a tall, evergreen, shade-enduring
C. Methyl black tree. The needles are solid, prickly,
D. Potassium dichromate quadrangular in cross-section, spirally
E. Thymol blue arranged. This tree is:
186. Clinical practice involves measurement A. Picea abies
of ESR. What are the components of B. Larix sibirica
blood plasma that mainly determine the C. Pinus sylvestris
ESR value? D. Juniperus communis
E. Ephedra equisetina
A. Globulins
B. Urea 192. Early pregnancy can be detected
C. Bilirubin by using the appropriate test. A positi-
D. Inorganic ions ve pregnancy test is based on the presence
E. Phospholipids of the following hormone in urine:
187. The patient uses a daily basis for A. Chorionic gonadotropin
several raw eggs, which contain antivi- B. Progesterone
tamin biotin - avidin. Violations of any C. Oestradiol
phase of lipid metabolism might arise? D. Prolactin
E. Oxytocin
A. Fatty acid biosynthesis
B. Cholesterol biosynthesis 193. A patient has developed an attack of
C. Lipid absorption bronchial asthma: he has laboured respi-
D. Glycerol oxidation ration with the frequency of 24-26/min.,
E. Lipid transport in blood inspirations take turns with prolonged
expirations involving participation of
188. Aqueous solutions of CuSO4 are expiratory muscles. What form of respi-
used in ophthalmic and urological practice ratory failure has developed in the pati-
as an antiseptic, astringent and cauterant ent?
agent. What is the oxidation number of
cuprum in this compound? A. Expiratory dyspnea
B. Cheyne-Stokes
A. +2 C. Biot’s
B. +1 D. Inspiratory dyspnea
C. 0 E. Apneustic respiration
D. +3
E. -1 194. A 58-year-old male patient was found
to have a peripheral circulation disorder
189. Astragalus dasyanthus has sessile with a restricted arterial inflow, paleness
flowers gathered into inflorescences wi- of the respective region, drop of parti-
th a short thick axis. This inflorescence is al oxygen pressure in it. This disorder is
called: called:
A. Capitulum A. Ischemia
B. Cyme B. Arterial hyperemia
C. Truss C. Thrombosis
D. Spike D. Venostasis
E. Head E. Reperfusion syndrome
190. The technology of drug production 195. To prepare 600 g of 10% soluti-
widely uses the phenomena of absorption on of potassium hydroxide the following
and ion exchange. Which of the ions will amount of potassium must be taken:
be selectively adsorbed on the surface of
a silver chloride crystal from an aqueous A. 60 g
solution? B. 0,6 g
C. 6 g
A. Ag + D. 10 g
B. Cu2+ E. 12 g
C. NO 3−
D. H + 196. Half-life (half-reaction) is inversely
E. OH − proportional to the initial concentration
123
krok123.in.ua
Krok 1 Pharmacy 2012 21

for the reactions of: A. Urticaceae


B. Brassicaceae
A. Second order C. Fabaceae
B. First order D. Solanaceae
C. Fraction order E. Papaveraceae
D. Third order
E. Zeroth order 199. Seroprophylaxis and serotherapy of
infectious diseases involves using immune
197. A male received a radiation dose of sera. What type of immunity is thus acqui-
30 Gy. He presents with necrotic angi- red?
na, disorders of the gastrointestinal tract.
Blood tests revealed anemia, leukopenia A. Passively acquired artificial immunity
and thrombocytopenia. What period of B. Actively acquired artificial immunity
acute radiation sickness is observed in the C. Actively acquired natural immunity
patient? D. Passively acquired natural immunity
E. -
A. Height of disease
B. Primary reactions 200. Caffeine inhibits phosphodiesterase
C. Imaginary wellbeing which converts cAMP to AMP. The most
D. End of disease typical feature of caffeine intoxication is
E. - the reduced intensity of:
198. Examination of the leaf epidermis A. Glycogen synthesis
revealed cells containing cystoliths. B. Protein phosphorylation
Presence of cystoliths is typical for plants C. Pentose phosphate pathway
of the following family: D. Glycolysis
E. Lipolysis

124
krok123.in.ua
Krok 1 Pharmacy 2013 1

1. An excess of concentrated ammoni- ent developed gastric bleeding due to


um hydroxide is a group reagent for the ulceration of the gastric mucosa.
the cations of the VI analytical group Ulcerogenic effect of the drug is associ-
(acid-base classification), namely Co2+ , ated with a decrease in the activity of the
Ni2+ , Cd2+ , Cu2+ , Hg 2+ . As a result of following enzyme:
this reaction the following substances are
formed: A. Cyclooxygenase-1
B. Cyclooxygenase-2
A. Water-soluble ammonia complexes C. Lipoxygenase
B. Hydroxides of acid-soluble cations D. Thromboxane synthetase
C. Stained water-insoluble compounds E. Prostacyclin synthase
D. Hydroxides of alkali-soluble cations
E. Hydroxides of the cations insoluble in 5. A patient with hypertensive crisis has
the excess of ammonium hydroxide been given an intravenous injection of
clonidine. What mechanism underlies the
2. Which of the listed reactions antihypertensive effect of clonidine?
demonstrates the basic properties of pyri-
dine? A. Stimulation of presynaptic central
α2 -adrenoceptors
A. B. Blockade of peripheral α1 -
adrenoceptors
C. Blockade of β-adrenoceptors
D. Blockade of N-cholinergic receptors
E. Direct myotropic effect on blood vessels
6. A patient with ischemic heart disease
B. has been administered inosine which is an
intermediate metabolite in the synthesis
of:
A. Purine nucleotides
B. Metalloproteins
C. C. Lipoproteins
D. Glycoproteins
E. Ketone bodies
7. A patient with hypertensive crisis
should be administered a diuretic as a part
D. of complex therapy. What drug should be
given the patient?
A. Furosemide
B. Diacarb
C. Spironolactone
E. D. Triamterene
E. Amiloride
8. Rates of chemical reactions of the same
order are compared by:
A. Constant of chemical reaction rate
B. Chemical reaction rate
C. Endpoint of a reaction
3. A patient with stenocardia has been D. Change in the reactants concentration
administered acetylsalicylic acid for: E. Change in the concentration of the
A. Antiplatelet effect reaction products
B. Inhibition of blood fibrinolytic activity 9. Consult a patient on which antihistami-
C. Aggregate effect ne drug DOES NOT have sedative and
D. Anti-inflammatory effect hypnotic effect:
E. Increase in blood fibrinolytic activity
4. On the 4th day of treatment with
indomethacin a male 55-year-old pati-
125
krok123.in.ua
Krok 1 Pharmacy 2013 2

A. Loradatine A.
B. Diphenhydramine
C. Promethazine
D. Suprastinum
E. Tavegil (Clemastine)
B.
10. Solutions of some electrolytes are
used as medications. What is the maxi-
mum value of the isotonic coefficient for
MgSO4 solution?
C.
A. 2
B. 4
C. 3
D. 5
E. 7 D.

11. A patient with hypertension has been


prescribed a drug that blocks angiotensin
receptors. Specify this drug:
E.
A. Losartan
B. Nifedipine
C. Prazosin
D. Captopril
E. Apressin 14. After taking phenacetin a pati-
ent developed acute sore throat, fever.
12. A patient with gastric ulcer has been Examination enabled doctors to make
administered omeprazole. What is the a diagnosis of necrotic angina and
mechanism of its action? agranulocytosis. Agranulocytosis can be
A. Inhibition of H + K + -ATPase characterized by a decrease in the amount
B. Blockade of histamine H2 -receptors of the following WBCs:
C. M-cholinergic receptor blockade A. Neutrophils
D. Neutralization of HCl B. Eosinophils
E. Stimulation of mucus production C. Basophils
13. Which of these formulas corresponds D. Lymphocytes
with acetoacetic acid? E. Monocytes
15. A patient with a heart rhythm disorder
has been given lidocaine. Apart from the
local anesthetic effect, this drug has the
following pharmacological effect:
A. Antiarrhythmic
B. Hypnotic
C. Antipyretic
D. Antidepressant
E. Nootropic
16. A patient with chronic constipation
has been prescribed bisacodyl. After 3
weeks of treatment, the patient noticed a
reduction of laxative effect. This is caused
by the development of the following side-
effect:
A. Habituation
B. Dependence
C. Sensibilization
D. Cumulation
E. Dysbacteriosis

126
krok123.in.ua
Krok 1 Pharmacy 2013 3

17. It is not advisable to use antacids and A. Proteins


iron supplements at the same time for the B. Lipids
following reason: C. Polysaccharides
D. Nucleic acids
A. Malabsorption of iron E. Disaccharides
B. Increased binding to plasma proteins
C. Impaired deposition of iron in the body 23. Analysis of a plant revealed essential-
D. Increased intoxication with iron oil glands with several layers of cells
E. Accelerated elimination of iron arranged in pairs. This allows for the
possibility that the plant relates to the
18. A patient with acute heart failure and family:
cardiac glycosides intolerance was given
an injection of dobutamine. What is the A. Asteraceae
mechanism of its action? B. Scrofulariaceae
C. Solanaceae
A. Stimulation of β1 -adrenoceptors D. Apiaceae
B. Stimulation of α1 -adrenoceptors E. Lamiaceae
C. Blockade of K + -, Na+ -ATPase
D. Inhibition of phosphodiesterase activity 24. In an emergency situation a scuba di-
E. Stimulation of M-cholinergic receptors ver has quickly risen from the depths to
the surface, which is against the rule. He
19. A female patient asked a pharmacist is unconscious, exhibits respiratory failure
to recommend her a drug for headache and cardiac activity disorder as a result of
with antiplatelet effect. Specify this drug: decompression sickness. What complicati-
on may develop in the scuba diver?
A. Acetylsalicylic acid
B. Codeine phosphate A. Gas embolism
C. Promedol B. Fat embolism
D. Tramadol C. Air embolism
E. Fentanyl D. Cellular embolism
E. Thromboembolism
20. A plant under examination has papi-
lionaceous flowers. This plant belongs to 25. A patient has been hospitalized for
the following family: chronic heart failure. Objectively: skin
and mucous membranes are cyanotic, the
A. Fabaceae patient has tachycardia, tachypnea. What
B. Scrofulariaceae type of hypoxia has developed in the pati-
C. Ranunculaceae ent?
D. Lamiaceae
E. Asteraceae A. Circulatory
B. Anemic
21. Microscopic analysis of a root revealed C. Hemic
the following features: primary structure, D. Tissue
endodermal cells with horseshoe-shaped E. Hypoxic
areas, radial fascicle of the central cyli-
nder, more than six xylem rays. Such 26. What are the indications for the use of
root structure is typical for the followi- naloxone?
ng plants:
A. Acute intoxication with narcotic
A. Angiosperms, monocotyledons analgesics
B. Angiosperms, dicotyledons B. Heavy metal intoxication
C. Gymnosperms, conifers C. Intoxication with cardiac glycosides
D. Gymnosperms, gnetalians D. Intoxication with ergot alkaloids
E. Pteridosperms E. Atropine sulfate intoxication
22. Chronic pancreatitis is accompanied 27. Amino group of p-aminobenzoic acid
by the decreased synthesis and secretion is involved into reaction with the followi-
of trypsin. This impairs the hydrolysis and ng reagent:
absorption of the following substances:

127
krok123.in.ua
Krok 1 Pharmacy 2013 4

A. HCl A. Blood pressure fall


B. NH4 OH B. Excitation of the central nervous system
C. NaOH C. Arrhythmia
D. CH3 COONa D. Rise of intraocular pressure
E. KCN E. Allergic skin rash
28. A patient with renal colic has been 34. A solution contains cations of zinc
administered a spasmolytic from the and aluminium. Specify the reagent that
group of M-cholinergic antagonists as a makes it possible to detect cations of zinc
part of the complex therapy. Specify this in this solution:
drug:
A. Potassium hexacyanoferrate (II) soluti-
A. Atropine on
B. Proserin B. Sodium hydroxide solution
C. Galantamine C. Cobalt nitrate Co(NO3 )2
D. Dithylinum D. Excess of 6M sodium hydroxide in
E. Benzohexonium presence of hydrogen peroxide
E. Sulfuric acid solution
29. What drug is administered in case of
uterine inertia? 35. A medicament comprises sodium bi-
carbonate and sodium chloride. What
A. Oxytocin method is used for quantitative determi-
B. No-spa nation of sodium bicarbonate?
C. Progesterone
D. Vikasolum A. Acid-base titration
E. Fenoterol B. Precipitation titration
C. Redox titration
30. A patient with myocardial infarction D. Complexometric titration
has been administered intravenously a di- E. Coulometric titration
rect anticoagulant, namely:
36. Concentration of magnesium sulfate
A. Heparin in a drug can be determined by
B. Neodicumarinum complexometric titration. Select an indi-
C. Vikasol cator for fixing the end point of titration:
D. Thrombin
E. Calcium gluconate A. Chromogen black
B. Phenolphtalein
31. What drug should be administered for C. Methyl orange
individual prevention of malaria? D. Diphenylcarbazone
E. -
A. Chingamin
B. Rifampicin 37. Select a pair of electrodes for potenti-
C. Ampicillin ometric pH measurement of a solution:
D. Gentamicin
E. Biseptol (Co-Trimoxazolel) A. Glass and silver chloride
B. Calomel and silver chloride
32. A sample section of an axial body C. Quinhydrone and antimonial
shows a complex consisting of phellogen D. Mercury sulphate and silver chloride
and its derivatives - cork and phelloderm. E. Glass and antimonial
This tissue is called:
38. The mass percentage of ascorbic acid
A. Periderm can be determined by the cerimetric
B. Colenchyma analysis in the presence of the following
C. Sclerenchyma redox indicator:
D. Epiblema
E. Epidermis A. Ferroin
B. Methylene red
33. A pharmacy dispenses glaucine C. Eosin
hydrochloride to a patient with chronic D. Fluorescein
bronchitis. The patient must be warned E. Methylene orange
about the following typical side effect of
the drug: 39. An injured person exhibits the followi-
ng signs at the site of trauma: skin
redness, throbbing small arteries, elevated
128
krok123.in.ua
Krok 1 Pharmacy 2013 5

local temperature, increased tissue turgor. has been found to include flagellated
What local blood circulation disorder are bacteria. What is the most likely mi-
these presentations typical for? croorganism that has been found by the
doctor?
A. Arterial hyperemia
B. Venous hyperemia A. Trihomonas vaginalis
C. Thrombosis B. Leishmania donovani
D. Embolism C. Trypanosoma gambiense
E. Ischemia D. Trihomonas hominis
E. Lamblia intestinalis
40. Microscopic examination of a
perennial stem revealed the secondary 46. A patient has been taking diclofenac
integumentary tissue that was formed as a sodium for a long time. The family physi-
result of cell division of: cian withdrew this drug and administered
celecoxib. What disease was the reason
A. Phellogen for the drug substitution?
B. Procambium
C. Cambium A. Peptic ulcer
D. Pericycle B. Bronchial asthma
E. Protoderma C. Urolithiasis
D. Arterial hypertension
41. What synthetic drug of the hydrazi- E. Chronic hepatitis
de group is typically prescribed for
pulmonary tuberculosis? 47. The rate of a chemical reaction DOES
NOT DEPEND on the concentration of
A. Isoniazid the reactants. Specify the order of such
B. Rifampicin reaction:
C. Acyclovir
D. Metronidazole A. Zeroth
E. Doxycycline hydrochloride B. First
C. Second
42. A female student with a cold has D. Third
been prescribed an antipyretic medicati- E. Fraction
on. Specify this drug:
48. A patient has been diagnosed with
A. Paracetamol bronchial asthma. Specify a medicament
B. Ascorbic acid that can be administered for asphyxiation:
C. Oxytocin
D. Famotidine A. Salbutamol
E. Cyanocobalamin B. Diclofenac sodium
C. Paracetamol
43. Specify the drug that constricts pupils D. Anapriline
and reduces intraocular pressure: E. Acetylcysteine
A. Pilocarpine hydrochloride 49. Sulfanilamides inhibit the growth and
B. Fenofibrate development of bacteria. The mechanism
C. Nitrazepamum of their action is based on the impairment
D. Atropine sulfate of the following acid synthesis:
E. Dithylinum
A. Folic
44. A plant under study has stipules fused B. Lipoic
together and thus forming a tight tube - C. Nicotinic
ochrea, that is a diagnostic feature of the D. Pantothenic
following family: E. Pangamic
A. Polygonaceae 50. Sulfanilamides are widely used as
B. Gramineae bacteriostatic agents. The mechanism of
C. Rosaceae antimicrobial action of sulfanilamides is
D. Papaveraceae based on their structural similarity to:
E. Clusiaceae
45. A 42-year-old female has foamy-
purulent vaginal discharges. The smear
stained by Romanovsky-Giemsa’s method
129
krok123.in.ua
Krok 1 Pharmacy 2013 6

A. Para-aminobenzoic acid 56. Some medications are colloidal soluti-


B. Glutamic acid ons. What size of the colloidal particles is
C. Folic acid typical for the colloidal dispersion?
D. Nucleic acid
E. Antibiotics A. 10−7 - 10−9 м
B. 10−5 - 10−7 м
51. Specify the standard solution for the C. 10−10 - 10−11 м
iodometric determination of reducing D. 10−5 - 10−3 м
agents (direct titration): E. > 10−3 м
A. I2 57. To maintain a certain level of pH-
B. KMnO4 medium, the buffer solutions are used.
C. Na2 S2 O3 Specify a composition of substances that
D. K2 Cr2 O7 DOES NOT EXHIBIT buffer properties:
E. KI
A. NaOH + NaCl
52. The fourth group of cations includes B. CH3 COOH + CH3 COONa
the cations Al3+ , Sn2+ , Sn(IV ), As(V ), C. NH4 Cl + NH3 ·H2 O
As(III), Zn2+ , C 3+ . The group reagent D. HCOOH + HCOONa
for the fourth group of cations is the E. NaH2 P O4 + Na2 HP O4
solution of:
58. What non-narcotic centrally-acting
A. NaOH, H2 O2 antitussive drug can be used for dry
B. HCl cough?
C. NH3 , H2 O2
D. H2 C2 O4 A. Glaucine
E. H2 SO4 , H2 O2 B. Codeine
C. Acetylcysteine
53. In a qualitative analysis, when an D. Ambroxol
excess of the group reagent (NH3 soluti- E. Mucaltinum
on) reacts with the cations of the si- 59. What analytical effect is observed after
xth analytical group (Cu2+ , Co2+ , Ni2+ , fixing the endpoint during the titration by
Cd2+ , Hg 2+ ), the following compounds Mohr method?
are formed:
A. Brick-red precipitate
A. Metal ammine complexes B. Red colour of solution
B. Metal hydroxides C. Yellow colour of solution
C. Basic metal salts D. White precipitate
D. Aqua complexes of metals E. Yellow precipitate
E. Hydroxocomplexes of metals
60. What solution can be determined
54. Sodium fluoride is one of the by photocolorimetric method by self-
components of the drugs used in the absorbance?
treatment of dental caries. NaF reacts wi-
th the following compound: A. Potassium chromate
B. Potassium chloride
A. H2 SO4 C. Potassium sulphate
B. CO2 D. Potassium nitrate
C. NaCl E. Potassium phosphate
D. KI
E. CH3 COOH 61. What substance in a solution can be
determined in two ways - by the polari-
55. The most common technology in metric or refractometric method?
pharmaceutical production is maintaining
constant temperature and pressure. What A. Ascorbic acid
is this process called? B. Sodium benzoate
C. Benzoic acid
A. Isobaric-isothermal D. Calcium gluconate
B. Isochoric-isothermal E. Magnesium sulfate
C. Isobaric
D. Isochoric 62. A patient with symptoms of
E. Isothermal chronic bronchitis has been administered
acetylcysteine. What is the mechanism of
130
krok123.in.ua
Krok 1 Pharmacy 2013 7

its expectorant action?


A. Depolymerization of sputum
mucopolysaccharides
B. Stimulation of adrenergic receptors
C. Inhibition of cough center A. Addition
D. Stimulation of respiratory center B. Substitution
E. Anesthesia of respiratory mucosa C. Reduction
63. During the gastric secretion, D. Oxidation
proteolytic enzymes are secreted in form E. Rearrangement
of zymogens. What enzyme is activated by 66. The combined use of furosemide with
hydrochloric acid? aminoglycoside antibiotics causes:
A. Pepsin A. Hearing impairment
B. Trypsin B. Increased blood pressure
C. Amylase C. Cramps
D. Lipase D. Hyperhidrosis
E. Chymotrypsin E. Increased intraocular pressure
64. Specify the reaction product of purine 67. A child exhibits physical and mental
reacting with sodium hydroxide: retardation. Urine analysis revealed high
concentration of orotic acid. This disease
can be addressed by the constant use of:
A. Uridine
B. Adenine
C. Guanine
A. D. Glutamine
E. Alanine
68. To distinguish between phenol and
salicylic acid, the following reagent is
used:
B.

C.
A. Sodium bicarbonate solution
B. Iron (III) chloride solution
C. Sodium hydroxide solution
D. Sodium chloride solution
E. Bromine solution
D. 69. Ethane is the product of the following
reaction:

E.

65. What class of reactions does this


reaction relate to?
131
krok123.in.ua
Krok 1 Pharmacy 2013 8

A. condition. Respiration is rapid, superfi-


H2 ,cat.
CH2 = CH2 −−−
o
−→ cial, with periods of apnea (Biot’s respi-
t ,p
ration). What was the main cause of the
B. development of periodic breathing in the
H O patient?
Al4 C3 −−2→
A. Inhibition of the respiratory center
C. function
F e,to B. Impaired function of spinal cord
CO + 2H2 −−−→ motoneurons
C. Impaired function of the neuromuscular
D. system
k.H2 SO4 ,to D. Diminished chest mobility
C2 H5 OH −−−−−−→
E. Pulmonary dysfunction
E. - 75. Specify the number of electrons
involved into formation of the isolated
70. What titrants are used for the titration conjugated system in the pyrimidine
in the non-aqueous medium? molecule:
A. Perchloric acid and sodium ethylate
B. Sulfuric acid and barium hydroxide
C. Hydrochloric acid and potassium
ethylate
D. Nitric acid and sodium hydroxide
E. Perchloric acid and barium hydroxide
A. 6
71. There are areas where humans or ani- B. 4
mals are exposed to the constant risk C. 10
of contracting certain types of bacteria. D. 2
What feature of these bacteria is responsi- E. 8
ble for their long viability in the soil?
76. In course of long-term treatment of
A. Spore formation an infectious patient with penicillin, the
B. Capsule formation pathogen transformed into the L-form.
C. Ability to multiply in the plant remains What changes occur in the pathogen cell
D. Thick cell wall in case of L-transformation?
E. Plasmids
A. Absence of a cell wall
72. What reagents are used to separate the B. Absence of flagella
cations of the IV analytical group from the C. Absence of a capsule
cations of the V and VI analytical groups D. Absence of a spore
in the analysis of their composition? E. Absence of inclusions
A. NaOH and H2 O2 77. Dosage forms produced as coarse di-
B. H2 SO4 spersion systems with the liquid dispersi-
C. Dithizone on medium and the solid phase are called:
D. NH4 OH
E. Na2 S A. Suspension
B. Emulsion
73. Specify the standard substance for the C. Aerosol
standardization of 0,1 M of hydrochloric D. Powder
acid solution: E. Foam
A. Sodium carbonate 78. Ethanol can be distinguished from
B. Ammonium hydroxide glycerol by the reaction with the following
C. Oxalic acid reagent:
D. Sodium chloride
E. Zinc sulphate
74. A patient with a diagnosis of drug poi-
soning has been admitted to the resusci-
tation department. The patient is in grave
132
krok123.in.ua
Krok 1 Pharmacy 2013 9

A. Cu(OH)2 A. Concentrated
B. HBr B. Diluted
C. F eCl3 C. Highly concentrated
D. KMnO4 D. W/O type
E. Ag2 O E. O/W type
79. A patient was found to have a 84. Bacteriological inspection of disi-
tumor of the pancreatic head, which is nfection quality at a pharmacy revealed
accompanied by the impaired patency a microorganism in an utility room (in the
of the common bile duct. Blood test wi- sink). The microorganism has the followi-
ll reveal an increase in the following ng properties: mobile nonspore-forming
substance level: gram-negative bacteria that form capsular
substance, grow well on ordinary nutrient
A. Bilirubin media, secrete the blue-green pigment.
B. Urea This microorganism is most likely to be
C. Hemoglobin of the following genus:
D. Insulin
E. Adrenaline A. Pseudomonas
B. Proteus
80. On the 2nd day after developing acute C. Clostridium
inflammation of the knee joint, the patient D. Shigella
exhibits the joint enlargement, swelling of E. Vibrio
the skin. At what stage of inflammation
are these signs typically observed? 85. Microbiological analysis of medicinal
raw materials revealed capsular bacteria.
A. Exudation What stain method was used to detect the
B. Alteration capsules?
C. Proliferation
D. Regeneration A. Gin’s
E. Sclerosis B. Ziehl-Neelsen’s
C. Neisser’s
81. Quite often, the soil may contain a D. Gram’s
number of pathogenic microorganisms. E. Ozheshko’s
The causative agents of the following di-
sease may stay viable in the soil for a long 86. At pH value 5,0 and isoelectric
time: point 4,0, the protein will migrate
toward the following electrode during
A. Anthrax electrophoresis:
B. Diphtheria
C. Viral hepatitis A. Anode
D. Pertussis B. Cathode
E. Dysentery C. Calomel
D. Silver chloride
82. As a result of an accident (snakebi- E. Platinum
te) a male patient has the following blood
values: Hb- 80 g/l, RBC- 3, 0 · 1012/l; WBC- 87. The intracellular metabolism of
5, 5·109/l. What type of anemia is observed glycerol starts with its activation. What
in this case? compound is formed as a result of the first
reaction of its conversion?
A. Hemolytic
B. Folic acid-deficiency A. Alpha-glycerolophosphate
C. Posthemorrhagic B. Pyruvate
D. Aplastic C. Lactate
E. Iron-deficiency D. Choline
E. Acetyl coenzyme A
83. Emulsions of 0,1 - 74% dispersed-
phase volume relate to: 88. A 40-year-old patient has a history of
bronchial asthma and bradyarrhythmia.
In order to eliminate bronchospasm, the
drugs of the following pharmacological
group should be administered:

133
krok123.in.ua
Krok 1 Pharmacy 2013 10

A. M-anticholinergics A. Hot water


B. β-adrenergic blocking agents B. Hydrochloric acid
C. M-cholinergic agents C. Alkalis
D. Anticholinesterase agents D. Ammonia solution
E. Muscle relaxants E. Sulfuric acid
89. Which representative of the Rosaseae 94. A student analyzes an axial plant
family has spring bloom in form of whi- organ characterized by radial symmetry,
te, fragrant flowers gathered in pendulous unlimited growth, positive geotropism. It
racemes at the ends of short shoots? provides nutrition, vegetative propagati-
on, anchorage of plant in the soil. This
A. Padus rasemosa (P.avia) organ should be identified as:
B. Potentilla erecta
C. Sorbus aucuparia A. Root
D. Cerasus vulgaris B. Stem
E. Crataegus sanquinea C. Leaf
D. Rhizome
90. A patient had been diagnosed wi- E. Seed
th right lung cancer and administered
surgical treatment. After right-sided 95. Cross section of a root conducting
pulmonectomy the patient developed evi- zone shows pericycle that gives rise to:
dent dyspnea. What form of respiratory
failure developed in this patient? A. Lateral roots
B. Trichomes
A. Pulmonary restrictive C. Adventitious roots
B. Central D. Root fibrilla
C. Peripheral E. Root cap
D. Pulmonary obstructive
E. Thoracodiaphragmal 96. Select a name that corresponds with
the formula: CH3 − C ≡ N:
91. A 22-year-old male was stung by bees,
the affected region became hyperemic and A. Acetic acid nitrile
edematous. What is the leading mechani- B. Acetamide
sm of edema development in this patient? C. Acetic anhydryde
D. Acetoxime
A. Increased permeability of the capillaries E. Ethyl isocyanide
B. Decreased hydrostatic blood pressure
in the capillaries 97. What data is required to measure the
C. Increased oncotic pressure of tissue activation energy?
fluid
D. Impaired lymphatic efflux A. Constants of reaction rate at two
E. Reduced oncotic pressure of blood temperatures
B. Thermal energy of the reaction
92. A patient has obstruction of the C. Energy change of the system
common bile duct. Which of these D. Internal energy of the system
substances is usually found in urine in E. Reaction order
such cases?
98. A patient with chronic renal fai-
A. Bilirubin lure exhibits azotemia, hypo- and
B. Ketone bodies isosthenuria. What is the main factor in
C. Uric acid the pathogenesis of these symptoms in the
D. Creatinine patient?
E. Glucose
A. Reduction of existing nephrons mass
93. A ground for separating lead (II) B. Increase in glomerular filtration rate
chloride from the other chlorides of the C. Reduction of tubular secretion
II analytical group (acid-base classificati- D. Disturbance of the permeability of the
on) is its different solubility in: glomerular membrane
E. Decrease in glomerular filtration rate in
each nephron
99. Which of the given compounds WILL
NOT decolorize the bromine water?
134
krok123.in.ua
Krok 1 Pharmacy 2013 11

A. CH3 − CH3 A. Renin-angiotensin


B. CH2 = CH2 B. Hypothalamic-pituitary
C. CH ≡ CH C. Kallikrein-kinin
D. CH3 − CH = CH2 D. Sympathoadrenal
E. E. Parasympathetic
104. The products of the toluene nitration
are mainly:

100. Chemically, ethers are quite inert


compounds. Ethers decompose even at
a room temperature under the effect of
the following haloid acid:
A. HI A.
B. HBr
C. HCl
D. HF
E. HClO
101. Specify the number of existing
stereoisomeric aldopentoses: B.

C.

A. 8
B. 2
C. 4
D. 6
E. 16 D.
102. This scheme of nitroalkane synthesis
is called the reaction of:

E.

A. Konovalov
B. Zinin
C. Kucherov 105. Halogen atoms in an organic
D. Tishchenko compound can be detected by means of:
E. Chichibabin
A. Beilstein test
103. A hospital admitted a patient wi- B. Molisch’s test
th arterial hypertension induced by renal C. Bayer’s test
artery stenosis. The patient complains of D. Lucas’ test
persistent nausea and headache. The main E. Iodoform test
element in the pathogenesis of hypertensi-
on is the activation of the following 106. Before a surgical operation, a
system: surgeon treated his hands with an alcohol-
containing solution. Which group of drugs
does this solution relate to?

135
krok123.in.ua
Krok 1 Pharmacy 2013 12

A. Antiseptics A. CH3 − CH2 − CH = CH − CH3


B. Disinfectants B. CH3 − CH2 − CH2 − CH2 − CH3
C. Sterilizing solutions C. CH3 − CH2 − CH2 − CH = CH2
D. Detergents D. CH3 − CH2 − CH2 − CH3
E. Surface-active substances E. CH3 − CH = CH − CH3
107. Aniline can be converted into the 110. Specify a compound having the most
water-soluble salt through the treatment pronounced basic properties in the gas
with a solution of: phase:
A. Hydrochloric acid A.
B. Sodium hydroxide
C. Sodium sulfate
D. Ethanol
E. Dimethylamine
108. Select the correct product of the B.
reaction:

C.

D. NH3
E.
A.

111. Specify the correct name for the


B. product of the acetaldehyde reacting with
hydrazine:

C.
A. Acetaldehyde hydrazone
B. Acetaldehyde oxime
C. Acetaldimine
D. Acetaldehyde phenylhydrazone
D. E. Acetaldehyde semicarbazone
112. Select a conjugated diene from the li-
st of diene hydrocarbons:
A. CH2 = CH  CH = CH2
B. CH2 = C = CH  CH3
C. CH2 = CH  CH2 CH = CH2
D.
E.

E. CH2 = CH  CH2 CH2  CH = CH2


113. Under the given conditions, the
109. Select the formula for pentene-2 from unsaturated organic compounds are
the list: reduced with the following reagent:

136
krok123.in.ua
Krok 1 Pharmacy 2013 13

A. H2 , Ni, t which may belong to:


B. HNO3 , p, t
C. NaOH, H2 O A. Lycopodiophyta
D. K2 Cr2 O7 , H + B. Equisetiphyta
E. H2 O, Hg 2+ , H + C. Bryophyta
D. Polypodiophyta
114. Which of the following compounds E. Pinophyta
relates to the conjugated dienes?
120. Many species of wild rose are a source
A. CH3  CH2  CH = CH  CH = CH2 of vitamins, fatty oils and herbal materi-
B. CH2 = C = CH  CH2 CH2  CH3 al. Specify the juicy pseudocarps that are
C. CH2 = CH  CH2  CH2 CH = CH2 procured as herbal raw material:
D. CH3  CH = C = CH  CH2 CH3
E. A. Rose hips
B. Coenobia
C. Hesperides
D. Aggregate-accessory fruits
E. Cenocarp stone-fruits
115. Which of the following solutions wi-
th the same molar concentration has the 121. Comparison of the underground
maximum osmotic pressure? organs of herbaceous plants revealed that
in the bipartite annuals the following
A. Aluminium nitrate organ prevails:
B. Glucose
C. Sodium chloride A. Main root system
D. Magnesium sulfate B. Adventitious root system
E. Potassium iodide C. Rhizome
D. Bulb
116. What class of organic compounds is E. Corm
characterized by the presence of C ≡ N
group? 122. Growth of some cancer cells is caused
by a certain growth factor. Treatment of
A. Nitriles leukemia involves applying an enzyme
B. Amines that destroys this essential factor. Speci-
C. Nitro compounds fy this enzyme:
D. Alcohols
E. Aldehydes A. Asparaginase
B. Glutaminase
117. If the amount of a high-molecular C. Succinate dehydrogenase
substance added to the sol is very small, it D. Citrate synthase
may not increase but decrease its stability. E. Aspartate aminotransferase
This phenomenon is called:
123. The anti-tumor preparation
A. Sensibilization Methotrexate is a structural analogue of
B. Solubilization folic acid. The mechanism of its action is
C. Mutual coagulation based on the inhibition of the following
D. Colloidal protection enzyme:
E. Sol habituation
A. Dihydrofolate reductase
118. Which medicinal plant of the B. Xanthine oxidase
Asteraceae family has only disk flowers C. Hexokinase
in the flowerhead? D. Creatine kinase
E. Lactate dehydrogenase
A. Three-part beggarticks (Bidens triparti-
ta) 124. A sample of a finished dosage form
B. Dandelion (Taraxacum officinale) was found to be contaminated with some
C. Echinacea purpurea microorganisms exhibiting the following
D. Cornflower (Centaurea cyanus) properties: greenish fluorescent colonies
E. Common yarrow (Achillea millefolium) of gram-negative nonsporeforming bacilli
that grew on the medium for the detection
119. Spore and pollen analysis revealed in of pyocyanin. The bacilli release the blue-
the pollen some tetrahedral spores with a green pigment into the medium. What mi-
semi-circular base and a reticular surface, croorganisms contaminated the finished
137
krok123.in.ua
Krok 1 Pharmacy 2013 14

dosage form? A. Bilabiate


B. Tubular
A. Pseudomonas aeruginosa C. Funnelform
B. Enterobacteriaceae D. Ligulate
C. Staphylococcus aureus E. Pseudoligulate
D. Staphylococcus epidermidis
E. Staphylococcus saprophyticus 130. Datura stramonium has dry many-
seeded fruits formed by syncarpous
125. Vaccines are the artificial or natural gynoecium that dehisce when the valves
preparations produced from bacteria, vi- are broken off. Specify the fruit type:
ruses and other microorganisms, their
chemical components and waste products. A. Capsule
They are used for the active immunization B. Follicle
of humans and animals for the prevention C. Siliqua
and treatment of infectious diseases. The D. Coenobium
attenuated vaccines consist of: E. Hesperidium
A. Viable microbes 131. A sample of water used in drug
B. Dead microbes production has been sent to a laboratory
C. Anatoxin for sanitary and virological analysis.
D. Dead microbes and toxoid Presence of what virus group will be indi-
E. Immunoglobulins cative of faecal contamination of water
and thus the need for its additional purifi-
126. What method of sterilization should cation?
be used during the manufacturing liquid
dosage forms containing proteins? A. Picornaviridae
B. Herpesviridae
A. Filtering C. Orthomyxoviridae
B. Boiling D. Retroviridae
C. Gas sterilization E. Flaviviridae
D. Autoclaving
E. Pasteurization 132. Specify the order of the reaction, for
which К=1/t (1/c - 1/c0):
127. Blood pressure is regulated by a
number of biologically active compounds. A. Second
What peptides that enter the bloodstream B. Third
can affect the vascular tone? C. First
D. Zeroth
A. Kinins E. Fractional
B. Leukotrienes
C. Enkephalins 133. After an insulin injection a 45-year-
D. Iodothyronines old female with a long history of di-
E. Endorphins abetes mellitus has developed weakness,
paleness, palpitation, anxiety, double visi-
128. Addison’s (bronze) disease is treated on, numbness of lips and the tip of tongue.
with glucocorticoids. Their effect is provi- Blood glucose is at the rate of 2,5 mmol/l.
ded by the potentiation of the following What complication has developed in the
process: patient?
A. Gluconeogenesis A. Hypoglycemic coma
B. Glycolysis B. Hyperosmolar coma
C. Pentose phosphate cycle C. Hyperglycemic coma
D. Glycogenolysis D. Hyperketonemic coma
E. Ornithine cycle E. Uremic coma
129. Representatives of Asteraceae fami- 134. A 45-year-old male patient was di-
ly have various types of flowers EXCEPT agnosed with stomach ulcer. After the
FOR: conservative treatment the pain and
heartburn disappeared, the function of
the gastrointestinal tract was normali-
zed. Endoscopic examination of stomach
revealed cicatrization of the ulcer. Qualify
this course of the disease:
138
krok123.in.ua
Krok 1 Pharmacy 2013 15

A. Lactase
A. Remission B. Maltase
B. Relapse C. Aldolase
C. Latent period D. Hexokinase
D. Recovery E. Glycosidase
E. Prodromal stage
140. A patient has developed megaloblastic
135. An older patient exhibits low levels anemia on a background of alcoholic ci-
of red blood cells and hemoglobin in rrhosis. The main cause of anemia in this
blood, but the color index is 1,3. Blood patient is the following vitamin deficiency:
smear analysis revealed megaloblasts.
What type of anemia is observed in this A. Folic acid
case? B. Lipoic acid
C. Biotin
A. B12 -folic acid deficiency D. Thiamin
B. Iron-deficiency E. Pantothenic acid
C. Acquired hemolytic
D. Hereditary hemolytic 141. The fruit of black locust is dry, formed
E. Chronic posthemorrhagic of a single carpel, dehisces by the ventral
and dorsal sutures on two sides, the seeds
136. Morphological analysis of poplar are attached along the ventral suture. Such
inflorescence showed that it is a simple fruit is called:
monopodial inflorescence: main axis is
drooping, the flowers are sessile, uni- A. Legume
sexual. Specify the type of inflorescence: B. Siliqua
C. Follicle
A. Catkin D. Capsule
B. Head E. Silicula
C. Capitulum
D. Cyme 142. At a chemical analytical laboratory, a
E. Panicle technician examines a solution of the VI
analytical group cations. After the additi-
137. During the morphologic analysis of on of ammonium thiocyanate and amyl
various plant leaves the students found alcohol, the organic layer turned blue.
the leaves, whose length of the leaf blade What cation is present in the solution?
is 5 times more than its width. Specify the
shape of the leaf blade: A. Co2+
B. Ni2+
A. Linear C. Cu2+
B. Elliptical D. Hg 2+
C. Lanceolate E. Cd2+
D. Ovoid
E. Reniform 143. Microscopic study of soybean seeds
stained with Sudan III revealed some
138. Food rich in carbohydrates at fi- droplets of various sizes. They are:
rst increases the blood sugar and then
decreases its rate due to the insulin acti- A. Lipids
on. What process is activated by this B. Proteins
hormone? C. Starch
D. Inulin
A. Synthesis of glycogen E. Glycogen
B. Gluconeogenesis
C. Breakdown of glycogen 144. An analytical chemist determi-
D. Breakdown of proteins nes sodium cations by ion-exchange
E. Breakdown of lipids chromatography. In order to prepare the
cation-exchange resin in the H + form, the
139. After drinking milk a 1-year-old chi- analyst uses:
ld developed diarrhea, flatulence. The
baby is likely to have the deficiency of A. HCl
the following enzyme: B. CH3 COOH
C. C2 H5 OH
D. H3 P O4
E. CH3 OH
139
krok123.in.ua
Krok 1 Pharmacy 2013 16

145. When studying the diagnostic A. Potassium permanganate, iron (II)


features of Origanum vulgare, the sulfate
students noticed that the plant had a B. Potassium dichromate, sodium thi-
compound monopodial inflorescence. It osulfate
is called: C. Potassium bromate, sodium thiosulfate
D. Potassium iodate, sodium thiosulfate
A. Corymbose panicle E. Cerium (IV) sulfate, iron (II) sulfate
B. Cluster of heads
C. Cincinnus 151. What indicator is used for fixing the
D. Bostyx endpoint of mercurimetric titration?
E. Head
A. Thiocyanate complexes of iron (III)
146. While preparing a solution, a B. Fluorescein
pharmaceutical analyst converted a C. Eosin
freshly formed precipitate into a sol by D. Murexide
treating it with an electrolyte soluti- E. Potassium chromate
on. What method of obtaining disperse
systems was used by the analyst? 152. Vitamin B1 deficiency has a negative
effect on a number of processes. This is
A. Peptization caused by the dysfunction of the followi-
B. Physical condensation ng enzyme:
C. Chemical condensation
D. Solvent exchange method A. Pyruvate dehydrogenase complex
E. Condensation from steam B. Aminotransferase
C. Succinate dehydrogenase
147. Sol of iron (III) hydroxide is positi- D. Glutamate
vely charged. Specify the ion which has E. Lactate dehydrogenase
the lowest coagulation threshold:
153. A patient with tuberculosis has been
A. SO42− prescribed some anti-TB preparations.
B. Cl− Which of the following chemotherapeutic
C. Cu2+ drugs has an effect on the tuberculosis
D. Na+ pathogen?
E. J −
A. Ftivazide
148. The method of treating people with B. Furacilinum
serious diseases and intoxications is based C. Methisazonum
on the absorption of toxic substances from D. Sulfadimezinum
the blood. What is this method called? E. Phtalazolum

A. Hemosorption 154. Many diseases of medicinal plants are


B. Electrophoresis caused by bacteria of the Pseudomonas
C. Hemadsorption genus. Select the bacteria relating to this
D. Dialysis genus:
E. Ultrafiltration
A. Blue pus bacillus
149. It is known that malonyl CoA is B. Colon bacillus
formed from acetyl CoA and carbon di- C. Proteus
oxide under the influence of acetyl CoA D. Mycoplasma
carboxylase. What vitamin is a coenzyme E. Micrococci
of this enzyme?
155. Microbiological studies of air in the
A. Biotin pharmacy room revealed the presence
B. Folic acid of pathogenic staphylococci. Select the
C. Pantothenic acid medium in which you can detect the
D. Ascorbate lecithinase activity of the isolated mi-
E. Thiamine croorganism:

150. Specify the standard solutions that A. Yolk-salt agar


are used in permanganatometry to quanti- B. Blood agar
fy the oxidants by the residual titration C. Bismuth sulfite agar
method: D. Sugar agar
E. Meat-extract agar
140
krok123.in.ua
Krok 1 Pharmacy 2013 17

156. Colloid silver preparations 161. To isolate the lead (II) chloride from
Protargolum and Collargolum are wi- the other cations of the II analytical group
dely used in medical practice as bacteri- in the systematic analysis, the chloride
cidal drugs. In addition to the active precipitate should be processed with:
ingredients, these drugs contain protein
compounds. What is the function of A. Hot water
proteins in these preparations? B. Ammonia solution
C. Nitric acid solution
A. Prevention of coagulation of the colloi- D. Acetate acid solution
dal solution E. Alkali solution
B. Prolongation of shelf-life
C. Reduction of the side effects 162. What reagent is used to separate
D. Improvement of the drug technology the cations of copper (II) and mercury
E. Potentiation of the bactericidal action from the other cations of the VI analytical
of silver group?
157. Therapeutic preparations for topi- A. Sodium thiosulfate
cal use (transdermal, vaginal, etc.) do not B. Sodium sulfate
require sterility. However, the total permi- C. Bromine water
ssible number of microbial cells and fungi D. Potassium sulfide
in 1 g (ml) of a drug should not exceed: E. Excess of the concentrated ammonia
solution
A. 100
B. 10 163. For cultivation of Brucella, pure
C. 500 cultures should be incubated in CO2 enri-
D. 1000 ched atmosphere. What type of breathing
E. 10 0000 is typical for Brucella?

158. P.Ehrlich is considered to be the A. Capnophilic


founder of modern chemotherapy. What B. Facultative anaerobic
chemotherapy drug was developed by this C. Obligate anaerobic
scientist? D. Obligate aerobic
E. Any
A. Salvarsan
B. Solusurminum 164. Some success in reducing malaria
C. Calomel transmission was achieved through the
D. Novarsenolum mass destruction of transmitting mosqui-
E. Osarsolum toes and their larvae. The measures aimed
at the destruction of insects are called:
159. A patient was admitted to a hospital
in a state of hypoglycemic coma. It occurs A. Disinfestation
at the following level of blood glucose: B. Disinfection
C. Deratization
A. 2,5 mmol/l or less D. Sterilization
B. 4,0 mmol/l E. Decontamination
C. 3,3 mmol/l
D. 4,5 mmol/l 165. On the photomicrograph of a
E. 5,5 mmol/l herbaceous plant stem the bicollateral
vascular bundles are clearly visible. The
160. A patient who had been continuously microspecimen represents the stem of the
treated with glucocorticoids was found to following plant:
have a duodenal ulcer. What mechanism
plays a major part in its development? A. Pumpkin
B. Rye
A. Increase of gastric juice secretion and C. Flax
acidity D. Corn
B. Acceleration of histamine inactivation E. Solomon’s seal
in the stomach
C. Inhibition of gastrin secretion in the 166. What type of conducting bundles is
stomach typical for all root zones of one-seeded
D. Excess production of prostaglandin E plants?
E. Hyperglycemia

141
krok123.in.ua
Krok 1 Pharmacy 2013 18

A. Radical is called:
B. Central phloem
C. Central xylem A. Capitulum
D. Bilateral B. Cyme
E. Collateral C. Raceme
D. Head
167. Antidepressants can increase the E. Calathidium
concentartion of catecholamines in the
synaptic cleft. What is the mechanism of 173. The technology of drug production
action of these drugs? widely uses the phenomena of absorption
and ion exchange. Which of the ions will
A. Inhibition of monoamine oxidase be selectively adsorbed on the surface of
B. Activation of monoamine oxidase a silver chloride crystal from an aqueous
C. Inhibition of xanthine oxidase solution?
D. Activation of acetylcholinesterase
E. Inhibition of acetylcholinesterase A. Ag +
B. Cu2+
168. The figwort family Scrophulariaceae C. NO3−
includes a biennial plant up to 1,5 m hi- D. H +
gh, with golden-yellow flowers gathered E. OH −
in spiked inflorescences. The flowers have
five stamens. Specify this plant: 174. A patient with acute myocarditis
exhibits rapid fatigability, shortness of
A. Verbascum flomoides breath, edemata of legs, hepatomegaly.
B. Digitalis purpurea Classify the type of heart failure by the
C. Digitalis grandiflora mechanism of its development:
D. Digitalis lanata
E. Digitalis Ferruginea A. Myocardial
B. Overload
169. Permanganometric titration of C. Compensated
hydrogen peroxide is carried out in the D. Subcompensated
following medium: E. Combined
A. Sulfate 175. A child with PKU has an unpleasant
B. Alkaline mouse-like odor, growth retardation,
C. Nitrate mental retardation.These symptoms are
D. Hydrochloric associated with the high concentration of
E. Alcohol the following substance in blood:
170. Quantitative analysis of zinc salts A. Phenylpyruvic acid
is performed by method of trilonometry. B. Glucose
What indicator is used for this purpose? C. Cholesterol
D. Adrenaline
A. Eriohrome black T E. Uric acid
B. Phenolphtalein
C. Methyl black 176. After a contact with a person havi-
D. Potassium dichromate ng an infectious diseases, the disease
E. Thymol blue pathogens entered the patient’s body and
started to multiply, but the symptoms of
171. In response to the administration the disease were not yet observable. What
of protein drugs, a patient developed an period of the disease is this typical for?
allergic reaction. The development of the
allergic reaction is caused by the increased A. Latent
synthesis of the following compound: B. Prodromal
C. Manifest illness stage
A. Histamine D. Clinical outcome
B. Choline E. Relapse
C. Adrenaline
D. Histidine 177. Extraction is commonly used
E. Serotonin in pharmacy for separating mixtures,
increasing the concentration of any solute
172. Astragalus dasyanthus has sessile and extracting lipophilic compounds from
flowers gathered into inflorescences with a the herbal material. This process is based
short thick axis. This kind of inflorescence
142
krok123.in.ua
Krok 1 Pharmacy 2013 19

on: A. Acetyl-CoA
B. Isocitrate
A. Nernst distribution law C. Lactate
B. Konovalov’s first law D. Malate
C. Dalton’s second law E. Ketoglutarate
D. Third law of thermodynamics
E. Hess’s Law 183. Allantoic fluid of a chicken embryo
contaminated with nasopharyngeal flush
178. The rate of extraction of a drug of a patient was found to contain a virus.
substance depends on the value of its di- What diagnostic agents should be used to
stribution coefficient. If the distributed identify it?
substance is characterized by different
rates of dissociation or association in di- A. Standard antiviral sera
fferent phases, the distribution coefficient B. Viral diagnosticums
is calculated by: C. Serum preparations
D. Diagnosticums produced of standard
A. Shilov-Lepin equation virus strains
B. Nernst distribution law E. Polyvalent immune diagnostic sera
C. Gibbs’ phase rule
D. The first Raoult’s law 184. A patient complains of severe
E. Van’t Hoff rule abdominal pain, cramps, blurred vision.
His relatives exhibit the same symptoms.
179. A male patient developed fever up The urine is of red colour. The patient has
to 40o C, there are vomiting, diarrhea, been hospitalized for acute intermittent
the patient is in grave condition. Blood porphyria. This disease might have been
osmolality is 270 mOsm/l. What disorder caused by the impaired synthesis of the
of water-salt metabolism is observed in following compound:
the patient?
A. Heme
A. Hypoosmolar hypohydration B. Insulin
B. Isoosmolar hypohydration C. Bile acids
C. Hyperosmolar hypohydration D. Prostaglandins
D. Isoosmolar hyperhydration E. Collagen
E. Hypoosmolar hyperhydration
185. Common nettle, hop, black
180. A 40-year-old male presented to elderberry relate to the plants that require
the endocrinology department with di- soils rich in nitrogen compounds, that is,
sproprortionate enlargement of limbs, such plants are called:
mandible and nose. These manifestations
are caused by the overproduction of the A. Nitrophytes
following hormone: B. Nitrophobes
C. Calciphiles
A. Somatostatin D. Calciphobes
B. Corticotropin E. Halophytes
C. Aldosterone
D. Adrenaline 186. A 40-year-old female farmworker
E. Vasopressin has been diagnosed with brucellosis and
administered causal chemotherapy. What
181. Surfactants are commonly used in group of drugs will be used for this
pharmaceutical production. What kind of purpose?
surfactant is potassium oleate?
A. Antibiotic
A. Anionic B. Donor immunoglobulin
B. Cationic C. Inactivated therapeutic vaccine
C. Nonionic D. Polyvalent bacteriophage
D. Amphoteric E. Antitoxic serum
E. None of the above
187. A male received a radiation dose of
182. Diabetes and starvation cause the 30 Gy. He presents with necrotic angi-
excess production of ketone bodies that na, disorders of the gastrointestinal tract.
are used as an energy source. They are Blood tests revealed anemia, leukopenia
produced from the following compound: and thrombocytopenia. What phase of
acute radiation syndrome is observed in
143
krok123.in.ua
Krok 1 Pharmacy 2013 20

the patient? A. Transferrin (siderophilin)


B. Transcobalamin
A. Manifest illness stage C. Haptoglobin
B. Primary reactions D. Ceruloplasmin
C. Latent E. Albumin
D. Outcome of disease
E. - 193. A laboratory received a food product
that had been taken from the focus of
188. As a result of hypothermia a food poisoning and presumably contained
male patient developed acute diffuse botulinum toxin. To identify the type of
glomerulonephritis. What type of allergic toxin, the neutralization reaction must be
reaction caused damage to the glomerular performed on white mice. What biological
capillaries in the patient? product is used in this reaction?
A. Immunocomplex A. Antitoxic serum
B. Anaphylactic B. Normal serum
C. Cytotoxic C. Antibacterial serum
D. Cell-mediated D. Diagnosticum
E. Stimulating E. Allergen
189. A patient with hyperproduction 194. For the specific prevention of
of thyroid hormones has been prescri- influenza, the employees of an enterpri-
bed Merkazolilum. This drug inhibits se were vaccinated with "Influvac". What
the following enzyme of iodothyronine type of immunity will develop in the body
synthesis: of the vaccinated?
A. Iodide peroxidase A. Artificial active
B. Aromatase B. Innate congenital
C. Reductase C. Artificial passive
D. Decarboxylase D. Natural active
E. Aminotransferase E. Natural passive
190. A patient with Parkinson’s disease 195. Suspension is a form of pharmaceuti-
exhibits low level of dopamine which cals used in medical practice. Which pair
is produced from dihydroxyphenylalani- of substances is able to form a suspension?
ne (DOPA). What enzyme catalyzes this
conversion? A. Water-clay
B. Water-oil
A. Decarboxylase C. Water-ethanol
B. Deaminase D. Ethanol-ethyl acetate
C. Hydrolase E. Ethanol-diethyl ether
D. Aminotransferase
E. Carboxypeptidase 196. In a sample studied under a
microscope the multilayer palisade
191. As a result of an emergency situation (columnar) parenchyma can be clearly
(shipwreck) a man had to drink sea (salty) seen. Such structure is typical for:
water. What form of water-salt imbalance
may occur in this case? A. Leaf
B. Root
A. Hyperosmolar hyperhydration C. Dicotyledon stem
B. Hypoosmolar hyperhydration D. Rhizomes of ferns
C. Hypotonic hyperhydration E. Adventitious roots
D. Isoosmolar hyperhydration
E. Isotonic hyperhydration 197. A patient exhibits small (petechial)
hemorrhages under the skin and mucous
192. Hemoglobin catabolism results in membranes, bleeding gums, tooth decay,
release of iron which is transported to general weakness, edemata of the lower
the bone marrow by a certain transfer extremities. What vitamin deficiency can
protein and used again for the synthesis of be suspected?
hemoglobin. Specify this transfer protein:

144
krok123.in.ua
Krok 1 Pharmacy 2013 21

A. C the representatives of pathogenic mi-


B. B1 croorganisms. Specify the diseases, whose
C. A causative agents may say viable in the soil
D. D for a long time:
E. E
A. Tetanus and gas anaerobic infection
198. A parturient woman diagnosed wi- B. Tuberculosis and mycobacterioses
th uterine inertia has been delivered to C. Colibacillosis and cholera
the maternity ward. The doctor gave her D. Leptospirosis and plague
an injection of the drug that activates E. Typhoid fever and dysentery
the contraction of smooth muscles of the
uterus. What hormone is a component of 200. The one-way penetration of solvent
this drug? molecules into the polymer phase resulti-
ng in complete dissolution of the polymer
A. Oxytocin is called:
B. Gastrin
C. Secretin A. Unlimited swelling
D. Angiotensin B. Limited swelling
E. Bradykinin C. Coacervation
D. Salting-out
199. Soil microflora often includes E. Thixotropy

145
krok123.in.ua
Кrok 1 Pharmacy (англомовнi студенти) 2014 рiк 1

1. Calculation of chemical reacti- detected in its maturation zone, where


ons thermal effects at pharmaceutical xylem and phloem areas interchange
production is based on the Hess law, radially. It can be concluded that this
stating that reaction thermal effect is bundle type is:
determined by:
A. Radial
A. Initial and final state of system B. Collateral
B. Mechanism by which the chemical C. Bicollateral
change occurs D. Amphicribal
C. Route by which the chemical change E. Amphivasal
occurs
D. Number of intermediate stages 6. In large intestine microorganisms
E. Process duration synthesize vitamins that participate
in organism’s biochemical processes.
2. Dobutamine has been administered What vitamins are mainly synthesized
to the 49-year-old-patient with acute by microflora?
cardiac failure and cardiac glycosi-
de intolerance. What is this drug’s A. K , B12
mechanism of action? B. A, C
C. E , P P
A. Stimulation of β1 -adrenoreceptors D. B1 , B2
B. Stimulation of α1 -adrenoreceptors E. B6 , E
C. Blockade of K + -, Na+ -
adenosinetriphosphatase 7. Parents of the 10-year-old child have
D. Suppression of phosphodiesterase made an appointment with endocri-
activity nologist due to complaints of child’s
E. Stimulation of M -cholinergic low height. The child’s appearance is
receptors corresponding with that of 5-year-old
child. What hormon secretion disorder
3. During calculous cholecystitis causes such physical development
attack the patient has developed the changes?
following symptoms: saponated feces
and steatorrhea. What stage of fats A. Somatotropic hormone
metabolism is disrupted according to B. Adrenocorticotropic hormone
those symptoms? C. Thyroxin
D. Testosterone
A. Fat digestion, absorption and E. Insulin
secretion
B. Fat absorption 8. The 55-year-old patient has been
C. Intermediary metabolism of fats hospitalised due to chronic cardiac
D. Fats metabolism in adipose tissue failure. Objectively: skin and mucosa
E. Depositing disruption are cyanotic, tachycardia, tachypnea.
What kind of hypoxia does the patient
4. If aromatic secretory-downy plant have?
has square in cross section stem, spike
inflorescence made up from whorled A. Circulatory
dichasia, bilabiate corolla and its fruit B. Anemic
consists of four nutlets, it probably C. Hemic
belongs to the following family: D. Tissue
E. Hypoxic
A. Lamiaceae
B. Scrofulariaceae 9. The patient has been hospitalised
C. Brassicaceae with pneumonia. What kind of respi-
D. Apiaceae ratory failure does the patient have?
E. Solanaceae
5. When root is studied under mi-
croscope, one leading bundle is
146
krok123.in.ua
Кrok 1 Pharmacy (англомовнi студенти) 2014 рiк 2

A. Restrictive A. Tetracytic
B. Obstructive B. Diacytic
C. Central C. Anisocytic
D. Peripheral D. Anomocytic
E. Thoracic diaphragm E. Paracytic
10. What naloxone indications are 15. Аrctostaphylos uva ursi, Vaccini-
there? um vitis ideae, Vaccinium myrtillus life
forms can be defined as:
A. Narcotic analgetics acute poisoning
B. Heavy metals poisoning A. Small shrub (fruticulus)
C. Cardiac glycosides poisoning B. Vine
D. Ergot alkaloids poisoning C. Grass
E. Atropine sulphate poisoning D. Shrub (frutex)
E. Subshrub (suffrutex, semifrutex)
11. During gastric secretory functi-
on research decrease of hydrochloric 16. In the course of plant cells
acid concentration in gastric juice was treatment with phloroglucinol wi-
detected. What enzyme will be less th concentrated sulfuric acid their
active in such a condition? cell walls became crimson-red, which
means:
A. Pepsin
B. Amylase A. Lignification
C. Lipase B. Suberization
D. Dipeptidase C. Mucification
E. Hexokinase D. Cutinization
E. Mineralization
12. What reagent does p-aminobenzoic
acid amino group react with? 17. When root was being studied under
microscope, root hairs were detected,
which are cell growths of:
A. Epiblema
B. Epidermis
A. HCl C. Endoderm
B. NH4 OH D. Exoderm
C. NaOH E. Mesoderm
D. CH3 COONa
18. Pharmacy sells glaucine hydrochlori-
E. KCN de to the patient with chronic bronchi-
13. The patient with hepatic colic tis. What common side effect should
has been prescribed spasmolytic of he be warned about?
muscarinic receptor antagonists group A. Decrease of arterial pressure
as a part of his complex therapy. What B. Excitation of central nervous system
drug is it? C. Disruption of cardiac rate
A. Atropine D. Increase of intraocular pressure
B. Proserin E. Allergic skin rashes
C. Galantamine 19. Potassium iodide solution has been
D. Dithylin added to the solution containing cati-
E. Benzohexonium
ons of the sixth analytical group (acid-
14. Microscopy of monocotyledon base classification). It resulted in red
leaf epidermis revealed that stomatal precipitate soluble in excess of reagent.
complex has four accessory cells. That What cations are present in the soluti-
means stomatal apparatus belongs to on?
the following type:

147
krok123.in.ua
Кrok 1 Pharmacy (англомовнi студенти) 2014 рiк 3

A. Mercury (II) ons. What is the group reagent for the


B. Nickel sixth group of cations?
C. Cobalt (II)
D. Bismuth A. Excess of NH3
E. Cadmium B. Solution of H2 SO4
C. Solution of NaOH
20. The Mohr method is used to defi- D. Excess of KOH solution
ne sodium chloride mass concentration E. Solution of HCl
in isotonic solution. Titration is carri-
ed out with the following indicator 26. Research of reaction rate
present: dependance from various factors
allows to intensify technological
A. Potassium chromate processes. What factor HAS NO effect
B. Fluorescein on reaction rate constant?
C. Ammonium iron (III) sulfate
D. Diphenylcarbazone A. Reacting agents concentration
E. Ferroin B. Temperature
C. Reagents nature
21. The student with cold has been D. Solvent nature
prescribed antipyretic medicine. Name E. Solid substance dispersion degree
this medicine.
27. What substance is surface-inactive
A. Paracetamol regarding water-air interface?
B. Ascorbic acid
C. Oxytocin A. Saccharose
D. Famotidine B. Acetic acid
E. Cyanocobalamin C. Ethanol
D. Methylamine
22. Name the drug that causes miosis E. Acetone
and lowers intraocular pressure.
28. Medicines used in treatment of
A. Pilocarpine hydrochloride dental caries contain sodium fluori-
B. Fenofibrate de. Which one of the compounds given
C. Nitrazepam below does NaF react with?
D. Atropine sulphate
E. Suxamethonium chloride A. H2 SO4
B. CO2
23. The patient with acute cardi- C. NaCl
ac failure has developed dyspnea, D. KI
tachycardia and cyanosis during physi- E. CH3 COOH
cal exertion. Name the type of hypoxia.
29. Epinephrine is used to prolong the
A. Circulatory effect of novocaine during infiltration
B. Respiratory anesthesia. What epinephrine action is
C. Hemic this effect caused by?
D. Hypoxic
E. Tissue A. Vasoconstriction
B. Potentiation of novocaine action at
24. Choose the surfactant out of the CNS level
substances listed below: C. Suppression of nerve endings and
conductors functioning
A. C2 H5 OH D. Vasodilatation
B. H2 O E. Suppression of tissue esterases
C. NaCl
D. HNO3 30. Potentiometric method of pH
E. K4 F e[(CN)6 ] measuring is used during pharmaceuti-
cal substances research. What
25. Cations Cu2+ , Co2+ , Ni2+ , Cd2+ , electrode can be used as indicator
Hg 2+ belong to the sixth group of cati- (working electrode) in solution pH
148
krok123.in.ua
Кrok 1 Pharmacy (англомовнi студенти) 2014 рiк 4

measuring? 34. Point out number of π -electrons in


benzol molecule.
A. Glass
B. Standard hydrogen A. 6
C. Zinc B. 2
D. Calomel C. 4
E. Silver-chlorine D. 5
E. Benzol has no π -electrons
31. What analytical effect is observed
when titration end point in the Volhard 35. Point out the product resulting
method is registered? from interaction of purine with sodi-
um hydroxide:
A. Red coloration of solution
B. Red precipitate
C. Yellow coloration of solution
D. Brown precipitate
E. Yellow precipitate
32. Information transfer from
peptide hormones to intracellular A.
second messengers occures involving
adenylate cyclase. What reaction is
catalyzed by adenylate cyclase?
A. Cyclic adenosine monophosphate B.
production
B. ATP breakdown into ADP and
inorganic phosphate
C. ATP synthesis from adenosine
monophosphate and pyrophosphate
D. ADP breakdown with adenosi- C.
ne monophosphate and inorganic
phosphate production
E. ATP breakdown into adenosine
monophosphate and pyrophosphate
33. Point out radical among active
intermediate particles given below. D.
A.
CH3 − ĊH2

B.
+ E.
CH3 − CH2

C.

CH3 − CH2
36. At the sixth month of pregnancy
D. the female patient has been diagnosed
− with severe iron-deficiency anemia. Di-
OH agnostic character was the appearance
of the following in blood:
E.
N̈H3

149
krok123.in.ua
Кrok 1 Pharmacy (англомовнi студенти) 2014 рiк 5

A. Hypochromic erythrocytes A. Anaphylactic shock


B. Macrocytes B. Traumatic shock
C. Megalocytes C. Cardiogenic shock
D. Reticulocytes D. Septic shock
E. Erythroblasts E. Burn shock
37. Botulism agent causes severe food 42. Virological laboratory has received
toxicoinfection. Point out the most patient’s nasopharyngeal lavage. What
characteristic morphologic feature of can be used to single out influenza vi-
botulism agent. rus from the patient’s lavage?
A. Gram-positive spore-forming bacilli A. Chick embryo
with subterminal spore B. Meat infusion agar
B. Thick gram-positive non-spore- C. Meat infusion broth
forming bacilli D. Endo’s medium
C. Gram-positive spore-forming bacilli E. Lowenstein–Jensen medium
with terminal spore
D. Thin mobile spore-forming bacilli 43. When hydrogen peroxide solution
with central spore is administered to bleeding wounds,
E. Thick gram-positive non-spore- it is broken up by one of the blood
forming non-filament-forming bacilli enzymes. Point out this enzyme.
38. What reagent is used to separate A. Catalase
AgCl precipitate from AgI precipitate? B. Monoamine oxidase
C. Cytochrome oxidase
A. Aqueous solution of ammonia D. Aspartate aminotransferase
B. Concentrated nitric acid E. Lactate dehydrogenase
C. Diluted nitric acid
D. Concentrated solution of potassium 44. Water solution of CaCl2 with
chloride 10% mass concentration is used for
E. Sulfuric acid solution intravenous injections. What is the
maximum value of CaCl2 isotonic
39. Nitrite ions can be detected in coefficient in water solution?
the presence of nitrate ions using the
following: A. 3
B. 4
A. Crystalline antipyrine in the C. 2
presence of diluted HCl D. 5
B. Crystalline sodium thiosulfate E. 1
C. Dimethylglyoxime
D. Crystalline iron (III) sulfate 45. What product results from propi-
E. Diphenylcarbazone onic aldehyde and P Cl5 interaction?

40. Choose the colloid surfactant out of


the substances listed below:
A. Potassium oleate
B. Iodine
C. Sodium chloride
D. Polyethylene
E. Gelatin
41. 1 minute after the patient had
been administered penicillin the pati-
ent’s arterial pressure sharply dropped,
pulse became thready, cold sweating
and clonic convulsions began. Name
this condition.
150
krok123.in.ua
Кrok 1 Pharmacy (англомовнi студенти) 2014 рiк 6

A. A. Processing precipitate with 30%


ammonium acetate solution
B. Precipitate recrystallization
C. Processing precipitate with
B. concentrated sulfate acid
D. Processing precipitate with acetate
acid solution
E. Processing precipitate with ammonia
solution
C.
49. In the process of conductometric
titration of HCl and CH3 COOH aci-
ds mixture 0,1 М solution of NaOH is
used to measure:
D. A. Electrical conduction in solution
B. pH of medium
C. Potential difference
D. Rotation angle of polarized light
E. H3 C − CH2 − CH2 − Cl plane
E. Refractive index
46. What qualitative reaction
proves linoleic acid nonsaturation? 50. Gas chromatography has been used
CH3 (CH2 )4 CH = CH − CH2 − CH = for ethanol quantitative determinati-
CH(CH2 )7 COH on. What parameter is measured?
A. Decoloration of bromine water A. Chromatographic peak height or
(Br2 ; H2 O) area
B. Hydrohalogenation (HCl) B. Retention time
C. Reaction with F eCl3 C. Retention volume
D. "Silver mirror"reaction with D. Chromatographic peak width
[Ag(NH3 )2 ]OH E. Chromatographic peak half-width
E. Decarboxylation
51. Pharmaceutical synthesis requires
47. Staphylococci grow well in ordinary studying complex reaction kinetics. If
media but inoculation of blood and the first stage product is the second
egg-yolk salt agar should be done to stage initial substance, then such reati-
separate pure bacterial cultures from on is called:
diseased tissue. What is the purpose of
those media? A. Consecutive
B. Inverse
A. To define disease-producing factor C. Concerted
B. To define tinctorial properties D. Second order
C. To study antigenic properties E. Parallel
D. To define bacterial mobility
E. To define antibiotic susceptibility 52. The following spore-forming
bacteria can be preserved in soil
48. How to separate P bSO4 from mi- over a long period of time: clostri-
xture of the 3rd analytical group cation dia of tetanus, botulism, anaerobic gas
sulphates in the process of systematic infection. Name the way with which
analysis? these microorganisms get into soil.
A. With feces
B. With urine
C. With water
D. With industrial waters
E. With expectoration
53. Meat infusion broth is prepared
151
krok123.in.ua
Кrok 1 Pharmacy (англомовнi студенти) 2014 рiк 7

for sterilization in bateriological A. Venous hyperemia


laboratory. What sterilization method B. Arterial hyperemia
is advisable? C. Neuroparalytic arterial hyperemia
D. Pulmonary artery thrombosis
A. Autoclaving E. Ischemia
B. Ignition
C. Boiling 57. Knee joint enlargement and
D. Filtering cutaneous edema has developed in
E. Dry heat the 46-year-old patient with acute
knee joint inflammation on the
54. Point out benzene structure: second day. What stage of inflammati-
on progressing are these symptoms
A. usually observed at?
A. Exudation
B. Alteration
C. Proliferation
B. D. Regeneration
E. Sclerosis
58. What data is necessary to calculate
activation energy of drug synthesis
C. reaction?
A. Reaction rate constant for two
temperatures
B. Thermal effect
D. C. Change of Gibbs energy of system
D. Internal energy of system
E. Reaction order
59. In the process of chemical soluti-
on preparation laboratory assistant’s
E. forearm was exposed to concentrated
hydrochloric acid. There are burni-
ng pain, hyperemia and swelling of
the damaged area. What pathologic
process are these symptoms evidenti-
55. What reagent action allows to disti- al of?
nguish ethanol from glycerine? A. Inflammation
B. Tumor
C. Embolism
D. Thrombosis
E. Lymphostasis
A. Cu(OH)2 60. Capsuliferous bacteria has been
B. HBr detected during microbiological
C. F eCl3 inspection of crude drugs. What
D. KMnO4 method of staining has been used to
E. Ag2 O detect capsules?
56. The patient with acute left ventri- A. Burri-Gins
cular failure has developed edema of B. Ziehl–Neelsen
lungs. What peripheral circulation di- C. Neisser
sorder taking place in the lungs has D. Gram
caused this complication? E. Aujeszky
61. Antibiotics prodused by fungi
152
krok123.in.ua
Кrok 1 Pharmacy (англомовнi студенти) 2014 рiк 8

belonging to Penicillium and Aspergi- A. To ensure full precipitation of all


llus genera are widely used in medici- cations of this group
ne. What class do these genera belong B. To further dissolve obtained precipi-
to? tate
C. For fractional precipitation of cati-
A. Ascomycetes ons
B. Basidiomycetes D. To change pH of medium
C. Zygomycetes E. To prevent complexing
D. Deuteromycetes
E. Chytridiomycetes 65. In 1915 Japanese scientists
Katsusaburo Yamagiwa and Koi-
62. The following parameter is used in chi Ichikawa became the first, who
thin-layer chromatography to identify induced experimental tumors, by pai-
pharmaceutical composition: nting ears of rabbits with coal tar. What
method of experimental tumor induci-
A. Rf ng did they use?
B. n
C. E , mV A. Chemical induction
D. I , A B. Transplantation
E. K C. Explantation
D. Cell-free filtrate induction
63. What reaction will produce sodium E. Radioisotope induction
nicotinate as a result?
66. The alleged diagnosis of the newly-
A. hospitalised in-patient is leukemia.
What symptom among those given
below is diagnostic character di-
fferentiating acute leukemia from
chronic leukemia?
B.
A. Leukemic hiatus
B. Significant increase of leucocytes
number
C. Leukosis rate
D. Eosinophil and basophil levels
C. E. Gumprecht’s shadows (smudge cells)
67. A student analyses plant organ with
radial symmetry, unlimited growth and
positive geotropism, which provides
nourishment, vegetative reproduction
D. and plant fastening in soil. This organ
is:
A. Root
B. Stem
E. C. Leaf
D. Rhizome
E. Seed
68. In root transverse section layi-
ng and formation from pericycle of
the following organs can be seen in
64. Why is ethyl alcohol used along maturation zone:
with the group reagent of the third
analytical group?

153
krok123.in.ua
Кrok 1 Pharmacy (англомовнi студенти) 2014 рiк 9

A. Lateral roots A. Picric acid


B. Trichome B. Valeric acid
C. Additional roots C. Tartaric acid
D. Root hairs D. Lactic acid
E. Root cap E. Malic acid
69. What chemical compound WILL 72. What reaction proves acid properti-
NOT decolorize bromine water? es of pyrrole?
A. CH3 − CH3 A.
B. CH2 = CH2
C. CH ≡ CH
D. CH3 − CH = CH2
E.
B.

70. γ -Butyrolactone is produced duri-


ng γ -hydroxybutyric acid heating. Poi- C.
nt it out among the compounds given
below:

D.

A.

E.

B.

C. 73. Products obtained from toluene ni-


triding will mostly consist of:

D.

E.

71. What compound has no carboxyl


group but nevertheless is called an
acid?

154
krok123.in.ua
Кrok 1 Pharmacy (англомовнi студенти) 2014 рiк 10

A. A.
2C2 H5 Cl+2Na → CH3 CH2 −CH2 CH3 +NaCl

B.
hv
B. C2 H6 + Cl2 −→ C2 H2 Cl + HCl

C.
C2 H5 OH + Cl → CH3 CH2 Cl + H2 O

C. D.
(HOH)
C2 H5 Cl + NaOH −−−−→

CH3 − CH2 − OH + NaCl

D. E.
(C2 H5 OH)
C2 H5 Cl + NaOH −−−−−−−→

H2 C = CH2 + H2 O + NaCl
E.

75. What products result from the


process of heating oxalic acid (HOOC −
COOH) with concentrated sulfate acid
(H2 SO4 )?
74. Which of the reactions given below
(concentr.)H2 SO4 ,to
is called the Wurtz reaction? HOOC − COOH −−−−−−−−−−−−→?

A. CO2 + CO + H2 O
B. 2CO2 + H2
C. H2 O + CO2
D. 2CO + H2 + O2
E. C2 H2 + 2O2
76. Point out the substance produced
during the following reaction:
HOH,Hg 2+
CH ≡ CH −−−−−−−→?

A. Ethanal
B. Ethanol
C. Propionaldehyde
D. Propanone
E. Acetate acid
77. When substances interact according
to the scheme given in
155
krok123.in.ua
Кrok 1 Pharmacy (англомовнi студенти) 2014 рiк 11

it results in the following:


A. Ethyl formate
B. Ethyl acetate A.
C. Methyl ethanoate
D. Methyl acetate
E. Methyl formate
78. Point out the compound that has B.
amphoteric properties, that is, reacts
both with acids and bases producing
relevant salts.
A.
C.

B.
D.

C.
E.

D.

81. Point out the compound that allows


to synthesize acetonitrile CH3 C ≡ N
E. in one stage.
A. CH3 I
B. CH4
C. CH3 OH
79. Point out the correct name of D. C2 H5 Cl
product resulting from interaction of E. C6 H5 Cl
acetaldehyde with hydrazine:
82. What reagent is used to perform
the following transformation:

A. Acetaldehyde hydrazone
B. Acetaldoxime
C. Acetaldimine A. NaNO2 (HCl)
D. Acetaldehyde phenylhydrazine B. K2 Cr2 O7
E. Acetaldehyde semicarbazone C. NaNO3 (H2 SO4 )
D. KOH
80. Point out the product of the reacti- E. Cu(OH)2
on given below:
83. Which of the alcohols given in
156
krok123.in.ua
Кrok 1 Pharmacy (англомовнi студенти) 2014 рiк 12

produces acetone during oxidation? A. Pinnatilobate


B. Trilobate
C. Pinnatipartite
D. Palmatilobate
E. Palmatipartite

A. 88. Catabolism of body’s own ti-


ssue proteins is intensified during
such diseases as thyrotoxicosis and
tuberculosis. This process is attended
B. by intensive synthesis in liver and
subsequent excretion with urine of the
following:
A. Urea
C. CH3 − CH2 − CH2 − OH B. Glucose
D. C2 H5 − OH C. Acetone bodies
E. CH3 OH D. Fatty acids
E. Nucleotides
84. What reagent will allow for
unsaturated organic compounds 89. Natural peptides can carry out vari-
reduction under the conditions given ous functions. What biologically active
below? peptide is one of the main antioxidants
and carries out coenzyme functions?
A. H2 , Ni, t
B. HNO3 , p, t A. Glutathione
C. NaOH , H2 O B. Bradykinin
D. K2 Cr2 O7 , H + C. Oxytocin
E. H2 O , Hg 2+ , H + D. Releasing hormone (Liberine)
E. Anserine
85. What reagent allows to si-
multaneously detect aldehyde group 90. The patient has been prescri-
and glycol fragment presence in bed oral drug to treat diarrhea.
glucose molecule? In accordance with WHO and
Pharmacopoeia demands 1 g (ml)
A. Cu(OH)2 of drug has to contain the following
B. Br2 number of microorganisms:
C. AlCl3
D. F eCl3 A. 1000 bacteria and 100 mold fungi
E. KMnO4 B. 100 bacteria and 10 mold fungi
C. 10 bacteria and no mold fungi
86. In spring birch and poplar buds are D. No bacteria and no mold fungi
gathered. They essentially are: E. 1000 bacteria and 200 mold fungi
A. Embryonic shoots 91. Microbiological purity of tableted
B. Sporophyte embryos drugs had been tested at factory.
C. Gametophyte embryos Samples cultivation in mannitol salt
D. Reduced sporophytes agar resulted in growth of golden-
E. Reduced gametophytes yellow colonies, microscopic exami-
87. Quercus robur leaves have the nation of colonies detected gram-
following type of lamina shape and di- positive globular bacteria positioned
vision: in clusters; microorganisms had plasma
coagulation prorerties. What pure
bacterial culture was obtained?

157
krok123.in.ua
Кrok 1 Pharmacy (англомовнi студенти) 2014 рiк 13

A. Staphylococcus aureus A. Tetanus serum


B. Enterobacteriaceae B. BCG vaccine
C. Staphylococcus epidermidis C. Tetanus anatoxin
D. Staphylococcus saprophyticus D. DPT vaccine
E. Pseudomonas aeruginosa E. -
92. Leaves damage by mosaic di- 97. Reaction rate constant numeri-
scoloration has been detected at medi- cally equals reaction rate, if molar
cinal plantations. What microorgani- concentrations of:
sms are the cause?
A. Reagents equal 1
A. Plant-pathogenic viruses B. Reagents differ by 1
B. Plant-pathogenic bacteria C. Products are identical
C. Plant-pathogenic fungi D. Products differ by 1
D. Protozoa E. -
E. Rickettsia
98. Tetanic spasms of skeletal muscles
93. Bacteria eventually become resi- occur under low calcium concentration
stant to antibacterial agents. What in blood. What endocrine disorder can
enables gram-positive bacteria’s resi- this condition be associated with?
stance to penicillin antibiotics?
A. Hypofunction of parathyroid glands
A. Beta-lactamases production B. Hyperfunction of adrenal cortex
B. Cell wall permeability C. Hypofunction of adrenal cortex
C. Active synthesis of peptidoglycane D. Hyperthyroidism
D. Active transport of antibiotics E. Hypothyroidism
E. Protein synthesis
99. Eicosanoids, - hormone-like
94. The following have been compounds, - are used to stimulate
detected in hand lavage of the ki- labor and for contraception. What
ndergarten chef: colibacilli, ray fungi, substances have such an effect?
staphylococci, bacilli, mold fungi.
What microbes are evidential of fecal A. Prostaglandines
contamination of hands? B. Interleukines
C. Endorphines
A. Colibacilli D. Angiotensines
B. Ray fungi E. Enkephalines
C. Staphylococci
D. Bacilli 100. When studying white mistletoe,
E. Mold fungi - perennial medicinal semiparasi-
te plant, - it was revealed that its
95. A person has been in contact with embryonic root buries into higher
influenza patient. What drug should plant stem tissue and reaches vascular
be administered for specific passive tissue system. This type of roots is
influenza prophylaxis? called:
A. Antigrippal immunoglobulin A. Haustorial roots
B. Vaccine influenza virus inactivated B. Photosynthetic roots
C. Leukocytic interferon C. Aerating roots
D. Amizon D. Contractile roots
E. Anaferon E. Aerial roots
96. Which one of the listed substances 101. It is known that cells of
causes formation of acquired artificial Chlorophyta division representati-
passive immunity? ves have chromatophores of various
shapes. We can observe ribbon-like
chromatophores in the species of the
following genus:
158
krok123.in.ua
Кrok 1 Pharmacy (англомовнi студенти) 2014 рiк 14

A. Spyrogyra 106. In pharmaceutical producti-


B. Volvox on oxyethylated derivatives of fatty
C. Сlorella acid esters (FAEs) are used, which
D. Chlamidomonas undergo colloid dissolution in suffi-
E. Spirulina ciently concentrated solutions. This
process is called:
102. During practical field session
students have detected plant wi- A. Solubilization
th diversity of leaves that differ B. Sensitization
by their placement on stem, parts C. Synergism
development, size, shape, lamina di- D. Colloid protection
vision. This phenomenon is called: E. Syneresis
A. Heterophylly 107. A newborn infant has hemolytic
B. Phyllotaxy jaundice caused by rhesus incompati-
C. Metamorphosis bility. What bile pigment will be
D. Leaf mosaic concentrated highest in the blood of
E. Venation this infant?
103. It is known that leaves of most A. Unconjugated bilirubin
gymnosperm species are represented B. Conjugated bilirubin
by needles. Which one of the speci- C. Urobilinogen
es listed below has macropodous D. Stercobilinogen
leathery leaves with solid flabellate E. Bile acids
lamina, dichotomous venation and one
or several notches along the upper 108. The patient with acute cardi-
margin? ac insufficiency has decreased urine
excretion caused by reduction of fi-
A. Ginkgo biloba ltering taking place in glomerules.
B. Cedrus libani What causes this drop in filtration?
C. Juniperus communis
D. Picea abies A. Decrease of arterial pressure
E. Abies sibirica B. Increase of hepatic blood flow
C. Exsiccosis
104. Leaves of Aesculus hi- D. Duct lumen obstruction
ppocastanum are composed of 5-7 E. Decrease in number of functioning
assidenous folioles that are oblong- glomerules
obovate shaped with dentate-serrated
margin, are attached to petiole (leaf 109. The 49-year-old female patient
rachis), and therefore are: suffering long-term from pancreatic
diabetes has developed the following
A. Palmately compound symptoms after administering insulin:
B. Pinnately compound weakness, facial pallor, palpitation,
C. Pinnatisected anxiety, double vision, numbness of
D. Palmatisected lips and tongue apex. Glucose molar
E. Palmatilobed concentration in blood was 2,5 mmol/l.
What complication has developed in
105. In potentiometric titration the the patient?
following indicator electrode is used
for chloride and borate acids quanti- A. Hypoglycemic coma
tative determination in their mixture: B. Hyperosmolar coma
C. Hyperglycemic coma
A. Glass D. Hyperketonemic coma
B. Silver-chlorine E. Uremic coma
C. Silver
D. Platinum 110. The 40-year-old patient has been
E. Calomel diagnosed with gastric ulcer, disease
symptoms making reappearance after
159
krok123.in.ua
Кrok 1 Pharmacy (англомовнi студенти) 2014 рiк 15

prolonged period of dormancy. How 115. Milk intake has resulted in the
can this kind of disease progression be one-year-old child having diarrhea and
qualified? abdominal distension. What enzyme
deficiency does the child have?
A. Relapse
B. Remission A. Lactase
C. Recovery B. Maltase
D. Latent period C. Aldolase
E. Prodromal stage D. Hexokinase
E. Glycosidase
111. The 55-year-old female pati-
ent has developed a case of acute 116. The 56-year-old patient has
pancreatitis caused by greasy food. developed megaloblastic anemia in the
What is the main pathogenesis step of course of alcoholic cirrhosis. What vi-
this disorder? tamin deficiency is the main cause of
anemia in this patient?
A. Premature activation af enzymes in
gland ducts and cells A. Folic acid
B. Pancreatic juice deficiency B. Lipoic acid
C. Low bile production in liver C. Biotin
D. Fats digestion disruption D. Thiamine
E. Acute bowel obstruction E. Pantothenic acid
112. As the result of taking herbal 117. During morphological analysis of
medicine the 30-year-old patient has lily-of-the-valley (Convallaria majalis)
developed anaphylactic allergic reacti- leaf it was noted that lamina has wide
on and blood leukocytosis. What kind elliptic shape and numerous veins are
of leukocytosis is characteristic of this parallel to leaf margin and merge only
case? at the leaf point. What is this venation
type called?
A. Eosinophilia
B. Monocytosis A. Arcuate
C. Lymphocytosis B. Parallel
D. Basophilia C. Palmate
E. Heutrophilia D. Pinnate-reticulate
E. Dichotomous
113. During morphological descripti-
on of common periwinkle it was defi- 118. In the process of silver cations
ned that it has shoot that trails on the identification reaction HCl and then
ground and takes root. It allows to ammonia solution have been added to
chatacterize such shoot as: the solution. What compound has been
produced as a result?
A. Creeping
B. Recumbent A. [Ag(NH3 )2 ]Cl
C. Twining B. [Ag2 (NH3 )3 ]Cl
D. Scandent C. AgOH
E. Tenent D. AgCl
E. [Ag(NH3 )3 ]Cl
114. L-DOPA and its derivatives
are used in treatment of Parki- 119. In chemico-analytical laboratory
nson’s disease. What aminoacid is this a specialist studies the mixture of the
substance made of? 5th analytical group cations. When thi-
ocyanate ions are added the solution
A. Tyrosine becomes red-colored. This analytical
B. Asparagine effect indicates presence of the followi-
C. Glutamate ng cation:
D. Tryptophan
E. Arginine
160
krok123.in.ua
Кrok 1 Pharmacy (англомовнi студенти) 2014 рiк 16

A. F e3+ A. Tetradynamous
B. F e2+ B. Didynamous
C. Mg 2+ C. Diadelphous
D. Bi3+ D. Monadelphous
E. Mn2+ E. Polydelphous
120. Dispensing chemist conducts 124. The dispensing chemist has been
quantitative determination of studying properties of certain disperse
pharmaceutical substance with system classes, namely, aerosols. What
restorative properties through direct optical phenomenon is characteristic
bromate titration. What solution is the of this disperse system class?
titrant?
A. Light scattering
A. Potassium bromate B. Light absorption
B. Iodine solution in potassium iodide C. Opalescence
C. Sodium thiosulfate D. Light reflection
D. Potassium iodide E. Light refraction
E. Chloride acid
125. Selective solvents are used in
121. In the chemico-analytical laboratories and factories to isolate
laboratory the dispensing chemist and refine essential oils, alkaloids,
studies solution of anion mixture. antibiotics and other pharmaceutical
When antipyrin solution is added it subctances. This process is called:
becomes emerald-green colored. This
analytical effect signifies presence of A. Extraction
the following anions: B. Sedimentation
C. Coagulation
A. Nitrite D. Flocculation
B. Nitrate E. Flotation
C. Acetate
D. Tartrate 126. When rare dosage forms are
E. Citrate produced, colloid surfactants are
added to increase certain components
122. In microbiology class students had solubility. What physicochemical
been growing pure bacterial culture. phenomenon is this process based on?
Bacterial inoculation of solid medium
was performed to obtain separate visi- A. Solubilization
ble colonies, resulting in two coloni- B. Coagulation
es, R-type and S-type, grown in C. Extraction
thermostat after one day of incubation. D. Diffusion
What microorganism properties were E. Sedimentation
described by students? 127. A pharmacist has been adding
A. Cultural small portions of electrolyte to silver
B. Tinctorial chloride sol, with resulting coagulati-
C. Biochemical on occuring under higher electrolyte
D. Morphologic concentration, if compared to single
E. Antigenic instance of adding electrolyte. This
phenomenon is called:
123. Androecium of Brassica oleracea
flower has six stamens, with four A. Sol acclimatization
stamens of inner circle longer than two B. Antagonism
stamens of outer circle. What is this C. Synergism
type of androecium called? D. Additivity
E. Desensitization
128. Ketoacidosis occurs during
starvation. What metabolite blood
concentration increase is symptomatic
161
krok123.in.ua
Кrok 1 Pharmacy (англомовнi студенти) 2014 рiк 17

of this medical condition? A. Ovicell


B. Synergides
A. Acetoacetate C. Antipodes
B. Oxaloacetate D. Nucellus
C. Malonate E. Chalaza
D. Beta-hydroxy-beta-methylglutaryl-
CoA 133. The dispensing chemist’s arteri-
E. Acetyl-CoA al pressure has increased (160/110 mm
Hg) due to his conducting long-term
129. The patient with myocardial analytical analysis (neurosis). What
infarction has been prescribed stati- neurohumoral regulation changes can
nes, cholesterol synthesis inhibitors, to cause increased arterial pressure in the
prevent complications. What enzyme given case?
activity is suppressed by these medici-
nes? A. Sympathoadrenal system activation
B. Activation of aldosterone producing
A. Beta-GHB-reductase and secretion
B. Hydroxylase C. Renin-angiotensin system activation
C. Lecithin-cholesterol acyltransferase D. Kallikrein-kinin system activation
D. Esterase E. Sympathoadrenal system inhibition
E. Oxygenase
134. The patient with alcoholic ci-
130. What compound is added along rrhosis complains of general weakness
with murexide indicator when detecti- and dyspnea. The following is
ng calcium cations in order to reach revealed: decrease of arterial pressure,
рН>12? ascites, dilation of stomach anterior
A. Sodium hydroxide wall superficial veins, esophageal vari-
B. Acetate buffer cose veins dilatation, splenomegaly.
C. Urotropin What haemodynamics disorder does
D. Ammoniac buffer the patient suffer from?
E. Ammonium hydroxide A. Portal hypertension
131. To make diaphoretic herbal B. Left ventricular failure
tea the following inflorescences are C. Right ventricular failure
used: 3-15 corymbose dichasia wi- D. Cardiac insufficiency
th light-yellow oblong wing-shaped E. Collapse
membranous recaulescent squamella 135. Pharmacy has received viricides.
that fuses halfway with floral axis. Choose the viricide used for influenza
Flowers are fragrant, yellowish. These treatment from the list given below.
inflorescences belong to:
A. Rimantadine
A. Tilia cordata B. Metisazone
B. Viburnum opulus C. Levamisole
C. Robinia pseudoacacia D. Azidothimidine
D. Mentha piperita E. Acyclovir
E. Padus avium
136. Infectious agents of various
132. S.H.Navashyn, Ukrainian scienti- ultrastructures can be etiological
st, was the first one, who described agents of infectious diseases. Which
double fertilization. In the process of of the groups named below HAS NO
fertilization first spermium conjugates cellular structure, protein synthesizing,
with central nucleus of embryo sac and enzyme and energy systems?
second spermium with:

162
krok123.in.ua
Кrok 1 Pharmacy (англомовнi студенти) 2014 рiк 18

A. Viruses
B. Fungi A. Type I pancreatic diabetes
C. Bacteria B. Type II pancreatic diabetes
D. Protozoa C. Diabetes insipidus
E. Rickettsia D. Steroidogenic diabetes
E. Glycogenosis
137. Emulsions are widely used in
pharmaceutical practice. What is the 142. The patient with mushroom poi-
process of spontaneous merging of soning has developed the following
drops in emulsions called? symptoms: yellow coloring of skin and
sclera, dark-colored urine. Hemolytic
A. Coalescence jaundice was diagnosed. What pigment
B. Flocculation causes such coloring of the patient’s
C. Sedimentation urine?
D. Flotation
E. Coagulation A. Stercobilin
B. Conjugated bilirubin
138. Streptomycin and other ami- C. Biliverdin
noglycosides by binding with 30S- D. Unconjugated bilirubin
subunit of ribosome prevents E. Verdohemoglobin
formylmethionyl-tRNA joining. What
process is disrupted due to this effect? 143. Phenomenon of decreasing system
volume resulting from polymer swelli-
A. Translation initiation ng is called:
B. Translation termination
C. Transcription initiation A. Contraction
D. Transcription termination B. Solvation
E. Replication initiation C. Sedimentation
D. Dissolution
139. Production of injections in E. Coagulation
pharmacies requires srtict control of
sterilization quality. What is placed 144. If the amount of high-molecular
in autoclave sterilization chamber to substance added to the given sol is
ensure proper control? extremely small, it is possible its stabi-
lity will decrease, instead of increase.
A. Ampoule with microbe spores What is this phenomenon called?
B. Ampoule with staphylococcus
culture A. Sensitization
C. Ampoule with colibacillus culture B. Solubilization
D. Ampoule with fungi spores C. Syneresis
E. Ampoule with viruses D. Sedimentation
E. Synergism
140. The patient has hypovitaminosis
PP. What amino acid taken with meals 145. Ammonia solution has been
partially compensates patient’s need added to the solution being studi-
for vitamin PP? ed. Black precipitate has formed.
That means the following cations are
A. Tryptophan present in the solution:
B. Phenylalanine
C. Valine A. Mercury (I)
D. Arginine B. Copper (II)
E. Methionine C. Iron (III)
D. Iron (II)
141. The 13-year-old female patient E. Silver (I)
having suffered from measles complai-
ns of dry mouth, thirst, body weight 146. Silver nitrate solution has been
loss, polyuria, her glucose concentrati- added to the solution containing ani-
on in blood is 16 mmol/l. What disease ons of the first analytical group. It
can be suspected?
163
krok123.in.ua
Кrok 1 Pharmacy (англомовнi студенти) 2014 рiк 19

resulted in yellow precipitate. That A. Umbel


means the following are present in the B. Corymb
solution: C. Raceme
D. Spike
A. Arsenite ions E. Head
B. Arsenate ions
C. Sulphate ions 151. Representatives of this divisi-
D. Iodide ions on propagate vegetatively by speci-
E. Bromide ions al formations: isidia, soredia, lobules.
Name this division.
147. The child has been hospitali-
sed with scalded skin syndrome. A. Lichenes
Staphylococcus aureus was detected in B. Basidiomycota
blisters. What virulence factor causes C. Equisetophyta
exfoliation and necrosis of epidermis? D. Lycopodiophyta
E. Polypodiophyta
A. Exfoliative toxin
B. Enterotoxin 152. The Volhard method is used
C. Hemolysin to define sodium chloride mass
D. Toxic shock syndrome toxin concentration. Name titrant of this
E. Hyaluronidase method.
148. During long-term carbon A. Ammonium thiocyanate
tetrachloride poisoning of animals si- B. Mercury (I) nitrate
gnificant activity drop of aminoacyl C. Sodium tetraborate
tRNA synthetase in hepatocytes was D. Mercury (II) nitrate
detected. What metabolic process is E. Sodium hydroxide
disrupted in this case?
153. Mass fraction of pharmaceuti-
A. Protein biosynthesis cal preparations that contain aromatic
B. DNA replication amino groups is defined through nitri-
C. RNA transcription te titration. What external indicator is
D. Post-translational modification of used in this case?
peptides
E. Post-transcriptional modification of A. Starch-iodide paper
RNA B. Methylene red
C. Eriochrome Black T
149. Nucleoproteins contain signifi- D. Phenolphthalein
cant amount of alkaline proteins. What E. Eosin
propteins carry out structural function
in chromatin? 154. The poultry factory worker, who
has been consuming 5 or more raw
A. Protamines and histones eggs daily, complains of weakness,
B. Albumines and globulines drowsiness, muscle pain, loss of hair,
C. Prolamines and glutenins seborrhea. What vitamin deficiency
D. Hemoglobin and myoglobin causes such condition?
E. Interferones and mucin
A. H (biotin)
150. Cherry (Prunus cerasus) B. C (ascorbic acid)
inflorescence has short floral axis C. A (retinol)
and approximately same length pedi- D. B1 (thiamine)
cles emerging from one point. It E. B2 (riboflavin)
is characteristic of the following
inflorescence organisation: 155. The patient has icteritous skin;
unconjugated bilirubin content in
blood is high; conjugated bilirubin
in urine is not detected. There is si-
gnificant amount of urobilin in uri-
164
krok123.in.ua
Кrok 1 Pharmacy (англомовнi студенти) 2014 рiк 20

ne and stercobilin in feces. Name A. Reaction of peroxochromate acid


the pathology characterized by given formation after previous chromium
symptoms. oxidation
B. Apply sodium hydroxide
A. Hemolytic jaundice C. Apply potassium permanganate
B. Obstructive jaundice D. Apply ammonia
C. Jaundice of the newborn E. Apply sodium hydroxide and
D. Hepatocellular jaundice hydrogen peroxide
E. Atherosclerosis
160. When determining drug expi-
156. Having been studied, conifer ration date, it is supposed that drug
wood is determined to be composed decomposition reaction is reaction of
of cells with pointed ends and lignified the following order:
ring-porous cell wall. Therefore, this ti-
ssue of conifers is represented only by: A. First
B. Fractional
A. Tracheids C. Third
B. Vessels D. Zero
C. Sieve tubes E. -
D. Companion cells
E. Bast fibers 161. Identical analytical effect is
observed when NO3− and NO2− ions
157. There is a suspicion of active interact with:
tuberculosis development in patient.
The doctor has appointed Mantoux A. Diphenylamine and concentrated
test to make a diagnosis. What H2 SO4
immunobiological agent has to be B. Solution of KMnO4
administered? C. Solution of I2 у KI
D. Solution of AgNO3
A. Tuberculine E. Solution of BaCl2
B. BCG vaccine
C. DPT vaccine 162. What anion of the 2nd analytic
D. Tularin test group produces black precipitate with
E. DT vaccine group reagent AgNO3 ?
158. At the sixth day since illness onset A. S 2−
the patient has been diagnosed with B. I −
leptospirosis. What sample should be C. Cl−
taken for microbiological study of di- D. Br−
sease? E. NCS −
A. Blood and cerebrospinal fluid 163. When studying five herbarium
B. Nasopharyngeal lavage specimen of medicinal plants, it was
C. Ulcer secretion determined that one of them belongs
D. Lymph node puncture sample to Fabaceae family. Which one is it?
E. Urine and feces
A. Ononis arvensis
159. Chemist-analyst should use the B. Atropa belladonna
following reaction to detect chromium C. Hyoscyamus niger
(III) ions during preliminary tests: D. Datura stramonium
E. Solanum dulcamara
164. You are studying the silvery
downy plant of Asteraceae family, whi-
ch is rich with essential oils and bitters.
Harvested are apical sprouts with pani-
cle of small round flower heads. This
plant is:
165
krok123.in.ua
Кrok 1 Pharmacy (англомовнi студенти) 2014 рiк 21

patient with pancreatic diabetes. What


A. Artemisia absinthium process intensification has caused
B. Arctium lappa lenticular opacity?
C. Bidens tripartita
D. Calendula officinalis A. Protein glycosylation
E. Chamomilla recutita B. Lipolysis
C. Ketogenesis
165. Oligomycin antibiotic is prescri- D. Protein proteolysis
bed to the patient with tuberculosis. E. Gluconeogenesis
What mitochondrial process is slowed
down by this medicine? 170. Stinging nettle (Urtica dioica), hop
(Humulus lupulus) and common elder
A. Oxidative phosphorylation (Sambucus nigra) are plants that requi-
B. Substrate-linked phosphorylation re high nitrogen content in soil, which
C. Microsomal oxidation means that they are:
D. Lipid peroxidation
E. Oxidative decarboxylation A. Nitrophilous
B. Nitrophobous
166. Group reagent of the second C. Calciphilous
analytical group anions (Cl− ,Br− ,I − , D. Calciphobous
S 2− ) is the solution of AgNO3 with the E. Halophytic
addition of the following substance:
171. Fluorography examination of the
A. Nitric acid 59-year-old patient has revealed well-
B. Hydrobromic acid defined shadow, which is characteri-
C. Acetic acid stic to tumor, in the lower part of the
D. Hydrochloric acid left lung. What trait is characteristic of
E. Hydrosulphuric acid benign tumor?
167. Emulsions containing less then A. Expansive growth
0,1% of dispersed phase (in volume) B. Metastasis
are classified as: C. Cancer cachexia
D. Invasion in surrounding tissues
A. Diluted E. Infiltrating growth
B. Concentrated
C. High-concentration 172. The patient has been prescri-
D. Water-in-oil type bed drug with antibacterial effect on
E. Oil-in-water type tuberculosis mycobacteria. What drug
is used in tuberculosis treatment and is
168. The patient has been admi- pyridoxine antivitamin?
tted to the hospital with complai-
nts of general fatigue, headache, A. Isoniazid
lumbago, edema of face and extremi- B. Heparin
ties. Urine analysis revealed protei- C. Trimethoprim/sulfamethoxazole
nuria, hematuria and cylindruria. What (Co-trimoxazole)
is the main pathogenetic mechani- D. Streptomycin
sm of edema formation during E. Sulfanilamide
glomerulonephritis?
173. Detoxication rate is 4 times
A. Decrease of oncotic blood pressure lower in children than in adults. What
B. Increase of vascular permeability enzyme necessary for toxic compounds
C. Increase of hydrodynamic blood conjugation has low activity in chi-
pressure ldren?
D. Hormonal disbalance
E. Lymph flow disruption
169. Cataract (lenticular opacity) has
developed in the 52-year-old female
166
krok123.in.ua
Кrok 1 Pharmacy (англомовнi студенти) 2014 рiк 22

A. Glucuronosyltransferase colloidal systems and high-molecular


B. ALAT compound solutions by semi-
C. AspAT permeable membrane diffusion is
D. Creatine phosphokinase called:
E. LDH1
A. Dialysis
174. Certain drugs can stimulate liver B. Electrodialysis
to synthesize enzyme systems taking C. Ultrafiltration
part in drugs and toxines metaboli- D. Decantation
sm. What compound stimulates drug E. Compensatory dialysis
metabolism in liver microsomes?
179. What particles of the micelle
A. Phenobarbital described by the following formula:
B. Heparin {m(AgCl) nAg + (n − x) NO3− }x+ xNO3−
C. Menadione sodium bisulfite are situated in diffusion layer?
D. Sulfanilamide
E. Aspirin A. NO3−
B. AgCl
175. Barbiturates are used as sopori- C. Ag +
fics. These substances, similarly to D. AgCl and Ag +
rotenone, are tissue respiration inhi- E. Ag + and NO3−
bitors. What complex level do these
compounds suppress respiratory chain 180. The 32-year-old patient has been
at? taking antituberculosis drugs. Later he
noticed that his urine had become red-
A. NADH-coenzyme Q reductase orange in color. What drug is conducti-
B. Cytochrome oxidase ve to this phenomenon?
C. Cytochrome C reductase
D. Adenosine triphosphate synthetase A. Rifampicin
E. Succinate dehydrogenase B. Isoniazid
C. Pyrazinamide
176. Inhibitors of one of the amides D. Ethambutol
metabolism enzymes are used to treat E. Streptomycin sulphate
depression. What enzyme inhibition
has such an effect? 181. The 33-year-old female patient,
who undergoes long-term treatment
A. Flavin adenine dinucleotide (FAD)- due to her chronic polyarthri-
containing monoamine oxidase (MAO) tis, complains of increased arterial
B. Acetylcholinesterase pressure, adipose tissue redistributi-
C. Formylkynureninase (Arylformami- on and menstrual irregularities. What
dase) drug does the patient take?
D. Kynurenine 3-hydroxylase
E. Lactate dehydrogenase A. Prednisolone
B. Indometacin
177. Pathogenic microorganisms C. Phenylbutazone
produce various enzymes in order to D. Fluocinolone acetonide
penetrate body tissues and spread E. Diclofenac sodium
there. Point out these enzymes among
those named below. 182. Due to prolonged taking of
phenobarbital the epileptic patient
A. Hyaluronidase, lecithinase has developed tolerance for this drug.
B. Lyase, ligase What is this phenomenon based on?
C. Transferase, nuclease
D. Oxydase, catalase A. Biotransformation acceleration
E. Esterase, protease B. Absorption process weakening
C. Increase of receptor sensitivity
178. Method consisting in removal D. Biotransformation suppression
of low-molecular impurities from E. Substance accumulation in body
167
krok123.in.ua
Кrok 1 Pharmacy (англомовнi студенти) 2014 рiк 23

183. What side effect is characteristic A. Doxycycline


of captopril? B. Paracetamol
C. Ascorbic acid
A. Dry cough D. Acetylcysteine
B. Increase of arterial pressure E. Codeine phosphate
C. Hyperglycemia
D. Cardiac rate disorder 189. What drug is more advisable for
E. Hypokaliemia the patient with amebic dysentery?
184. The patient with acute poisoning A. Metronidazole
needs forced diuresis. What drug can B. Pyrantel
be used for this purpose? C. Levamisole
D. Bicillin-5
A. Furosemide E. Benzylpenicillin sodium salt (Penici-
B. Caffeine and sodium benzoate llin G sodium salt)
C. Galantamine hydrobromide
D. Enalapril 190. The student asks the pharmacist
E. Piracetam to recommend him the drug to relieve
allergic rhinitis symptoms he suffers
185. The patient with parkinsonism has from when lime tree is in bloom. What
been prescribed a drug - dopamine drug can be recommended in this case?
precursor - to relieve muscular rigidity.
Name this drug. A. Loratadine
B. Epinephrine
A. Levodopa C. Propranolol
B. Aminazine D. Ambroxol
C. Paracetamol E. Losartan
D. Scopolamine hydrobromide
E. Atropine sulphate 191. The patient with rheumatoid
arthritis and concomitant duodenal
186. The patient with neurosis has ulcer has to be prescribed nonsteroid
been prescribed anxiolytic derivative antiinflammatory drug. Which one of
of benzodiazepine. Name this drug. the drugs listed below is a drug of choi-
ce in the given case?
A. Diazepam
B. Atropine sulphate A. Celecoxib
C. Piroxicam B. Acetylsalicylic acid
D. Nandrolone C. Paracetamol
E. Trihexyphenidyl D. Metamizole
E. Diclofenac sodium
187. The elderly patient suffers from
constipation caused by large intesti- 192. Which of the drugs listed below
ne hypotonia. What drug should be quickly arrests angina pectoris attack
prescribed? when taken sublingually?
A. Bisacodyl A. Nitroglycerine
B. Sodium sulfate B. Digoxin
C. Castor oil C. Amiodarone
D. Atropine sulphate D. Lisinopril
E. Procainamide E. Convallariae glycoside
188. In the course of bronchi- 193. What chemotherapeutic agent is a
tis pharmacotherapy, the patient drug of choice for treatment of herpes?
has developed dyspeptic disorders,
photodermatitis and hepatic failure.
What drug can cause such disorders?

168
krok123.in.ua
Кrok 1 Pharmacy (англомовнi студенти) 2014 рiк 24

A. Acyclovir for acute cardiac failure treatment.


B. Rifampicin
C. Doxycycline hydrochloride A. Intravenous
D. Chingamin B. Intramuscular
E. Metronidazole C. Subcutaneous
D. Internal
194. What local anesthetic is given E. Inhalational
to patients with cardiac rhythm di-
sturbance? 198. Which one of the drugs listed
below is potentially addictive?
A. Lidocaine
B. Paracetamol A. Trimeperidine
C. Morphine hydrochloride B. Acetylsalicylic acid
D. Caffeine and sodium benzoate C. Naloxone
E. Nitrazepam D. Diclofenac sodium
E. Paracetamol
195. The patient with bronchial asthma
had been prescribed salbutamol, whi- 199. The 55-year-old patient had
ch led to disappearance of bronchi- been diagnosed with angina pectoris.
ospasm symptoms. It happened due to Calcium channel-blocking agent was
stimulation of: prescribed for treatment. Name this
agent.
A. β2 -adrenoreceptors
B. α1 -adrenoreceptors A. Amlodipine
C. Muscarinic acetylcholine receptors B. Atenolol
D. Acetylcholine synthesis C. Guanethidine
E. β1 -adrenoreceptors D. Reserpine
E. Labetalol
196. What drug is advisable for indivi-
dual malaria prophylaxis? 200. The patient has mucosal dryness
and mesopic vision disorder. What
A. Chingamin vitamin deficiency causes these
B. Rifampicin symptons?
C. Ampicillin
D. Gentamicin A. A
E. Trimethoprim/sulfamethoxazole B. P
(Co-trimoxazole) C. E
D. C
197. Choose the most efficient way of E. D
convallariae glycoside administration

169
krok123.in.ua
Krok 1 Pharmacy 2015 1

1. Calculation of thermal effects of below are all of the same molarity, the hi-
chemical reactions at pharmaceutical ghest boiling temperature will belong to:
production is based on the Hess law stati-
ng, that reaction thermal effect is determi- A. Sodium sulfate
ned by: B. Promedol (Trimeperidine)
C. Nicotinamide
A. Initial and final state of system D. Resorcin
B. Mechanism by which the chemical E. Iodine
change occurs
C. Route by which the chemical change 7. Which of these formulas corresponds wi-
occurs th acetoacetic acid?
D. Number of intermediate stages
E. Process duration
A.
2. A victim of a traffic accident is hospi-
talized at a resuscitation unit. Objectively:
the patient is unconscious, BP is 90/60 mm
Hg, high blood content of creatinine and B.
urea is observed, diurnal diuresis is 80 ml.
Characterize the patient’s diurnal diuresis:
C.
A. Anuria
B. Oliguria
C. Polyuria
D. Pollakiuria D.
E. Nocturia
3. In snake venom there is a substance
that causes erythrocyte hemolysis when E.
it is introduced into a human organi-
sm. Blood test revealed a large amount 8. Universal system of biological oxidati-
of lysolecithin (lysophosphatidylcholi- on of nonpolar compounds (numerous
ne). What enzyme leads to accumulating drugs, toxic agents, steroid hormones,
lysolecithin in blood? cholesterol) is microsomal oxidation.
A. Phospholipase A2 Name the cytochrome that is included in
B. Phospholipase A1 oxygenase chain of microsomes:
C. Phospholipase C A. Cytochrome Р-450
D. Phospholipase D B. Cytochrome C
E. Neuraminidase C. Cytochrome A3
4. Microbe survial in environment is faci- D. Cytochrome A
litated by spore formation. What mi- E. Cytochrome C1
croorganisms of those listed below are 9. A patient with chronic constipation had
spore formers: been prescribed bisacodyl. After 3 weeks
A. Clostridium of treatment, the patient noticed a reducti-
B. Bacteroides on of laxative effect. This is caused by the
C. Staphylococcus development of the following side-effect:
D. Peptococcus A. Tolerance
E. Peptostreptococcus B. Dependence
5. Bacteria rapidly become resistant C. Sensibilization
to drugs in the course of antibacterial D. Cumulation
treatment. What structural components of E. Dysbacteriosis
bacteria provide for their resistance? 10. A newborn child born from Rh-
A. R-plasmids negative mother in the result of her thi-
B. Spores rd pregnancy presents with gradually
C. Capsule worsening jaundice, irritated central
D. Flagella nervous system, anemia. What type of
E. Volutine granules jaundice does the infant suffer from?

6. If aqueous solutions of the drugs listed


170
krok123.in.ua
Krok 1 Pharmacy 2015 2

A. Hemolytic A. Open, heterogeneous


B. Hepatocellular B. Isolated, heterogeneous
C. Obstructive C. Closed, homogeneous
D. Parasitic D. Closed, heterogeneous
E. Toxic E. Open, homogeneous
11. Structure of proteins includes protei- 16. What drug should be administered for
nogenic amino acids. What is the position individual prevention of malaria?
of the amino group in the structure of these
amino acids? A. Chingamin
B. Rifampicin
A. α-position C. Ampicillin
B. β -position D. Gentamicin
C. γ -position E. Biseptol (Co-Trimoxazolel)
D. δ -position
E. -position 17. A sample section of an axial body shows
a complex consisting of phellogen and its
12. In an emergency situation a scuba di- derivatives - cork and phelloderm. This ti-
ver has quickly risen from the depths to ssue is called:
the surface, thus breaking safety rules. He
is unconscious, presents with respiratory A. Periderm
failure and cardiac activity disorder as the B. Colenchyma
result of decompression sickness. What C. Sclerenchyma
complication may develop in the scuba di- D. Epiblema
ver? E. Epidermis

A. Gas embolism 18. When root was being studied under mi-
B. Fat embolism croscope, root hairs were detected, which
C. Air embolism are cell growths of:
D. Cellular embolism
E. Thromboembolism A. Epiblema
B. Epidermis
13. The patient has been hospitalised with C. Endoderm
pneumonia. What kind of respiratory fai- D. Exoderm
lure does the patient have? E. Mesoderm

A. Restrictive 19. The Mohr method is used to determi-


B. Obstructive ne sodium chloride mass concentration in
C. Central isotonic solution. Titration is carried out
D. Peripheral with the following indicator present:
E. Thoracic diaphragm
A. Potassium chromate
14. Thermodynamic calculations allow B. Fluorescein
us to determine the possibility and di- C. Ammonium iron (III) sulfate
rection of spontaneous processes. In an D. Diphenylcarbazone
isolated system the change of the followi- E. Ferroin
ng thermodynamic function is used for this
purpose: 20. Choose the pair of electrodes for
potentiometric pH measurement of a
A. Entropy solution:
B. Gibbs energy
C. Helmholtz energy A. Glass and silver chloride
D. Internal energy B. Calomel and silver chloride
E. Enthalpy C. Quinhydrone and antimonial
D. Mercury sulphate and silver chloride
15. Organisms of plants and animals E. Glass and antimonial
belong to biological systems that perform
substance and energy exchange with their 21. What disorder of local circulation is
environment. These systems are: characterized by pallor, local temperature
drop, pain, local sensitivity disorder,
reduction of the organ volume?

171
krok123.in.ua
Krok 1 Pharmacy 2015 3

A. Ischemia Ba2+ ) the solubility of sulphates can be


B. Venostasis reduced by adding:
C. Thrombosis
D. Embolism A. Ethyl alcohol
E. Arterial hyperemia B. Distilled water
C. Benzene
22. Racemose clusters of calcium D. Chloroform
carbonate crystals are detected among E. Amyl alcohol
the waste products of a protoplast. These
crystals are: 28. In a surgical unit an outbreak
of purulent infections has been regi-
A. Cystoliths stered. The infections are caused by
B. Isolated crystals Staphylococcus aureus with multiple resi-
C. Raphides stance to antibiotics. What plasmid has
D. Styloids provided this property?
E. Druses
A. R
23. Heart rate of a person at rest is 40/min. B. F
What structure is the pacemaker of heart C. Col
in this man? D. T ox
E. Hly
A. Atrioventricular node
B. Sinoatrial node 29. What standard solution (titrant) is used
C. His’ bundle in Folgard’s direct titration method?
D. His’ bundle branches
E. Purkinje fibers A. Ammonium thiocyanate
B. Sodium chloride
24. The volume of air exhaled by a C. Silver nitrate
healthy person during quiet breathing was D. Potassium chromate
measured with a spirometer and determi- E. Potassium dichromate
ned to be 0,5 liter. What is this volume
called? 30. Uric acid is a derivative of:
A. Tidal volume
B. Inspiratory reserve volume
C. Expiratory reserve volume
D. Vital capacity of lungs
E. Residual volume
25. A patient has been taking diclofenac A. Purine
sodium for a long time. A family physi- B. Indole
cian withdrew this drug and prescribed C. Pyrazine
celecoxib. What disease was the cause of D. Pyrazole
drug substitution? E. Pyridine

A. Peptic ulcer 31. Select structural isomers among the gi-


B. Bronchial asthma ven compound pairs:
C. Urolithiasis
D. Arterial hypertension
E. Chronic hepatitis
26. Specify the standard solution (titrant)
for the iodometric determination of oxi-
dants:
A. Na2 S2 O3
B. KMnO4
C. I2
D. K2 Cr2 O7
E. KBrO3
27. In the qualitative analysis which
involves precipitation of sulphates of the
third analytical group cations (Ca2+ , Sr 2+ ,
172
krok123.in.ua
Krok 1 Pharmacy 2015 4

36. In terms of water-air interface, the


following substance acts as a surface-active
A. substance:
A. Valeric acid
B. HCl
C. NaOH
B. D. Urea
E. -
37. At the sixth month of pregnancy
a woman has been diagnosed with
C. severe iron-deficiency anemia. Diagnostic
character was the appearance of the
following in blood:
D. A. Hypochromic erythrocytes
B. Macrocytes
C. Megalocytes
E. D. Reticulocytes
E. Erythroblasts
32. Optical activity of monosaccharides 38. Nitrite ions in presence of nitrate ions
can be explained by their: can be detected by means of:
A. Asymmetric carbon atoms in a molecule A. Crystalline antipyrine in presence of
B. Asymmetric crystal dilute HCl
C. Complicated rotation around σ -bond B. Crystalline sodium thiosulfate
D. Aldehyde or ketone group C. Dimethylglyoxime
E. Number of hydroxyl groups in a D. Crystalline iron (III) sulfate
molecule E. Diphenylcarbazone
33. Name the disaccharide with the 39. Choose the colloid surfactant among
following structure: the substances listed below:
A. Potassium oleate
B. Iodine
C. Sodium chloride
D. Polyethylene
E. Gelatin
40. Analysis of the cerebrospinal fluid of
A. β -lactose a child with signs of purulent lesion of
B. α-lactose brain tunics revealed gram-negative bean-
C. β -maltose shaped diplococci. What provisional di-
D. β -cellobiose agnosis can be made on the basis of the
E. Saccharose analysis results?
34. Which of the ligands is bidentate? A. Meningitis
B. Gonorrhea
A. Ethylenediamine C. Cholera
B. Thiocyanate ion D. Plague
C. Cyanide ion E. Anthrax
D. Pyridine
E. Hydroxide ion 41. Given the ability of iodine to dissolve
in nonpolar solvents, determine the type
35. Combined administration of furosemi- of chemical bond in an I2 molecule:
de with aminoglycoside antibiotics causes:
A. Nonpolar covalent
A. Hearing impairment B. Ionic
B. Increased blood pressure C. Polar covalent
C. Cramps D. Metal
D. Hyperhidrosis E. Intermolecular interaction
E. Increased intraocular pressure
42. What product results from propionic
173
krok123.in.ua
Krok 1 Pharmacy 2015 5

aldehyde and P Cl5 interaction? an infectious patient with penicillin, the


pathogen transformed into the L-form.
What changes occur in the pathogen cell
in case of L-transformation?
A. Absence of a cell wall
B. Absence of flagella
A. C. Absence of a capsule
D. Absence of a spore
E. Absence of inclusions
45. Interaction of aniline with bromine
B. water resulted in white precipitate. What
substance was produced?

C.

D.
A.

E.
B.

43. Which of these reactions can be used to


identify the primary amino group?
C.

A. D.

B.
E.

46. During feces analysis of a 3-month-


old child with signs of enteric infection,
numerous dark-red colonies has grown on
Endo agar. What microorganisms can be
C. the cause of such enteric infection?
A. Escherichia
B. Streptococci
C. Gonococci
D. D. Salmonellae
E. Shigella
47. During assessment of air purity in an
aseptic unit of a phamacy, sedimentation
analysis had been applied. Test resulted
E. in growth of the small colonies with areas
of hemolysis. What medium was used for
inoculation?
44. In the course of long-term treatment of
174
krok123.in.ua
Krok 1 Pharmacy 2015 6

A. Blood agar A. Vitamin PP


B. Levine’s formulation (Eosin Methylene B. Vitamin C
Blue agar) C. Vitamin A
C. Endo agar D. Vitamin B1
D. Ploskirev’s agar E. Vitamin D
E. Egg-yolk salt agar
54. After application of chlorine-zinc-
48. What enzyme allows for synthesys iodine to the thick colourless cell
of various genes from template-RNA to membranes of collenchyme they became
DNA in genetic engineering (this enzyme violet. That means the membranes are:
catalyzes the process detected in RNA-
A. Cellulose
viruses)? B. Lignificated
A. Reverse transcriptase C. Cutinized
B. Exonuclease D. Mineralized
C. DNA-ligase E. Suberinized
D. Helicase 55. Calomel electrode is listed in the State
E. Endonuclease Pharmacopoeia of Ukraine as auxiliary
49. According to van’t Hoff rule, when the electrode for pH measurement. What type
temperature is raised by 10 degrees, the of electrodes is it?
reaction rate increases by: A. Second kind
A. 2-4 times B. First kind
B. 1,5 times C. Gas
C. 5 times D. Redox
D. 10 times E. Ion-selective
E. Temperature does not affect reaction rate 56. Specify the name of the compound
50. What anions form brown ring with iron according to the IUPAC nomenclature:
(II) salts in the presence of concentrated
sulfuric acid?
A. Nitrate ions
B. Acetate ions
C. Bromate ions
D. Citrate ions
E. Thiocyanate ions
A. 2,3,5-Trimethyl heptadiene-3,4
51. Specify the precipitating agent to be B. 3,5,6,6-Tetramethyl heptadiene-3,4
used in gravimetric determination of calci- C. 3,5,6,6-Trimethyl hepten-3
um salts: D. 2-Ethyl-4,5-dimethyl hexadiene-2,3
E. 2-Ethyl-4,5,5-trimethyl pentadiene-2,3
A. (NH4 )2 C2 O4
B. K2 C2 O4 57. Thiocyanatometric titration method
C. Na2 C2 O4 requires secondary standard solution of
D. Na2 CO3 potassium thiocyanate that is standardized
E. K2 CO3 with standard solution of:
52. Chromatin contains positively charged A. Silver nitrate
histone proteins. What amino acid is B. Hydrochloric acid
contained in histone proteins in large C. Sulfuric acid
amounts? D. Iron (II) sulfate
E. Copper (II) nitrate
A. Lysine
B. Alanine 58. To identify a drug by thin-layer
C. Valine chromatography the following parameter
D. Threonine is used:
E. Serine
53. Diet of a human must contain vitami-
ns. What vitamin is usually prescribed for
treatment and prevention of pellagra?
175
krok123.in.ua
Krok 1 Pharmacy 2015 7

A. Rf A. Autoimmune disorder of the nephron


B. n glomerulus
C. E , mV B. Inflammatory disorder of the nephron
D. I , A tubule
E. Kp C. Ishemic disorder of the nephron tubule
D. Increased concentration of blood protei-
59. A 22-year-old man was stung by bees; ns
the affected area became hyperemic and E. Disorder of the urinary tracts
edematous. What is the leading mechani-
sm of edema development in this patient? 64. To determine sodium iodide with
Fajans method the following indicator is
A. Increased permeability of the capillaries required:
B. Decreased hydrostatic blood pressure in
the capillaries A. Eosin
C. Increased oncotic pressure of tissue fluid B. Methyl-orange
D. Impaired lymphatic efflux C. Diphenylcarbazone
E. Reduced oncotic pressure of blood D. Potassium chromate
E. Iron ammonium alum
60. A patient has obstruction of the
common bile duct. Which of these 65. Which of the given compounds WILL
substances is usually found in urine in such NOT decolorize bromine water?
cases?
A. CH3 − CH3
A. Bilirubin B. CH2 = CH2
B. Ketone bodies C. CH ≡ CH
C. Uric acid D. CH3 − CH = CH2
D. Creatinine E.
E. Glucose
61. A student analyzes an axial plant
organ characterized by radial symmetry,
unlimited growth, positive geotropism. It
provides nutrition, vegetative propagation, 66. Which of the following reactions is
anchorage of plant in the soil. This organ required in order to obtain an azo dye out
can be identified as: of an aromatic amine?
A. Root A. Diazotization and azo compound
B. Stem B. Reduction and diazotization
C. Leaf C. Diazotization and interaction with
D. Rhizome potassium cyanide
E. Seed D. Salt formation and nitration
E. Alkylation and nitrosation
62. Choose the name that corresponds wi-
th the formula: CH3 − C ≡ N : 67. What is the mechanism of bromination
of toluene methyl group?
A. Acetic acid nitrile
B. Acetamide
C. Acetic anhydryde
D. Acetoxime
E. Ethyl isocyanide
63. A patient with systemic lupus A. SR
erythematosus has developed diffuse B. AE
affection of kidneys followed by protei- C. SE
nuria, hypoproteinemia, extensive swelli- D. SN
ng. What mechanism of proteinuria E. AN
development is the most likely in this case?
68. Quinoline reacts with the following
reagent if heteroatom is present:

176
krok123.in.ua
Krok 1 Pharmacy 2015 8

A. Radicle
A. B. Apical meristem
C. Pericycle
B. Na2 SO4 D. Lateral meristem
C.  E. Intercalary meristem
D. NaNH2
E. NaNO3 74. Silver mirror reaction can be
characterized by:
69. Aniline can be converted into the
water-soluble salt through its treatment wi- A. Production of metal
th the solution of: B. Smell
C. Red colouring of precipitate
A. Hydrochloric acid D. Blue colouring of solution
B. Sodium hydroxide E. Decolouration
C. Sodium sulfate
D. Ethanol 75. The end product of starch hydrolysis is:
E. Dimethylamine A. D-glucose
70. Specify the reagent necessary for the B. D-fructose
following transformation: C. Saccharose
D. Maltose
E. D-galactose
76. Specify the electronic effects of the
carboxylic group (-СООН) in a benzoic
acid molecule:
A. NH2 NH2
B. NH2 OH A. -I, -M
C. CH3 NH2 B. -I
D. C6 H5 NHNH2 C. +I, -M
E. NH3 D. +I
E. -I, +M
71. One of the cyclic glucose forms is as
follows: 77. Which of the alcohols given below
produces acetone when oxidated?

Name this compound:

A. α-D-glucopyranose A.
B. β -D-glucopyranose
C. α-L-glucopyranose
D. α-D-glucofuranose B.
E. β -D-glucofuranose
72. Select the formula for pentene-2 from C.
the list:
D.
A. CH3 − CH2 − CH = CH − CH3
B. CH3 − CH2 − CH2 − CH2 − CH3 E.
C. CH3 − CH2 − CH2 − CH = CH2
D. CH3 − CH2 − CH2 − CH3
E. CH3 − CH = CH − CH3 78. What reagent allows to simultaneously
detect both aldehyde group and glycol
73. The study of the main root ontogenesis fragment presence in glucose molecule?
shows that it has developed from:
A. Cu(OH)2
B. Br2
C. AlCl3
D. F eCl3
E. KMnO4
177
krok123.in.ua
Krok 1 Pharmacy 2015 9

79. What class of organic compounds is A. 3’,5’-phosphodiester


characterized by the presence of C ≡ N B. Peptide
group? C. Glycosidic
D. Disulfide
A. Nitriles E. Amide
B. Amines
C. Nitro compounds 85. Quercus robur leaves have the followi-
D. Alcohols ng type of lamina shape and division:
E. Aldehydes
A. Pinnatilobate
80. If the amount of a high-molecular B. Trilobate
substance added to the sol is very small, C. Pinnatipartite
it may not increase but decrease its stabili- D. Palmatilobate
ty. This phenomenon is called: E. Palmatipartite
A. Sensibilization 86. Microbiological purity of tableted
B. Solubilization drugs had been tested at a factory. Samples
C. Mutual coagulation cultivation in mannitol salt agar resulted
D. Colloidal protection in growth of golden-yellow colonies, mi-
E. Sol adaptation croscopic examination of colonies detected
gram-positive globular bacteria situated
81. Spore and pollen analysis revealed in clusters; microorganisms had plasma
tetrahedral spores in the pollen, whi- coagulation prorerties. What pure bacteri-
ch have semi-circular base and reticular al culture was obtained?
surface. The spores can belong to:
A. Staphylococcus aureus
A. Lycopodiophyta B. Enterobacteriaceae
B. Equisetiphyta C. Staphylococcus epidermidis
C. Bryophyta D. Staphylococcus saprophyticus
D. Polypodiophyta E. Pseudomonas aeruginosa
E. Pinophyta
87. Leaves affected by mosaic discoloration
82. In the practice of harvesting herbal have been detected at medicinal plantati-
raw material of Asteraceae family the term ons. What microorganisms are the cause?
”flowers” means both individual flowers
and inflorescences. However, the notion of A. Plant-pathogenic viruses
”flowers” is botanically correct only for: B. Plant-pathogenic bacteria
C. Plant-pathogenic fungi
A. Centaurea cyanus D. Protozoa
B. Gnaphalium uliginosum E. Rickettsia
C. Arnica montana
D. Echinops ritro 88. A dry-heat box is used for sterilization
E. Bidens tripartita of various materials and instruments in a
bacteriological laboratory. This sterilizati-
83. A woman noticed that a cut on her on method can be applied to the following
skin was still bleeding even after 20 mi- objects:
nutes had passed. What vitamin deficiency
causes such condition? A. Glass test tubes
B. Rubber gloves
A. Vitamin K C. Simple nutrient medium
B. Vitamin A D. Wire inoculating loops
C. Vitamin D E. Physiological solution
D. Vitamin E
E. Vitamin B12 89. A 3,5-year-old child has been di-
agnosed with dysbacteriosis in the form
84. The primary structure of nucleic acids of critical reduction of gram-positive
is a polynucleotide chain that has a certain anaerobic bacteria and increased number
composition and order of the nucleotides. of staphylococci and yeast fungi. What
What bonds stabilize this structure? preparation should be used for the
correction of dysbacteriosis?

178
krok123.in.ua
Krok 1 Pharmacy 2015 10

A. Bifidumbacterin A. Lactase
B. Colibacterin B. Maltase
C. Coli-Proteus bacteriophage C. Aldolase
D. Furazolidone D. Hexokinase
E. Lactoglobulin E. Glycosidase
90. A sample of water used in drug 95. A man presents with signs of albi-
production has been sent to a laboratory nism: blonde hair, extreme photosensi-
for sanitary and virological analysis. tivity, impaired vision. What amino acid
Presence of what virus group will be indi- metabolism is disrupted in the patient?
cative of faecal contamination of water and
thus the need for its additional purificati- A. Tyrosine
on? B. Methionine
C. Proline
A. Picornaviridae D. Histidine
B. Herpesviridae E. Valine
C. Orthomyxoviridae
D. Retroviridae 96. In the process of silver cations identi-
E. Flaviviridae fication reaction HCl and then ammonia
solution have been added to the solution.
91. Specify the indicator of the protective What compound has been produced as a
properties of high-molecular compounds result?
of body, which promote the retention of
calcium, phosphate and carbonate in blood A. [g(N 3 )2 ]l
plasma: B. [g2 (N 3 )3 ]l
C. g 
A. Protective value D. g l
B. Coagulation threshold E. [g(N 3 )3 ]l
C. Critical micelle concentration
D. Hydrophilic-lipophilic balance 97. Microscopic study of soybean seeds
E. Volume of sol coagulated by 1 mol of the stained with Sudan III revealed droplets
electrolyte substance of various sizes. They are:
92. A 40-year-old man diagnosed with A. Lipids
gastric ulcer has developed the symptoms B. Proteins
anew after a long period of dormancy. Such C. Starch
disease course can be characterized as a: D. Inulin
E. Glycogen
A. Recurrence
B. Remission 98. In a chemico-analytical laboratory a di-
C. Recovery spensing chemist studies the solution of
D. Latency anion mixture. When antipyrin solution is
E. Prodromal phase added to the solution, it becomes emerald-
green in colour. This analytical effect signi-
93. An elderly patient exhibits low levels of fies presence of the following anions:
red blood cells and hemoglobin in blood,
but the color index is 1,3. Blood smear A. Nitrite
analysis revealed megaloblasts. What type B. Nitrate
of anemia is observed in this case? C. Acetate
D. Tartrate
A. B12 -folic acid deficiency E. Citrate
B. Iron-deficiency
C. Acquired hemolytic 99. Selective solvents are used in
D. Hereditary hemolytic laboratories and factories to isolate and
E. Chronic posthemorrhagic refine essential oils, alkaloids, antibiotics
and other pharmaceutical subctances. This
94. After drinking milk a 1-year-old child process is called:
has developed diarrhea and flatulence. The
baby is likely to have the deficiency of the A. Extraction
following enzyme: B. Sedimentation
C. Coagulation
D. Flocculation
E. Flotation
100. Sol of iron (III) hydroxide is positi-
179
krok123.in.ua
Krok 1 Pharmacy 2015 11

vely charged. Specify the ion which has the A. Rimantadine


lowest coagulation threshold: B. Metisazone
C. Levamisole
A. SO42− D. Azidothimidine
B. Cl− E. Acyclovir
C. Cu2+
D. Na+ 106. Colloid silver preparations
E. J − Protargolum and Collargolum are widely
used in medical practice as bactericidal
101. Specify the standard solutions that are drugs. In addition to the active ingredients,
used in permanganatometric titration to these drugs contain protein compounds.
quantify the oxidants by the residual ti- What is the function of proteins in these
tration method: preparations?

A. Potassium permanganate, iron (II) A. Prevention of coagulation of the colloi-


sulfate dal solution
B. Potassium dichromate, sodium thi- B. Prolongation of shelf-life
osulfate C. Reduction of the side effects
C. Potassium bromate, sodium thiosulfate D. Improvement of the drug technology
D. Potassium iodate, sodium thiosulfate E. Potentiation of the bactericidal action of
E. Cerium (IV) sulfate, iron (II) sulfate silver

102. What indicator is used for fixing the 107. Mass fraction of F e2+ ions in Mohr’s
endpoint of mercurimetric titration? salt can be determined by gravimetric sedi-
mentation method using:
A. Thiocyanate complexes of iron (III)
B. Fluorescein A. NH4 OH
C. Eosin B. Na2 S
D. Murexide C. K3 P O4
E. Potassium chromate D. BaCl2
E. ZnCl2
103. Substrate-linked phosphorylation
occurs in the cycle of tricarboxylic acids. 108. Silver nitrate solution has been added
What compound takes part in this reacti- to the solution containing anions of the fi-
on? rst analytical group. It resulted in yellow
precipitate. That means the following are
A. Succinyl coenzyme A present in the solution:
B. α-ketoglutarate
C. Acetyl coenzyme A A. Arsenite ions
D. Succinate B. Arsenate ions
E. Malate C. Sulphate ions
D. Iodide ions
104. The patient with alcoholic ci- E. Bromide ions
rrhosis complains of general weakness
and dyspnea. The following is revealed: 109. Inhibition of the synthesis of bile aci-
decrease of arterial pressure, ascites, di- ds from cholesterol in liver of an experi-
lation of the superficial veins of the mental animals has caused maldigestion of
stomach anterior wall, esophageal vari- lipids. What is the role of these acids in the
cose veins dilatation, splenomegaly. What enteral lipidic metabolism?
haemodynamics disorder does the patient
suffer from? A. They emulsify dietary lipids
B. They keep balance of alkaline envi-
A. Portal hypertension ronment in the intestines
B. Left ventricular failure C. They participate in the synthesis of lipids
C. Right ventricular failure D. They are part of LDL
D. Cardiac insufficiency E. They activate the formation of chylomi-
E. Collapse crons

105. Pharmacy has received viricides. 110. Name the above-ground sprout modi-
Choose the viricide used for influenza fications that develop from lateral buds,
treatment from the list given below: are situated in leaf angles or inflorescences,
and take part in vegetative reproduction:

180
krok123.in.ua
Krok 1 Pharmacy 2015 12

A. Bulbils allergic reaction. The development of the


B. Above-ground tubers allergic reaction is caused by the increased
C. Cladodes synthesis of the following compound:
D. Tendrils
E. Thorns A. Histamine
B. Choline
111. What type of conducting bundles is C. Adrenaline
characteristic of all root zones in one- D. Histidine
seeded plants? E. Serotonin
A. Radical 117. The patient has icteric skin;
B. Amphivasal (Lepto centric) unconjugated bilirubin content in blood
C. Amphicribal (Hadro centric) is high; conjugated bilirubin in urine is not
D. Bilateral detected. There is significant amount of
E. Collateral urobilin in urine and stercobilin in feces.
Name the pathology characterized by the
112. A patient with atherosclerosis has given symptoms:
been prescribed Linaetholum containi-
ng essential fatty acids. Which of the A. Hemolytic jaundice
following acids is an essential part of the B. Obstructive jaundice
preparation? C. Jaundice of the newborn
D. Hepatocellular jaundice
A. Linolenic E. Atherosclerosis
B. Palmitic
C. Crotonic 118. A patient complains of pain in the
D. Stearic small joints. High concentration of uric
E. Oleic acid is detected in his blood plasma. What
pathology causes such changes?
113. The Volhard method is used
to determine sodium chloride mass A. Gout
concentration. Name the titrant of this B. Diabetes mellitus
method: C. Phenylketonuria
D. Lesch-Nyhan syndrome
A. Ammonium thiocyanate E. Diabetes insipidus
B. Mercury (I) nitrate
C. Sodium tetraborate 119. Corolla of a zygomorphic
D. Mercury (II) nitrate hermaphroditic flower consists of 5 petals:
E. Sodium hydroxide the largest one is called the banner, the two
lateral petals are called the wings, and the
114. Mass fraction of pharmaceutical two fused petals forming the keel. Such
preparations that contain aromatic ami- corolla is characteristic of medicinal plants
no groups is determined through nitrite of Leguminosae family. Name the type of
titration. What external indicator is used corolla:
in this case?
A. Papilionaceous
A. Starch-iodide paper B. Labiate
B. Methylene red C. Saucer-shaped
C. Eriochrome Black T D. Funnelform
D. Phenolphthalein E. Tubular
E. Eosin
120. What ions have maximal coagulative
115. What cation of the 4th analytical effect, when added into positive sols?
group is present in a solution, if it is known
that the reaction with a group reagent A. P O43−
causes formation of yellow precipitate? B. Al3+ ; F e3+
C. K + ; Na+
A. Cr 3+ D. SO42−
B. Zn2+
C. Sn2+ E. Cl−
D. Al3+ 121. What anion of the 2nd analytic group
E. Sn(IV ) produces black precipitate with group
116. In response to the administration reagent AgNO3 ?
of protein drugs, a patient developed an
181
krok123.in.ua
Krok 1 Pharmacy 2015 13

A. S 2− vered into a contagious isolation ward of


B. I − a hospital. He is diagnosed with toxic di-
C. Cl− phtheria of the pharynx. What drug should
D. Br − be administered immediately for specific
E. NCS − treatment and prevention of complicati-
ons?
122. A patient suffering from coronary
heart disease, who had had two myocardial A. Antidiphtheric serum
infarctions of left ventricular wall, presents B. Diphtheria and tetanus toxoids and
with bubbling breathing and dyspnea. pertussis adsorbed vaccine
Pulmonary auscultation reveals numerous C. Diphtheria anatoxin
moist crackles. What kind of heart failure D. Penicillin antibiotic
is it? E. -
A. Left ventricular 128. Cataract (lenticular opacity) has
B. Right ventricular developed in a 52-year-old woman wi-
C. Compensated th pancreatic diabetes. What process has
D. Subcompensated intensified and thus caused lenticular
E. Combined opacity?
123. You are studying the silvery downy A. Protein glycosylation
plant of Asteraceae family, which is rich wi- B. Lipolysis
th essential oils and bitters. Harvested are C. Ketogenesis
apical sprouts with panicle of small round D. Protein proteolysis
flower heads. This plant is: E. Gluconeogenesis
A. Artemisia absinthium 129. Common nettle, hop, black elderberry
B. Arctium lappa are the plants that require soils rich in ni-
C. Bidens tripartita trogen compounds, that is, such plants are
D. Calendula officinalis called:
E. Chamomilla recutita
A. Nitrophytes
124. A 46-year-old patient was found to B. Nitrophobes
have hyperactivity of creatine kinase in his C. Calciphiles
blood serum. What kind of pathology can D. Calciphobes
be suspected? E. Halophytes
A. Myocardial infarction 130. A man received a radiation dose of
B. Acute pancreatitis 30 Gy. He presents with necrotic angi-
C. Chronic hepatitis na, disorders of the gastrointestinal tract.
D. Haemolytic anemia Blood tests revealed anemia, leukopenia
E. Renal failure and thrombocytopenia. What period of
acute radiation sickness is observed in the
125. Microcrystalloscopic reactions of patient?
potassium ions detection include the
reaction with: A. Height of disease
B. Primary reactions
A. Sodium lead hexanitrocuprate (II) C. Imaginary wellbeing
B. Sodium hydrotartrate D. End of disease
C. Sodium hexanitrocobaltate E. -
D. Sodium tetraphenylborate
E. Flame test 131. In the course of an experiment in
the mesenteric vein of a toad a trombus
126. Emulsions containing less then 0,1% was created with a crystal of common salt.
of dispersed phase (in volume) are classifi- What processes occurred during the first
ed as: stage of trombus formation?
A. Diluted A. Adhesion, aggregation, agglutination of
B. Concentrated platelets
C. High-concentration B. Production of active thromboplastin
D. Water-in-oil type C. Production of thrombin
E. Oil-in-water type D. Production of fibrin monomer
E. Production of fibrin polymer
127. A student in severe condition was deli-
182
krok123.in.ua
Krok 1 Pharmacy 2015 14

132. A patient with hyperproduction ffusion is called:


of thyroid hormones has been prescri-
bed Merkazolilum. This drug inhibits A. Dialysis
the following enzyme of iodothyronine B. Electrodialysis
synthesis: C. Ultrafiltration
D. Decantation
A. Iodide peroxidase E. Compensatory dialysis
B. Aromatase
C. Reductase 138. Neuroleptanalgesia has been appli-
D. Decarboxylase ed in the case of cardiac infarction. What
E. Aminotransferase neuroleptic is most often applied along wi-
th fentanyl?
133. Hemoglobin catabolism results in
release of iron which is transported to the A. Droperidol
bone marrow by a certain transfer protein B. Perphenazine (Aethaperazinum)
and is used again for the synthesis of C. Levomepromazine
hemoglobin. Specify this transfer protein: D. Clozapine
E. Sulpiride
A. Transferrin (siderophilin)
B. Transcobalamin 139. Due to prolonged taking of
C. Haptoglobin phenobarbital the epileptic patient has
D. Ceruloplasmin developed tolerance for this drug. What
E. Albumin is this phenomenon based on?
134. For the specific prevention of A. Biotransformation acceleration
influenza the employees of an enterprise B. Absorption process weakening
were vaccinated with ”Influvac”. What type C. Increase of receptor sensitivity
of immunity will develop in the body of D. Biotransformation suppression
those vaccinated? E. Substance accumulation in body
A. Artificial active 140. What side effect is characteristic of
B. Innate congenital captopril?
C. Artificial passive
D. Natural active A. Dry cough
E. Natural passive B. Increase of arterial pressure
C. Hyperglycemia
135. A patient has a mental disorder due D. Cardiac rate disorder
to the insufficient synthesis of gamma- E. Hypokaliemia
aminobutyric acid in the brain. Such
pathological changes might be caused by 141. During treatment of chronic cardi-
the deficiency of the following vitamin: ac failure with digitoxin a patient
developed the drug-specific signs of intoxi-
A. Pyridoxine cation. A doctor prescribed Unithiol
B. Tocopherol (Dimercaptopropansulfonate sodium).
C. Cyanocobalamin Explain its mechanism of action of Unithi-
D. Folic acid ol in case of cardiac glycoside intoxication:
E. Riboflavin
A. Restoration of + -Na+ -adenosine
136. Soil microflora often contai- triphosphatase activity
ns representatives of pathogenic mi- B. Binding of calcium ions
croorganisms. Specify the diseases with C. Increase of sodium concentration in
causative agents that can remain viable in cardiac hystiocytes
the soil for a long time: D. Increase of calciun permeability of
cardiac hystiocytes
A. Tetanus and gas anaerobic infection E. Binding of glycosides into complex
B. Tuberculosis and mycobacterioses compound
C. Colibacillosis and cholera
D. Leptospirosis and plague 142. An elderly patient suffers from consti-
E. Typhoid fever and dysentery pation caused by colon hypotonia. What
drug should be prescribed?
137. The method consisting in removal
of low-molecular impurities from colloi-
dal systems and high-molecular compound
solutions by semipermeable membrane di-
183
krok123.in.ua
Krok 1 Pharmacy 2015 15

A. Bisacodyl channel-blocking agent was prescribed for


B. Sodium sulfate treatment. Name this agent:
C. Castor oil
D. Atropine sulfate A. Amlodipine
E. Novocainamide (Procainamide) B. Atenolol
C. Guanethidine
143. In the course of bronchitis D. Reserpine
pharmacotherapy a patient has developed E. Labetalol
dyspeptic disorders, photodermatitis and
hepatic failure. What drug can cause such 149. A patient suffers from mucosal
disorders? dryness and mesopic vision disorder. What
vitamin deficiency causes these symptons?
A. Doxycycline
B. Paracetamol A. 
C. Ascorbic acid B. 
D. Acetylcysteine C. 
E. Codeine phosphate D. 
E. D
144. The patient with rheumatoid arthritis
and concomitant duodenal ulcer has to be 150. Microorganisms that reach blood and
prescribed a nonsteroid antiinflammatory other biological systems have negative
drug. Which one of the drugs listed below surface charge. What surfactants are used
is the drug of choice in the given case? as antibacterial agents to suppress the acti-
on of microorganisms?
A. Celecoxib
B. Acetylsalicylic acid A. Cationic
C. Paracetamol B. Anionic
D. Metamizole C. Lyophilic
E. Diclofenac sodium D. Lyophobic
E. Micellar
145. The patient with bronchial asthma
had been prescribed salbutamol, which 151. A ready-made drug was inoculated
led to disappearance of bronchiospasm on Sabouraud’s agar and incubated under
symptoms. It happened due to stimulati- 22oC for 5 days. This nutrient medium was
on of: used to determine the following:
A. β2 -adrenoreceptors A. Number of mold and yeast fungi
B. α1 -adrenoreceptors B. Total number of bacteria
C. Muscarinic acetylcholine receptors C. Presence of E. coli
D. Acetylcholine synthesis D. Presence of S. aureus
E. β1 -adrenoreceptors E. Presence of Salmonella
146. Choose the most efficient way of 152. The second stage of detoxificati-
convallariae glycoside administration for on involves joining certain chemical
acute cardiac failure treatment: compounds with functional groups of toxi-
nes. Select one such compound:
A. Intravenous
B. Intramuscular A. Glucuronic acid
C. Subcutaneous B. Higher fatty acids
D. Internal C. Cholesterol
E. Inhalational D. Glucose
E. Pyruvate
147. A doctor has prescribed a nonsteroi-
dal anti-inflammatory drug to relieve 153. Fatty acids arrive into mitochondria,
inflammation and pain syndrome. Name and there their oxidation occurs. Name the
this drug: vitamin-like substance that takes part in
transportation of fatty acids through mi-
A. Diclofenac sodium tochondrial membrane:
B. Glibenclamide
C. Loratadine A. Carnitine
D. Prednisolone B. Choline
E. Calcium chloride C. Biotin
D. Pantothenic acid
148. A 55-year-old patient had been di- E. Folic acid
agnosed with angina pectoris. Calcium
184
krok123.in.ua
Krok 1 Pharmacy 2015 16

154. A 5-year-old child presents with A. Renin


abdominal distension, abdominal cramps, B. Cortisol
and diarrhea occurring 1-4 hours after C. Vasopressin
drinking milk. Described symptoms are D. Noradrenaline
caused by the lack of enzymes that break E. Thyroxin
up:
160. Micelle of a colloid surfactant will
A. Lactose have the following structure in a certain
B. Glucose solvent: polar groups are turned towards
C. Maltose the solvent, while radicals are facing the
D. Saccharose micelle center. What solvent is it?
E. Fructose
A. Water
155. Fajans titration with fluorescein is B. Toluene
performed within the following pH range C. Benzene
of a medium: D. Tetrachloromethane
E. Hydrogen sulfide
A. 7-10
B. 1-3 161. What cations of the V analytical group
C. 3-5 can be detected by hydrolysis?
D. 5-7
E. 10-13 A. Antimony and bismuth
B. Manganese
156. Electrode made according to the C. Iron (II)
scheme Red, Ox, H + | Pt belongs to the D. Magnesium
following type: E. Iron (III)
A. Complex redox electrode 162. A solution of magnesium mixture was
B. Ion-selective electrode added into solution with anions of the 1st
C. Electrode of the second kind analytical group. White crystalline precipi-
D. Electrode of the first kind tate was produced. What anions cause such
E. Gas electrode analytical effect?
157. A plant has erect stem with only A. P O43− and AsO43−
one leaf growing from each node. What B. AsO33−
phyllotaxy is characteristic of this plant? C. S2 O32−
A. Alternate D. SO32−
B. Opposite E. C2 O42−
C. Verticillate
D. Dichotomous 163. Albumine, blood serum protei-
E. Parallel ns, and gastric juice pepsin consist of
macromolecules of polypeptide chains
158. Plant pathogens are represented by that are joined with hydrogen bonds into
various microorganisms: bacteria, fungi, hydrophilic spheres. These proteins are
actinomycetales, viruses. Name the main named:
location of plant pathogens in the natural
environment: A. Globular
B. Fibrillar
A. Soil C. Structural
B. Water D. Synthetic
C. Air E. Artificial
D. Plant parts
E. Plant vascular system 164. What drug group has the most
pronounced vasodilatory action, and has
159. During ultrasound investigation a little effect on cardiac conduction system
patient was diagnosed with bilateral and miocardial activity?
renal artery stenosis of atherosclerotic
genesis. Specify the bioactive substance A. Dihydropyridine derivatives
that due to its excessive secretion is the B. Phenylalkylamine derivatives
key component of arterial hypertension C. Benzodiazepine derivatives
pathogenesis in the given case: D. Sulfonylurea preparations
E. β -adrenoceptor agonist
165. A woman is to be prescribed a
185
krok123.in.ua
Krok 1 Pharmacy 2015 17

narcotic analgesic for labor pain relief. gastric ulcer disease was prescribed a
What drug is indicated in this case? medicine as a part of his multimodality
therapy. The medicine lowers acidity of
A. Promedol (Trimeperidine) gastric juice, inhibits + , + -adenosine
B. Morphine triphosphatase, decreases the volume of
C. Papaveretum (Omnopon) gastric secretion and pepsinogen producti-
D. Codeine on. It is a prodrug. Name this medicine:
E. Fentanyl
A. Omeprazole
166. A woman, who during the 5th-10th B. Famotidine
weeks of her pregnancy had been taki- C. Gastrozepin (Pirenzepine)
ng sodium valproate for treatment of D. Ranitidine
her epilepsy, gave birth to a child with E. Phosphalugel (Aluminium phosphate)
pathology of the vertebral column (split
spine). What side effect of the drug caused 171. A cultivated plant has green berrylike
such malformation? fruit and underground sprout modificati-
ons - tubers. The described plant is:
A. Teratogenic
B. Mutagenic A. Solanum tuberosum
C. Embryotoxic B. Соnvalaria majalis
D. Fetotoxic C. Polygonatum odoratum
E. Sensitizing D. Atropa belladonna
E. Solanum lycopersicum
167. A patient with acute bronchitis was
prescribed an expectorant that caused 172. Elongated narrow prismatic crystals
bronchial spasm after the patient had with sharpened points were detected
taken it. What drug of those listed below during microscopic investigation of
can cause such side effect? Convallaria majalis mesophile. These
crystals are:
A. Acetylcysteine
B. Salbutamol A. Styloids
C. Validol (Menthyl isovalerate) B. Druses
D. Platyphyllin C. Crystalline sand
E. Prenoxdiazine (Libexin) D. Cystoliths
E. Perigonium
168. A patient was visiting a pharmacy,
when he suddenly felt unwell. He 173. Students should identify the following
developed palpitations, rapid heart rate, to determine the sex of a flower:
pain in the chest that after several minutes A. Stamens and pistils
spread to the left scapula and left side of B. Flower cup and corolla
the head. What condition should be consi- C. Pedicle and receptacle
dered first? D. Symmetry
A. Ischemic heart disease E. Colour and type of indumentum
B. Peptic gastric ulcer disease 174. A doctor prescribed a herbal drug
C. Dysphagia with flavonoid complex of Silybum mari-
D. Pneumonia anum to a patient suffering from chronic
E. Somatoform autonomic dysfunction hepatitis. This hepatic protector stimulates
169. Prolonged application of broad protein synthesis, normalizes phospholipid
spectrum antibacterial drugs resulted in metabolism, acts as an antioxidant. Name
the patient being hospitalised with di- this drug:
agnosis of candidiasis. What side effect A. Silymarin
of antibiotic therapy has developed in the B. Essentiale
patient? C. Galstena
A. Disbacteriosis D. Thiotriasoline
B. Endotoxic reaction E. Ursodeoxycholic acid
C. Toxic reaction 175. A pharmaceutical manufacture
D. Allergic reaction produces a drug, that is an animal antibi-
E. Formation of resistant microorganism otic. Point out this drug among those listed
strains below:
170. A 37-year-old patient with peptic
186
krok123.in.ua
Krok 1 Pharmacy 2015 18

A. Lysozyme A. Salbutamol
B. Gramicidin B. Metoprolol
C. Novobiocin C. Atenolol
D. Phaseolin D. Anaprilin (Propranolol)
E. Chloramphenicol E. Nebivolol
176. A patient has been receiving 181. A 25-year-old-patient with the II
Theophylline - inhibitor of cyclic adenosi- degree thermal burns addressed a doctor.
ne monophosphate phosphodiesterase - Objectively: there are large blisters on the
for a week. What hormone can increase upper limbs; the blisters are filled with
its action due to such treatment and cause clear exudate containing mostly water and
hyperglycemia? albumines with isolated leukocytes. Name
the type of the exudate:
A. Glucagon
B. Testosterone A. Serous
C. Aldosterone B. Catarrhal (mucous)
D. Insulin C. Fibrinous
E. Estradiol D. Purulent
E. Hemorrhagic
177. To detect anions in a solution by fracti-
onal method a reaction with iron (III) 182. A seed of a legume contains proteins
chloride was performed in acid medium. and fatty oil. Name this legume:
The solution coloured red-violet. What
anion is the cause of such analytical effect? A. Glycine hispida
B. Vaccinium myrtillus
A. Salicylate C. Sinapis alba
B. Chloride D. Astragalus dasyanthus
C. Nitrate E. Datura stramonium
D. Bromate
E. Phosphate 183. Rhizome of a species belonging to
the Asteraceae family is polycephalous,
178. An impression smears of the dead succulent, has lysigenous cavities,
person’s brain and salivary glands revealed accumulates inulin. Such underground
Negri bodies, when coloured with Mann organ is characterisic of:
methyl blue-eosin stain. These results
confirm the presence of the following di- A. Inula helenium
sease: B. Hyoscyamus niger
C. Digitalis grandiflora
A. Hydrophobia D. Sorbus aucuparia
B. Influenza E. Helianthus annuus
C. Parotitis
D. Parainfluenza 184. What drug is used in treatment regi-
E. Encephalitis men for peptic ulcer disease to eliminate
Helicobacter pylori?
179. Preparations of colloid silver -
Protargol (silver proteinate) and Collargol A. Clarithromycin
(colloid silver) - contain proteine B. Tienam
compounds besides their active substance. C. Biseptol
What is the function of proteins in these D. Chloridine
preparations? E. Sulfalene

A. Protection of colloid solution against 185. Sodium hydroxide was added to a


coagulation solution. Precipitation occurred. The preci-
B. Increased storage time pitate was initially white and became
C. Decreased side effects brown later. It indicates the presence of
D. Improved preparation technology the following in the solution:
E. Increased bactericidal action of silver A. Manganese (II) cations
180. Adrenomimetic agents are differenti- B. Lead (II) cations
ated into selective and non-selective. C. Potassium cations
What drug is an agonist of β2 -adrenergic D. Calcium cations
receptors and can be used for treatment of E. Barium cations
bronchial asthma? 186. What group of drugs is characterized
by development of drug addiction as a side
187
krok123.in.ua
Krok 1 Pharmacy 2015 19

effect? A. Paresis
B. Paralysis
A. Psychosedatives C. Myasthenia
B. Cholinergic antagonists D. Hyperkinesia
C. Adrenergic drugs E. Tremor
D. Diuretics
E. Emetics 193. A 32-year-old patient with cerebellar
tumor was delivered to an admission room
187. Some drugs have the form of colloid of a hospital. The patient presents with
solutions. What size of dispersed phase ataxia that can be characterized by:
particles corresponds with colloidal di-
spersion? A. Disrupted coordination of movements
B. Involuntary contraction of skeletal
A. 10−7 − 10−9 m muscles
B. 10−5 − 10−7 m C. Increased muscle tone
C. 10−10 − 10−11 m D. Pathological reflexes
D. 10−5 − 10−3 m E. Irregular force and direction of
E. >10−3 m movements
188. A local general practitioner 194. A doctor prescribed diazepam to
recommends taking interferon for a patient with anxiety disorders. What
influenza prevention. What is the mechani- pharmacological effect is the reason for
sm of action of this drug? such a prescription?
A. Blocks virus protein synthesis A. Anxiolytic
B. Blocks virus stripping B. Anticonvulsant
C. Inhibits virion exit from cells C. Anti-inflammatory
D. Prevents adsorption of virus in cell D. Antianginal
receptors E. Hypotensive
E. Disrupts the process of virus assembly
195. A 40-year-old man presents with rapid
189. What cation can be detected with weight gain after he had suffered a severe
Chugaiev’s agent (Dimethylglyoxime)? craniocerebral trauma. At doctor’s exami-
nation the patient’s weight was 125 kg, with
A. Ni2+ his weight being 175 cm. What mechanism
B. Ca2+ of obesity development is the most likely
C. + in this case?
D. Mn2+
E. Co2+ A. Hypothalamic
B. Alimentary
190. Dry many-seeded monocarp fruit C. Hormonal
opens along its ventral suture. It can be D. Hereditary
identified as: E. -
A. Follicle 196. Specify the parameters that
B. Legume characterize the sensitivity of analytical
C. Nutlet reaction:
D. Drupe
E. Capsule A. All the parameters
B. Minimal volume of borderline diluted
191. Determination of silver salts by solution
ammonium thiocyanate titration is C. Borderline dilution
performed in the presence of the following D. Absolute sensitivity
indicator: E. Borderline concentration
A. (NH4 )F e(SO4)2 197. Specify the name of the carbohydrate
B. F eSO4 given in
C. F eCl2
D. (NH4 )2 SO4
E. NH4 SCN
192. After ishemic stroke a 67-year-old
patient developed reduced mobility of the
left leg. Name this condition: according to the systematic nomenclature:
188
krok123.in.ua
Krok 1 Pharmacy 2015 20

A. 5,6,6-trimethyl-1-hepten
B. 2,2,3-trimethyl-6-hepten A. С1
C. 5-tert-Butyl-1-hexene B. С2
D. 2-tert-Butyl-5-hexene C. С3
E. 2,2,3-trimethyl-6-hexene D. С4
198. M. I. Konovalov’s reaction is as E. С5
follows: 200. To chemically distinguish between
A. 4 + HNO3 (diluted) = CH3 NO2 + glycerine and ethylene glycol
H2 O
B. 4 + H2 SO4 = CH3 SO2 OH + H2 O
C. 22 5 Cl + 2Na = C4 H10 + 2NaCl
D. 6 6 + HNO3 (concentrated) = it is nececcary to apply:
C6 H5 NO2 + H2 O
E. 4 + Cl2 = CH3 Cl + HCl
199. Hydrogen atom attaches to the A. KHSO4
following carbon-1-pentene atom during B. CuOH
electrophilic addition of hydrogen bromi- C. Cu(OH)2
de: D. NaOH
E. NaCl

189
krok123.in.ua
Кrok 1 Pharmacy (англомовнi студенти) 2016 рiк 1

1. Calculation of thermal effects of A. Glomerular filtration


chemical reactions at pharmaceutical B. Obligate reabsorption
production is based on the Hess law. C. Facultative reabsorption
This law states that thermal effect of D. Tubular secretion
a reaction is determined by: E. -
A. Initial and final states of system 6. In snake venom there is a
B. Mechanism of chemical change substance that causes erythrocyte
C. Route of chemical change hemolysis, when it is introduced
D. Number of intermediate stages into a human organism. Blood test
E. Process duration revealed a large amount of lysoleci-
thin (lysophosphatidylcholine). What
2. A chemical laboratory received a enzyme leads to accumulation of
drug that is a mixture of glucose and lysolecithin in blood?
mannose. To identify these substances
in the mixture the following method A. Phospholipase A2
can be applied: B. Phospholipase A1
C. Phospholipase C
A. Thin-layer sorbent D. Phospholipase D
chromatography E. Neuraminidase
B. Polarimetry
C. Spectrophotometry 7. Sulfanilamide drugs contain pri-
D. Polarography mary aromatic amides in their
E. Amperometric titration structure. Specify the method of
quantitative determination of these
3. Microscopy of leaf epidermis of compounds:
Lamiaceae (Labiatae) family plants
revealed that both accessory cells A. Nitritometry
are perpendicular to a stoma. Such B. Iodometry
stomata are called: C. Dichromatometry
D. Permanganatometry
A. Diacytic E. Cerimetry
B. Paracytic
C. Anisocytic 8. Microbe survival within envi-
D. Anomocytic ronment is facilitated by spore
E. Tetracytic formation. What microorganisms of
those listed below are spore formers:
4. Stem thickening occurs due
to functioning of the following A. Clostridia
structures: B. Bacteroides
C. Staphylococci
A. Lateral meristem D. Peptococci
B. Apical meristem E. Peptostreptococci
C. Wound meristem
D. Intercalary meristem 9. Dissociation degree in 0,01 M
E. Endoderm water solution is the same for all the
strong electrolytes listed below. Name
5. A patient has developed anuria. the substance with the highest boiling
Blood pressure is 50/20 mm Hg. What temperature:
process of uropoiesis caused acute
decrease of urination? A. Al2 (SO4 )3
B. KCl
C. N a3 P O4
D. Cu(N O3 )2
E. K3 P O4
10. Proteins are of great importance
190
krok123.in.ua
Кrok 1 Pharmacy (англомовнi студенти) 2016 рiк 2

for vital functions. What value of pH


results in zero electrophoretic mobili- A. Vaccinium vitis-idaea
ty of gelatin (gelatin isoelectric point B. Arctostaphilos uva-ursi
equals 4,7)? C. Vaccinium oxycoccus
D. Vaccinium myrtillus
A. 4,7 E. Ledum palustre
B. 7,0
C. 14,0 15. In an emergency situation a scuba
D. 5,5 diver has quickly risen from the
E. 9,4 depth to the surface in violation of
safety regulations. He is unconsci-
11. Enzymes are widely used as drugs ous, presents with respiratory fai-
in pharmacy. What is the main di- lure and cardiac activity disorder as
fference that separates enzymes from the result of decompression sickness.
non-biological catalysts? What complication can develop in the
scuba diver?
A. High specificity and selectivity
B. High universality A. Gas embolism
C. Low universality B. Fat embolism
D. High dispersion C. Air embolism
E. High homogeneity D. Cellular embolism
E. Thromboembolism
12. A patient with chronic constipati-
on had been prescribed bisacodyl. 16. Parents of a 10-year-old chi-
After 3 weeks of treatment the pati- ld have made an appointment wi-
ent noticed a reduction of laxati- th endocrinologist due to complai-
ve effect. This is caused by the nts of the child’s low height. The
development of the following side- child’s appearance is correspondi-
effect: ng with that of a 5-year-old. What
hormone causes such changes in
A. Tolerance physical development, if its secreti-
B. Dependence on is disrupted?
C. Sensibilization
D. Cumulation A. Somatotropic hormone
E. Disbacteriosis B. Adrenocorticotropic hormone
C. Thyroxin
13. Solution under analysis received D. Testosterone
chloroform and, drop by drop, chlori- E. Insulin
ne water. Chloroform layer colored
orange, which indicates the presence 17. A patient complains of
of: tachycardia, insomnia, weight loss,
irritability, sweating. Objectively:
A. Bromide ions the patient has goiter and sli-
B. Iodide ions ght exophthalmos. What gland is
C. Sulfite ions affected, and what functional di-
D. Sulfate ions sorder is it?
E. Nitrate ions
A. Hyperthyroidism
14. A species of Ericaceae family is B. Hypothyroidism
characterized by the following type C. Hyperparathyroidism
of leaves: alternate leaf arrangement, D. Hypoparathyroidism
short footstalk, leathery, elliptic or E. Adrenomedullary hyperfunction
obovate with retuse tip, downturned
edges; upper surface is dark-green, 18. In titration analysis aimed
lower surface is light-green with at determining the substances by
punctate glandules. Name this speci- means of mercurimetry the followi-
es: ng substance can be used as the indi-
191
krok123.in.ua
Кrok 1 Pharmacy (англомовнi студенти) 2016 рiк 3

cator:
A. Chingamin
A. Diphenylcarbazide B. Rifampicin
B. Potassium chromate C. Ampicillin
C. Eriochrome black T D. Gentamicin
D. Starch E. Biseptol (Co-Trimoxazole)
E. Tropeolin OO
24. A sample section of an axial
19. A patient has been hospitalised body shows a complex consisting of
with pneumonia. What kind of respi- phellogen and its derivatives - cork
ratory failure does the patient have? and phelloderm. This tissue is called:
A. Restrictive A. Periderm
B. Obstructive B. Colenchyma
C. Central C. Sclerenchyma
D. Peripheral D. Epiblema
E. Thoracic diaphragm E. Epidermis
20. In the state of fright the followi- 25. The Mohr method is used to
ng signs can be observed: acute pallor determine mass concentration of
of face, tremor of extremities. What sodium chloride in an isotonic soluti-
kind of ischemia can be observed in on. Titration is carried out with the
such a condition? following indicator:
A. Angiospastic A. Potassium chromate
B. Compression B. Fluorescein
C. Obstructive (thrombus) C. Ammonium iron (III) sulfate
D. Metabolic D. Diphenylcarbazone
E. Obstructive (vascular wall thickeni- E. Ferroin
ng)
26. An injured person exhibits
21. What are the indications for the the following signs at the site of
use of naloxone? trauma: skin redness, throbbing small
arteries, elevated local temperature,
A. Acute intoxication with narcotic increased tissue turgor. What local
analgesics blood circulation disorder are these
B. Heavy metals intoxication presentations typical of?
C. Intoxication with cardiac glycosides
D. Intoxication with ergot alkaloids A. Arterial hyperemia
E. Atropine sulfate intoxication B. Venous hyperemia
C. Thrombosis
22. Thermodynamic calculations D. Embolism
allow us to determine the possibi- E. Ischemia
lity and direction of spontaneous
processes. In an isolated system 27. Racemose clusters of calci-
the change of the following um carbonate crystals are detected
thermodynamic function is used for among the waste products of a
this purpose: protoplast. These crystals are:
A. Entropy A. Cystoliths
B. Gibbs energy B. Isolated crystals
C. Helmholtz energy C. Raphides
D. Internal energy D. Styloids
E. Enthalpy E. Druses
23. What drug should be administered 28. Purine ring biosynthesis occurs
for individual prevention of malaria? in ribose-5-phosphate by gradual
192
krok123.in.ua
Кrok 1 Pharmacy (англомовнi студенти) 2016 рiк 4

accumulation of nitrogen and carbon A. Temperature


atoms and closing the rings. The B. Pressure
source of ribose phosphate is the C. Volume
process of: D. Concentration
E. Reaction time
A. Pentose phosphate cycle
B. Glycolysis 33. Uric acid is a derivative of:
C. Glyconeogenesis
D. Gluconeogenesis
E. Glycogenolysis
29. Number of freedom degrees at the
point of intersection of liquidus with
Y-axis on the fusibility chart of a two-
component system would equal: A. Purine
A. 0 B. Indole
B. 2 C. Pyrazine
C. 3 D. Pyrazole
D. 4 E. Pyridine
E. 1 34. In terms of water-air interface the
30. The fourth group of cations following substance is a surfactant:
includes the following cations: Al3+, A. Valeric acid
Sn2+ , Sn(IV ), As(V ), As(III), B. HCl
Zn2+ , C 3+. The group reagent for the C. N aOH
fourth group of cations is the solution D. Urea
of: E. -
A. N aOH , H2 O2 35. At the sixth month of pregnancy
B. HCl a woman has been diagnosed wi-
C. N H3 , H2 O2 th severe iron-deficiency anemia.
D. H2 C2 O4 Appearance of the following
E. H2 SO4 , H2 O2 elements in her blood became the di-
agnostic character:
31. In the qualitative analysis that
involves precipitation of sulfates of A. Hypochromic erythrocytes
the third analytical group cations B. Macrocytes
(Ca2+ , Sr2+ , Ba2+ ) the solubility of C. Megalocytes
sulfates can be reduced by adding: D. Reticulocytes
E. Erythroblasts
A. Ethyl alcohol
B. Distilled water 36. What compound produces
C. Benzene phthalic acid during oxidation?
D. Chloroform
E. Amyl alcohol
32. Reaction rate constant values
allow to draw conclusions regardi-
ng processes of synthesis of various
drugs. What factor affects reaction
rate constant?

193
krok123.in.ua
Кrok 1 Pharmacy (англомовнi студенти) 2016 рiк 5

A. of a child with signs of purulent lesi-


on of brain tunics revealed gram-
negative bean-shaped diplococci.
What provisional diagnosis can be
made based on the analysis results?
B.
A. Meningitis
B. Gonorrhea
C. Cholera
D. Plague
E. Anthrax

C. 40. Aqueous solution of CaCl2 with


10% mass concentration is used for
intravenous injections. What is the
maximum value of isotonic coeffici-
ent of CaCl2 in an aqueous solution?
D. A. 3
B. 4
C. 2
D. 5
E. 1
E.
41. Which of these reactions can be
used to identify the primary amino
group?

37. To distinguish between phenol and


salicylic acid the following reagent is
used:

A. Sodium bicarbonate solution


B. Iron (III) chloride solution
C. Sodium hydroxide solution
D. Sodium chloride solution
E. Bromine solution
38. Nitrite ions in presence of nitrate
ions can be detected with:
A. Crystalline antipyrine in presence
of diluted HCl
B. Crystalline sodium thiosulfate
C. Dimethylglyoxime
D. Crystalline iron (III) sulfate
E. Diphenylcarbazone
39. Analysis of the cerebrospinal fluid
194
krok123.in.ua
Кrok 1 Pharmacy (англомовнi студенти) 2016 рiк 6

A. 44. Sedimentation analysis has been


applied for assessment of air purity in
an aseptic unit of a phamacy. The test
resulted in growth of the small coloni-
es with areas of hemolysis. What
medium was used for inoculation?
B.
A. Blood agar
B. Levine’s agar (Eosin Methylene
Blue agar)
C. Endo agar
C. D. Ploskirev’s agar
E. Egg-yolk salt agar
45. What enzyme allows for synthesis
of various genes from template-RNA
to DNA in genetic engineering (this
enzyme catalyzes the process di-
scovered in RNA-viruses)?
D.
A. Reverse transcriptase
B. Exonuclease
C. DNA-ligase
D. Helicase
E. E. Endonuclease
46. Smears from tonsillar coating of
a patient were stained by Neisser’s
method. Microscopy revealed thin
yellow V-shaped bacilli with dark-
blue grains at their ends. Make the
42. Bacterial culture obtained from preliminary diagnosis:
patient DOES NOT grow when
exposed to oxygen. Conditions sui- A. Diphtheria
table for bacterial culture growth can B. Measles
be created in: C. Tuberculosis
D. Whooping cough
A. Anaerobic culture jar E. Influenza
B. Serum-supplemented medium
C. Pasteur oven 47. According to van’t Hoff rule,
D. Krotov apparatus when the temperature is raised by 10
E. Oxidative medium degrees, the reaction rate increases
by:
43. In the course of long-term
treatment of an infectious pati- A. 2-4 times
ent with penicillin, the pathogen B. 1,5 times
transformed into the L-form. What C. 5 times
changes occur in the pathogen cell in D. 10 times
case of L-transformation? E. Temperature does not affect reacti-
on rate
A. Absence of a cell wall
B. Absence of flagella 48. Potassium dichromate solution
C. Absence of a capsule was added into a solution obtai-
D. Absence of a spore ned after the precipitate consisti-
E. Absence of inclusions ng of group II chloride cations was
processed with hot water. Yellow
195
krok123.in.ua
Кrok 1 Pharmacy (англомовнi студенти) 2016 рiк 7

precipitate was produced; the preci- regulations non-sterile drugs can


pitate is insoluble in acetic acid, but contain certain microorganisms.
soluble in alkali. What cations were Name the microorganisms that
present in the solution under investi- CANNOT be present:
gation?
A. Enterobacteriaceae
A. Lead (II) B. Yeast fungi
B. Mercury (II) C. Micrococci
C. Barium D. Mold fungi
D. Silver (I) E. Sarcinae
E. Calcium
54. According to IUPAC nomenclature
49. Diet of an individual must the following is the name of nicotinic
contain vitamins. What vitamin is acid:
usually prescribed for treatment and
prevention of pellagra?
A. Vitamin P P
B. Vitamin C
C. Vitamin A
D. Vitamin B1
E. Vitamin D A. Pyridine-3-carboxylic acid
B. Pyridine-2-carboxylic acid
50. Thiocyanatometric titration C. Pyridine-4-carboxylic acid
method requires secondary standard D. 4-carboxypyridine
solution of potassium thiocyanate. E. 2-carboxypyridine
This solution is standardized with
standard solution of: 55. Select a name that corresponds
with the formula: CH3 − C ≡ N :
A. Silver nitrate
B. Hydrochloric acid A. Acetic acid nitrile
C. Sulfuric acid B. Acetamide
D. Iron (II) sulfate C. Acetic anhydryde
E. Copper (II) nitrate D. Acetoxime
E. Ethyl isocyanide
51. To identify a drug by means
of thin-layer chromatography the 56. Tryptophan amino acid is a deri-
following parameter is used: vative of:
A. Rf
B. n
C. E , mV
D. I , A
E. Kp
52. Intracellular metabolism of
glycerol starts with its activation. A. Indole
What compound is formed in the fi- B. Coumarin
rst reaction of its conversion? C. Pyridine
D. Imidazole
A. α-glycerolophosphate E. Purine
B. Pyruvate
C. Lactate 57. Flax seeds are used in medicine as
D. Choline coating agents, due to the following
E. Acetyl coenzyme A ability of their secondary membranes:

53. According to the Pharmacopoeia


196
krok123.in.ua
Кrok 1 Pharmacy (англомовнi студенти) 2016 рiк 8

A. Sliming A. Trichloracetic acid


B. Suberization B. Dichloroacetic acid
C. Gummosis C. Chloroacetic acid
D. Lignification D. Acetic acid
E. Mineralization E. Propionic acid
58. Which of the following reactions 62. Reaction of benzene sulfonation
is required to obtain an azo dye out produces:
of an aromatic amine?
A. Diazotization and azo compound
B. Reduction and diazotization
C. Diazotization and interaction with
potassium cyanide
D. Salt formation and nitration A.
E. Alkylation and nitrosation
59. How many asymmetric carbon
atoms and stereoisomers are there in
tartaric acid? B.

C.

A. Two asymmetric atoms and three


stereoisomers
B. One asymmetric atom and two
stereoisomers
C. Two asymmetric atoms and four D.
stereoisomers
D. No asymmetric atoms and no
stereoisomers
E. Two asymmetric atoms and two
stereoisomers E.

60. Chemically, ethers are quite inert


compounds. Ethers decompose even
at a room temperature under the
effect of the following haloid acid:
63. Select the formula of diazonium
A. HI salt:
B. HBr
C. HCl
D. HF
E. HClO
61. Introduction of an electron
acceptor substitute into molecule
is known to increase acid strength.
What substance demonstrates the
most explicit acidic properties?

197
krok123.in.ua
Кrok 1 Pharmacy (англомовнi студенти) 2016 рiк 9

A. A. N H2 N H2
B. N H2 OH
C. CH3 N H2
D. C6 H5 N HN H2
E. N H3
B.
68. Select the compound with
amphoteric properties (which reacts
both with acids and bases and
produces salts):
C. C6 H5 − N = O
D. C6 H5 − N H − C(O) − CH3 A.
E. (CH3 )2 N − N = O
64. A pharmaceutical enterprise
produces a tetanus-specific preventi-
ve drug. Which drug of those listed B.
below is it?
A. Anatoxin
B. Dead vaccine
C. Live vaccine C.
D. Immunoglobulin
E. Recombinant vaccine
65. Inoculation in a nutrient medi-
um was performed to determine
probable contamination of a drug D.
with fungi. The colonies are large,
resembling sour cream. What nutrient
medium had been used in this case?
A. Sabouraud
B. Loewenstein-Jensen medium E.
C. Roux
D. Loeffler
E. FINN-II
66. Aniline can be converted into the
water-soluble salt, if processed with 69. Select the formula of pentene-2
the solution of: from the list:

A. Hydrochloric acid A. CH3 − CH2 − CH = CH − CH3


B. Sodium hydroxide B. CH3 − CH2 − CH2 − CH2 − CH3
C. Sodium sulfate C. CH3 − CH2 − CH2 − CH = CH2
D. Ethanol D. CH3 − CH2 − CH2 − CH3
E. Dimethylamine E. CH3 − CH = CH − CH3

67. Specify the reagent necessary for 70. The end product of starch
the following transformation: hydrolysis is:
A. D-glucose
B. D-fructose
C. Saccharose
D. Maltose
E. D-galactose

198
krok123.in.ua
Кrok 1 Pharmacy (англомовнi студенти) 2016 рiк 10

71. Determine the compounds of X A.


and Y in the reaction:

A. B.

B.

C.

C. CH3 CHBr2 and HC ≡ CH


D.

D.
E.

E.
72. What reagent can help distinguish
between starch and glucose?
A. I2
B. Br2
C. KMnO4
D. K2 Cr2O7
E. F eCl3 74. What reagent allows to si-
multaneously detect the presence
73. Select the formula, where carbon of both aldehyde group and glycol
atoms numbering complies with fragment in glucose molecule?
IUPAC replacement nomenclature:
A. Cu(OH)2
B. Br2
C. AlCl3
D. F eCl3
E. KMnO4
75. What class of organic compounds
is characterized by the presence of
C ≡ N group?
A. Nitriles
B. Amines
C. Nitro compounds
D. Alcohols
E. Aldehydes
199
krok123.in.ua
Кrok 1 Pharmacy (англомовнi студенти) 2016 рiк 11

76. Accidental ingestion of death cap


mushrooms containing α-amanitin A. A
causes intoxication. What enzyme is B. C
inhibited with this toxine? C. K
D. B1
A. RNA polymerase II E. B6
B. DNA polymerase
C. DNA synthetase 82. A 70-year-old patient presents wi-
D. Peptidyl transferase th cardiac and cerebral atherosclerosis.
E. Translocase Examination revealed changes of
blood lipid spectre. Increase of
77. Specify the substance that results the following lipoproteins plays a
from the following reaction: significant role in atherosclerosis
HOH, Hg 2+ pathogenesis:
CH ≡ CH −−−−−−→?
A. Low-density lipoproteins
A. Ethanal B. Very low-density lipoproteins
B. Ethanol C. Intermediate density lipoproteins
C. Propanal D. High-density lipoproteins
D. Propanone E. Chylomicrons
E. Acetic acid
83. During containment measures
78. Choose the indicator and titrati- following Chornobyl Nuclear Power
on method to determine hydrogen Plant disaster a worker has been
carbonate ions in a drug: exposed to a dose of ionizing emi-
A. Methyl-orange, acidimetry ssion of 6 Gy (600 R). The worker
B. Phenolphthalein, acidimetry complains of general fatigue, nausea,
C. Methyl-orange, alkalimetry dizziness, labile blood pressure and
D. Phenolphthalein, alkalimetry heart rate, short-term leukocytosis
E. Murexide, acidimetry with lymphopenia. What stage of
acute radiation sickness can be
79. What titrant is used in characterized by such presentations?
bromatometry?
A. Prodromal
A. KBrO3 B. Manifest
B. KBr C. Latent
C. Br2 D. Recovery
D. KBrO4 + KCl E. Long-term consequences
E. KBrO4
84. During influenza epidemic a
80. If the amount of a high-molecular patient with severe case of disease
substance added to the sol is very developed hacking cough and chest
small, it can not increase but decrease pain; signs of focal pneumonia were
its stability. This phenomenon is visible on X-ray. Microscopy of
called: sputum detected large number of
pneumococci. What type of infecti-
A. Sensibilization on is it?
B. Solubilization
C. Mutual coagulation A. Secondary
D. Colloidal protection B. Superinfection
E. Sol adaptation C. Abortive
D. Relapse
81. An ophthalmologist has detected E. Reinfection
increased time of dark adaptation in a
patient. What vitamin deficiency can 85. A factory producing typhoid fever
result in such symptom? vaccine cultivates bacteria of virulent
strain in optimal nutrient medium.
200
krok123.in.ua
Кrok 1 Pharmacy (англомовнi студенти) 2016 рiк 12

Then the cells are separated from A. 3, 5-phosphodiester


culture fluid by means of centrifugati- B. Peptide
on and processed with formalin. What C. Glycosidic
type of vaccine is it? D. Disulfide
E. Amide
A. Inactivated
B. Attenuated 90. Quercus robur leaves have the
C. Chemical following type of lamina shape and
D. Anatoxin division:
E. Autovaccine
A. Pinnatilobate
86. Spore and pollen analysis revealed B. Trilobate
tetrahedral spores with a semi- C. Pinnatipartite
circular base and reticular surface in D. Palmatilobate
the pollen. It is the pollen of: E. Palmatipartite
A. Lycopodiophyta 91. Natural peptides can perform
B. Equisetiphyta various functions. What bioactive
C. Bryophyta peptide is a major antioxidant and
D. Polypodiophyta performs coenzyme functions?
E. Pinophyta
A. Glutathione
87. A patient demonstrates milky- B. Bradykinin
white color of blood plasma due to C. Oxytocin
high content of chylomicrons. Di- D. Liberin
sintegration of triacylglycerol is di- E. Anserine
srupted. Deficiency of the following
enzyme activity is observed: 92. Microbiological purity of tableted
drugs has been tested at a factory.
A. Lipoprotein lipase Samples cultivation in mannitol salt
B. Amylase agar resulted in growth of golden-
C. Tripsin yellow colonies, microscopic exami-
D. Cholesterol esterase nation of colonies detected gram-
E. Lactase positive globular bacteria positi-
oned in clusters; microorganisms had
88. A woman noticed that a cut on plasma coagulation prorerties. What
her skin was still bleeding even after pure bacterial culture was obtained?
20 minutes had passed. What vitamin
deficiency causes such condition? A. Staphylococcus aureus
B. Enterobacteriaceae
A. Vitamin K C. Staphylococcus epidermidis
B. Vitamin A D. Staphylococcus saprophyticus
C. Vitamin D E. Pseudomonas aeruginosa
D. Vitamin E
E. Vitamin B12 93. Mosaic discoloration of leaves has
been detected at medicinal plantati-
89. Primary structure of nucleic acids ons. What microorganisms are the
is a polynucleotide chain that has a cause of such damage?
certain composition and order of the
nucleotides. What bonds stabilize this A. Plant-pathogenic viruses
structure? B. Plant-pathogenic bacteria
C. Plant-pathogenic fungi
D. Protozoa
E. Rickettsia
94. Contrykal is used to prevent
pancreatic autolysis. This drug is the
201
krok123.in.ua
Кrok 1 Pharmacy (англомовнi студенти) 2016 рiк 13

inhibitor of the following enzymes: A. B12 -folic acid deficiency


B. Iron-deficiency
A. Proteases C. Acquired hemolytic
B. Lipases D. Hereditary hemolytic
C. Glycosidases E. Chronic posthemorrhagic
D. Nucleases
E. Synthetases 99. Morphological analysis of poplar
inflorescence shows that it is a si-
95. A dry-heat box is used for mple monopodial inflorescence: main
sterilization of various materials axis is drooping, the flowers are sessi-
and instruments in a bacteriological le, unisexual. Specify the type of
laboratory. This sterilization method inflorescence:
can be applied to the following
objects: A. Catkin
B. Head
A. Glass test tubes C. Capitulum
B. Rubber gloves D. Cyme
C. Simple nutrient medium E. Panicle
D. Wire inoculating loops
E. Physiological solution 100. After drinking milk a 1-year-old
child developed diarrhea, flatulence.
96. A sample of water used in The baby is likely to have deficiency
drug production has been sent to a of the following enzyme:
laboratory for sanitary and virological
analysis. Presence of what virus group A. Lactase
will be indicative of faecal contami- B. Maltase
nation of water and, thus, the need C. Aldolase
for its additional purification? D. Hexokinase
E. Glycosidase
A. Picornaviridae
B. Herpesviridae 101. Patients with severe depressi-
C. Orthomyxoviridae on demonstrate decreased serotonin
D. Retroviridae levels in brain and cerebrospinal
E. Flaviviridae fluid. What aminoacid is a serotonin
precursor?
97. In potentiometric titration the
following indicator electrode is used A. Tryptophan
for quantitative determination of B. Threonine
chloride and borate acids in their mi- C. Tyrosine
xture: D. Glutamic acid
E. Aspartic acid
A. Glass
B. Silver-chlorine 102. During reaction of silver cati-
C. Silver ons identification first HCl and then
D. Platinum ammonia solution have been added
E. Calomel to the solution. What compound was
produced as the result?
98. An elderly man exhibits low levels
of red blood cells and hemoglobin A. [Ag(N H3 )2 ]Cl
in blood; however, his color index is B. [Ag2 (N H3 )3 ]Cl
1,3. Blood smear analysis revealed C. AgOH
megaloblasts. What type of anemia D. AgCl
is observed in this case? E. [Ag(N H3)3 ]Cl
103. Microscopic study of soy bean
seeds stained with Sudan III revealed
202
krok123.in.ua
Кrok 1 Pharmacy (англомовнi студенти) 2016 рiк 14

droplets of various sizes. These A. SO42−


droplets are: B. Cl−
C. Cu2+
A. Lipids D. N a+
B. Proteins E. J −
C. Starch
D. Inulin 108. Fatty acids synthesis occurs in
E. Glycogen human body. What compound is initi-
al in this process?
104. In a chemical analytical
laboratory a chemist investigates a A. Acetyl coenzyme A
solution of anion mixture. When B. Vitamin C
antipyrin solution is added it colors C. Glycine
emerald-green. This analytical effect D. Succinate
signifies presence of the following E. Cholesterol
anions:
109. Specify the standard solutions
A. Nitrite that are used in permanganatometry
B. Nitrate to quantify the oxidants by means of
C. Acetate back titration:
D. Tartrate
E. Citrate A. Potassium permanganate, iron (II)
sulfate
105. Selective solvents are used in B. Potassium dichromate, sodium
laboratories and factories to isolate thiosulfate
and refine essential oils, alkaloids, C. Potassium bromate, sodium thi-
antibiotics and other pharmaceutical osulfate
substances. This process is called: D. Potassium iodate, sodium thi-
osulfate
A. Extraction E. Cerium (IV) sulfate, iron (II)
B. Sedimentation sulfate
C. Coagulation
D. Flocculation 110. What indicator is used to fix the
E. Flotation endpoint of mercurimetric titration?
106. When preparing a solution, an A. Thiocyanate complexes of iron
analytical pharmacist converted a (III)
freshly formed precipitate into a sol B. Fluorescein
by treating it with an electrolyte C. Eosin
solution. What method of obtaini- D. Murexide
ng disperse systems was used by the E. Potassium chromate
pharmacist?
111. A patient with alcoholic cirrhosis
A. Peptization complains of general weakness and
B. Physical condensation dyspnea. The following is revealed:
C. Chemical condensation decrease of blood pressure, asci-
D. Solvent exchange tes, dilation of superficial vei-
E. Condensation from steam ns of the stomach anterior wall,
107. Sol of iron (III) hydroxide is posi- esophageal varicose veins dilatation,
tively charged. Specify the ion that splenomegaly. What hemodynamics
has the lowest coagulation threshold: disorder does the patient suffer from?

203
krok123.in.ua
Кrok 1 Pharmacy (англомовнi студенти) 2016 рiк 15

A. Portal hypertension um chloride. Name the titrant of this


B. Left ventricular failure method:
C. Right ventricular failure
D. Cardiac insufficiency A. Ammonium thiocyanate
E. Collapse B. Mercury (I) nitrate
C. Sodium tetraborate
112. Pharmacy has received virici- D. Mercury (II) nitrate
des. Choose the viricide for influenza E. Sodium hydroxide
treatment from the list given below:
117. What cation of the 4th analyti-
A. Rimantadine cal group is present in a solution, if
B. Metisazone its reaction with the group reagent
C. Levamisole results in formation of yellow precipi-
D. Azidothimidine tate?
E. Acyclovir
A. Cr3+
113. Colloid silver preparations B. Zn2+
Protargolum and Collargolum are C. Sn2+
widely used in medical practice as D. Al3+
bactericidal drugs. In addition to E. Sn(IV )
the active ingredients, these drugs
contain protein compounds. What 118. A patient has icteric skin;
is the function of proteins in these unconjugated bilirubin content in
preparations? blood is high; conjugated bilirubin
in urine is not detected. There is si-
A. Prevention of colloidal solution gnificant amount of urobilin in uri-
coagulation ne and stercobilin in feces. Name
B. Prolongation of shelf-life the pathology characterized by given
C. Reduction of side effects symptoms:
D. Improvement of drug technology
E. Potentiation of bactericidal action A. Hemolytic jaundice
of silver B. Obstructive jaundice
C. Jaundice of the newborn
114. Pharmacies receive large D. Hepatocellular jaundice
amounts of sterile medical products E. Atherosclerosis
(dressing, rubber gloves, catheters,
etc.). What ensures their sterility duri- 119. What anion of the 2nd analytic
ng production? group produces black precipitate wi-
th the group reagent AgN O3 ?
A. Alpha irradiation
B. Beta irradiation A. S 2−
C. Gamma irradiation B. I −
D. Infrared irradiation C. Cl−
E. Ultraviolet irradiation D. Br−
E. N CS −
115. What type of conducting bundles
is characteristic of all root zones of 120. You are studying a silvery downy
one-seeded plants? plant of Asteraceae family, which is
rich with essential oils and bitters.
A. Radical Harvested are apical sprouts with
B. Central phloem (Amphivasal) panicle of small round flower heads.
C. Central xylem (Amphicribal) This plant is:
D. Bilateral
E. Collateral
116. The Volhard method is used to
define mass concentration of sodi-
204
krok123.in.ua
Кrok 1 Pharmacy (англомовнi студенти) 2016 рiк 16

A. Artemisia absinthium A. Protein glycosylation


B. Arctium lappa B. Lipolysis
C. Bidens tripartita C. Ketogenesis
D. Calendula officinalis D. Protein proteolysis
E. Chamomilla recutita E. Gluconeogenesis
121. A 46-year-old patient was found 126. Common nettle, hop, and black
to have hyperactivity of creatine elderberry require soils rich in ni-
kinase in the blood serum. What trogen compounds. Such plants are
pathology can be suspected? called:
A. Myocardial infarction A. Nitrophyles
B. Acute pancreatitis B. Nitrophobes
C. Chronic hepatitis C. Calciphiles
D. Hemolytic anemia D. Calciphobes
E. Renal failure E. Halophytes
122. Emulsions containing under 127. A man received a radiation
0,1% of dispersed phase (in volume) dose of 30 Gy. He presents with
are classified as: necrotic angina, disorders of the
gastrointestinal tract. Blood tests
A. Diluted revealed anemia, leukopenia and
B. Concentrated thrombocytopenia. What stage of
C. High-concentration acute radiation sickness is observed
D. Water-in-oil type in the patient?
E. Oil-in-water type
A. Manifest illness stage
123. A patient with pulmonary carci- B. Prodromal stage
noma has developed a case of pleuri- C. Latent stage
sy. Large amount of hemorrhagic D. Recovery
exudate was obtained for analysis. E. -
What component is specific for
hemorrhagic exudate? 128. A patient with hyperproduction
of thyroid hormones has been prescri-
A. Erythrocytes bed Merkazolilum. This drug inhibits
B. Leukocytes the following enzyme participating in
C. Platelets iodothyronine synthesis:
D. Fibrin
E. Pus A. Iodide peroxidase
B. Aromatase
124. A pharmaceutical enterprise C. Reductase
offers wide range of antimicrobial D. Decarboxylase
agents. Select the broad spectrum E. Aminotransferase
antimicrobial agent:
129. A patient with croupous
A. Tetracycline pneumonia presents with sharp
B. Rimantadine increase of body temperature up to
C. Nystatin 39oC , which persisted for 9 days
D. Griseofulvin with daily amplitude of 1 degree.
E. Phthalazolum What temperature curve could be
observed?
125. Cataract (lenticular opacity) has
developed in a 52-year-old woman wi-
th diabetes mellitus. Lenticular opaci-
ty was caused by intensification of the
following processes:
205
krok123.in.ua
Кrok 1 Pharmacy (англомовнi студенти) 2016 рiк 17

A. Stable an admission room. The skin is


B. Hectic hyperemic and dry; pupils are dilated;
C. Septic tachycardia is observed. The pati-
D. Recurrent ent was diagnosed with belladonna
E. Atypical alkaloids intoxication. What drug
would be advisable?
130. Employees of an enterprise were
vaccinated with ”Influvac” for specific A. Proserin
prevention of influenza. What type of B. Aceclidine
immunity will develop in those vacci- C. Pilocarpine
nated? D. Armin
E. Dipiroxim
A. Artificial active
B. Innate congenital 135. A patient with signs of cardiac
C. Artificial passive glycosides intoxication was prescri-
D. Natural active bed Unithiol. What is the mechanism
E. Natural passive of drug action in this case?
131. Soil microflora often includes A. Reactivation of membrane +,
representatives of pathogenic mi- N a+ -adenosine triphosphatase
croorganisms. Specify the diseases wi- B. Binding of ionized Ca2+
th causative agents that remain viable C. Increased permeability of K + into
in the soil for a long time: myocardiocytes
A. Tetanus and gas anaerobic infecti- D. Increased N a+ content in
on myocardium
B. Tuberculosis and mycobacterioses E. Induction of cardiac glycoside
C. Colibacillosis and cholera metabolism
D. Leptospirosis and plague 136. A woman suffering from
E. Typhoid fever and dysentery neurosis has disturbed sleep. What
132. Removal of low-molecular drug is optimal for insomnia
impurities from colloidal systems treatment?
and high-molecular compound soluti- A. Nitrazepam
ons by means of semipermeable B. Phenobarbital
membrane diffusion is called: C. Aethaminalum-natrium
A. Dialysis (Pentobarbital)
B. Electrodialysis D. Bromisoval
C. Ultrafiltration E. Valerian tincture
D. Decantation 137. Select the halogenated antiseptic
E. Compensatory dialysis that would be preferable for a child
133. Neuroleptanalgesia has been to pack in the first aid kit, when going
applied in the case of cardiac infarcti- to a summer camp:
on. What neuroleptic is most often A. Iodine alcoholic solution
applied along with fentanyl? B. Brilliant green
A. Droperidol C. Copper sulfate
B. Perphenazine (Aethaperazinum) D. Methylene blue
C. Levomepromazine E. Formaldehyde solution
D. Clozapine 138. A patient consulted an
E. Sulpiride ophthalmologist about deterioration
134. A patient complaining of dry of twilight vision and xerophthalmus.
mouth, photophobia, and visual What drug should the doctor prescri-
impairment has been delivered into be?
206
krok123.in.ua
Кrok 1 Pharmacy (англомовнi студенти) 2016 рiк 18

144. An elderly patient suffers from


A. Retinol constipation caused by large intesti-
B. Pyridoxine ne hypotonia. What drug should be
C. Tocopherol prescribed?
D. Ascorbic acid
E. Cocarboxylase A. Bisacodyl
B. Sodium sulfate
139. Due to prolonged taking of C. Castor oil
phenobarbital an epileptic patient has D. Atropine sulphate
developed tolerance for this drug. E. Procainamide
What is this phenomenon based on?
145. A 25-year-old woman with signs
A. Biotransformation acceleration of acute morphine intoxication was
B. Absorption process weakening administered naloxone, which rapidly
C. Increase of receptor sensitivity improved her condition. What is the
D. Biotransformation suppression mechanism of action of this drug?
E. Substance accumulation in body
A. Opioid receptor blockade
140. Decreased absorption of B. GABA receptor blockade
tetracyclines, if they are taken si- C. Serotonin receptor blockade
multaneously with antacids, is an D. Dopamine receptor blockade
example of their: E. Benzodiazepine receptor blockade
A. Pharmacokinetic incompatibility 146. A patient with rheumatoid
B. Pharmaceutical incompatibility arthritis and concomitant duodenal
C. Pharmacodynamic incompatibility ulcer has to be prescribed nonsteroid
D. Synergism anti-inflammatory drug. Which one
E. Functional antagonism of the drugs listed below is a drug of
choice in the given case?
141. A patient with epilepsy is prescri-
bed a diuretic. Name this drug: A. Celecoxib
B. Acetylsalicylic acid
A. Diacarb (Acetazolamide) C. Paracetamol
B. Verospiron D. Metamizole
C. Furosemide E. Diclofenac sodium
D. Hypothiazid (Hydrochlorothiazi-
de) 147. A 48-year-old patient has been
E. Mannitol intravenously administered predni-
solone solution to arrest severe attack
142. What side effect is characteristic of bronchial asthma. What group of
of captopril? hormonal agents does prednisolone
A. Dry cough belong to?
B. Increased blood pressure A. Glucocorticoids
C. Hyperglycemia B. Gestagenic drugs
D. Cardiac rate disorder C. Estrogenic drugs
E. Hypokalemia D. Mineralocorticoid
143. Paracetamol belongs to the E. Anabolic steroids
following pharmacological group: 148. Choose the most efficient way of
A. Nonnarcotic analgetics convallariae glycoside administration
B. Soporifics for acute heart failure treatment:
C. Diuretics
D. Hypotensive drugs
E. Antianginal drugs

207
krok123.in.ua
Кrok 1 Pharmacy (англомовнi студенти) 2016 рiк 19

A. Intravenous A. Amlodipine
B. Intramuscular B. Atenolol
C. Subcutaneous C. Guanethidine
D. Internal D. Reserpine
E. Inhalational E. Labetalol
149. What drug should be admini- 154. Colored or white component of
stered in case of acute cardiac insuffi- double perianth, which consists of
ciency? petals, is a:
A. Corglycon A. Corolla
B. Salbutamol B. Flower cup
C. Pilocarpine hydrochloride C. Androecium
D. Naloxone D. Gynoecium
E. Heparin E. Perigonium
150. A patient with signs of mercury 155. Weeds can be harmful for
poisoning has been delivered into populace’s wellbeing. Particularly,
an admission room. What antidote allergic reactions are often caused by
should be prescribed in this case? the following plant in its period of
blossoming:
A. Unithiol
B. Atropine sulfate A. Ambrosia artemisiifolia
C. Proserin B. Equisetum arvense
D. Naloxone C. Stellaria media
E. Calcium chloride D. Erigeron canadensis
E. Taraxacum officinale
151. Diuretic should be prescribed to
treat cerebral edema. What drug is to 156. Aurococcus culture was obtained
be administered? from the nasal cavity of a child sufferi-
ng from chronic tonsillitis. Causative
A. Furosemide agent’s sensitivity towards a number
B. Hydrochlorothiazide of antibiotics was tested to choose the
C. Caffeine and sodium benzoate optimal drug. What drug WAS NOT
D. Diacarb (Acetazolamide) included in antibiotic susceptibility
E. Spironolactone testing?
152. A doctor has prescribed a A. Nystatin
nonsteroidal anti-inflammatory drug B. Ampicillin
to relieve inflammation and pain C. Tetracycline
syndrome. Name this drug: D. Levomycetin (Chloramphenicol)
A. Diclofenac sodium E. Erythromycin
B. Glibenclamide 157. Interferons are natural antiviral
C. Loratadine and antitumor agents. What is their
D. Prednisolone mechanism of action?
E. Calcium chloride
A. Protein synthesis depression
153. The 55-year-old patient has B. Protein synthesis increase
been diagnosed with angina pectoris. C. Replication activation
Calcium channel-blocking agent was D. Transcription activation
prescribed for treatment. Name this E. Repair activation
agent:
158. A patient of a neurology unit
suffers from paralysis of all limbs.
Name this condition:

208
krok123.in.ua
Кrok 1 Pharmacy (англомовнi студенти) 2016 рiк 20

A. Tetraplegia A. Potassium dichromate


B. Paraplegia B. Potassium chloride
C. Hemiplegia C. Potassium iodide
D. Paresis D. Potassium nitrate
E. Hypodynamia E. Sodium hydroxide
159. The most severe and dangerous 163. The second stage of detoxificati-
complication of diabetes mellitus is on involves joining certain chemical
hypoglycemic coma that is characteri- compounds with functional groups of
zed by loss of consciousness and toxines. Select one such compound:
is lethal, unless efficient emergency
treatment is received by patient. A. Glucuronic acid
What is the main pathogenetic B. Higher fatty acids
component of hypoglycemic coma? C. Cholesterol
D. Glucose
A. Carbohydrate deficiency and low E. Pyruvate
energy of cerebral neurons
B. Carbohydrate deficiency and low 164. What forms from an ovule after
energy of myocardium cells fertilization of flowering plants?
C. Blood hyperosmia
D. Noncompensated ketoacidosis A. Seed
E. Respiratory alkalosis B. Gametophyte
C. Sporophyte
160. A patient demonstrates D. Fruit
symmetrical dermatitis on the palms. E. Endosperm
A doctor made a diagnosis of
pellagra. What vitamin deficiency can 165. A woman complains of nausea,
result in such symptoms? vomiting, skin itch. She was di-
agnosed with mechanical jaundice.
A. Nicotinic acid What is the possible cause of skin itch
B. Cobalamin in such a condition?
C. Ascorbic acid
D. Folic acid A. Bile acids accumulating in the
E. Cholecalciferol blood
B. Increased blood content of indirect
161. A ready-made drug was bilirubin
inoculated on Sabouraud’s agar and C. Cholesterol accumulating in the
incubated under 22o C for 5 days. blood
This nutrient medium was used to D. Direct bilirubin appearing in the
determine the following: blood
E. Erythrocyte disintegration products
A. Number of mold and yeast fungi accumulating in the blood
B. Total number of bacteria
C. Presence of E. coli 166. When determining oxidizing
D. Presence of S. aureus agents by means of iodometry in
E. Presence of Salmonella the presence of starch the following
phenomenon can be observed at the
162. Specify the reagent allowing titration end point:
to determine barium cations in the
presence of calcium and strontium A. Blue coloring disappears
cations: B. Red coloring appears
C. Green coloring of precipitate
appears
D. Green coloring of solution di-
sappears
E. White precipitate occurs
209
krok123.in.ua
Кrok 1 Pharmacy (англомовнi студенти) 2016 рiк 21

167. Corolla of a zygomorphic bi- A. Paracetamol


sexual flower consists of 5 petals: B. Acetylsalicylic acid
the largest one is called banner, two C. Diclofenac
lateral - wings, and two fused together D. Ketorolac
- keel. This corolla is characteristic of E. Ibuprofen
Fabacea family and is called:
172. A woman is to be prescribed a
A. Papilionaceous narcotic analgesic for labor pain reli-
B. Lingulate ef. What drug is indicated in this case?
C. Rotate
D. Funnelform A. Promedol (Trimeperidine)
E. Tubular B. Morphine
C. Papaveretum (Omnopon)
168. During ultrasound investigati- D. Codeine
on a patient has been diagnosed wi- E. Fentanyl
th bilateral stenosis of renal artery
with atherosclerotic genesis. Speci- 173. A woman, who during the 5th-
fy the bioactive substance that due 10th weeks of her pregnancy had
to its excessive secretion is the key been taking sodium valproate for
component of arterial hypertension treatment of epilepsy, gave birth to a
pathogenesis in the given case: child with pathology of the vertebral
column (split spine). What side effect
A. Renin of the drug caused such malformati-
B. Cortisol on?
C. Vasopressin
D. Noradrenaline A. Teratogenic
E. Thyroxin B. Mutagenic
C. Embryotoxic
169. A plant producing essential oil D. Fetotoxic
has square stem, bilabiate corolla, E. Sensitizing
coenobium fruit. These features are
characteristic of: 174. Analysis of a dry substance
always begins with preliminary tests.
A. Lamiaceae Sample under investigation is green
B. Papaveraceae in color, which allows to conclude the
C. Polygonaceae presence of:
D. Solanaceae
E. Scrophulariaceae A. Chrome (III)
B. Manganese (II)
170. Blood contains erythrocytes with C. Cobalt (II)
sizes of 10−6 m degree as its consti- D. Iron (III)
tuent parts. What type of disperse E. Barium (II)
system is blood?
175. Students should identify the
A. Microheterogeneous following to determine the sex of a
B. Homogeneous flower:
C. Coarse dispersion
D. Colloidal dispersion A. Stamens and pistils
E. Heterogeneous B. Flower cup and corolla
C. Pedicle and receptacle
171. A patient with headache D. Symmetry
consulted a pharmacist. The patient E. Color and type of indumentum
was prescribed a cyclooxygenase inhi-
bitor - an aminophenol derivative. 176. Disintegration of adenosine
What drug was prescribed? nucleotides results in release of
ammonia. What enzyme plays the key
210
krok123.in.ua
Кrok 1 Pharmacy (англомовнi студенти) 2016 рiк 22

role in ammonia synthesis from these A. N2 O4(gas) ↔ 2N O2(gas)


compounds? B. C(solid) + O2(gas) ↔ CO2(gas)
C. 4HCl(gas) + O2 ↔ 2H2 O(gas) +
A. Adenosine deaminase
B. Alcohol dehydrogenase 2Cl2(gas)
C. Lactate dehydrogenase D. N2(gas) + 3H2(gas) ↔ 2N H3(gas)
D. Alanine transaminase E. CO2(gas) + H2(gas) ↔ CO(gas) +
E. Amylase H2 O(gas)
177. An analytical chemist performs 181. What substance is not a
qualitative analysis of cations of the surfactant at the water solution-air
II analytical group. The following interface?
solution is used to separate silver and
mercury chlorides: A. Sodium chloride
B. Acetic acid
A. Ammonia C. Ethyl alcohol
B. Hydrochloric acid D. Sodium stearate
C. Sodium hydroxide E. Glucose
D. Sodium nitrate
E. Potassium chloride 182. A drop of oil-water emulsion had
been applied to a plate cowered wi-
178. Both scientific and folk medici- th paraffin; no wetting was observed.
ne uses medicinal plant Glycyrrhiza Such phenomenon characterizes this
glabra L. What part of the plant is emulsion as:
harvested?
A. Direct
A. Roots and rhizomes B. Concentrated
B. Foliage C. Diluted
C. Inflorescence D. Stable
D. Grass E. Invert
E. Seeds
183. A patient has been recei-
179. A patient undergoes chemotherapy ving Theophylline (inhibitor of
with 5-fluorouracil that is a competiti- cyclic adenosine monophosphate
ve inhibitor of thymidilate synthase. phosphodiesterase) for a week. What
What process is inhibited by this hormone can increase its action
drug? due to such treatment and cause
hyperglycemia?
A. Thymidine monophosphate
synthesis A. Glucagon
B. Purine nucleotides disintegration B. Testosterone
C. Adenosine triphosphate synthesis C. Aldosterone
D. Purine nucleotides salvage D. Insulin
E. Glucose synthesis E. Estradiol
180. Upon increase of pressure the 184. Burner’s flame colors carmine-
system’s chemical equilibrium will red in the presence of salts of an
shift towards parent substances. Point unknown cation. Name this cation:
out such a system:
A. Strontium
B. Ammonium
C. Sodium
D. Potassium
E. Iron
185. Reaction of sodium ions with
potassium hexahydroxoantimonate
211
krok123.in.ua
Кrok 1 Pharmacy (англомовнi студенти) 2016 рiк 23

(V) in neutral medium produces A. Psychosedatives


precipitate. Specify the color of this B. Cholinergic antagonists
precipitate: C. Adrenergic drugs
D. Diuretics
A. White E. Emetics
B. Red
C. Yellow 190. What cation can be detected wi-
D. Green th Chugaiev’s agent (Dimethylglyoxi-
E. Blue me)?
186. Rhizome of an Asteraceae fami- A. N i2+
ly species is polycephalous, succulent, B. Ca2+
has lysigenous cavities, accumulates C. +
inulin. Such underground organ is D. Mn2+
characterisic of: E. Co2+
A. Inula helenium 191. A substance performs mechani-
B. Hyoscyamus niger cal function; its cells are covered with
C. Digitalis grandiflora uniformly thick lignified membranes.
D. Sorbus aucuparia This substance is:
E. Helianthus annuus
A. Sclerenchyma
187. Osmotic pressure is an important B. Collenchyme
characteristic of biological flui- C. Periderm
ds. Semipermeable membranes D. Cambium
are necessary for penetration of E. Sieve tubes
solvent molecules. What substance
CANNOT be used as a semi- 192. A patient in the state of ketoaci-
permeable membrane? dotic coma presents with loud rapid
respiration: labored expiration with
A. Glass tension of expiratory muscles occurs
B. Biological membrane after deep inspiration. Name the type
C. Collodion film of pathologic respiration:
D. Parchment
E. Gelatine A. Kussmaul’s
B. Cheyne-Stokes’
188. Modelling of immobilization C. Gasping
stress is performed on a test animal - D. Stenotic
guinea pig - that starved for a day. Di- E. Biot’s
ssection revealed hyperemic gastric
mucosa with multiple erosions. What 193. Air contamination with
theory of ulcer formation is confi- pathological microorganisms can be
rmed by this test? anticipated by the presence of indi-
cator bacteria. Specify the bacteria
A. Corticovisceral (stress) that indicate immediate epidemi-
B. Vascular ologic danger:
C. Inflammatory
D. Mechanical A. Hemolytic streptococci
E. Peptic B. Sarcinae
C. Mold fungi
189. What group of drugs is D. Yeast fungi
characterized by development of E. Micrococci
drug addiction as a side effect?
194. Barium carbonate sol has been
obtained from the reaction of excessi-
ve amount of barium chloride soluti-
on with ammonium carbonate soluti-
212
krok123.in.ua
Кrok 1 Pharmacy (англомовнi студенти) 2016 рiк 24

on. The micelle aggregate of obtai- A. Br2 , hν , 20o C


ned sol consists of the following mi- B. H2 SO4 concentrated
crocrystals: C. HN O3 concentrated
D. Cl2 , F eCl3
A. BaCO3 E. CH3COON O2
B. N H4 Cl
C. (N H4 )2 CO3 199. Specify the name of the
D. BaCl2 carbohydrate given in the figure
E. Mixture of BaCO3 with N H4 Cl
195. A 40-year-old man presents
with rapid weight gain after he
had suffered a severe craniocerebral
trauma. On examination the patient’s
weight was 125 kg, with his height bei- according to the systematic
ng 175 cm. What mechanism of obesi- nomenclature:
ty development is the most likely in A. 5,6,6-trimethyl-1-hepten
this case? B. 2,2,3-trimethyl-6-hepten
A. Hypothalamic C. 5-tert-Butyl-1-hexene
B. Alimentary D. 2-tert-Butyl-5-hexene
C. Hormonal E. 2,2,3-trimethyl-6-hexene
D. Hereditary 200. What reaction proves that
E. - phenol has acidic properties?
196. Different structures of a bacteri- A.
al cell perform different functions.
What dispensable component of a
cell ensures its survival within hostile
environment?
A. Spores B.
B. Flagella
C. Capsule
D. Cilia
E. Inclusions
197. Isoniazid
C.

is an antituberculous drug derivative D.


from:
A. Isonicotinic acid
B. Nicotinic acid
C. Picolinic acid
D. Pyrrole-2-carboxylic acid E.
E. N-aminobenzoic acid
198. What substance will react with
propane under the given conditions?

213
krok123.in.ua
Krok 1 Pharmacy (англомовнi студенти) 2017 рiк 1

1. In a child-rearing facility there was A. Lateral meristem


an outbreak of measles. What speci- B. Apical meristem
fic urgent prophylaxis should be admi- C. Wound meristem
nistered to contact UNVACCINATED D. Intercalary meristem
children? E. Endoderm
A. Gamma globulin against measles 6. To determine mass fraction of sodium
B. Measles virus vaccine live chloride in a drug, the Fajans method
C. DPT vaccine should be applied. Titration is to be
D. Medical screening of the children performed in the presence of the followi-
E. Isolation and treatment of infected ng indicator solution:
children
A. Fluorescein
2. Bouguer-Lambert-Beer law is the B. Methyl red
basis of molecular absorption analysis. C. Potassium chromate
According to this law, optical density of a D. Ammonium iron (III) sulfate
solution is: E. Phenolphthalein
A. Directly proportional to layer thi- 7. Sulfanilamide drugs contain primary
ckness and concentration of a substance aromatic amides in their structure. Speci-
B. Directly proportional to layer thi- fy the method of quantitative determi-
ckness and absorption coefficient nation of these compounds:
C. Inversely proportional to layer thi-
ckness and concentration of a substance A. Nitritometry
D. Directly proportional to concentration B. Iodometry
and inversely proportional to layer thi- C. Dichromatometry
ckness D. Permanganatometry
E. Directly proportional to concentration E. Cerimetry
and inversely proportional to absorption 8. Microbe survival within environment
coefficient is facilitated by spore formation. What
3. Within folded parenchyma of a fir microorganisms of those listed below are
needle there are cavernous structures spore formers:
filled with galipot and lined with live A. Clostridia
thin-walled secretory cells. Name these B. Bacteroides
structures: C. Staphylococci
A. Resin ducts D. Peptococci
B. Laticifers E. Peptostreptococci
C. Hydatodes 9. Synthesis of a medicinal substance
D. Glandules occurs in an isolated system. What is
E. Nectar glands a direction criterion of spontaneous
4. A chemotherapeutic agent has processes?
bactericidal effect against streptococci, A. Entropy change
staphylococci, bacilli, and clostridia. B. Gibbs energy
According to its action spectrum this C. Helmholtz energy
drug belongs to the following group: D. Intrinsic energy
A. Broad spectrum antibacterial agents E. Enthalpy
B. Narrow spectrum antibacterial agents 10. Dissociation degree in 0,01 M
C. Broad spectrum antifungal agents water solution is the same for all the
D. Antiviral agents strong electrolytes listed below. Name
E. Antituberculous agents the substance with the highest boiling
5. Stem thickening occurs due to functi- temperature of such solution:
oning of the following structures:

214
krok123.in.ua
Krok 1 Pharmacy (англомовнi студенти) 2017 рiк 2

A. Al2 (SO4 )3 A. Somatotropic hormone


B. KCl B. Adrenocorticotropic hormone
C. Na3 P O4 C. Thyroxin
D. Cu(NO3 )2 D. Testosterone
E. K3 P O4 E. Insulin
11. Proteins are of great importance 16. Dry residue received after evaporati-
for vital functions. What value of pH on of the investigated solution turns
results in zero electrophoretic mobility previously colorless burner flame yellow,
of gelatin (gelatin isoelectric point equals which is observed as violet through blue
4,7)? glass. What cations are there in the dry
residue?
A. 4,7
B. 7,0 A. Na+ , K +
C. 14,0 B. a2+ , K +
D. 5,5 C. Na+ , Sr 2+
E. 9,4 D. Li+ , Ba2+
E. Na+ , Ca2+
12. Enzymes are widely used as drugs
in pharmacy. What is the main di- 17. A smear of purulent excharge from
fference that separates enzymes from urethra contains gram-negative bean-
non-biological catalysts? shaped diplococci with both extra- and
intracellular positions. Make the provisi-
A. High specificity and selectivity onal microbiological diagnosis:
B. High universality
C. Low universality A. Gonorrhea
D. High dispersion B. Syphilis
E. High homogeneity C. Chlamydiosis
D. Trichomoniasis
13. Chloroform and sodium nitrite soluti- E. Candidiasis
on were added into the acidulous investi-
gated solution. The chloroform layer 18. Under isobaric-isothermal condi-
colored red-violet, which indicates the tions the possibility and direction of
presence of: spontaneous processes can be predicted
through change of:
A. Iodide ions
B. Carbonate ions A. Gibbs energy
C. Chloride ions B. Helmholtz energy
D. Sulfate ions C. Enthalpy
E. Fluoride ions D. Entropy
E. Intrinsic energy
14. The following should be used for
sterilization of laboratory glassware in 19. Investigated solution contains
a microbiological laboratory: potassium and ammonium ions. Speci-
fy the reagent that can indicate the
A. Hot-air sterilizer presence of potassium ions in this soluti-
B. Bacteria-excluding filters on:
C. Koch’s steam sterilizer
D. Disinfectant A. Potassium tetraiodomercurate
E. Bactericidal lamps B. Sodium chloride
C. Sodium acetate
15. Parents of a 10-year-old child have D. Potassium hexacyanoferrate (II)
made an appointment with an endocri- E. Uranyl zinc acetate
nologist due to complaints of the chi-
ld’s low height. The child’s appearance 20. An injured person exhibits the
is corresponding with that of a 5-year- following signs at the site of trauma:
old. What hormone causes such changes skin redness, throbbing small arteries,
in physical development, if its secretion elevated local temperature, increased ti-
is disturbed? ssue turgor. What local blood circulation
disorder are these presentations typical
of?
215
krok123.in.ua
Krok 1 Pharmacy (англомовнi студенти) 2017 рiк 3

A. Arterial hyperemia A. Ethylenediamine


B. Venous hyperemia B. Thiocyanate ion
C. Thrombosis C. Cyanide ion
D. Embolism D. Pyridine
E. Ischemia E. Hydroxide ion
21. During investigation of bacterial 26. Name the pH value, under which
contamination of air it is necessary to occurs the most intense color change of
take into account both total amount an indicator:
of microorganisms in a certain volume
and qualitative content of microflora. A. pT value
What microorganisms are the sanitary B. pK value
indicators of air contamination within C. Color change interval
enclosed spaces? D. Equivalence point
E. Titration end point
A. Staphylococcus aureus
B. Colibacillus 27. What compound produces phthalic
C. Hay bacillus acid during oxidation?
D. Yeast fungi
E. Mold fungi
22. Purine ring biosynthesis occurs
in ribose-5-phosphate through gradual
accumulation of nitrogen and carbon
atoms and closing of the rings. The
source of ribose phosphate is the process
of:
A.
A. Pentose phosphate cycle
B. Glycolysis
C. Glyconeogenesis
D. Gluconeogenesis
E. Glycogenolysis B.
23. Number of freedom degrees at the
point of intersection of liquidus with
Y-axis on the fusibility chart of a two-
component system would equal:
A. 0
B. 2 C.
C. 3
D. 4
E. 1
24. In the process of qualitative analysis
to determine strontium ions, so-called D.
”gypseous water” is used. This substance
can be defined as:
A. Concentrated aqueous solution of
CaSO4 E.
B. Solution of Ca(OH)2
C. Concentrated aqueous solution of CO2
D. Aqueous solution of Ba(NO3 )2
E. Solution of (OH)2
25. Which of the ligands is bidentate?
28. To distinguish between phenol and
salicylic acid the following reagent is
used:

216
krok123.in.ua
Krok 1 Pharmacy (англомовнi студенти) 2017 рiк 4

culture growth can be created in:


A. Anaerobic culture jar
B. Serum-supplemented medium
C. Pasteur oven
D. Krotov apparatus
E. Oxidative medium
A. Sodium bicarbonate solution
B. Iron (III) chloride solution 31. Specify the number of electrons
C. Sodium hydroxide solution that participate in creation of closed
D. Sodium chloride solution conjugated system within the pyrimidi-
E. Bromine solution ne molecule:

29. Reaction of benzaldehyde with


chlorine produces:

A. 6
B. 4
C. 10
D. 2
E. 8
A.
32. An oncological patient was prescri-
bed fluorouracil that is a competitive
inhibitor of thymidine synthase. It inhi-
bits the process of:

B. A. Pyrimidine nucleotides synthesis


B. Carbohydrate disintegration
C. Purine nucleotides synthesis
D. Purine nucleotides disintegration
E. Lipids synthesis
33. During feces analysis of a 3-month-
C. old child with signs of enteric infection,
numerous dark-red colonies has grown
on Endo agar. What microorganisms can
be the cause of such enteric infection?
A. Escherichia
B. Streptococci
C. Gonococci
D. D. Salmonellae
E. Shigella
34. Sedimentation analysis has been
applied for assessment of air purity in
an aseptic unit of a phamacy. The test
E. resulted in growth of the small colonies
with areas of hemolysis. What medium
was used for inoculation?
A. Blood agar
B. Levine’s agar (Eosin Methylene Blue
agar)
C. Endo agar
D. Ploskirev’s agar
30. Bacterial culture obtained from pati- E. Egg-yolk salt agar
ent DOES NOT grow when exposed to
oxygen. Conditions suitable for bacterial 35. Reaction of urea with nitrous acid
217
krok123.in.ua
Krok 1 Pharmacy (англомовнi студенти) 2017 рiк 5

produces the following: A. Increased permeability of the capillari-


es
B. Decreased oncotic blood pressure
C. Increased hydrostatic blood pressure
D. Decreased osmotic blood pressure
E. Disturbed lymphatic efflux

A. CO2 + N2 + H2 O 41. During identification of fruits of


B. CO2 + NO2 + H2 O Datura family they were determined to
C. CO + NH3 + CO2 be a:
D. CO2 + NO + NH3 A. Thorned quadrivalve capsule
E. CO2 + NH3 + NO2 B. Glossy black berry
36. Surface tension is an important C. Urceolate capsule with a lid
characteristic of a liquid. What substance D. Juicy globular cynarodium
has maximal surface tension? E. Berry in an orange cup

A. Water 42. During examination of a patient with


B. Ethanol intestinal infection, inoculation in Endo
C. Benzene medium resulted in multi-colored coloni-
D. Acetone es: red and colorless. According to its
E. Chloroform purpose this medium can be determined
as:
37. What standard solution can be used to
standardize the solution of I2 ? A. Differential diagnostic
B. Universal
A. Sodium thiosulfate solution C. Special
B. Potassium iodide solution D. Selective
C. Potassium dichromate solution E. -
D. Potassium permanganate solution
E. Sodium nitrite solution 43. Formation enthalpy equals zero for
the following substance:
38. Catalysts are widely used in producti-
on of drugs. How can reaction accelerati- A. O2
on in the presence of a catalyst be explai- B. H2 O2
ned? C. H2 SO4
D. CaCO3
A. Activation energy decreases E. O2
B. Total collision frequency increases
C. Activation energy increases 44. Nicotinic acid amide fulfills important
D. Collision frequency decreases metabolic function. What disorder
E. Molecule speed increases develops, when it is deficient in the
organism?
39. Microorganisms in the environment
are being affected by various physi- A. Pellagra
cal factors. What is the effect of high B. Rickets
temperature on a microbial cell? C. Anemia
D. Xerophthalmia
A. Irreversible degradation of all cellular E. Beriberi
structures
B. Mutagenic effect 45. At the triple point of the water phase
C. Transition into anabiosis state diagram:
D. Albuminolysis A. f=0
E. Fats saponification B. f=2
40. A person has been stung by a bee. C. P=3; f=1
The stung area developed redness and D. f=1
edema. What is the main mechanism of E. P=3; n=1
edema development? 46. Reaction of benzene sulfonation
produces:

218
krok123.in.ua
Krok 1 Pharmacy (англомовнi студенти) 2017 рiк 6

A.

A. B.

C.
B.

D.
C.

E.
D.

48. Select the formula of diazonium salt:


E.
A.

47. Choose the reaction of ester producti- B.


on among those listed below:

C. C6 H5 − N = O
D. C6 H5 − NH − C(O) − CH3
E. (CH3 )2 N − N = O
49. A patient was delivered into a resusci-
tation unit with signs of alcohol poisoni-
ng. The patient developed hypoxia of the
following pathogenesis:
A. Tissue
B. Hypoxic
C. Hemic
D. Circulatory
E. Mixed
50. Select the compound with
amphoteric properties (which reacts both
with acids and bases and produces salts):

219
krok123.in.ua
Krok 1 Pharmacy (англомовнi студенти) 2017 рiк 7

A. 54. Specify the substance that results


from the following reaction:
HOH, Hg 2+
CH ≡ CH −−−−−−−→?
A. Ethanal
B. B. Ethanol
C. Propanal
D. Propanone
E. Acetic acid

C. 55. Choose the indicator and titrati-


on method to determine hydrogen
carbonate ions in a drug:
A. Methyl-orange, acidimetry
B. Phenolphthalein, acidimetry
D. C. Methyl-orange, alkalimetry
D. Phenolphthalein, alkalimetry
E. Murexide, acidimetry
56. Pharmacopoeia reaction to determi-
ne phosphate ions is a reaction with
E. magnesia mixture. It results in producti-
on of white crystalline precipitate
gNH4P O4. Magnesia mixture consists
of the following:
A. MgCl2 , NH3 · H2 O , NH4 Cl
B. MgCl2 , NaOH , NaCl
51. Since 2005 in Asian and European C. MnCl2 , NH3 · H2 O , NaCl
countries there was recorded unusually D. MgCl2 , MnSO4 , NH4 Cl
high avian flu morbidity. Such spread of E. MgCl2 , NH4 Cl
epidemic process can be determined as:
57. An ophthalmologist has detected
A. Pandemia increased time of dark adaptation in
B. Epidemic a patient. What vitamin deficiency can
C. Endemia result in such symptom?
D. Sporadic
E. Epizooty A. A
B. C
52. Pharmacopoeia reaction to determi- C. K
ne benzoate ions requires interaction wi- D. B1
th the solution of: E. B6
A. Iron (III) chloride 58. Prolonged taking of cytostatic agents
B. Potassium chloride resulted in development of necrotic
C. Resorcin tonsillitis in the patient. It can be associ-
D. Acetic anhydride ated with the following changes in the
E. Diphenylamine leukocyte content:
53. Tricarboxylic acid cycle is a general A. Agranulocytosis
way of carbohydrates, amino acids, and B. Neutrophilic leukocytosis
fatty acids oxidation. Specify the acid wi- C. Lymphopenia
th which acetyl-CoA reacts first in tri- D. Eosinopenia
carboxylic acid cycle: E. Lymphocytosis
A. Oxaloacetic 59. During photosynthesis within plant
B. Citric cell chloroplasts there is short-term retai-
C. Isocitric ned starch being produced, which rapi-
D. Fumaric dly hydrolyzes into glucose. This starch is
E. Malic called:
220
krok123.in.ua
Krok 1 Pharmacy (англомовнi студенти) 2017 рiк 8

65. Specify the indicator of the protective


A. Primary properties of high-molecular compounds
B. Secondary of body, which promote the retention
C. Transitory of calcium, phosphate and carbonate in
D. Resistant blood plasma:
E. Reserve
A. Protective value
60. A patient with type I diabetes B. Coagulation threshold
mellitus developed hyperketonemic C. Critical micelle concentration
coma. What acid-base imbalance will be D. Hydrophilic-lipophilic balance
observed in the patient? E. Volume of sol coagulated by 1 mol of
the electrolyte substance
A. Nongaseous acidosis
B. Gaseous acidosis 66. In a research center there is a live
C. Nongaseous alkalosis vaccine against dysentery being created.
D. Gaseous alkalosis What property of attenuated vaccine
E. There will be no acid-base imbalances strain should coincide with the properti-
es of original virulent strain of dysentery
61. Reaction rate constant of a hypotheti- bacillus?
cal reaction is measured as s−1 . What is
the order of this reaction? A. Antigenic structure
B. Morphology
A. First-order C. Biochemical activity
B. Zero-order D. Antibiotic susceptibility
C. Second-order E. Toxin production
D. Third-order
E. Fractional-order 67. Plantago major inflorescence grows
at the apex, its rachis is long, with sessile
62. Species character of Thymus flowers. Name this type of inflorescence:
serpyllum includes: apical inflorescences
(flower heads), dark punctate glands on A. Spike
the inferior surface of a leaf, long hairs B. Panicle
along the edge of leaf base, and: C. Spadix
D. Capitulum
A. Creeping stems E. Thyrse
B. Thorns
C. Stems with prickles 68. Microbiological investigation of vagi-
D. Climbing stems nal suppositories determined them to
E. Short decumbent stems be CONTRARY to the Pharmacopoeia
demands. What microflora was detected
63. Natural peptides can perform various in the suppositories, resulting in such a
functions. What bioactive peptide is a conclusion?
major antioxidant and fulfills coenzyme
functions? A. Blue pus bacillus
B. Sarcina
A. Glutathione C. Micrococcus
B. Bradykinin D. Tetracoccus
C. Oxytocin E. Citrobacter
D. Liberin
E. Anserine 69. A number of hereditary diseases
is caused by mutations in gene areas
64. Gout develops when purine nucleoti- that determine beginning or end of an
de metabolism is disturbed. A doctor intron. What process results in removal
prescribed the patient allopurinol that of introns and joining of exons?
is a competitive inhibitor of:
A. Splicing
A. Xanthine oxidase B. Transcription
B. Succinate dehydrogenase C. Recombination
C. Alcohol dehydrogenase D. Replication
D. Lactate dehydrogenase E. Translation
E. Hexokinase
70. Hyperlipemia can be observed in 2-
221
krok123.in.ua
Krok 1 Pharmacy (англомовнi студенти) 2017 рiк 9

3 hours after eating fatty food. 9 hours A. Progression


later lipid content normalizes again. How B. Promotion (activation)
can this condition be characterized? C. Transformation
D. Exudation
A. Alimentary hyperlipemia E. Inactivation
B. Transport hyperlipemia
C. Hyperplastic obesity 76. You are studying a silvery downy
D. Retention hyperlipemia plant of Asteraceae family, which is rich
E. Hypertrophic obesity with essential oils and bitters. Harvested
are apical sprouts with panicle of small
71. Name the mercurimetry titrant: round flower heads. This plant is:
A. 0,1 М solution of Hg2 (NO3 )2 A. Artemisia absinthium
B. 0,1 М solution of NaNO2 B. Arctium lappa
C. 0,1 М solution of AgNO3 C. Bidens tripartita
D. 0,1 М solution of KSCN D. Calendula officinalis
E. 0,1 М solution of NH4 SCN E. Chamomilla recutita
72. What inflorescences are characteristic 77. Among dosage forms there are
of Cruciferae (Brassicaceae) family? numerous disperse systems. Specify the
free disperse system:
A. Raceme or panicle
B. Capitulum or corymb A. Emulsion
C. Capitulum or umbel B. Gel
D. Corymb or spike C. Jelly
E. Spadix or panicle D. Diaphragm
E. Membrane
73. What cation of the 4th analytical
group is present in a solution, if its reacti- 78. A 55-year-old man, who had been
on with the group reagent results in suffering from mitral insufficiency for
formation of yellow precipitate? many years, developed acute heart fai-
lure. What pathophysiological type of
A. Cr3+ heart failure can be observed in this
B. Zn2+ case?
C. Sn2+
D. Al3+ A. Due to cardiac overload
E. Sn(IV ) B. Due to hypoxic damage to the heart
C. Due to coronarogenic damage to the
74. A patient presents with icteric sclera heart
and mucous tunics; urine is dark; feces D. Due to neurogenic damage to the
are light-colored. Blood content of direct heart
and indirect bilirubin is increased, urine E. Due to acute cardiac tamponade
content of direct bilirubin is increased.
What pathology can be characterized by 79. Cataract (lenticular opacity) has
these signs? developed in a 52-year-old woman with
diabetes mellitus. Lenticular opacity was
A. Obstructive jaundice caused by intensification of the following
B. Hemolytic jaundice processes:
C. Hepatocellular jaundice
D. Jaundice of the newborn A. Protein glycosylation
E. Atherosclerosis B. Lipolysis
C. Ketogenesis
75. The patient presents with rapid D. Protein proteolysis
growth of a tumor node and its progressi- E. Gluconeogenesis
ng malignization. What stage of tumor
growth can be characterized by these 80. During practice in the laboratory the
presentations? students had been investigating in vi-
tro, how malonate affects enzymes of
tricarboxylic acid cycle. They detected
accumulation of the following metaboli-
te:
222
krok123.in.ua
Krok 1 Pharmacy (англомовнi студенти) 2017 рiк 10

A. Succinate insomnia treatment?


B. Malate
C. Isocitrate A. Nitrazepam
D. Fumarate B. Phenobarbital
E. Succinyl-CoA C. Aethaminalum-natrium (Pentobarbi-
tal)
81. Aqueous solution of the following D. Bromisoval
substance will have the smallest surface E. Valerian tincture
tension, if all the listed solutions are
taken in the same concentration: 86. What antiprotozoal drug can be
recommended to a woman with tri-
A. Sodium stearate chomoniasis?
B. Ethanol
C. Sodium chloride A. Metronidazole
D. Sodium hydroxide B. Primaquine
E. Sucrose C. Chloridine
D. Solusurminum (Sodium sti-
82. A 45-year-old woman, who for bogluconate)
two weeks has been taking neodi- E. Chiniofon
coumarin (ethyl biscoumacetate) due to
trombophlebitis, during a regular exami- 87. Select the halogenated antiseptic that
nation was detected to have decreased would be preferable for a child to pack in
blood content of prothrombin, in uri- the first aid kit, when going to a summer
ne there is microhematuria. What drug camp:
should be administered as a neodi-
coumarin antagonist? A. Iodine alcoholic solution
B. Brilliant green
A. Vicasol (Menadione) C. Copper sulfate
B. Protamine sulfate D. Methylene blue
C. Sodium citrate E. Formaldehyde solution
D. Heparin
E. Aminocapronic acid 88. Name the most typical symptom of
atropine poisoning:
83. A patient complaining of dry mouth,
photophobia, and visual impairment has A. Dilated pupils unresponsive to light
been delivered into an admission room. B. Constricted pupils unresponsive to
The skin is hyperemic and dry; pupils light
are dilated; tachycardia is observed. The C. Excessive sweating
patient was diagnosed with belladonna D. Bradycardia
alkaloids intoxication. What drug would E. Low intraocular pressure
be advisable?
89. A patient consulted an ophthalmologi-
A. Proserin st about deterioration of twilight vision
B. Aceclidine and xerophthalmus. What drug should
C. Pilocarpine the doctor prescribe?
D. Armin
E. Dipiroxim A. Retinol
B. Pyridoxine
84. A child accidentally took a drink from C. Tocopherol
the vial of grandmother’s medicine for D. Ascorbic acid
glaucoma. The medicine was identified E. Cocarboxylase
as pilocarpine hydrochloride. What drug
can be used as an antidote? 90. A woman with hypertension came to
a doctor complaining of dry cough that
A. Atropine developed against the background of her
B. Carbachol therapy. What antihypertensive drug was
C. Aceclidine she taking?
D. Benzohexonium (Hexamethonium)
E. Pentamin (Azamethonium bromide)
85. A woman suffering from neurosis has
disturbed sleep. What drug is optimal for
223
krok123.in.ua
Krok 1 Pharmacy (англомовнi студенти) 2017 рiк 11

A. Lisinopril her condition. What is the mechanism of


B. Atenolol action of this drug?
C. Nifedipine
D. Furosemide A. Opioid receptor blockade
E. Dichlothiazide (Hydrochlorothiazide) B. GABA receptor blockade
C. Serotonin receptor blockade
91. Decreased absorption of tetracycli- D. Dopamine receptor blockade
nes, if they are taken simultaneously with E. Benzodiazepine receptor blockade
antacids, is an example of their:
97. Name the ability of a drug to
A. Pharmacokinetic incompatibility accumulate within the patient’s body:
B. Pharmaceutical incompatibility
C. Pharmacodynamic incompatibility A. Cumulation
D. Synergism B. Antagonism
E. Functional antagonism C. Synergism
D. Habituation
92. A patient after craniocerebral trauma E. Allergy
has been prescribed piracetam. What
pharmacological group does this drug 98. What pharmacological effect of
belong to? acetylsalicylic acid allows its application
in patients with ischemic heart disease
A. Nootropic agents for prevention of thromboses?
B. Non-narcotic analgesics
C. Tranquilizers A. Antiaggregant
D. General anesthetics B. Analgesic
E. Antipsychotics C. Antipyretic
D. Ulcerogenic
93. Recommend the patient with E. Anti-inflammatory
glaucoma an M-cholinomimetic agent:
99. A patient developed herpetic rashes.
A. Pilocarpine hydrochloride What drug should be prescribed in this
B. Ephedrine hydrochloride case?
C. Sulfacyl-sodium (Sulfacetamide)
D. Atropine sulfate A. Acyclovir
E. Levomycetin (Chloramphenicol) B. Gentamicin
C. Clotrimazole
94. A patient with hypertension D. Benzylpenicillin
was prescribed a nonselective beta- E. Biseptol (Co-trimoxazole)
adrenergic blocking agent. Name this
drug: 100. A woman complaining of sleep di-
sturbance, fearfulness, and anxiety came
A. Anaprilin (Propranolol) to a neurologist. What drug should be
B. Prazosin prescribed in this case?
C. Proserin
D. Adrenalin hydrochloride A. Diazepam
E. Labetalol B. Levodopa
C. Nitroglycerine
95. In the course of bronchi- D. Oxytocin
tis pharmacotherapy a patient E. Lisinopril
has developed dyspeptic disorders,
photodermatitis and hepatic failure. 101. A man developed cardiac arrest due
What drug can cause such disorders? to thoracic trauma. Name the drug that
should be introduced into the cavity of
A. Doxycycline the left ventricle in this case:
B. Paracetamol
C. Ascorbic acid A. Adrenalin hydrochloride
D. Acetylcysteine B. Salbutamol
E. Codeine phosphate C. Lisinopril
D. Proserin
96. A 25-year-old woman with signs of E. Metoprolol
acute morphine intoxication was admini-
stered naloxone, which rapidly improved 102. A patient with acute heart failure
224
krok123.in.ua
Krok 1 Pharmacy (англомовнi студенти) 2017 рiк 12

was administered corglycon. What effect A. The investigated substance does not
of this drug results in improvement of the interact with the titrant
patient’s condition? B. The reaction runs very quickly
C. It is impossible to select the indicator
A. Increased heart force to determine titration end point
B. Decreased heart force D. Side reactions are possible
C. Coronary vessels dilatation E. The reaction runs slowly
D. Increased heart rate
E. Decreased oxygen demand of 108. Colored or white component of
myocardium double perianth, which consists of petals,
is a:
103. Explain to an intern, what is
the mechanism of analgesic action of A. Corolla
morphine hydrochloride: B. Flower cup
C. Androecium
A. Opiate receptors stimulation D. Gynoecium
B. Histamine receptors blockade E. Perigonium
C. Phosphodiesterase blockade
D. Adenylate cyclase stimulation 109. A 54-year-old man requested a
E. Choline esterase blockade pharmacist’s advice on drug prescripti-
on. The patient has 4-year-long history of
104. What pharmacological effect of di- chronic glomerulonephritis and 2-year-
azepam allows its application for termi- long history of persistent hypertension.
nation of convulsions? What substance synthesized in the ki-
dneys has important role in development
A. Anticonvulsant of arterial hypertension?
B. Analgesic
C. Antipyretic A. Renin
D. Anti-inflammatory B. Nitric oxide
E. Hypnotic C. Aldosterone
D. Histamine
105. A patient suffers from hyperchromic E. Catecholamines
B12 -deficiency anemia. What vitamin
preparation should be prescribed in this 110. A patient complains of general
case? weakness, muscle weakness in the
extremities (if the patient is asked to
A. Cyanocobalamin make a fist several times in a row, for
B. Riboflavin example, the patient is capable of doing
C. Vicasol (Menadione) it only once), facial muscles are weak,
D. Thiamine chloride swallowing is disturbed. Administration
E. Retinol acetate of acetylcholinesterase drugs removes
106. A man is diagnosed with Parkinson’s these disturbances to a certain degree.
disease. What drug should be prescribed Determine the pathology:
in this case? A. Myasthenia
A. Levodopa B. Paralysis
B. Nitrazepam C. Paresis
C. Paracetamol D. Hemiplegia
D. Aminazine E. Monoplegia
E. Anaprilin (Propranolol) 111. A drug solution sterilized by
107. Direct titration CANNOT be means of boiling was tested for sterili-
applied for quantitative determinati- ty. Inoculation on Kitt-Tarozzi medium
on of calcium chloride by means of revealed clostridia. Clostridia survived
permanganatometry, because: the boiling because they are:
A. Spore-formers
B. Thermophilic
C. Anaerobic
D. Prototrophic
E. Acid-fast
225
krok123.in.ua
Krok 1 Pharmacy (англомовнi студенти) 2017 рiк 13

112. A 71-year-old woman developed bacteriological laboratory. What medium


mechanical jaundice due to obstruction of those listed below can be determined
of the bile duct with a chololith. Decrease as selective?
of blood pressure and bradycardia are
detected. These changes in functioning A. Alkaline peptone water
of the patient’s cardiovascular system are B. Meat infusion broth
caused by increased blood content of the C. Meat infusion agar
following substance: D. Hiss’ serum water medium
E. Endo agar
A. Bile acids
B. Direct bilirubin 118. A 5-year-old child presents wi-
C. Indirect bilirubin th abdominal distension, abdominal
D. Urobilin cramps, and diarrhea occurring 1-4 hours
E. Stercobilin after drinking milk. Described symptoms
are caused by the lack of enzymes that
113. A sanitary-epidemic station break up:
employee has been poisoned when
the premices were processed wi- A. Lactose
th an organophosphorous insectici- B. Glucose
de. What enzyme is inhibited by C. Maltose
organophosphorous compounds? D. Saccharose
E. Fructose
A. Acetylcholinesterase
B. Lactate dehydrogenase 119. If pH of a solution is lower than its
C. Xanthine oxidase isoelectric point, it means in this solution:
D. Catalase
E. Pepsin A. Cation forms of amino acids are
prevalent
114. Indican excretion is a diagnostic cri- B. Anion and cation forms of amino acids
terion of intencified protein putrefaction are balanced
in the intestine. Name the end product C. Certain protein aggregates are formed
of tryptophan ”decay” occurring in the D. Irreversible protein precipitation
large intestine: occurs
E. Anion forms of amino acids are
A. Indole prevalent
B. Putrescine
C. Benzoic acid 120. Avidin - an egg white protein -
D. Mercaptan inhibits reception of biotin (carboxylase
E. Hydrogen sulfide coenzyme) by the body. What reaction
will be blocked by avidin administration?
115. The second stage of detoxificati-
on involves joining certain chemical A. CO2 attachment to pyruvate
compounds with functional groups of B. NH3 attachment to glutamate
toxines. Select one such compound: C. NH3 detachment from glutamine
D. Detachment of phosphate residuals
A. Glucuronic acid E. Beta-oxidation of fatty acids
B. Higher fatty acids
C. Cholesterol 121. During mercurimetric titration of
D. Glucose halogenide ions in the presence of di-
E. Pyruvate phenylcarbazone, at the titration end
point the precipitate is produced. This
116. Actinomorphic apopetalous corolla precipitate will be colored:
include:
A. Blue
A. Cruciform B. Red
B. Funnelform C. Yellow
C. Campanulate D. Green
D. Tubular E. Brown
E. Ligulate
122. When determining oxidizing agents
117. Selective medium can be used to by means of iodometry in the presence
separate various species of bacteria in a of starch the following phenomenon can
226
krok123.in.ua
Krok 1 Pharmacy (англомовнi студенти) 2017 рiк 14

be observed at the titration end point:


A. Oxidants
A. Blue coloring disappears B. Dyes
B. Red coloring appears C. Detergents
C. Green coloring of precipitate appears D. Alcohols
D. Green coloring of solution disappears E. Nitrofurans
E. White precipitate occurs
128. To relieve dry cough a patient wi-
123. To obtain exotoxins of some mi- th bronchitis was prescribed a drug that
croorganisms, these microorganisms are is an alkaloid of yellow horned-poppy.
inoculated into liquid nutrient medium, Name this drug:
where microbial cultivation occurs and
toxins are produced. At a certain stage it A. Glaucine hydrochloride
is necessary to remove the microbial cells B. Codeine phosphate
from the medium, that is, to separate C. Libexin (Prenoxdiazine)
the toxins from microbes. What method D. Oxeladin
should be applied in this case? E. Codterpin
A. Bacteria-excluding filters 129. Hydrochloric acid was added into
B. Boiling the solution under investigation. The
C. Autoclaving resulting precipitate was filtered, then
D. Ultraviolet irradiation this filter cake was processed with hot
E. Disinfectants (chloramine) water; after the filtrate cooled, KI soluti-
on was added into it. What cation was
124. A plant producing essential oil has present in the solution, if the precipitate
square stem, bilabiate corolla, coenobi- was colored yellow?
um fruit. These features are characteri-
stic of: A. P b2+
B. Ag +
A. Lamiaceae C. Hg 2+
B. Papaveraceae D. Ca2+
C. Polygonaceae E. Ba2+
D. Solanaceae
E. Scrophulariaceae 130. When cations are divided into
analytical groups according to the acid-
125. A plant under investigation base classification, group reagents can be
has compound uniform monopodium acids or bases. What acids can be used as
inflorescence - compound umbel. What group reagents?
plant is it characteristic of?
A. HCI , H2 SO4
A. Anethum graveolens B. HNO3 , CH3 COOH
B. Allium cepa C. H3 P O4, H2 C2 O4
C. Sorbus aucuparia D. HCIO4
D. Rosa canina E. H2 CO3
E. Centaurea cyanus
131. Main process of ammonia neutrali-
126. During field practice a student was zation occurs in the liver. Arginine
tasked with making a morphological decomposition reaction that produces
collection of coenocarpous fruits. What urea as a result is catalyzed with argi-
type of fruit belongs to this group? nase. What group of enzymes does argi-
A. Hesperidium nase belong to?
B. Aggregate-accessory fruit A. Hydrolases
C. Fragaria B. Synthetases
D. Cynarodium C. Oxidoreductases
E. Drupe D. Transferases
127. A patient with gingivitis was prescri- E. Isomerases
bed oral cavity irrigation with 0,02% 132. Connection between plant cell
potassium permanganate solution. What protoplasts and their metabolic function
group of antiseptics does this drug belong is provided by thin cytoplasmic threads
to?
227
krok123.in.ua
Krok 1 Pharmacy (англомовнi студенти) 2017 рiк 15

that pass through pores in the cell walls. A. Phospholipase A2


Name these threads: B. Cyclooxygenase
C. Phospholipase C
A. Plasmodesma D. Protein kinase
B. Fibrils E. Arginase
C. Microtubules
D. Microfilaments 138. Thermal analysis is used in
E. Cytoskeleton pharmacy to identify drugs and determi-
ne drug purity. What coordinates are
133. On autopsy there are numerous necessary to build a cooling curve?
suppurative foci within many of the
internal organs. What pathological A. Temperature-time
process is it characteristic of? B. Pressure-time
C. Volume-temperature
A. Septicopyemia D. Volume-time
B. Septicemia E. Temperature-volume
C. Sepsis
D. Bacteriemia 139. Both scientific and folk medicine
E. Toxemia uses medicinal plant Glycyrrhiza glabra
L. What part of the plant is harvested?
134. Isotonicity is required of infusi-
on solutions. What phenomenon A. Roots and rhizomes
occurs when a hypertensive solution is B. Foliage
introduced into blood plasma? C. Inflorescence
D. Grass
A. Plasmolysis E. Seeds
B. Osmosis
C. Hemolysis 140. Choose the formula of nonionic
D. Denaturation surfactant among those listed below:
E. Thixotropy
A. CH3 (OCH2 CH2 )10 OH
135. The third analitycal group of cations B. C15 H31 COONa
(acid-base classification) includes Ca2+ , C. C11 H23 OSO3Na
Sr 2+ , Ba2+ . What acid can function as D. C6 H13 NH2 COONa
a precipitator agent (group reagent) for E. C2 H5 NH2 · HCl
these cations? 141. A pregnant woman was diagnosed
A. H2 SO4 with vaginal dysbacteriosis. What drug
B. HNO3 should be prescribed in this case?
C. HCI A. Probiotic
D. CH3 COOH B. Antibiotic
E. HCIO4 C. Bacteriophage
136. During routine preventive exami- D. Interferon
nation the local pediatrician noti- E. Polyvitamins
ced a boy of short stature. Mental 142. A patient has been receiving
development of the child corresponds wi-
th his age. What endocrine disorder is it? Theophylline (inhibitor of cyclic adenosi-
ne monophosphate phosphodiesterase)
A. Pituitary nanism for a week. What hormone can increase
B. Cretinism its action due to such treatment and
C. Acromegalia cause hyperglycemia?
D. Gigantism
E. Rickets A. Glucagon
B. Testosterone
137. Eicosanoids synthesis begins with C. Aldosterone
freeing polyene acids from membrane D. Insulin
phospholipids by means of a specific E. Estradiol
phospholipase. Name this enzyme:
143. Reaction of sodium ions with
potassium hexahydroxoantimonate (V)
in neutral medium produces precipitate.
228
krok123.in.ua
Krok 1 Pharmacy (англомовнi студенти) 2017 рiк 16

Specify the color of this precipitate: A. Glucose


B. This method will suffice for any
A. White substance
B. Red C. Sulfuric acid
C. Yellow D. Sodium chloride
D. Green E. Benzene
E. Blue
149. A local general practitioner
144. Reaction with potassium permanganate recommends taking interferon for
is used to detect reducing anions. Speci- influenza prevention. What is the
fy the anion that decolorizes potassium mechanism of action of this drug?
permanganate:
A. Blocks virus protein synthesis
A. Sulfite B. Blocks virus stripping
B. Carbonate C. Inhibits virion exit from cells
C. Tetraborate D. Prevents adsorption of virus in cell
D. Sulfate receptors
E. Arsenate E. Disrupts the process of virus assembly
145. A 25-year-old-patient with the II 150. What cation can be detected with
degree thermal burns came to a doctor. Chugaiev’s agent (Dimethylglyoxime)?
Objectively: there are large blisters on
the upper limbs; the blisters are filled wi- A. Ni2+
th clear exudate containing mostly water B. Ca2+
and albumines with isolated leukocytes. C. +
Name the type of the exudate: D. Mn2+
E. Co2+
A. Serous
B. Catarrhal (mucous) 151. A solution of hydrogen peroxide in
C. Fibrinous an acid medium was added into investi-
D. Purulent gated solution, leading to blue coloring
E. Hemorrhagic of the resulting solution. This analyti-
cal effect indicates the presence of the
146. Rhizome of an Asteraceae family following anions:
species is polycephalous, succulent, has
lysigenous cavities, accumulates inulin. A. Cr2 O72−
Such underground organ is characterisic B. MnO4−
of:
C. C2 O42−
A. Inula helenium D. NO3−
B. Hyoscyamus niger E. Cl−
C. Digitalis grandiflora
D. Sorbus aucuparia 152. A patient presents with hypoxia.
E. Helianthus annuus What metabolic process activates when
oxygen supply is insufficient?
147. Osmotic pressure is an important
characteristic of biological fluids. Semi- A. Anaerobic glycolysis
permeable membranes are necessary for B. Urea cycle
penetration of solvent molecules. What C. Pentose-phosphate pathway
substance CANNOT be used as a semi- D. Oxidative decarboxylation of keto
permeable membrane? acids
E. Tricarboxylic acid cycle
A. Glass
B. Biological membrane 153. A child had been administered anti-
C. Collodion film diphtheric serum. What resistance was
D. Parchment formed in the child?
E. Gelatine
148. Specify the substance that can be
determined by means of polarimetry:

229
krok123.in.ua
Krok 1 Pharmacy (англомовнi студенти) 2017 рiк 17

A. Passive its survival within hostile environment?


B. Active
C. Primary A. Spores
D. Pathologic B. Flagella
E. Physiological C. Capsule
D. Cilia
154. A patient with brain edema presents E. Inclusions
with respiration that is characteri-
zed by periods of several respiratory 159. Total content of chloride, bromide,
movements of equal amplitude alternati- and iodide ions in the investigated soluti-
ng with periods of apnea. What on can be quantitatively determined with
pathologic respiration is it characteristic the following titrant:
of?
A. Silver nitrate solution
A. Biot’s respiration B. Potassium dichromate solution
B. Gasping respiration C. Sodium thiosulfate solution
C. Apneustic respiration D. Potassium permanganate solution
D. Cheyne-Stokes’ respiration E. Sodium nitrite solution
E. Kussmaul’s respiration
160. Specify the reaction product of
155. A patient in the state of ketoacidotic complete aniline hydrogenation:
coma presents with loud rapid respi-
ration: labored expiration with tension A.
of expiratory muscles occurs after deep
inspiration. Name the type of pathologic
respiration: B.
A. Kussmaul’s
B. Cheyne-Stokes’
C. Gasping
D. Stenotic C.
E. Biot’s
156. In a nursery-garden some medici-
nal plants developed signs of a disease: D.
there are yellow spots and necrotic foci
on the leaves. Sap of the diseased plants
remained infectious even after passing E.
through a bacteria-excluding filter. No
microorganisms growth was detected on
the nutrient medium. What microorgani-
sms could be the cause of this disease?
161. Name the isomer of diethyl ether
A. Viruses functional group:
B. Fungi CH3 − CH2 − O − CH2 − CH3 :
C. Ray fungi
D. Bacteria A. Butanol
E. Mycoplasma B. Ethanol
C. Butanal
157. Bark of a thornless xylophyte of the D. Ethyl acetate
Rhamnaceae family has laxative effect. E. Dimethyl ether
Name this plant:
162. What substance can be identified wi-
A. Frangula alnus th aqueous solution of F eCl3 ?
B. Aronia melanocarpa
C. Hippophaё rhamnoides
D. Rubus idaeus
E. Crataegus sanguinea
158. Different structures of a bacterial
cell perform different functions. What
dispensable component of a cell ensures
230
krok123.in.ua
Krok 1 Pharmacy (англомовнi студенти) 2017 рiк 18

A. on of sulfates in potable water. What


substance should be used as precipitator
for sulfates?
A. BaCI2
B. B. KCI
C. MgCI2
D. NaCI
E. NH4 NO3
C.
165. In 9 days after administration of a
therapeutic serum the patient developed
urticaria, itching, edemas, and lymph
nodes enlargement. What type of allergic
D. reaction has occurred in the patient?
A. Immune complex
B. Cytotoxic
E. C. Anaphylactic
D. Stimulating
E. Cellular
166. Which of the listed compounds is an
alpha-amino acid?
163. What reaction proves that phenol
has acidic properties? A.
A.

B.

B.
C.

D.
C.

E.

D.

167. Exudation is characteristic of


inflammation. What factors cause
exudation and local edema of the
inflamed area?
E.
A. Increased permeability of vessel wall
B. Hyperglycemia
C. Ischemia
D. Leukocyte adhesion to endothelial
cells
E. Decreased permeability of vessel wall
164. Gravimetry (precipitation method)
is used for quantitative determinati- 168. For cardiovascular disease preventi-
231
krok123.in.ua
Krok 1 Pharmacy (англомовнi студенти) 2017 рiк 19

on the patient was recommended to take A. Alpha-tocopherol


vitamin F . What is the chemical nature B. Glucose
of this vitamin? C. Calciferol
D. Cobalamine
A. Complex of polyunsaturated fatty E. Glicerol
acids
B. Cholesterol derivative 174. During excursion into a conifer
C. Polysaccharide complex forest the students noticed that bilberry
D. Amino acids complex (Vaccinium myrtillus) stems are lignifi-
E. Carotin derivative ed only partially in their lower part, the
upper part of the stem retains the form of
169. Wetting occurs when a drop of a li- caulis. Therefore, this plant can be classi-
quid comes into contact with the surface fied as:
of a solid substance. The degree of wetti-
ng is measured through: A. Suffrutex
B. Annual grass
A. Contact angle C. Liana
B. Drop density D. Perennial grass
C. Surface tension E. Tree
D. Drop size
E. Work of adhesion 175. Antibiotics derived from various
species of actinomycetes are widely used
170. A perennial plant has white flower in medical practice. Point out these drugs
heads grouped in compound corymbs among those listed below:
and bipinnatisected or tripinnatisected
leaves. Name this plant: A. Aminoglycosides (streptomycin,
monomycin)
A. Аchillea millefolium B. Penicillin, cephalosporin, griseofulvin
B. Melilotus officinalis C. Polymyxin, bacitracin
C. Potentilla erecta D. Chloreline, arenarinum
D. Phaseolus vulgaris E. Lysozyme, erytrinum
E. Taraxacum officinale
176. In a human body there occur
171. What organic compounds are numerous reactions of direct interacti-
produced in the result of intramolecular on between substrate and molecular
dehydration of monohydric alcohols? oxygen. What enzyme catalyzes
A. Alkenes attachment of two oxygen atoms to the
B. Esters substrate?
C. Aldehydes A. Dioxygenase
D. Alkanes B. Catalase
E. Arenes C. Monooxygenase
172. A group of children in the D. Superoxide dismutase
E. Glutathione reductase
kindergarten (6-year-olds) received
Mantoux test; 15 children presented with 177. Seeds of rye, corn, and other crops
negative results. What measures should have small corymb-shaped cotyledon
be taken towards these children? and accumulate nutrients in the:
A. BCG vaccination A. Endosperm
B. Tuberculosis antitoxin B. Perisperm
C. Isolation C. Shell
D. Repeat the test D. Gemmule
E. Referral for fluorography E. Embryo root
173. Increased concentration of acti- 178. A 55-year-old man came to a doctor
ve oxygen forms is a mechanism of with complaints of acute pain in his big
pathogenesis in a number of diseases. toes. Meat and wine remain permanently
To prevent this process, antioxidants are in his diet. The doctor suspects gout.
prescribed. Select an antioxidant from What substance must be measured in the
the list below: patient’s blood to confirm this diagnosis?
232
krok123.in.ua
Krok 1 Pharmacy (англомовнi студенти) 2017 рiк 20

A. Uric acid 183. The patient’s large-focal myocardial


B. Urea infarction is complicated with pulmonary
C. Lactate edema. What disturbance of cardi-
D. Bilirubin ohemodynamics contributed to the
E. Ketone bodies pulmonary edema development?
179. Isotonic glucose solution is widely A. Acute left ventricular failure
used as a solvent or infuson medium for B. Acute right ventricular failure
introduction of various drugs. What mass C. Autoimmune myocarditis
fraction is characteristic of this solution? D. Cardiogenic shock
E. Reperfusion syndrome
A. 5%
B. 10% 184. The fruit is a thorned many-seeded
C. 15% capsule that opens into four flaps when
D. 20% ripe. It is characteristic of:
E. 1%
A. Datura stramonium
180. Name the process of cell membrane B. Papaver somniferum
saturation with a fat-like substance - C. Hyoscyamus niger
suberin: D. Digitalis purpurea
E. Linum usitatissimum
A. Suberization
B. Lignification 185. An interhospital pharmacy received
C. Mineralisation a short-acting narcotic analgesic that is
D. Cutinization 100 times more active than morphine.
E. Sliming Name this drug:
181. A man has been suffering from A. Fentanyl
rheumatoid arthritis for 10 years. Due B. Naltrexone
to its exacerbation he had been taki- C. Naloxone
ng acetylsalicylic acid and predni- D. Analgin (Metamizole)
solone. The patient complains of E. Ketanov (Ketorolac)
stomachache, eructation, nausea, sensati-
on of full epigastrium, and meteorism. 186. Hemoglobin break-up begins in the
On gastroscopy there was an erosion cells of reticuloendothelial system. What
(0,5x0,5 cm) of gastric mucosa detected. enzyme catalyzes the reduction reaction
What is the cause of gastric mucosa of biliverdine into bilirubin?
defect development?
A. Biliverdine reductase
A. Prolonged taking of aspirin and B. Beta-glucuronidase
hormones C. Xanthine oxidase
B. Immune-mediated destruction of D. Heme oxygenase
gastric mucosa E. Hexokinase
C. Dysbacteriosis development
D. Age-related changes of mucosa 187. To stop diarrhea the doctor prescri-
E. Prolonged hypersthenia of gastric bed a drug that affects opiate receptors
muscles of the intestine and decreases its peri-
stalsis. Name this drug:
182. Treatment of a number of pathologic
changes in human body is based on A. Loperamide
the peptization process, particularly di- B. Heparin
sintegration of thrombi within blood C. Levorin
vessels. The most important condition D. Voltaren (Diclofenac)
for efficient peptization can be determi- E. Tetracycline
ned as: 188. What formula corresponds with
A. Timely introduction of anticoagulant alpha-nitropyrole?
B. Introduction of excessive solvent
C. Ultrasound effect
D. Shaking
E. Heating
233
krok123.in.ua
Krok 1 Pharmacy (англомовнi студенти) 2017 рiк 21

A. A. Hydrogen bonds
B. Ionic bonds
C. Polar covalent bonds
D. Nonpolar covalent bonds
E. Donor-acceptor bonds
B.
192. Specify the functional group of
isoniazide (an isonicotinic acid derivati-
ve) molecule:
C.

D.
A. Hydrazide group
B. Amide group
C. Carboxyl group
E. D. Pyridine heterocyclic group
E. Ester group
193. What compound is synthesized by
189. Essential oils are used both in means of beta-picoline oxidation?
pharmaceutical and cosmetic industry.
To extract essential oils from herbal
raw material the following technology is
used:
A. Steam distillation A. Nicotinic acid
B. Calorimetry B. Benzoic acid
C. Colorimetry C. Uric acid
D. Potentiometry D. Barbituric acid
E. Conductometry E. Ascorbic acid
190. Specify the mechanism of the given 194. Specify quinoline among the
reaction: compounds given below:
A.
+ o
H , t>170 C
C2 H5 OH −−−−−−−−→ H2 C = CH2 +H2 O

B.

A. Е (elimination)
B. SN (nucleophilic substitution) C.
C. N (nucleophilic attachment)
D. SR (radical substitution)
E. S (electrophilic substitution) D.
191. What type of bonds participates
in creation of both linear and cyclic
carboxylic acid associates in the form of
dimers? E.

195. Name the substance B in the followi-


234
krok123.in.ua
Krok 1 Pharmacy (англомовнi студенти) 2017 рiк 22

ng transformation: A.

B.

C.
2+
HOH,Hg [0]
CH ≡ CH −−−−−−−→ A −→ B
D.

E.

A. Acetic acid 198. Among the given substances choose


B. Ethanol the one that is used for oxidation of
C. Propanal organic compounds:
D. Propanone
E. Ethanal A. KMnO4
B. CH3 − CH3
196. What substance produces ketone, C. NaOH
when oxidized with potassium D. HCl
permanganate? E. NH2 − NH2
A. 199. What substance will be produced as
the result of the following reaction:
6 5 − N + ≡ NCl− + KJ −→? to

B. CH3 − CH − CH2 OH
C. CH3 − CH2 OH A. 6 5 J
D. B. C6 5 
C. 6 6
D. 6 5 − Cl
E. C6 5 K
200. What reagent can simultaneously
E. detect aldehyde group and glycol
fragment in a glucose molecule?
A. Cu(OH)2
B. Br2
197. Among the given isomer pairs C. AlCl3
choose the position isomers of the functi- D. F eCl3
onal group: E. KMnO4

235
krok123.in.ua
Krok 1 Pharmacy (англомовний варiант, iноземнi студенти) 2018 рiк 1

1. What titrimetric method of analysis allopurinol - a competitive inhibitor of


requires the use of both external and xanthine oxidase. Xanthine oxidase is a
internal indicators? terminal enzyme of catabolism of:
A. Nitritometry A. Purine nucleotides
B. Alkalimetry B. Glycoproteins
C. Complexometric titration C. Phospholipids
D. Permanganatometry D. Higher fatty acids
E. Argentometry E. Heteropolysaccharides
2. By means of photoelectrocolorimetric 8. Many drugs must be manufactured under
analysis the concentration of the followi- strictly aseptic conditions. One such possi-
ng can be determined: ble source of microbiological contaminati-
on of drugs is laboratory glassware. What
A. Colored solution method should be used to sterilize the
B. Turbid solution glassware?
C. Optically active substance
D. Colorless solution A. Dry heat
E. Any type of solution B. Ignition
C. Boiling
3. A patient suffers from Down’s di- D. Tyndallization
sease that manifests as mental retardation, E. Pasteurization
shortness of stature, pathologically short fi-
ngers and toes, and eyes with mongoloid 9. To determine qualitative content of a
slant. Karyotype analysis revealed tri- drug, the drug sample was processed wi-
somy 21. What group of diseases does this th 2M solution of HCl. White precipitate
pathology belong to? soluble in aqueous ammonia solution was
formed. This analytical effect indicates the
A. Chromosomal disorders presence of the following cations:
B. Molecular genetic disease
C. Gametopathy A. Silver(I)
D. Fetopathy B. Lead(II)
E. Blastopathy C. Mercury(I)
4. Knowing the temperature of phase D. Mercury(II)
transformation under a certain pressure is E. Tin(II)
extremely important in the process of drugs 10. Name the process of spontaneous
manufacturing. This temperature can be adhesion of drops in an emulsion to each
calculated using the: other:
A. Clausius-Clapeyron equation A. Coalescence
B. Trouton’s rule B. Flocculation
C. Gibbs’ phase rule C. Sedimentation
D. Mendeleev-Clapeyron equation D. Flotation
E. Konovalov’s laws E. Coagulation
5. At the triple point on the water phase di- 11. Inheritable genetic disorders can result
agram, water is: in disturbed enzyme synthesis in the human
A. An invariant system body. What enzyme deficiency results in di-
B. A monovariant system sturbed break-up of lactose:
C. A bivariant system A. Lactase
D. A trivariant system B. Maltase
E. A quadrivariant system C. Invertase
6. Bioavailability of a powder depends on D. Lipase
the degree of comminution of the substance. E. Peptidase
The following value must be measured: 12. To treat the patients with purulent
A. Dispersion wounds, a dressing with a certain immobili-
B. Concentration zed enzyme is used. Name this enzyme:
C. Particle volume
D. Particle mass
E. Solution density
7. A patient with gout was prescribed
236
krok123.in.ua
Krok 1 Pharmacy (англомовний варiант, iноземнi студенти) 2018 рiк 2

A. Tripsin be the most advisable in cases of the followi-


B. Arginase ng type of infections:
C. Catalase
D. Alkaline phosphatase A. Viral
E. Acid phosphatase B. Helminthic
C. Protozoal
13. Cerebrospinal fluid of a patient di- D. Microbioses
agnosed with meningitis was taken for E. Fungal
analysis. To detect the causative agent the
sample was inoculated in a nutrient medi- 19. A chemotherapeutic agent has
um. Prior to that a serum had been added bactericidal effect against streptococci,
to the medium. What causative agent is staphylococci, bacilli, and clostridia.
expected to be obtained in this case? According to its action spectrum this drug
belongs to the following group:
A. Meningococcus
B. Mycobacteria A. Broad spectrum antibacterial agents
C. Staphylococcus B. Narrow spectrum antibacterial agents
D. Viruses C. Broad spectrum antifungal agents
E. Rickettsia D. Antiviral agents
E. Antituberculous agents
14. Hormones regulate numerous metabolic
processes. What hormone activates 20. It is known that in plants the synthesis
glycogen synthesis? of secondary reserve starch occurs in:
A. Insulin A. Amyloplasts
B. Adrenaline B. Chloroplasts
C. Vasopressin C. Chromoplasts
D. Thyroxine D. Elaioplasts
E. Oxytocin E. Proteinoplasts
15. A structural analog of vitamin P P (ni- 21. A patient has developed anuria. Blood
cotinic acid) is used as an antituberculous pressure is 50/20 mm Hg. What process of
medicine. Name this medicine: uropoiesis was disturbed resulting in acute
decrease of urine output?
A. Isoniazid
B. Streptocide A. Glomerular filtration
C. Riboflavin B. Obligate reabsorption
D. Tetracycline C. Facultative reabsorption
E. Aspirin D. Tubular secretion
E. -
16. A food plant of Polygonaceae family is
being studied. The plant has reddish stalk, 22. A 12-year-old boy is of short stature,
cordate-sagittate leaves, its fruit is a tri- but his mental development corresponds
hedral nutlet. Name this plant: with that of his age group. What hormone
deficiency is the most likely to cause this
A. Fagopyrum esculentum pathology?
B. Persicaria bistorta
C. Persicaria hydropiper A. Somatotropin
D. Polygonum aviculare B. Insulin
E. Rumex confertus C. Oxytocin
D. Vasopressin
17. Microscopy of subterranean organs of an E. Adrenaline
Аsteraceae family plant shows articulated
laticifers with anastomoses filled with whi- 23. A 10-year-old child has height of 178 cm
te latex. It is characterisic of the following and body mass of 67 kg. These presentations
plant: are caused by the functional disturbance of
the:
A. Taraxacum officinale
B. Helianthus annuus A. Pituitary gland
C. Artemisia absinthium B. Thyroid gland
D. Bidens tripartitа C. Gonads
E. Achillea millefolium D. Adrenal glands
E. Parathyroid glands
18. The defensive mechanisms against some
infectious diseases can be greatly reinforced 24. A certain infection leads to fetus
with interferon. Interferon preparations will malformation if a pregnant woman is
237
krok123.in.ua
Krok 1 Pharmacy (англомовний варiант, iноземнi студенти) 2018 рiк 3

affected. What vaccine should be used for


prevention of this infection? A. Transferases
B. Isomerases
A. Rubella virus vaccine C. Oxidoreductases
B. Influenza virus vaccine D. Ligases
C. Mumps vaccine E. Hydrolases
D. Poliovirus vaccine
E. Antirabic vaccine 31. A patient with high risk of hemorrhages
is recommended to take vicasol (menadi-
25. Quantitative content of hydrogen one) by his physician. This drug is the
peroxide can be determined by means of structural analog of:
the following self-indicator method:
A. Vitamin K
A. Permanganatometry B. Vitamin A
B. Bromatometry C. Vitamin B5
C. Iodometry D. Vitamin B12
D. Nitritometry E. Vitamin B6
E. Argentometry
32. Enzymes are widely used as drugs in
26. Coumarins, vitamin K antagonists, pharmacy. What is the main feature that
suppress the processes of blood coagulati- separates enzymes from non-biological
on. What protein synthesis is blocked by catalysts?
coumarins?
A. High specificity and selectivity
A. Prothrombin B. High universality
B. Gamma globulin C. Low universality
C. Albumin D. High dispersion
D. Transferrin E. High homogeneity
E. Ceruloplasmin
33. Any damage to the patient’s vessels
27. HIV-infection occupational risk groups results in persistent hemorrhage. Blood
include people of various professions, clotting factor VIII is deficient in the pati-
healthcare workers included. Specify the ent’s blood. What disease does this patient
mosl likely route of infection transmission suffer from?
for healthcare workers:
A. Hemophilia
A. Parenteral transmission B. Acute vascular purpura
B. Fecal-oral transmission C. Thrombocytopenic purpura
C. Droplet transmission D. Anemia
D. Transmission via airborne dust particles E. Radiation sickness
E. Vector-borne transmission
34. Name the structural unit of a colloidal
28. Microbial survival within environment solution of a medicinal substance:
is facilitated by spore formation. What mi-
croorganisms of those listed below are spore A. Micelle
formers: B. Molecule
C. Atom
A. Clostridia D. Ion
B. Bacteroides E. Zwitterion
C. Staphylococci
D. Peptococci 35. Protein structure includes proteinogenic
E. Peptostreptococci amino acids. What is the position of the ami-
no group in the structure of these amino aci-
29. Synthesis of a medicinal substance ds?
occurs in an isolated system. What is a di-
rection criterion of spontaneous processes? A. α-position
B. β -position
A. Entropy change C. γ -position
B. Gibbs energy D. δ -position
C. Helmholtz energy E. -position
D. Intrinsic energy
E. Enthalpy 36. To induce diabetes mellitus in a
rabbit, β -cells of pancreatic islets (islets of
30. An enzyme transports structure Langerhans) were selectively damaged with
fragments from one substrate into another. alloxan. What method of diabetes induction
Name this class of enzymes: was used in this experiment?
238
krok123.in.ua
Krok 1 Pharmacy (англомовний варiант, iноземнi студенти) 2018 рiк 4

general water hardness of potable water:


A. Shutdown
B. Irritation A. Complexometric titration
C. Introduction of enzymes, hormones B. Acidimetry
D. Isolated organs C. Precipitation
E. Stimulation D. Oxidimetry
E. Alkalimetry
37. A sample section of an axial body shows
a complex consisting of phellogen and its 43. Name the reaction producing a golden-
derivatives - cork and phelloderm. Name yellow precipitate (”golden rain” reaction):
this tissue:
A. Precipitation of P bI2
A. Periderm B. Precipitation of P bCl2
B. Colenchyma C. Precipitation of AgI
C. Sclerenchyma D. Precipitation of HgI2
D. Epiblema E. Precipitation of Hg2 I2
E. Epidermis
44. Name the reactions and reagents that
38. The Mohr method is used to determine under certain conditions allow determinati-
mass concentration of sodium chloride in on of certain ions in the presence of other
an isotonic solution. Titration is carried out ions:
with the following indicator:
A. Specific
A. Potassium chromate B. Selective
B. Fluorescein C. Group
C. Ammonium iron(III) sulfate D. Characteristic
D. Diphenylcarbazone E. General
E. Ferroin
45. In Friedel-Crafts alkylation of aromatic
39. A patient presents with persistent hydrocarbons the following substance is
tachycardia, exophthalmos, high excitabi- used as the catalyst:
lity, increased basal metabolic rate. What
disorder can lead to the development of this
syndrome?
A. Hyperthyroidism
B. Hypoparathyroidism
C. Hypothyroidism
D. Hyperparathyroidism A. Anhydrous AlCl3
E. Adrenal hypofunction B. P t
40. Racemose clusters of calcium carbonate C. Cr2 O3
crystals are detected among the waste D. Alcoholic NaOH solution
products of a protoplast. These crystals are: E. P Cl5

A. Cystoliths 46. What compound of those listed below is


B. Isolated crystals a condensed arene?
C. Raphides
D. Styloid crystals
E. Crystal druses
41. After the pus sample taken from the
urethra had been inoculated on ascitic agar,
it resulted in growth of round transparent
colonies. Microscopy of the colonies shows
gram-negative kidney bean-shaped di-
plococci. What causative agent is it?
A. Gonococcus
B. Pneumococcus
C. Meningococcus
D. Micrococcus
E. Streptococcus
42. Laboratories of various specialization
use the following method to determine
239
krok123.in.ua
Krok 1 Pharmacy (англомовний варiант, iноземнi студенти) 2018 рiк 5

A. A.

B. B.

C. C.

D.
D.

E.

E.

47. How does increasing temperature affects


physical adsorption of substances? 51. What drug is produced as the result of
reaction between salicylic acid and acetic
A. Decreases anhydride?
B. Increases
C. Transforms into chemical adsorption
D. Decreases in heterogeneous systems
E. Increases in homogeneous systems
48. Gelatin expands the most in the followi-
ng solvent: A. Aspirin
B. Salicylamide
A. Water C. Phenyl salicylate
B. Acetic acid solution D. Benzyl salicylate
C. Ethanol E. Sodium salicylate
D. Diethyl ether
E. Benzene 52. Glucose oxidation with a potent oxidi-
zing agent (concentrated HNO3 ) results in
49. Azo dyes are produced as the result of: production of:
A. Azo coupling
B. Diazotization
C. Amination
D. Nitration
E. Nitrosation
50. What reaction results in production of
phthalic anhydride?

240
krok123.in.ua
Krok 1 Pharmacy (англомовний варiант, iноземнi студенти) 2018 рiк 6

A. A. Pyrimidine nucleotides synthesis


B. Carbohydrate disintegration
C. Purine nucleotides synthesis
D. Purine nucleotides disintegration
E. Lipids synthesis
55. During assessment of air purity in an
aseptic unit of a phamacy, sedimentation
D-glucaric acid analysis had been applied. Test resulted
B. in growth of the small colonies with areas
of hemolysis. What medium was used for
inoculation?
A. Blood agar
B. Levine’s formulation (Eosin Methylene
Blue agar)
C. Endo agar
D. Ploskirev’s agar
D-gluconic acid E. Egg-yolk salt agar
C.
56. According to van’t Hoff rule, when the
temperature is raised by 10 degrees, the
reaction rate increases by:
A. 2-4 times
B. 1.5 times
C. 5 times
D. 10 times
D-glucuronic acid E. Temperature does not affect reaction rate
D.
57. What standard solution can be used to
standardize the solution of I2 ?
A. Sodium thiosulfate solution
B. Potassium iodide solution
C. Potassium dichromate solution
D. Potassium permanganate solution
D-galactaric (mucic) acid E. Sodium nitrite solution
E.
58. Catalysts are widely used in production
of drugs. How can reaction acceleration in
the presence of a catalyst be explained?
A. Activation energy decreases
B. Total collision frequency increases
C. Activation energy increases
D-arabinaric acid D. Collision frequency decreases
53. A patient, who lives in the area E. Molecule speed increases
with specific geochemical conditions, 59. On examination the patient’s sclera and
was diagnosed with endemic goiter. oral mucosa are icteric. What biochemi-
What microelement deficiency results in cal blood value can be expected to be
development of this pathology? increased?
A. I A. Bilirubin
B. Cl B. Amylase
C. Br C. Glucose
D. F D. Albumin
E. Na E. Cholesterol
54. An oncological patient was prescribed 60. Microorganisms in the environment are
fluorouracil that is a competitive inhibitor of being affected by various physical factors.
thymidine synthase. It inhibits the process What is the effect of high temperature on a
of: microbial cell?

241
krok123.in.ua
Krok 1 Pharmacy (англомовний варiант, iноземнi студенти) 2018 рiк 7

A. Irreversible degradation of all cellular A. To distribute between the mobile and


structures stationary phases
B. Mutagenic effect B. To distribute between two mobile phases
C. Transition into anabiosis state C. To distribute between two stationary
D. Albuminolysis phases
E. Fats saponification D. To dissolve
E. To precipitate
61. Extreme therapeutic effect of activated
carbon is due to its high specific surface 67. Quantitative content of oxalic
area. Name the phenomenon when gases acid can be determined by means of
are absorbed only by the surface of a solid permanganatometry. How to determine
body: equivalence point for this kind of titration?
A. Adsorption A. When titrate changes its color after
B. Adhesion another drop of process solution is added
C. Desorption B. With redox indicator diphenylamine
D. Cohesion C. With pH indicator
E. Recuperation D. With specific indicator
E. With adsorption indicator
62. Pastes are used in medicine to treat skin
diseases. What type of disperse systems are 68. Surfactants and high-molecular
they? compounds are added into concentrated
emulsions to stabilize them. These
A. Suspensions substances are:
B. Emulsions
C. Powders A. Emulsifiers
D. Foams B. Activators
E. Aerosols C. Catalysts
D. Solvents
63. In photometric analysis the series of 6-8 E. Absorbents
standard solutions is used:
69. Reaction of benzene sulfonation
A. To build a calibration curve produces:
B. To assess determination method
C. To simplify working method
D. To choose cuvettes
E. To choose optical filter
64. Ammonium iron(III) sulfate can be used
as an indicator in:
A. Argentometry, Volhard method
B. Argentometry, Mohr method
C. Alkalimetry
D. Acidimetry
E. Complexometric titration
65. Quantitative determination of iodine is
done by means of:
A. Redox titration
B. Alkalimetry
C. Complexometric titration
D. Acidimetry
E. Precipitation titration
66. Separation of substances in
chromatography is based on the ability of
solutes:

242
krok123.in.ua
Krok 1 Pharmacy (англомовний варiант, iноземнi студенти) 2018 рiк 8

A. trofurazone) the patient developed signi-


ficant labial edema. What type of allergic
reaction is observed in this case?
A. Anaphylactic
B. Delayed-type hypersensitivity
B. C. Cytolytic
D. Stimulated
E. Immune complex
72. A patient with gastric carcinoma has
undergone several courses of radiation
C. therapy. What system is the first to become
functionally disturbed after the body was
exposed to ionizing radiation?
A. Blood
B. Nervous
C. Digestive
D. D. Urinary
E. Respiratory
73. A patient with acute myocardial infarcti-
on received anticoagulation therapy. What
compound will have anticoagulation effect?
E.
A. Heparin
B. Hyaluronic acid
C. Chondroitin sulfate
D. Dermatan sulfate
E. Keratan sulfate

70. Choose the reaction of ester production 74. Specify what method of redox titration
among those listed below: requires the use of specific indicator - starch
- to fix the end point:
A.
A. Iodometry
B. Permanganatometry
C. Nitritometry
D. Cerimetry
E. Bromatometry
B.
75. Causative agents of infectious diseases
can be carried both by humans and animals.
Name the group of infections that affect ani-
mals and can be passed onto humans:
C.
A. Zooanthroponoses
B. Sapronoses
C. Anthroponoses
D. Zoonoses
D. E. Mixed
76. During regular check-up a patient
presents with enlarged thyroid gland,
exophthalmos, increased body temperature,
E. heart rate up to 110/min. What hormone
should be measured in the patient’s blood
in this case?
A. Thyroxin
B. Testosterone
C. Glucagon
71. A 25-year-old man has an appointment D. Insulin
with the dentist. Several minutes after his E. Cortisol
oral cavity was lavaged with furacilin (ni-
243
krok123.in.ua
Krok 1 Pharmacy (англомовний варiант, iноземнi студенти) 2018 рiк 9

77. A fruit tree of Rosaceae family has short conclusion additionally, a leaf of this plant
thorny shoots; the fruit is a distinctively- was examined under the microscope and a
shaped pome with stone cells in its pulp. search for the following crystalline inclusi-
Name this plant: ons was conducted:
A. Pyrus communis A. Raphides
B. Malus sylvestris B. Single crystals
C. Cerasus vulgaris C. Druse crystals
D. Prunus armeniaca D. Styloid crystals
E. Prunus spinosa E. Crystal sand
78. Pharmacopoeia reaction to determine 84. What type of conducting bundle
benzoate ions requires interaction with the is characteristic of primary anatomical
solution of: structure of a root?
A. Iron(III) chloride A. Radial
B. Potassium chloride B. Concentric
C. Resorcin C. Closed collateral
D. Acetic anhydride D. Bicollateral
E. Diphenylamine E. Open collateral
79. What solution can be used to detrmine 85. Calendula officinalis as a representative
the presence of chloride ions in the potable of Asteraceae family can be characterized
water? by the following type of inflorescence:
A. Silver nitrate A. Anthodium
B. Iodine B. Umbel
C. Potassium bromate C. Catkin
D. Sodium hydroxide D. Capitulum
E. Ammonia E. Corymb
80. Insulin production in β -cells involves 86. In the age of 5 months the child had
many substances. What substance gives the measles antibodies in the blood. By the age
main signal for insulin synthesis when its of 1 year these antibodies disappeared from
concentration changes? the child’s blood. Why were these antibodi-
es present in the child’s blood?
A. Glucose
B. Carbon dioxide A. Acquired natural passive immunity
C. Heparin B. Non-specific resistance
D. Hemoglobin C. Acquired natural active immunity
E. Urea D. Innate immunity
E. Artificial immunity
81. Specify the substance that results from
the following reaction: 87. A Gram stained smear shows large
HOH, Hg 2+ oval violet cells that form pseudomycelium.
CH ≡ CH −−−−−−−→? Name these microorganisms:
A. Ethanal A. Candida fungi
B. Ethanol B. Mucor fungi
C. Propanal C. Plasmodium vivax
D. Propanone D. Actinomycetales
E. Acetic acid E. Penicillium fungi
82. An ophthalmologist has detected 88. Herbarium specimens of medicinal
increased time of dark adaptation in a pati- plants are being studied. Which one of them
ent. What vitamin deficiency can result in belongs to Rosaceae family?
such symptom?
A. Crataegus sanguinea
A. A B. Melilotus officinalis
B. C C. Conium maculatum
C. K D. Capsella bursa-pastoris
D. B1 E. Polygonum persicaria
E. B6
89. When working in the garden, a man
83. Morphologically the herbaceous accidentally cut his hand. The wound
plant being studied can be identified remained untreated. Shortly after that the
as Convallaria majalis. To confirm this
244
krok123.in.ua
Krok 1 Pharmacy (англомовний варiант, iноземнi студенти) 2018 рiк 10

wounded area developed inflammation wi- A. Heating in the burner flame


th accumulation of exudate that contained B. Boiling under 60o C five times
numerous viable and degenerate neutrophi- C. Soaking in 1% chloramine-B solution
ls. What type of exudate is it? D. Dry heat sterilization under 160o C for
120-150 minutes
A. Purulent E. Formaldehyde vapor sterilization
B. Serous
C. Fibrinous 95. A dry-heat box is used for sterilization
D. Hemorrhagic of various materials and instruments in a
E. Catarrhal bacteriological laboratory. This sterilizati-
on method can be applied to the following
90. A patient presents with persistent fever, objects:
with the difference between evening and
morning temperature not exceeding 1o C . A. Glass test tubes
What type of fever curve is present in this B. Rubber gloves
patient? C. Simple nutrient medium
D. Wire inoculating loops
A. Continuous E. Physiological saline
B. Remittent
C. Hectic 96. A specialist of the analytical laboratory
D. Recurrent performs direct iodometric determination
E. Intermittent of ascorbic acid. What indicator is used in
this case?
91. Natural peptides can carry out various
functions. What bioactive peptide is a major A. Starch
antioxidant and functions as a coenzyme? B. Methyl orange
C. Diphenylamine
A. Glutathione D. Phenolphthalein
B. Bradykinin E. Methyl red
C. Oxytocin
D. Liberin 97. A certain herbaceous plant grows on
E. Anserine the meadows of the Carpathian Mountai-
ns. It has orange anthodium inflorescences,
92. Gout develops when purine nucleoti- upright stem, and a rosette of basal leaves.
de metabolism is disturbed. The doctor Name this plant:
prescribed the patient allopurinol that is a
competitive inhibitor of: A. Arnica montana
B. Cychorium intybus
A. Xanthine oxidase C. Calendula officinalis
B. Succinate dehydrogenase D. Echinacea purpurea
C. Alcohol dehydrogenase E. Centaurea cyanus
D. Lactate dehydrogenase
E. Hexokinase 98. What indicator is used for fixing the
endpoint of mercurimetric titration?
93. Name the serums made from blood
donated by volunteers or reconvalescent A. Thiocyanate complexes of iron(III)
donors: B. Fluorescein
C. Eosin
A. Homologous D. Murexide
B. Heterologous E. Potassium chromate
C. Autoimmune
D. Attenuated 99. Ultraviolet irradiation is used in medici-
E. Corpuscular ne in various physiotherapeutic procedures.
What mechanism of medicinal action is
94. During skill building session in the fi- characteristic of ultraviolet rays?
eld of microbiology, a student performed
inoculation of microorganisms into the solid A. Activation of vitamin D synthesis
nutrient medium to obtain isolated colonies. B. Activation of drug action
How should inoculation loops be sterilized C. Decrease of melanin synthesis in the skin
after that? D. Intensification of cell division
E. Activation of lipid peroxidation
100. Human body assimilates fats only as
emulsions. Vegetable oils and animal fats
contained in food are emulsified when
exposed to bile (an emulsifier). How does
245
krok123.in.ua
Krok 1 Pharmacy (англомовний варiант, iноземнi студенти) 2018 рiк 11

interface tension change in this case? A. Temperature increase


B. Temperature decrease
A. Decreases C. Continuous fever
B. Increases D. Compensation
C. Remains unchanged E. Latent stage
D. First increases, than decreases
E. First decreases, than increases 107. Ascarids were detected in a sick child.
What changes in leukogram will be the most
101. A patient has a gallstone lodged in the characteristic of helminthiasis?
common bile duct, which blocks bile supply
to the intestine. What digestive process will A. Eosinophilia
be disturbed in this case? B. Basophilia
C. Neutrophilia
A. Fat digestion D. Lymphocytosis
B. Protein absorption E. Monocytosis
C. Carbohydrate digestion
D. Carbohydrate absorption 108. A man received a radiation dose of
E. Protein digestion 30 Gy. He presents with necrotic angina
and disorders of the gastrointestinal tract.
102. You are studying a silvery downy plant Blood tests revealed anemia, leukopenia
of Asteraceae family, which is rich in essenti- and thrombocytopenia. What stage of acute
al oils and bitters. Harvested are the apical radiation sickness is observed in the pati-
sprouts with a panicle of small round flower ent?
heads. This plant is:
A. Manifest illness stage
A. Artemisia absinthium B. Prodromal stage
B. Arctium lappa C. Latent stage
C. Bidens tripartita D. Recovery
D. Calendula officinalis E. -
E. Chamomilla recutita
109. Neuroleptanalgesia has been appli-
103. A 46-year-old patient presents with ed in a case of cardiac infarction. What
hyperactivity of creatine kinase in his blood neuroleptic is most often administered
serum. What pathology can be suspected? along with fentanyl?
A. Myocardial infarction A. Droperidol
B. Acute pancreatitis B. Perphenazine (Aethaperazinum)
C. Chronic hepatitis C. Levomepromazine
D. Hemolytic anemia D. Clozapine
E. Renal failure E. Sulpiride
104. Among dosage forms there are 110. Select the halogenated antiseptic that
numerous disperse systems. Select a free would be preferable for a child to pack in
disperse system from the list: the first aid kit, when going to a summer
camp:
A. Emulsion
B. Gel A. Iodine alcoholic solution
C. Jelly B. Brilliant green
D. Diaphragm C. Copper sulfate
E. Membrane D. Methylene blue
E. Formaldehyde solution
105. How according to the Pharmacopoeia
is pH determined? 111. A patient, who was prescribed famoti-
dine to treat peptic ulcer disease, came to
A. Potentiometry the pharmacy. What is this drug’s mechani-
B. Spectrophotometry sm of action?
C. Indicator
D. Conductometry A. H2-histamine receptor blockade
E. Polarography B. H1-histamine receptor blockade
C. Muscarinic receptor blockade
106. A patient is pale, has goose bumps and D. Inhibition of hydrogen potassium ATPase
chills. What stage of fever is it characteristic E. Ganglionic receptor blockade
of?
112. What is represented by such a
pharmacokinetic value of a drug as its bi-
ological half-life (T1/2)?
246
krok123.in.ua
Krok 1 Pharmacy (англомовний варiант, iноземнi студенти) 2018 рiк 12

118. Name the ability of a drug to


A. Time period in which plasma drug accumulate within the patient’s body:
concentration decreases by 50%
B. Blood plasma volume cleared of drug A. Cumulation
within a time unit B. Antagonism
C. Period of total body clearance C. Synergism
D. Renal clearance rate D. Habituation
E. Correlation between the drug clearance E. Allergy
rate and plasma drug concentration
119. What pharmacological effect of
113. A doctor needs to prescribe the pati- acetylsalicylic acid allows its application
ent a drug for replacement therapy after in patients with ischemic heart disease for
thyreoidectomy. What drug would you prevention of thromboses?
recommend?
A. Antiaggregant
A. L-thyroxine B. Analgesic
B. Insulin C. Antipyretic
C. Prednisolone D. Ulcerogenic
D. Parathyroidin E. Anti-inflammatory
E. Thiamazole
120. A 48-year-old patient has been
114. What is the main mechanism of intravenously administered prednisolone
benzylpenicillin bactericidal action on the solution to arrest severe attack of bronchi-
coccal flora? al asthma. What group of hormonal agents
does prednisolone belong to?
A. Disturbed synthesis of microbial cell wall
B. Inhibition of protein synthesis A. Glucocorticoids
C. Disturbed cytoplasmic membrane B. Gestagenic drugs
permeability C. Estrogenic drugs
D. Activation of macroorganism immune D. Mineralocorticoid
system E. Anabolic steroids
E. Increased phagocytic activity of
leukocytes 121. A doctor has prescribed a nonsteroi-
dal anti-inflammatory drug to relieve
115. What mucolytic agent would you inflammation and pain syndrome. Name
recommend for the patient with acute this drug:
bronchitis to facilitate expectoration?
A. Diclofenac sodium
A. Acetylcysteine B. Glibenclamide
B. Glaucine C. Loratadine
C. Codeine D. Prednisolone
D. Libexin (Prenoxdiazine) E. Calcium chloride
E. Hydrocodone
122. A patient suffers from hyperchromic
116. A patient with allergic dermatitis came B12 -deficiency anemia. What vitamin
to the hospital. What anti-inflammatory and preparation should be prescribed in this
anti-allergic drug must be prescribed in this case?
case?
A. Cyanocobalamin
A. Prednisolone B. Riboflavin
B. Ethamide C. Vicasol (Menadione)
C. Oxytocin D. Thiamine chloride
D. Insulin E. Retinol acetate
E. Retabolil (Nandrolone)
123. To stimulate birth activity, a certain
117. A patient with bronchial asthma was neurohypophyseal hormone is used. Name
prescribed a drug with the mechanism of this hormone:
action that is primarily based on the sti-
mulation of β2 adrenergic receptors. Name A. Oxytocin
this drug: B. Insulin
C. Glucagon
A. Salbutamol D. Thyroxine
B. Adrenaline hydrochloride E. Testosterone
C. Droperidol
D. Clonidine 124. Sanitary microbiological investigation
E. Isadrine (Isoprenaline) of potable water has detected coliphages.
247
krok123.in.ua
Krok 1 Pharmacy (англомовний варiант, iноземнi студенти) 2018 рiк 13

What conclusion can be made about the


sanitary-hygienic status of this water? A. Polyadelphous
B. Monadelphous
A. Fecal contamination C. Diadelphous
B. The water is safe to drink D. Didynamous
C. The water is safe to drink after boiling E. Tetradynamous
D. Artesian water
E. The water is for industrial use only 131. Upon examination of a flower it is
determined to have one pistil made up of
125. After a traffic accident the driver single free carpel. Therefore, this gynoeci-
presents with increased blood glucose. What um can be identified as:
mechanism leads to hyperglycemia in this
case? A. Monocarpous
B. Apocarpous
A. Sympathoadrenal system activation C. Lysicarpous
B. Increased production of somatotropic D. Paracarpous
hormone E. Syncarpous
C. Decreased production of insulin
D. Decreased production of glucagon 132. Examination of a 45-year-old man, who
E. Decreased tone of parasympathetic for a long time kept to a vegetarian plant-
nervous system based diet, revealed him to have negative
nitrogen balance. What peculiarity of his di-
126. Quantitative determination of iodi- et has caused such developments?
des by Fajans method is performed with
adsorption indicators. The following can be A. Insufficient protein content
used as an adsorption indicator: B. Insufficient fat content
C. Insufficient vitamin content
A. Eosin D. Excessive water content
B. Methyl orange E. Excessive carbohydrate content
C. Phenolphthalein
D. Diphenylamine 133. After severe emotional strain a 53-
E. Murexide year-old man suddenly developed acute
pain in the heart area, which irradiates to
127. Pharmacopoeia reaction of potassium the left hand, to the neck, and under the
ferrocyanide with zinc cations produces: left scapula. He noted numbness of his
left hand. His face is pale and covered in
A. White precipitate cold sweat. Nitroglycerine administration
B. Red precipitate stopped the pain attack after 10 minutes
C. Violet precipitate had passed. What is the most likely disease
D. Yellow precipitate in this case?
E. Black precipitate
A. Angina pectoris
128. Rapid analysis of benzoate ions by B. Stroke
means of Pharmacopoeia reaction with C. Myocardial infarction
iron(III) chloride produces: D. Pulmonary embolism
E. Somatoform autonomic dysfunction
A. Pink-yellow precipitate
B. Green precipitate 134. According to the Pharmacopoeia,
C. Blue precipitate molecular weight of a high-molecular
D. Red precipitate substance should be determined by means
E. Black precipitate of:
129. The following is used to determine the A. Viscometry
titrant volume in the process of titrimetric B. Potentiometry
analysis: C. Nephelometry
D. Osmometry
A. Burettes E. Cryometry
B. Measuring flasks
C. Measuring glasses 135. In the process of coagulation by mi-
D. Cylinders xtures of different electrolytes, they seem
E. Measuring tubes to counteract each other’s effect. Name this
phenomenon:
130. A dissected flower has numerous
stamens that are united by the stamen fi-
laments into several bundles. What is this
type of androecium?
248
krok123.in.ua
Krok 1 Pharmacy (англомовний варiант, iноземнi студенти) 2018 рiк 14

A. Antagonism A. Angiotensin-receptor blockade


B. Additivity B. Inhibition of angiotensin-converting
C. Synergism enzyme
D. Mutual coagulation C. Inhibition of phosphodiesterase
E. Sedimentation D. Activation of central α-adrenoceptors
E. Calcium channel blockade
136. One of the important diagnostic
features of garden sage and motherwort is 141. Select the hepatoprotective drugs from
their shape of corolla. Their flowers have the list below:
the following type of corolla:
A. Essentiale (Phospholipides), Thiotriasoli-
A. Bilabiate ne
B. Thimble-shaped B. No-Spa (drotaverine), papaverine
C. Funnelform hydrochloride
D. Pseudoligulate C. Allochol, Cholenzym
E. Ligulate D. Festal, Panzinorm (Pancreatin)
E. Oxaphenamide (Osalmid), Nicodin
137. Investigation of bacterial contaminati-
on of indoor air in a pharmacy takes into 142. Hydrochloric acid was added into the
account the total number of microorgani- solution under investigation. The resulting
sms present in a certain air volume, as well precipitate was filtered, then this filter cake
as qualitative content of indoor air mi- was processed with hot water; after the fi-
croflora. Name the sanitary-indicative mi- ltrate cooled, KI solution was added into it.
croorganisms for indoor air: What cation was present in the solution, if
the precipitate was colored yellow?
A. Staphylococcus and streptococcus
B. Colibacillus A. P b2+
C. Sarcina B. Ag +
D. Chromobacterium C. Hg 2+
E. Fungi and yeasts
D. Ca2+
138. During ultrasound investigation a pati- E. Ba2+
ent was diagnosed with bilateral renal artery
stenosis of atherosclerotic genesis. Speci- 143. At the end of his shift a worker of
fy the bioactive substance that due to its the steel foundry felt dizziness, his body
excessive secretion is the key component of temperature increased to 38.5oC . What
arterial hypertension pathogenesis in the gi- condition does he present with?
ven case: A. Hyperthermia
A. Renin B. Decompression
B. Cortisol C. Fever
C. Vasopressin D. Hypothermia
D. Noradrenaline E. Hypertension
E. Thyroxin 144. The third analitycal group of cations
139. Preliminary disinfection of air and (acid-base classification) includes Ca2+ ,
working surfaces of the equipment was Sr 2+ , Ba2+ . What acid can function as a
conducted in the operating room of the precipitator agent (group reagent) for these
surgical inpatient unit. What method of cations?
sterilization would be the most advisable
in this case? A. H2 SO4
B. HNO3
A. Ultraviolet irradiation C. HCI
B. Irradiation sterilization D. CH3 COOH
C. High-frequency current E. HCIO4
D. Flowing steam
E. Formaldehyde vapor 145. Students should identify the following
to determine the sex of a flower:
140. A patient was prescribed losartan for
treatment of arterial hypertension. What A. Stamens and pistils
mechanism of action does this drug have? B. Flower cup and corolla
C. Pedicle and receptacle
D. Symmetry
E. Color and type of indumentum
146. In spring a perennial plant of
249
krok123.in.ua
Krok 1 Pharmacy (англомовний варiант, iноземнi студенти) 2018 рiк 15

Asteraceae family produces floral shoots wi- A. Collargol


th gloden-yellow flowers. After blossom- B. Sodium sulfate
fall, shoots with large leaves appear. Name C. Silver nitrate
this plant: D. Potassium gluconate
E. Sucrose
A. Tussilago farfara
B. Нiреriсum perforatum 152. Paracetamol has antipyretic and
C. Potentilla erecta analgesic effect. In the human body it is
D. Petroselinum crispum neutralized in the following organ:
E. Datura stramonium
A. Liver
147. Ultramicroscopy is used to determine B. Spleen
the radius of dispersed phase particles. The C. Intestine
following should be measured to make the D. Lungs
necessary calculations: E. Heart
A. Number of particles in a definite volume 153. Rhizome of an Asteraceae family speci-
B. Intensity of transmitted light es is polycephalous, succulent, has lysi-
C. Intensity of scattered light genous cavities, accumulates inulin. Such
D. Distance traveled by a tagged particle underground organ is characterisic of:
E. Time interval in which a tagged particle
travels a certain distance A. Inula helenium
B. Hyoscyamus niger
148. Permanganatometry is used in determi- C. Digitalis grandiflora
nation of many organic and inorganic D. Sorbus aucuparia
compounds. What are the main advantages E. Helianthus annuus
of permanganatometry over the other oxi-
dimetric methods? 154. Silver nitrate solution was added into
a solution with anions of the first analytical
A. Sufficiently high redox potential; it is group. A yellow precipitate was produced as
possible to determine titration end-point the result, which indicates that this solution
without indicator contained:
B. Sufficiently high stability of potassium
permanganate and its solutions A. Arsenite ions
C. High selectivity and sensitivity when B. Arsenate ions
determining compounds C. Sulfate ions
D. Pure potassium permanganate is easily D. Iodide ions
available and obtainable E. Bromide ions
E. Various types of indicators can be used; 155. Some medicinal plants are poisonous.
in some cases catalysts are necessary to Select a poisonous plant from the list below:
accelerate the reaction
A. Digitalis purpurea
149. Many organic compounds break up B. Origaum vulgare
in the cell into simple products. What C. Thymus serpilum
compounds break up into ammonia, carbon D. Salvia officinalis
dioxide, and water in the human body? E. Thymus vulgaris
A. Amino acids 156. What cation can be detected with
B. Monosaccharides Chugaiev’s agent (Dimethylglyoxime)?
C. Monohydric alcohols
D. Fatty acids A. Ni2+
E. Keto acids B. Ca2+
150. A fruit is a capsule with oblate li- C. +
ght brown smooth glossy seeds that mucify D. Mn2+
when moistened. This fruit belongs to: E. Co2+

A. Linum usitatissimum 157. A patient in the state of ketoacidotic


B. Hypericum perforatum coma presents with loud rapid respirati-
C. Ledum palustre on: labored expiration with tension of expi-
D. Linaria vulgaris ratory muscles occurs after deep inspiration.
E. Digitalis purpurea Name this type of pathologic respiration:

151. What substance forms colloid solution


when dissolved in water?

250
krok123.in.ua
Krok 1 Pharmacy (англомовний варiант, iноземнi студенти) 2018 рiк 16

A. Kussmaul’s 161. Specify the products of urea hydrolysis:


B. Cheyne-Stokes’
C. Gasping
D. Stenotic
E. Biot’s
158. In a nursery-garden some medicinal
plants developed signs of a disease: there A. CO2 + 2NH3
are yellow spots and necrotic foci on the B. CO2 + N2 + 3H2
leaves. Sap of the diseased plants remai- C. CO + 2NH3
ned infectious even after passing through D. CO + N2 + 3H2
a bacteria-excluding filter. No microorgani- E. CO2 + N2 + 3H2 O
sms growth was detected on the nutrient
medium. What microorganisms could be the 162. Specify the products obtained as the
cause of this disease? result of formic acid being heated with
concentrated sulfuric acid:
A. Viruses
B. Fungi
C. Ray fungi
D. Bacteria
E. Mycoplasma
A. CO + H2 O
159. A laboratory received ethanol and B. CO2 + H2 O
methanol. What reaction can be used to di- C. CO + CO2 + H2 O
stinguish between these two substances? D. CO2 + H2
A. Iodoform test (I2 + NaOH ) E. CO + H2
B. Beilstein test 163. Transformation C2 H4 (alkene) −→
C. Formation of a chelate complex with
copper hydroxide C2 H6 (alkane) occurs during the followi-
ng reaction:
D. Oxidation (CrO3 , H2 SO4 )
E. Reaction with halogen anhydrides of A. Hydrogenation
inorganic acids B. Dehydrogenation
C. Dehydration
160. Select pyridine from the listed D. Hydration
compounds: E. Dimerization
A. 164. Hydrolysis reaction will NOT occur wi-
th:
A. Glycerol
B. Starch
B. C. Cellulose
D. Fat
E. Protein
165. What reaction proves that phenol has
acidic properties?

C.

D.

E.

251
krok123.in.ua
Krok 1 Pharmacy (англомовний варiант, iноземнi студенти) 2018 рiк 17

A. A. Rimantadine
B. Methisazone
C. Levamisole
D. Idoxuridine
E. Doxycycline

B. 169. A woman complains of itching lips;


they are reddened and covered in scabs and
scales after she had been using new lipstick
for two weeks. What allergic reactions result
in this kind of disorders?
A. Delayed
B. Cytotoxic
C. C. Immune complex
D. Anaphylactic
E. Stimulating
170. Choose the potent fast-acting diuretic
to induce forced diuresis:
D.
A. Furosemide
B. Hydrochlorothiazide
C. Spironolactone
D. Triamterene
E. Acetazolamide
171. Name the psychostimulant with
E. analeptical action, which is a purine deri-
vative:
A. Caffeine and sodium benzoate
B. Tramadol
C. Medazepam
D. Sodium bromide
166. Select ketose from the monosacchari- E. Sulpiride
des listed below:
172. After ishemic stroke the patient was
A. Fructose prescribed a drug to improve his intellectual
B. Glucose functioning and memory. What drug would
C. Mannose he obtain in the pharmacy?
D. Arabinose
E. Ribose A. Piracetam
B. Metoclopramide
167. According to Hueckel’s rule an organic C. Tabex (Cytisine)
compound will have aromatic properties if: D. Diphenin (Phenytoin)
A. Its molecular structure contains a planar E. -
cycle with a closed conjugated system that 173. A patient with a small cut on the palm
contains (4n+2) of pi electrons, where n = came to the dispensing chemist. What anti-
0,1,2,3, etc. septic would be advisable in this case?
B. Its molecules are composed exclusively
of carbon and hydrogen atoms that form a A. Hydrogen peroxide
linear carbon chain B. Doxycycline hydrochloride
C. There is only one substituent in the C. Ketoconazole
molecule D. Lidocaine hydrochloride
D. There are condensed nuclei in the E. Flemoxin (Amoxicillin)
molecule
E. There is a cyclohexane ring in the 174. Mother of a 10-year-old child came to
molecule the pharmacy to obtain a drug for preventi-
on of upper respiratory tract infections.
168. A pharmacy has received a batch of What drug would be recommended by the
drugs for treatment of upper respiratory dispensing chemist?
tract infection. What drug is used to treat
influenza?

252
krok123.in.ua
Krok 1 Pharmacy (англомовний варiант, iноземнi студенти) 2018 рiк 18

A. Interferon
B. Benzoteph A.
C. Carvedilol
D. Tetracycline
E. Doxorubicin
175. A patient came to the pharmacy to
obtain an antidiarrheal agent. What drug B.
would be recommended by the dispensing
chemist?
A. Loperamide
B. Dicaine (Tetracaine) C.
C. Ranitidine
D. Picolax (Sodium picosulfate)
E. Anesthesin (Benzocaine)
176. A patient came to the pharmacy to
obtain a drug that contains pancreatic D.
enzymes and can be taked for chronic
pancreatitis. What drug would be
recommended by the dispensing chemist?
A. Pancreatine
B. Triamcinolone
C. Gordox (Aprotinin) E.
D. Pirenzepine
E. Omeprazole
177. Increased concentration of acti-
ve oxygen forms is a mechanism of
pathogenesis in a number of diseases.
To prevent this process, antioxidants are 181. When an isolated system spontaneously
prescribed. Select an antioxidant from the approaches its equilibrium, its entropy:
list below:
A. Approaches zero
A. α-tocopherol B. Approaches infinity
B. Glucose C. Reaches maximum
C. Calciferol D. Reaches minimum
D. Cobalamine E. Demonstrates linear magnification
E. Glicerol
182. A pharmacy produces a batch of vials
178. A 28-year-old man with peptic ulcer of with physiological saline for injections. How
the stomach was prescribed a drug that inhi- should they be sterilized?
bits gastric juice secretion. Specify this drug:
A. Under pressure in an autoclave
A. Omeprazole B. In a steam-jacketed autoclave chamber
B. Ethacrynic acid C. In a dry heat sterilizer
C. Duphalac (Lactulose) D. X-ray irradiation
D. Lidocaine E. Ultraviolet irradiation
E. Fenofibrate
183. A certain dioecious plant commonly
179. An engine driver complains of his grows at the forest edge. It is a shrub with
seasonal allergy symptoms. What non- thorned sprouts. Its fruit is a round black
sedating drug should be prescribed in this coenocarpous drupe (pyrenarium) with 3-4
case? seeds. Name this plant:
A. Loratadine A. Rhamnus cathartica
B. Novocaine B. Hippophae rhamnoides
C. Fenofibrate C. Crataegus sanguinea
D. Analgine (Metamizole) D. Rosa canina
E. Atenolol E. Sambucus nigra
180. Salicylic acid derivatives are widely 184. Choose the weakest carboxylic acid
used in medicine. Specify the formula of basing on its pKa value:
salicylic acid:
253
krok123.in.ua
Krok 1 Pharmacy (англомовний варiант, iноземнi студенти) 2018 рiк 19

A. Propionic acid (рКа = 4.9) A. Suberization


B. Acetic acid (рКа = 4.7) B. Lignification
C. Formic acid (рКа = 3.7) C. Mineralization
D. Lactic acid (рКа = 3.9) D. Cutinization
E. Butyric acid (рКа = 4.82) E. Mucification

185. Which of the amines listed below is a 191. Specify the analgesic that affects opi-
primary amine? ate receptors and can cause development of
tolerance and dependence:
A. C6 H5 CH2 NH2
B. C6 H5 CH2 NHCH3 A. Morphine
C. C6 H5 NHCH3 B. Phenobarbital
D. C6 H5 N(CH3 )2 C. Medazepam
E. C6 H5 CH2 N(CH3 )2 D. Voltaren (Diclofenac sodium)
E. Haloperidol
186. It can be safely assumed that the infants
born from the mothers with the history of 192. During furosemide therapy of a pati-
measles will not be affected by the measles ent with chronic edematous syndrome, his
outbreak during their stay in the materni- plasma-cation concentration was disturbed.
ty ward. What classes of antibodies provide What drug should be used in this case?
the infants with the resistance to this di- A. Potassium chloride
sease? B. Thiamine bromide
A. IgG C. Ascorutin (Ascorbic acid + Rutoside)
B. IgA D. Magne B6
C. IgD E. Aspirin
D. IgM 193. Upon examination the ophthalmologi-
E. IgE st diagnosed a 21-year-old woman with vi-
187. The children attending a kindergarten sual impairment - hemeralopia (”night bli-
were hospitalized with diagnosis of poli- ndness”). What drug should this patient
omyelitis. What was the route of infection take to restore her vision?
transmission in this case?
A. Retinol acetate
A. Fecal-oral transmission B. Ergocalciferol
B. Alimentary transmission C. Suprastin (Chloropyramine)
C. Direct contact transmission D. Cholecalciferol
D. Transmission via airborne dust particles E. Sustac forte (Nitroglycerin)
E. Vector-borne transmission
194. What bonds participate in creation of
188. What ion increases osmotic pressure in both linear and cyclic dimeric carboxylic
the focus of inflammation? acid associates?
A. Potassium
B. Calcium
C. Fluorine
D. Magnesium
E. Chlorine A. Hydrogen bonds
B. Ionic bonds
189. A 55-year-old man came to a doctor wi- C. Polar covalent bonds
th complaints of acute pain in his big toes. D. Nonpolar covalent bonds
Meat and wine are a permanent fixture in E. Donor-acceptor bonds
his diet. The doctor suspects gout. What
substance must be measured in the patient’s 195. Heating of sodium phenolate in
blood to confirm this diagnosis? CO2 stream results in production of a
certain carboxylic acid. Name the resulti-
A. Uric acid ng compound:
B. Urea
C. Lactate A. Salicylic acid
D. Bilirubin B. Ethyl salicylate
E. Ketone bodies C. Phenyl salicylate
D. Benzoic acid
190. Name the process of cell membrane E. Aminophenol
saturation with a fat-like substance -
suberin: 196. Which alkadiene of those listed below
254
krok123.in.ua
Krok 1 Pharmacy (англомовний варiант, iноземнi студенти) 2018 рiк 20

is a diene with cumulated double bonds?


A. CH2 = C = CH2
B. CH3 − CH = CH − CH2 − CH = CH2
C. CH2 = CH − CH2 − CH = CH2
D. CH2 = CH − CH2 − CH2 − CH = CH2
E. CH2 = CH − CH = CH2
A. 4-О-(α-D-glucopyranoside)-α-D-
197. What compound will be produced as glucopyranose
the result of toluene chlorination in the B. 4-О-(α-D-glucopyranoside)-β -D-
presence of F eCl3 catalyst? glucopyranose
C. 4-О-(β -D-glucopyranoside)-α-D-
glucopyranose
D. 4-О-(β -D-glucopyranoside)-β -D-
glucopyranose
E. α-D-glucopyranoside-β -D-
fructofuranoside

A. 199. What compound is synthesized as the


result of β -picoline oxidation?

A. Nicotinic acid
B. B. Benzoic acid
C. Uric acid
D. Barbituric acid
E. Ascorbic acid
200. Specify quinoline among the
compounds given below:
C.
A.

B.
D.

C.

E. D.

E.

198. Select α-maltose from the list:

255
krok123.in.ua

You might also like